physiology study

Lakukan tugas rumah & ujian kamu dengan baik sekarang menggunakan Quizwiz!

7) Antagonistic control of efferent output is typical of the __________ division. A) somatic B) sensory C) autonomic D) A and B E) A and C

Answer: C

7) The element 4He contains A) four protons. B) three protons and one neutron. C) two protons and two neutrons. D) two protons and two electrons. E) four neutrons.

Answer: C

29) If channels are permeable to A) Na+. B) K+. C) Ca2+. D) A and B E) A, B, and C

Answer: D

150) In typical women of reproductive age, the epithelial cells lining the uterus (the endometrium) die and are shed from the body roughly once every month, in the process of menstruation. The triggers in this process include chemical (hormonal) changes and contraction of the blood vessels in the lining. Cell death may be a result of either apoptosis or necrosis. Give the characteristics of each process. Devise an argument for classifying the process of menstruation as an example of apoptosis, and then argue for classifying it as necrosis. Which is correct?

...

53) What are the three levels of nervous system that control movement? Briefly describe their roles.

Answer: 1. the spinal cord, which integrates spinal reflexes and contains the central pattern generators 2. the brain stem and cerebellum, which control postural reflexes and hand and eye movements 3. the cerebral cortex and basal ganglia, which are responsible for voluntary movements

104) stimulates bile release

Answer: B

121) cilium

Answer: B

14) Periodic pulsatile secretion of GnRH appears to be important in A) delaying sexual maturation and puberty. B) preventing down-regulation of GnRH receptors. C) development of muscle responses. D) correct growth of coronary arteries.

Answer: B

21) Glucose and amino acids are reabsorbed by A) diffusion. B) symport with sodium. C) cotransport. D) countertransport.

Answer: B

21) Myogenic autoregulation means that A) increased blood pressure in a vessel triggers relaxation in that vessel. B) stretched smooth muscle in a blood vessel constricts reflexively. C) blood vessel diameter is adjusted by centers in the brain through monitoring blood pressure in areas throughout the body. D) A and C E) B and C

Answer: B

21) Which of the following is NOT true about angiotensin II? A) increases cardiac output B) activates parasympathetic output C) is a potent vasoconstrictor D) elevates blood pressure E) stimulates thirst

Answer: B

21) primary sympathetic neurotransmitter

Answer: B

22) A condition that produces a reduction in bone mass or density sufficient to compromise normal function is A) cretinism. B) osteoporosis. C) osteomyelitis. D) osteitis. E) acromegaly.

Answer: B

22) decreased surface area for gas exchange

Answer: B

34) Dominates during resting-and-digesting activities.

Answer: B

71) stretch

Answer: B

109) hormone that decreases iron absorption

Answer: C

130) basal lamina

Answer: C

140) Diffuse light illuminates both center and surround of on-surround cell.

Answer: C

51) the muscle cell membrane

Answer: C

61) Ventricular contraction A) begins during the first part of the P wave. B) begins just after the T wave. C) begins just after the Q wave. D) begins during the latter part of the P wave. E) none of the above

Answer: C

62) Dehydration may cause some ions to become concentrated. If a person was suffering from severe hyperkalemia, you would expect A) the potassium ion concentration of the interstitial fluid to be less than normal. B) the membrane potential of nerves and muscles to be more negative. C) the skeletal muscles to be unresponsive and cardiac arrest could occur. D) muscle weakness and increased strength of twitch contractions. E) all of the above

Answer: C

62) Information stored in the nucleus is translated into A) carbohydrates. B) lipids. C) proteins. D) phospholipids. E) none of the above

Answer: C

62) The heaviest organ in the body is the A) liver. B) brain. C) skin. D) stomach. E) urinary bladder.

Answer: C

64) Photoreceptors secrete the neurotransmitter A) acetylcholine. B) norepinephrine. C) glutamate. D) dopamine.

Answer: C

65) ventricular repolarization

Answer: C

71) their loss is characteristic of cancer

Answer: C

76) Harry suffers from cystic fibrosis and frequently has periods where he can hardly breathe. The problem is the result of A) inflammation of the bronchi. B) constriction of the trachea. C) thick secretions that exceed the ability of the mucus elevator to transport them. D) laryngospasms that occur in response to a toxic substance produced by the epithelial cells. E) collapse of one or both lungs.

Answer: C

77) In an experimental situation, a virus is injected into a rabbit and the rabbit is allowed to make antibodies for the viral antigen. These antibodies are then removed from the rabbit plasma and injected into a human to help combat the same viral disease. This is an example of A) innate immunity. B) active immunization. C) passive immunization. D) natural immunity. E) autoimmunity.

Answer: C

78) working on a dock, loading and unloading boxes into trucks all day

Answer: C

8) Cell membranes are said to be A) impermeable. B) freely permeable. C) selectively permeable. D) actively permeable. E) none of the above

Answer: C

8) The flexion reflex A) prevents a muscle from overstretching. B) prevents a muscle from generating damaging tension. C) moves a limb away from a painful stimulus. D) makes adjustments in other parts of the body in response to a particular stimulus. E) is an example of a monosynaptic reflex.

Answer: C

8) The most practical way to estimate a person's basal metabolic rate is to measure it when the person is A) sleeping. B) resting after a large meal. C) resting after a 12-hour fast.

Answer: C

9) Angiogenesis is A) an examination of the arteries and veins. B) having blood drawn into a tube for tests. C) the growth of new blood vessels. D) surgical restructuring of the coronary arteries. E) being able to detect a pulse in arteries.

Answer: C

9) The airway between the larynx and the primary bronchi is the A) pharynx. B) bronchiole. C) trachea. D) alveolar duct. E) laryngeal duct.

Answer: C

9) The driving force for blood flow is a(n) __________ gradient. A) osmotic B) volume C) pressure D) gravity

Answer: C

9) The nutrients that yield the most energy per gram when metabolized are A) carbohydrates. B) proteins. C) fats. D) nucleic acids. E) vitamins.

Answer: C

90) Arterioles: systemic dilate, pulmonary constrict.

Answer: C

153) Write the chemical formula for the molecule drawn below. Which class of organic molecule does it belong to? Is it most likely polar or nonpolar?

Answer: C11H12N2O2. The presence of the carboxylic acid (COOH) and amine (NH2) indicates this is an amino acid. Because of the R group structure, it is relatively nonpolar (this amino acid is tryptophan).

56) Sabrina's resting heart rate is 80 beats per minute, and her resting stroke volume is 50 mL per beat. What is her cardiac output? When she exercises, her heart rate increases to 100 beats per minute and her stroke volume increases to 55 mL per beat. What happens to her cardiac output? What is the advantage of the lower relative increase in stroke volume?

Answer: CO = HR × SV. CO = 80 beats/min × 50 mL/beat = 4000 mL/min at rest. During exercise: 100 beats/min × 55 mL/beat = 5500 mL/min. Overstretching of the heart muscle during venous return could damage it; thus a relatively higher heart rate is a safer way to increase cardiac output.

95) An adult with cretinism has incomplete manifestation of puberty (e.g., scant pubic hair), abnormal facial features, dwarfism, and mental retardation. Explain what causes these symptoms. How is cretin dwarfism different from pituitary dwarfism? How could cretinism be prevented?

Answer: Cretinism results from lack of thyroid hormones during early childhood. Thyroid hormones are permissive for growth hormones and play an important role in the development of the nervous system. The incomplete puberty may be related to lack of thyroid hormone and growth hormone effects on the sexual tissues. Pituitary dwarfism results from lack of growth hormone, unrelated to thyroid hormones. Prevention of thyroid cretinism is aimed at adequate prenatal care (especially dietary iodine) and administration of exogenous thyroid hormones to the infant.

10) Angiostatin and endostatin may be useful in the treatment of A) myocardial infarction. B) hypertension. C) hypotension. D) cancer. E) vasovagal syncope.

Answer: D

10) Hormones that increase in concentration in the plasma during exercise include A) cortisol. B) catecholamines. C) insulin. D) A and B E) A, B, and C

Answer: D

16) The common passageway shared by the respiratory and digestive systems is the A) larynx. B) glottis. C) vestibule. D) pharynx. E) esophagus.

Answer: D

16) The erectile tissue that surrounds the urethra is the A) membranous urethra. B) penile urethra. C) glans penis. D) corpus spongiosum. E) corpus cavernosum.

Answer: D

17) Juxtaglomerular cells in the nephron secrete A) angiotensinogen. B) angiotensin I. C) aldosterone. D) renin. E) angiotensin converting enzyme.

Answer: D

17) Sandwiched between the layer of circular and longitudinal muscle in the muscularis externa is the A) mucosa. B) submucosa. C) muscularis mucosa. D) myenteric plexus. E) submucosal plexus.

Answer: D

17) The Hering-Breuer reflex A) functions to increase ventilation with changes in blood pressure. B) alters pulmonary ventilation when the PO2 changes. C) alters pulmonary ventilation when the PCO2 changes. D) prevents overexpansion of the lungs. E) is an important aspect of normal, quiet breathing.

Answer: D

18) Lymphocytes and plasma cells A) mediate the acquired immune response. B) are found primarily in lymphoid tissue. C) all function and behave the same way. D) A and B E) A, B, and C

Answer: D

18) Which is NOT a kidney filtration barrier? A) glomerular capillary endothelium B) basal lamina C) Bowman's capsule epithelium D) juxtaglomerular apparatus

Answer: D

19) During exercise, sweating helps lose heat from the body through the process of A) radiation. B) conduction. C) convection. D) evaporation. E) concentration.

Answer: D

19) In humans, only __________ cells have flagella. A) respiratory tract lining B) intestine lining C) stomach lining D) sperm E) two of the above

Answer: D

19) Peyer's patches are characteristic of the A) stomach. B) esophagus. C) pancreas. D) small intestine. E) colon.

Answer: D

19) Stimuli for the activation of the RAAS pathway include A) low blood pressure in arterioles in the nephron. B) a decrease in fluid flow through the distal tubule. C) high blood pressure in the renal artery. D) A and B E) A, B, and C

Answer: D

19) Which organ is NOT part of the cardiovascular system and plays an important role in regulating blood pressure? A) liver B) spleen C) lung D) kidney E) skin

Answer: D

20) Features of the submucosa include A) blood and lymph vessels. B) a major nerve network. C) Peyer's patches. D) A and B E) A, B, and C

Answer: D

20) Interstitial cells produce A) sperm. B) inhibin. C) nutrients. D) androgens. E) androgen-binding protein.

Answer: D

20) The weak interactions between atoms that keep atoms near each other but don't tightly bind them together are called A) hydrogen bonds. B) van der Waals forces. C) ionic bonds. D) A and B E) B and C

Answer: D

21) A hemoglobin molecule is composed of A) two protein chains. B) three protein chains. C) four protein chains and nothing else. D) four protein chains and four heme groups. E) four heme groups but no protein.

Answer: D

22) An enzyme that joins monomers together is A) hydrolase. B) lipase. C) lyase. D) ligase. E) kinase.

Answer: D

23) Each amino acid differs from others in the A) number of central carbon atoms. B) size of the amino group. C) number of carboxyl groups. D) chemical structure of the R group. E) number of peptide bonds in the molecule.

Answer: D

23) The flattening of the action potentials of myocardial contractile cells, called the plateau phase, is due to a combination of __________ K+ permeability and __________ Ca2+ permeability. A) increasing, increasing B) decreasing, decreasing C) increasing, decreasing D) decreasing, increasing

Answer: D

24) organ systems

Answer: D

25) The average length of the menstrual cycle is A) 10 days. B) 14 days. C) 21 days. D) 28 days. E) 35 days.

Answer: D

25) The end of the plateau phase is due to the __________ of Ca2+ channels and __________ of K+ channels. A) opening, opening B) closing, closing C) opening, closing D) closing, opening

Answer: D

25) increased diffusion distance

Answer: D

32) Parathyroid hormone is A) a peptide. B) dissolved in plasma. C) bound to a carrier. D) A and B E) A and C

Answer: D

35) Stem cells that will form T lymphocytes are modified in the A) bone marrow. B) liver. C) spleen. D) thymus. E) kidneys.

Answer: D

4) Which statements apply to the parasympathetic division of the nervous system? A) It is dominant during "resting and digesting." B) Its ganglia are nearby, on or within their target organs. C) Epinephrine is the primary neurotransmitter of the parasympathetic division. D) A and B E) A, B, and C

Answer: D

40) All but one of the structures listed below are a type of cell junction. Identify the exception. A) desmosomes B) tight junctions C) gap junctions D) loose junctions E) adherens junctions

Answer: D

40) the minimum amount of air always present in the respiratory system, after blowing out all you can

Answer: D

44) epididymis

Answer: D

49) __________ between hair cell stereocilia cause ion channels to open and close. A) Kinocilia B) Otoliths C) Utricles D) Tip links

Answer: D

49) an intracellular structure that generates the force of muscle contraction

Answer: D

5) Cells need energy to A) import raw materials. B) make new molecules. C) repair or recycle aging parts. D) all of the above

Answer: D

5) Crossover effects are sometimes seen among A) most hormones. B) all hormones. C) peptide/protein hormones. D) steroid hormones.

Answer: D

67) These receptors respond to stimuli that are harmful or potentially harmful to body tissues.

Answer: D

74) competitive inhibitor

Answer: D

75) Damage to the type II cells of the lungs would contribute to A) a thickening of the respiratory membrane. B) an increased rate of gas exchange. C) alveolar rupture. D) alveolar collapse. E) decreased surface tension in the water lining the alveoli.

Answer: D

78) The respiratory rate times the tidal volume corrected for dead space is the A) vital capacity. B) respiratory minute volume. C) pulmonary ventilation rate. D) alveolar ventilation rate. E) external respiration rate.

Answer: D

79) The human immunodeficiency virus (HIV) that causes the disease known as AIDS selectively infects A) B cells. B) plasma cells. C) cytotoxic T cells. D) helper T cells. E) suppressor T cells.

Answer: D

8) Diffuse lymphoid tissue includes A) tonsils. B) GALT (gut-associated lymphoid tissue). C) the spleen. D) A and B E) A, B, and C

Answer: D

8) Isozymes A) are enzymes that catalyze the same reaction. B) play an important role in diagnosis of certain medical conditions. C) have complex structures with multiple protein chains. D) all of the above

Answer: D

8) Striated muscles are so-called because of a repeating pattern of light and dark bands. One repeating unit of the banding pattern is called a A) myofilament. B) myomere. C) sarcofibril. D) sarcomere. E) crossbridge.

Answer: D

82) Tom suffers from hypertension (high blood pressure). Which of the following might help deal with his problem? A drug that A) stimulates a1 receptors in cardiac muscle tissue. B) blocks a2 receptors in cardiac muscle tissue. C) stimulates cAMP formation in cardiac muscle tissue. D) blocks beta receptors in cardiac muscle tissue. E) blocks muscarinic receptors in cardiac muscle tissue.

Answer: D

83) eosinophils

Answer: D

84) A fluid sample contains a large amount of IgA type antibody. This fluid is probably A) blood. B) lymph. C) serum. D) tears. E) intracellular.

Answer: D

87) difficulty breathing

Answer: D

87) physiological role is unclear

Answer: D

89) a molecule that participates at a branch point

Answer: D

9) Chromium is A) a major essential element. B) a minor essential element. C) a dietary supplement with no natural role in the body. D) none of the above

Answer: D

9) The reflex that complements a withdrawal reflex by making compensatory adjustments on the opposite side of the body receiving the stimulus is the A) stretch reflex. B) tendon reflex. C) flexor reflex. D) crossed extensor reflex. E) reciprocal reflex.

Answer: D

CHAPTER 1 Introduction to Physiology 1) Physiology is the study of A) the structure of the body. B) the tissues and organs of the body at the microscopic level. C) growth and reproduction. D) the normal functions of the organ systems. E) the facial features as an indication of personality.

Answer: D

Human Physiology, 5e (Silverthorn) CHAPTER 12 Muscles 1) Identify the FALSE statement. A) There are only three types of muscle tissue in the human body. B) Skeletal muscle is always striated. C) Smooth muscle is often present in the walls of hollow organs. D) Skeletal muscle contraction is always under conscious control. E) Skeletal muscle contraction is not directly influenced by hormones.

Answer: D

Human Physiology, 5e (Silverthorn) CHAPTER 14 Cardiovascular Physiology 1) In the 16th century, William Harvey discovered evidence that A) the cardiovascular system transports blood and air. B) the cardiovascular system is an open system. C) arteries and veins are linked by capillaries. D) blood is recirculated instead of consumed. E) the liver manufactures blood.

Answer: D

Match the structure or compound to its function. A. ezetimbe B. ferroportin C. hepcidin D. DMT1 E. NPC1L1 107) transporter for iron absorption

Answer: D

Match the symptoms to the problem. A. damage to the cerebral cortex B. degeneration of hair cells C. ringing in the ears D. dizziness and nausea E. problems with structures of the middle ear 132) Ménière's disease

Answer: D

Match the type of reflex to its description. A. somatic B. autonomic C. spinal D. cranial E. innate F. acquired G. polysynaptic 18) integrated in the brain

Answer: D

The five functional types of epithelia are listed below. Match each type with its location and/or description. A. exchange B. transport C. ciliated D. protective E. secretory 123) Found in the epidermis, esophagus, and mouth, these stacked layers of cells prevent exchange, while they resist chemicals, bacteria, and other destructive forces.

Answer: D

143) Regulatory-protein transcription factors bind to __________ and activate __________, which tells the RNA polymerase __________.

Answer: DNA, promoter, where to bind to the DNA

97) What is Dalton's law? Why is it important?

Answer: Dalton's law states that the total air pressure in a mixture of gases is the sum of the pressures contributed by each individual gases (the partial pressures). It is important because the air that we breathe is a mixture of gases, gas pressure is related to amount of gas, and the respiratory system is regulated in part by the partial pressures of oxygen and carbon dioxide in the body.

139) Your study partner is having difficulty understanding dense fibrous connective tissue, tendons, and ligaments. Explain to her how they are related to each other and how they are different from other categories of connective tissue.

Answer: Dense fibrous connective tissue is a category of connective tissue, distinct from loose, adipose, blood, bone, and cartilage. It is not as dense as cartilage and bone but is denser than the other types listed. Like loose connective tissues, fibroblasts are the primary cell type, but unlike loose, the matrix consists of relatively more protein fibers and less ground substance. Like bone and cartilage, the fibers are primarily collagen. The fibers can be arranged randomly (irregular) or parallel to each other (regular). Tendons and ligaments are both composed of dense fibrous connective tissue. Tendons attach muscles to bones, whereas ligaments attach bones to bones; tendons lack elastic fibers, but they are present in ligaments; therefore ligaments are slightly stretchy.

177) You are constructing working models of the heart for extra credit. Your first attempt failed because the cells separated from each other during beating. What structures did you omit, and why did this allow the heart to fall apart? You omitted the gap junctions in your second attempt. How would a real heart fail to function normally without gap junctions?

Answer: Desmosomes were omitted. Without desmosomes, the contractile force of the heart would be large enough to separate the cells from each other. Without the gap junctions, the electrical currents that stimulate contraction would not be propagated from cell to cell, and the heart would fail to beat.

112) Describe what a nurse hears through the stethoscope while using a blood pressure cuff, and explain the significance of the sounds.

Answer: During a normal reading, the first thing the nurse hears is the absence of sound, because there is no blood flowing through the brachial artery; she has successfully compressed the artery with the inflated cuff. As soon as she has released enough pressure to allow blood to flow again, she hears the rumbling sounds of turbulent blood flow through a partially compressed artery. As she continues to release pressure, eventually the blood flows smoothly again as the artery is no longer compressed. The cuff pressure at the initial sound is the systolic blood pressure, and the cuff pressure when the sounds then disappear is the diastolic blood pressure.

136) conductive hearing loss

Answer: E

15) Most reflexes are regulated by A) the brain. B) the spinal cord. C) ganglia. D) positive feedback. E) negative feedback.

Answer: E

19) Protective reflexes of the lungs include A) coughing. B) bronchoconstriction. C) bronchodilation. D) A and C E) A and B

Answer: E

2) The first law of thermodynamics states that A) the total amount of energy in the universe is constant. B) energy can be neither created nor destroyed. C) what heats up must cool down. D) natural processes move from order to disorder, or entropy. E) A and B

Answer: E

2) Which statement about cardiac muscle is FALSE? A) It is found only in the heart. B) It is classified as a striated muscle. C) It is controlled involuntarily. D) It can contract spontaneously. E) It is easily fatigued.

Answer: E

20) Angiotensin II A) stimulates thirst. B) causes widespread vasoconstriction throughout the body. C) causes the synthesis and release of aldosterone from the adrenal cortex. D) A and B E) A, B, and C

Answer: E

20) Pain and itch A) can interact antagonistically. B) both involve A fibers. C) can result from different specific stimuli. D) A and B E) A and C

Answer: E

20) Parathyroid hormone A) stimulates osteoclast activity. B) increases the rate of calcium absorption. C) decreases the rate of calcium excretion. D) raises the level of calcium ion in the blood. E) all of the above

Answer: E

27) Active expiration is produced by contraction of A) abdominal muscles. B) diaphragm. C) internal intercostals. D) external intercostals. E) A and C

Answer: E

27) Cortisol A) suppresses the immune system. B) causes positive calcium balance. C) influences brain function. D) A and B E) A and C

Answer: E

29) During the first 15 seconds of intense exercise, such as sprinting or power lifting, most of the energy is provided by A) fat oxidation. B) carbohydrate oxidation. C) ATP in muscle fiber. D) phosphocreatine. E) C and D

Answer: E

29) The onset of fatigue in a muscle cell is influenced by A) the intensity and duration of the contractile activity. B) the use of aerobic or anaerobic metabolism. C) the fitness level of the individual. D) A and B E) A, B, and C

Answer: E

33) When comparing complete tetanus with unfused tetanus, which is true? A) In complete tetanus, no relaxation occurs between stimuli. B) In complete tetanus, maximum tension is developed. C) In complete tetanus, the muscle fiber is stimulated at a higher frequency. D) A and B E) A, B, and C

Answer: E

34) Polyspermy in humans results in A) twins. B) triplets. C) individuals with haploid cells. D) tetraploid adults. E) a nonfunctional zygote.

Answer: E

34) Which of the following is/are vasoconstrictors? A) platelet-activating factor B) serotonin C) adenosine diphosphate D) thromboxane A2 E) B and D

Answer: E

35) Sound waves are converted into mechanical movements by the A) auditory ossicles. B) cochlea. C) oval window. D) round window. E) tympanic membrane.

Answer: E

36) As a result of respiratory alkalosis, A) the respiratory rate increases. B) the tidal volume increases. C) the kidneys conserve bicarbonate. D) the kidneys secrete fewer hydrogen ions. E) the body retains less carbon dioxide.

Answer: E

60) pancreatic cells

Answer: E

61) In response to a rapid increase of organic acid in the body, you would expect to observe A) increased alveolar ventilation. B) decreased blood pressure. C) decreased heart rate. D) decreased blood pH. E) A and D

Answer: E

65) These degrade long chain fatty acids and toxic foreign molecules.

Answer: E

67) A rise in cortisol would cause each of the following except one. Identify the exception. A) enhanced rate of glucose synthesis by the liver B) increased rate of glycogen usage by the liver C) higher levels of fatty acids in the blood D) suppression of the immune system E) increased ACTH levels

Answer: E

73) Which of the following symptoms would you expect to observe in a person suffering from type I diabetes mellitus? A) glucosuria B) ketoacidosis C) thirst and polydipsia D) hyperglycemia E) all of the above

Answer: E

75) The continual movement of fluid through the interstitial space functions to A) accelerate the distribution of nutrients and hormones. B) assist the transport of insoluble substances that cannot enter the capillaries. C) help carry toxins and bacteria to cells of the immune system. D) flush hormones and wastes from the interstitial space. E) all of the above

Answer: E

75) aminase

Answer: E

77) On a tour of African countries, Don contracts a bad case of traveler's diarrhea. Because he can't eat very much, his body starts to use energy sources other than carbohydrates. This would result in A) increased levels of urea in the blood. B) ketosis. C) a decreased blood pH. D) increased gluconeogenesis in the liver. E) all of the above

Answer: E

79) Which of the following individuals would lose heat faster in a cold room? A) an adult man B) an adult woman C) an adolescent male D) a child E) a newborn infant

Answer: E

79) Which of the following is/are (a) possible treatment(s) for hypertension? A) calcium channel blockers B) ACE inhibitors C) ANP blockers D) chloride leak channel blockers E) A and B

Answer: E

8) Colony-stimulating factors are cytokines made by A) endothelial cells. B) fibroblasts from bone marrow. C) white blood cells. D) A and B E) A, B, and C

Answer: E

8) The only blood vessels whose walls permit exchange between the blood and the surrounding interstitial fluids are the A) arterioles. B) venules. C) capillaries. D) A and C E) B and C

Answer: E

89) A male body builder starts taking injections of testosterone (an anabolic steroid) on a daily basis. After three weeks, which of the following would you expect to observe? A) increased sex drive B) decreased levels of GnRH C) decreased levels of LH D) decreased levels of FSH E) all of the above

Answer: E

9) The adrenal medulla is important to the sympathetic branch of the autonomic nervous system because A) it is a source of catecholamines. B) it is considered a modified sympathetic ganglion. C) it releases epinephrine and norepinephrine directly into the blood. D) A and B E) A, B, and C

Answer: E

9) What is/are the major role(s) of the phospholipid bilayer? A) the absorption of fats B) the formation of a barrier that is selectively permeable to lipid-soluble molecules C) to provide a framework for membrane proteins D) to carry water-soluble molecules through a hydrophobic environment E) B and C

Answer: E

90) reactions that result in the breakdown of large biomolecules

Answer: E

Human Physiology, 5e (Silverthorn) CHAPTER 11 Efferent Division: Autonomic and Somatic Motor Control 1) Nicotine is thought to cause approximately __________ deaths per year, worldwide. A) 500 B) 5000 C) 50,000 D) 500,000 E) 5,000,000

Answer: E

Match the condition with the description. A. acromegaly B. adrenogenital syndrome C. cretinism D. dwarfism E. exophthalmus 43) associated with Graves' disease

Answer: E

Match the following structures with their functions. A. pancreas B. pylorus C. rectum D. small intestine E. stomach 47) chyme is released from here

Answer: E

Match the following with its description. A. acetylcholine B. norepinephrine C. cholinergic nicotinic receptor D. adrenergic receptor E. cholinergic muscarinic receptor 18) parasympathetic tissue receptor

Answer: E

47) Discuss changes in the cardiovascular and pulmonary systems that result from conditioning for exercise.

Answer: Pulmonary ventilation rate increases immediately with onset of exercise, due to feedforward responses that originate with muscle and joint proprioceptors. These receptors send signals to the motor cortex, which affects the respiratory control center. The cardiovascular control center is also affected, resulting in generalized sympathetic discharge, which causes widespread vasoconstriction, increasing systemic blood pressure. Other effects include increased cardiac output and vasodilation in vessels of skeletal muscle to increase blood flow.

141) What is the difference between cell movements in response to outside forces and cell- generated movements (in response to specific cell activity)? (Hint: Do red blood cells move because of RBC activity? What causes cell movements associated with cytokinesis?) Using the index of your text to guide you to relevant sections in other CHAPTERs (look up entries for the various cytoskeletal proteins), explain the different types of movements that cells generate.

Answer: Red blood cells and other cells in circulation move because they are suspended in a flowing fluid (blood or lymph); the axon of a nerve cell in the arm will move when the arm moves. Cell-generated movements involve the activity of actin microfilaments, intermediate filaments, microtubules, and myosin thick filaments (in muscle). The processes of chromosome alignment during mitosis/meiosis, cytoplasmic pinching during cytokinesis, beating of cilia and flagella, endocytosis and exocytosis, and muscle contraction are all examples of cell-generated movements.

38) What is the major problem with the deconstructionist view of biology?

Answer: Return to the topic of function and process. The deconstructionist view of biology predicted that once we uncovered the sequence of the human genome, the inner workings of the human body would be revealed. In reality, it is possible to know how a gene codes for a particular protein without knowing why that protein exists. Our knowledge of the human genome is only a piece of the puzzle.

167) Compare and contrast the effects of cholinergic and adrenergic fibers on the heart. Include a comparison of the various types of receptors they interact with, and their location on the heart.

Answer: See Fig. 14-27 in the CHAPTER.

165) Diagram at least seven different factors that affect the components of the equation that relates cardiac output to heart rate and stroke volume. Explain each factor involved.

Answer: See Fig. 14-31 in the CHAPTER.

87) Map the following terms: blood distribution, blood volume, cardiac output, diameter of the arterioles, diameter of the veins, fluid intake, fluid loss, heart rate, mean arterial pressure, passive, regulated at kidneys, resistance to blood flow, and stroke volume.

Answer: See Fig. 15-10 in the CHAPTER.

91) Summarize the steps of hemostasis.

Answer: See Fig. 16-10 in the CHAPTER.

92) Summarize the steps of the coagulation cascade.

Answer: See Fig. 16-12 in the CHAPTER.

84) Sketch and label a relaxed sarcomere, then draw the same sarcomere in a contracted state. The myofilaments may be represented as simple lines.

Answer: Sketch should resemble Fig. 12-8 in the CHAPTER.

92) Which single muscle contributes most to normal, resting inspiration? Which single muscle contributes most to normal, resting expiration?

Answer: The diaphragm muscle provides most of the force for inspiration. Normal expiration is primarily a result of the diaphragm relaxing, and expiratory muscles do not contribute.

96) List the three accessory glands of the male reproductive system and describe their function.

Answer: The prostate, seminal vesicle, and bulbourethral (Cowper's) gland all provide the components of the semen other than the sperm (i.e., fructose, mucus, etc.).

96) Describe the different types of smooth muscle.

Answer: The six major groups are vascular, gastrointestinal, urinary, respiratory, reproductive, and ocular.

32) What would the labels be for the graph axes?

Answer: The x-axis is horse name or number; the y-axis is finish time in minutes.

34) What would the labels be for the graph axes?

Answer: The x-axis is race number or date; the y-axis is finish time in minutes.

187) How many liters of blood are distributed between the pulmonary and systemic circuits in a "typical" person? Assume 20% of the blood is in the lungs at any moment. If the cardiac output is 5 L/min, how long will it take a drop of blood to flow from the right ventricle to the left ventricle?

Answer: There are about 5 liters of blood in a 70 kg man. If 20% of the blood is in the lungs, then 5 × 0.2 = 1 liter of blood in the lungs. 1 L × 1 min/5 L × 60 sec/1 min × 1 min/60 sec = 12 seconds.

104) What makes vegetable oils with trans fats similar in structure to saturated animal fats? Which type of fat is harmful, and in what way?

Answer: Trans fats have hydrogen atoms attached to make them more saturated and solid at room temperature, like animal fats. Both vegetable trans fats and saturated animal fats are associated with cardiovascular disease.

84) Compare and contrast the two types of diabetes mellitus.

Answer: Type I diabetes mellitus is a condition of insulin deficiency due to pancreatic beta cell destruction. Type 2 diabetes mellitus is insulin-resistant, due to a variety of causes. Fig. 22-16 in the CHAPTER provides a detailed summary of the characteristics of type I diabetes.

59) You are studying for your physiology test with your lab partner. She insists that the purpose of the knee-jerk reflex is to test for neural or muscular damage. State whether or not you agree with her, and why.

Answer: While reflexes can be exploited by medical personnel as a means to assess neural and muscular function, that is not their "purpose" in the sense of why they occur. Each reflex has a function; for example, the knee-jerk reflex helps control movement at the knee joint.

65) Water gain during a day comes from either __________ or __________. How is water lost during a day?

Answer: food, drink; metabolism Also see Fig. 20-2 in the CHAPTER.

153) Sympathetic stimulation to the pacemaker cells __________ heart rate by __________ ion flow through __________ and __________ channels.

Answer: increases, increasing, If , calcium

61) The largest collection of __________ tissue in the body is the gut-associated lymphoid tissue (GALT).

Answer: lymphoid 62) Digestion is essentially completed in the __________. (:0)Answer: small intestine

132) cytosol

Answer:A

18) Increased blood volume __________ blood pressure. A) increases B) decreases C) has no effect on

Answer:A

42) The identity of an element can be determined by the number of __________.

Answer:A

48) Glands that secrete hormones into the blood via tissue fluids are A) endocrine glands. B) mixed glands. C) exocrine glands. D) unicellular glands. E) none of the above

Answer:A

5) Kidneys respond relatively __________ to changes in blood volume. A) slowly B) quickly

Answer:A

53) Estimated to reach populations of one trillion in normal adults, most of these cells reside in the lymphoid tissues of the body. These cells look quite similar to one another under the microscope, yet function very differently.

Answer:A

53) fat breakdown

Answer:A

54) O2

Answer:A

54) Stimulation of the beta receptors on heart muscle results in A) the formation of cAMP. B) decreased rate of contraction. C) decreased force of cardiac contraction. D) increased sensitivity to acetylcholine. E) all of the above

Answer:A

54) a genetic condition where red blood cells have deficient cytoskeletons

Answer:A

55) An example of transamination is A) moving amino acids from an amino acid to another molecule to manufacture a different amino acid. B) removing an amino group and putting it into the bloodstream for the kidneys to discard. C) removing a phosphate group from a molecule. D) finding a brand new sports car in your driveway with your name on it. E) None of the above describes transamination.

Answer:A

55) Fat digestion begins here.

Answer:A

56) To change the ability of some proteins to function, it is necessary to alter the amino acid content by a minimum __________ amino acid(s). A) one B) two C) one hundred D) ten

Answer:A

57) Antibodies in breast milk protect a newborn baby from pathogens.

Answer:A

57) Autorhythmic cells A) are also called pacemakers because they set the rate of the heartbeat. B) are the same size as myocardial contractile cells. C) have organized sarcomeres. D) contribute to the force of contraction. E) none of the above

Answer:A

57) blocks sweating

Answer:A

58) Infected cells are killed by perforin and granzymes.

Answer:A

58) These are weak attractive forces between the nucleus of one atom and the electrons of another atom close by.

Answer:A

59) brush border

Answer:A

68) keratin

Answer:A

7) Kidneys regulate A) water loss. B) water gain. C) A and B

Answer:A

Match the stimulus to the type of receptor that typically responds to it. A. chemoreceptor B. mechanoreceptor 69) oxygen

Answer:A

Match the term to its definition. A. metabolism B. catabolism C. anabolism 40) the sum of all the body's chemical reactions

Answer:A

31) The motor neurons that innervate the normal contractile fibers of the muscle are the __________.

Answer:A lpha motor neurons 32) The motor neurons associated with intrafusal muscle fibers are referred to as __________ motor neurons. (:0)Answer: gamma

41) What is the potential yield of ATP molecules for each FADH2 molecule entering the electron transport system? A) 1 B) 1.5 C) 2 D) 2.5 E) 3

Answer: B

41) may result from adrenal tumor

Answer: B

44) What we perceive as the pitch of a sound is our sensory response to its A) amplitude. B) wavelength. C) frequency. D) intensity. E) duration.

Answer: C

44) When amino acids are metabolized for energy, the nitrogen is converted to urea and then is excreted from the body by the A) liver. B) skin. C) kidneys. D) sweat glands. E) spleen.

Answer: C

73) lipase

Answer: B

114) If a polypeptide contains 10 peptide bonds, how many amino acids does it contain? A) 0 B) 5 C) 10 D) 11 E) 12

Answer: D

39) Calcitonin may be an appropriate treatment.

Answer: D

41) When the baroreceptor reflex is triggered by a decline in blood pressure, A) sympathetic activity increases. B) cardiac output increases. C) peripheral resistance decreases. D) A and B E) A, B, and C

Answer: D

3) Cell volume (and therefore cell function) in most cells is dependent upon careful regulation of A) volume of extracellular fluid. B) blood pressure. C) osmolarity of extracellular fluid. D) permeability of cell membranes. E) resting membrane potential.

Answer: C

3) The brain neurotransmitter that is an important stimulus for food intake is A) ghrelin. B) leptin. C) neuropeptide Y. D) orexin. E) insulin.

Answer: C

3) The most abundant proteins in blood plasma are A) globulins. B) transport proteins. C) albumins. D) lipoproteins. E) fibrinogens.

Answer: C

3) The name for the process by which cells obtain and store energy in the chemical bonds of biomolecules is A) thermodynamics. B) energetics. C) metabolism. D) kinetic energy. E) potential energy.

Answer: C

3) The substance in muscle fiber cytoplasm that directly powers muscle contraction is A) glucose. B) fatty acids. C) ATP. D) phosphocreatine.

Answer: C

31) When eaten, nearly 90% of our dietary calories from fat are in the form of A) steroids. B) phospholipids. C) triglycerides. D) cholesterol. E) fat-soluble vitamins.

Answer: C

32) arterial during exercise compared to rest

Answer: C

81) Increased levels of carbon dioxide in the blood will result in A) decreased heart rate. B) decreased cardiac output. C) decreased blood flow to the lungs. D) decreased parasympathetic stimulation of the heart. E) all of the above

Answer: D

15) The function of the pericardial fluid is to A) provide oxygen to the heart. B) reduce friction between the heart and the pericardium. C) provide fuel to the heart. D) remove waste products from the heart. E) store calcium for the heart.

Answer: B

26) The cells responsible for the production of circulating antibodies are A) NK cells. B) plasma cells. C) helper T cells. D) cytotoxic T cells. E) suppressor T cells.

Answer: B

16) A unique aspect of hematopoiesis is that A) white blood cell production always results in the same proportion of leukocytes. B) white blood cell development varies with the specific needs of the body. C) neutrophils direct all development from the lymph nodes. D) lymphocytes never die.

Answer: B

26) There is clear evidence that strenuous exercise on a regular basis enhances the immune system. A) true B) false

Answer: B

26) When alveolar pressure is equal to atmospheric pressure, air flows into the lungs. A) true B) false

Answer: B

6) Technically, the kidneys are located A) in the thoracic cavity. B) in the abdominal cavity. C) behind the pleural membranes. D) behind the peritoneal membrane. E) in the pelvic cavity.

Answer: D

16) An increase in the level of carbon dioxide in the blood will A) decrease the rate of breathing. B) increase the rate of breathing. C) decrease pulmonary ventilation. D) decrease the alveolar ventilation rate. E) A, C, and D

Answer: B

16) Interneurons of the corticospinal tract synapse onto A) visceral efferents. B) somatic efferents. C) interneurons of central pattern generators.

Answer: B

2) Most obese humans are deficient in leptin. A) true B) false

Answer: B

25) According to the law of LaPlace, when comparing two alveoli lined with fluid, pressure in the one with the __________ diameter will be greater. A) larger B) smaller

Answer: B

25) Excess potassium ions are eliminated from the body by the A) sweat glands. B) kidneys. C) liver. D) digestive system. E) spleen.

Answer: B

25) The cell bodies of preganglionic neurons are found either in the brain stem or in the sacral region of the spinal cord.

Answer: B

25) The parathyroid glands produce a hormone that A) stimulates the formation of white blood cells. B) increases the level of calcium ions in the blood. C) increases the level of sodium ions in the blood. D) increases the level of potassium ions in the blood. E) increases the level of glucose in the blood.

Answer: B

25) The swallowing center in the brain, which coordinates the muscular reflexes, is located in the A) cerebrum. B) medulla oblongata. C) pons. D) cerebellum. E) hypothalamus.

Answer: B

6) The __________ of glycogen from many glucose molecules is an __________ reaction. A) decomposition, endergonic B) synthesis, exergonic C) decomposition, exergonic D) synthesis, endergonic

Answer: D

26) Which of the following events does NOT occur when you swallow? A) Respiration is inhibited. B) The upper esophageal sphincter closes. C) The glottis and epiglottis close. D) The lower esophageal sphincter relaxes.

Answer: B

35) The primary role of the carbonic acid-bicarbonate buffer system is A) to buffer stomach acid. B) to buffer carbonic acid formed by carbon dioxide. C) the prevention of pH changes caused by organic and fixed acids. D) to buffer the urine. E) to increase the amount of carbonic acid during ventilation.

Answer: C

35) hyperthyroidism

Answer: C

36) Chronic inhalation of fine particles that reach the alveoli leads to __________ lung disease. A) obstructive B) restrictive C) fibrotic D) compliant E) congestive

Answer: C

36) Stretch-sensitive mechanoreceptors known as __________ are located in some artery walls. A) nociceptors B) chemoreceptors C) baroreceptors D) elasticeptors

Answer: C

36) involves endocrine stimulation by antibodies

Answer: C

34) When energy is released during catabolism, it is temporarily trapped in high energy bonds or electrons of certain compounds. Which of the following is NOT associated with carrying energy? A) ATP B) DNA C) NADH D) FADH2 E) NADPH

Answer: B

35) Compartmentation refers to A) pairing together isozymes that perform similar reactions. B) grouping related enzymes into specific organelles. C) separating substrates into chemical classes. D) dividing metabolic reactions into groups for better understanding.

Answer: B

35) In electrocardiography, a lead is a/an A) electrode. B) pair of electrodes. C) cable that attaches between the ECG machine and the body.

Answer: B

35) Specific responses to changes in body temperature are regulated by the A) thermoreceptors. B) hypothalamus. C) medulla oblongata. D) skin. E) cardiac output.

Answer: B

35) Which of the following are the two anticoagulants produced by the body? A) heparin and plasmin B) antithrombin III and heparin C) antithrombin III and plasmin D) tissue plasminogen activator and plasmin E) protacyclin and heparin

Answer: B

35) Which of the following substances is most alkaline? A) lemon juice, pH = 2 B) urine, pH = 6 C) tomato juice, pH = 4 D) white wine, pH = 3 E) stomach secretions, pH = 1

Answer: B

36) If a substance has a pH that is less than 7, it is considered A) neutral. B) acidic. C) alkaline. D) a buffer. E) a salt.

Answer: B

36) plasma glucose concentration 30 minutes after glucose ingestion in diabetics who exercise, compared to those who don't

Answer: B

37) Poiseuille's law is summarized this way: A) PV = nRT B) R a Lh/r4 C) P = 2T/r D) P1V1 = P2V2

Answer: B

39) A person who suffers from hyperventilation will exhibit signs of A) respiratory acidosis. B) respiratory alkalosis. C) metabolic acidosis. D) metabolic alkalosis. E) none of the above

Answer: B

39) Immunoglobulins that are found on the surface of B lymphocytes and may play a role in regulation of the humoral immune response are A) IgA. B) IgD. C) IgE. D) IgG. E) IgM.

Answer: B

39) Movement of material between cells is known as the __________ pathway. A) transcellular B) paracellular C) metacellular D) transendothelial E) cisendothelial

Answer: B

39) The intrinsic pathway of coagulation is activated by the A) sticking of platelets to damaged tissue. B) activation of proenzyme exposed to collagen. C) release of tissue factor by a damaged endothelium. D) release of heparin from the liver. E) conversion of prothrombin to thrombin.

Answer: B

39) The structure that overlies the organ of Corti is the A) basilar membrane. B) tectorial membrane. C) stapedius. D) perilymph. E) endolymph.

Answer: B

3) A zygote with the genotype YO will A) develop into a male. B) develop into a female. C) have Turner's syndrome. D) have Kleinfelter's syndrome. E) die.

Answer: E

30) Angiotensin I is converted to angiotensin II by enzymes primarily located in the A) kidneys. B) liver. C) heart. D) lungs. E) blood vessels.

Answer: E

30) Epinephrine and norepinephrine increase ion flow through __________ channels. A) sodium B) potassium C) calcium D) If E) C and D

Answer: E

32) __________ and __________ transfer their high-energy electrons to the electron transport system. A) NADH, NAD+ B) NAD+, FADH2 C) FADH2, FAD+ D) B and C E) A and C

Answer: E

36) Platelets function in A) transporting chemicals important for clotting. B) forming temporary patches in injured areas. C) contraction after clot formation. D) initiating the clotting process. E) all of the above

Answer: E

36) The nucleus stores all the information needed to synthesize which of the following molecules? A) carbohydrates B) lipids C) proteins D) phospholipids E) all of the above

Answer: E

37) An enzyme that will digest proteins into peptides is A) lipase. B) amylase. C) nuclease. D) maltase. E) trypsin.

Answer: E

4) A zygote with the genotype XO will A) develop into a female. B) have Turner's syndrome. C) develop into a male. D) die. E) A and B

Answer: E

28) Melanocortins include A) ACTH. B) cortisol. C) growth hormone. D) MSH. E) A and D

Answer: E

41) Bicarbonate secretion A) neutralizes acid entering from the stomach into the duodenum. B) is secreted by apical Cl--HCO3- antiport. C) is not dependent on high levels of carbonic anhydrase to maintain bicarbonate production. D) is secreted by the acinar cells. E) A and B

Answer: E

42) Saliva is A) slightly acidic, with a pH of 6-7. B) controlled by the autonomic nervous system. C) not stimulated by sympathetic innervation. D) secreted from endocrine glands. E) A and B

Answer: E

46) The process of disassembling fatty acids into two-carbon units inside mitochondria is called A) oxidative phosphorylation. B) deamination. C) ketonization. D) chemiosmosis. E) beta-oxidation.

Answer: E

46) uterus

Answer: E

48) potassium

Answer: E

50) Phagocytic, these cells make up the majority of WBCs.

Answer: E

50) Which can serve as substrates for ATP production? A) glucose B) amino acids C) fatty acids D) glycogen E) all of the above

Answer: E

63) Draw a map that shows the renal compensation for acidosis. Draw another map for alkalosis.

Answer: Fig. 20-22 shows compensations for both acidosis and alkalosis.

76) Two individuals are the same age, sex, and ethnic background. While being tested for their BMR, Bill consumes 20 liters of oxygen/hour and Randy consumes 16 liters of oxygen/hour. Which of the two needs to consume the most calories in order to maintain proper health and constant weight? A) Bill B) Randy

Answer:A

16) The beta cells of the pancreas produce A) insulin. B) glucagon. C) renin. D) cortisol. E) digestive enzymes.

Answer:A

20) released by all autonomic preganglionic neurons

Answer:A

21) Beta-endorphin is produced from the same prohormone as A) ACTH. B) insulin. C) aldosterone. D) thyroxine. E) glucagon.

Answer:A

21) Cytokines function by A) acting as messengers for growth and activity of other cells. B) poisoning and killing invading cells. C) digesting invading cells. D) A and B E) A, B, and C

Answer:A

21) During the absorptive state, A) the liver forms glycogen. B) adipocytes contribute fatty acids to the circulation. C) skeletal muscles break down glycogen. D) glucagon levels are elevated. E) all of the above

Answer:A

21) thickened alveolar membrane and decreased lung compliance

Answer:A

31) The adrenal medulla is closely allied with this system.

Answer:A

7) Phasic receptors A) are rapidly adaptive receptors. B) cease firing unless the strength of the stimulus remains constant. C) are not attuned to changes in a parameter. D) include pressure sensitive baroreceptors. E) A, B, and D

Answer:A

71) dehydrogenase

Answer:A

79) Which is typically more important in regulating the respiratory system, PO2 or PCO2? Explain your

Answer and briefly discuss the receptors involved. Give examples of situations in which each of those factors changes enough to stimulate a reflex. How and why are these factors related to each other? (:0)Answer: PCO2 is the more important factor. For PCO2, there are central and peripheral receptors that respond to CO2 as well as to CO2-related pH. These receptors are very sensitive to routine changes in PCO2 and pH, such as those associated with an increase in physical activity. Peripheral chemoreceptors have been identified for O2, but these respond only to dramatic changes in PO2, such as those associated with high altitude or disease. Because CO2 is produced as a by-product of aerobic (oxygen-consuming) metabolism, an increase in CO2 is associated with a corresponding decrease in O2.

120) ribosome

Answer: B

76) osteoclast

Answer: B

133) tinnitus

Answer: C

28) glucose

Answer: C

22) sympathetic tissue receptor

Answer: D

50) organ where most digestion occurs

Answer: D

75) stretch

Answer: D

129) basolateral membrane

Answer: E

67) atrial depolarization

Answer:A

80) enlarging one's muscles through body-building exercises

Answer:A

49) location of the sphincter that prevents premature emptying of the stomach

Answer: B

70) the amount of blood left in the ventricle after it contracts

Answer: E

88) an end product that immediately becomes a substrate

Answer:A

5) Nicotine enhances the release of __________ in the brain. A) serotonin B) dopamine C) acetylcholine D) epinephrine E) glutamate

Answer: B

78) Which of the following is greater? A) heart rate normally B) heart rate during circulatory shock

Answer: B

26) A typical action potential of a myocardial contractile cell lasts __________ millisecond(s). A) less than 1 B) 1-5 C) 50-100 D) at least 200 E) at least 500

Answer: D

33) Vitamin D (calcitriol) A) is formed by sunlight. B) is bound to plasma protein for transport. C) synthesis is stimulated by high calcium levels. D) A and B E) A and C

Answer: D

34) A component of an important buffer in the human body is A) NaCl. B) H+. C) HCl. D) HCO3-. E) H2O.

Answer: D

53) The hair cells of utricle and saccule are clustered in A) ampullae. B) cristae. C) cupulae. D) maculae. E) otoliths.

Answer: D

53) human chorionic gonadotropin

Answer: D

54) Protein digestion is completed here.

Answer: D

9) Water reabsorption by the kidneys is a result of A) both passive and active transport processes. B) cotransport with ions. C) exchange with ions. D) osmosis.

Answer: D

91) Bronchioles and systemic arterioles constrict.

Answer: D

14) Renewal or modification of the cell membrane is a function of the A) microtubules. B) mitochondria. C) rough endoplasmic reticulum. D) ribosomes. E) Golgi complex.

Answer: E

Classify each scenario below as to primarily which type of work is being done. A. chemical work B. transport work C. mechanical work 77) maintaining a concentration gradient across a membrane

Answer: B

30) Dehydration can result from A) heat exhaustion. B) CCK. C) diabetes. D) gluconeogenesis. E) A and C

Answer: E

17) Somatostatin is another name for A) growth hormone. B) growth hormone-releasing hormone. C) growth hormone-inhibiting hormone. D) thyroid hormone. E) parathyroid hormone.

Answer: C

18) ACE converts A) renin to angiotensinogen. B) angiotensinogen to angiotensin I. C) angiotensin I to angiotensin II. D) angiotensin II to aldosterone. E) renin to aldosterone.

Answer: C

23) The majority of the protein inside a red blood cell is A) albumin. B) porphyrin. C) hemoglobin. D) immunoglobulin. E) fibrinogen.

Answer: C

23) increased airway resistance

Answer: C

23) organs

Answer: C

24) A hormone that can lower blood levels of calcium ion is A) parathyroid hormone. B) thyroxine. C) calcitonin. D) glucagon. E) oxytocin.

Answer: C

24) The structure that transports the ovum to the uterus is the A) uterosacral ligament. B) vagina. C) fallopian tube. D) infundibulum. E) myometrium.

Answer: C

24) With acclimatization, the body begins to do a better job of conserving A) water. B) heat. C) salt. D) A and B E) A, B, and C

Answer: C

24) movement from the nephron lumen to the blood

Answer: C

25) Acute phase proteins are produced by the A) macrophages. B) lymph nodes. C) liver. D) thymus. E) lymphocytes.

Answer: C

32) On average, one preganglionic neuron synapses with eight or nine postganglionic neurons, each innervating a different target.

Answer: C

57) What is the conclusion from the graph? Discuss the trends and discrepancies, explaining why they may occur.

Answer: (:0)Answers will vary, reiterating the results shown above, plus perhaps speculating on why the graph changes directions at various points.

33) peripheral vascular resistance during exercise

Answer: B

40) may be an autoimmune disease

Answer:A

103) inhibits gastric emptying

Answer: C

19) target receptor for preganglionic neurons

Answer: C

51) A type of granulocyte, these have red granules.

Answer: C

85) monocytes

Answer: C

24) fluid accumulation in interstitial spaces

Answer: D

38) associated with overactive osteoclasts

Answer: D

44) caused by growth hormone deficiency in childhood

Answer: D

105) stimulates insulin release

Answer: E

21) reflexes that one is born with; genetically determined

Answer: E

44) help(s) defend the body against germs

Answer: E

52) caused by elevation of TSH

Answer: E

50) lead

Answer: F 51) sodium (:0)Answer: B

119) mitochondrion

Answer:A

15) Fruits and vegetables are a good source of antioxidants. A) true B) false

Answer:A

55) blocks secretion of adrenal catecholamines

Answer:A

70) pH

Answer:A

For each question, select the corresponding category: A. membranous organelle B. inclusion 118) lysosome

Answer:A

15) The fold of skin that covers the tip of the penis is the A) glans penis. B) prepuce. C) corpus cavernosum. D) corpus spongiosum. E) penile urethra.

Answer: B

55) the lightest color bands of the sarcomere, occupied only by thin filaments

Answer: B

9) Without cortisol's permissive effect on __________, an animal would die. A) insulin B) glucagon C) thyroid hormone D) parathyroid hormone E) aldosterone

Answer: B

91) reactions that result in the synthesis of large molecules

Answer: B

Match the response to the description. A. increase B. decrease C. no change 31) percent fat metabolized for ATP production as exercise intensity increases

Answer: B

12) Motor neurons are sometimes inhibited by A) Golgi tendon organs. B) muscle spindle organs. C) joint receptors.

Answer:A

Match the response with the type of chemical. A. sympathetic antagonist B. parasympathetic antagonist 54) blocks secretion of pancreatic enzymes

Answer: B

Match the specific receptor to its typical stimulus. A. free nerve ending B. Merkel receptor C. Meissner's corpuscle D. Ruffini corpuscle E. Pacinian corpuscle 73) texture

Answer: B

Match the type of immunity to its description. A. specific B. nonspecific 54) present at birth

Answer: B

10) Fast pain, usually described as sharp and localized, is carried by A) large, unmyelinated C fibers. B) small, myelinated A-delta fibers. C) small, unmyelinated C fibers. D) large, myelinated A-beta fibers.

Answer: B

145) Blood pressure decreases during sleep. How does this affect velocity?

Answer: B

145) If 100 mL of water contains 5 grams of NaCl, m.w. 58.5 daltons, what is the molarity of the solution in moles/L? A) 0.05 B) 0.85 C) 2.92 D) 0.085 E) 0.25

Answer: B

30) The loudness or intensity of a sound wave is related to its A) amplitude. B) frequency. C) duration. D) decibels.

Answer:A

117) Compare and contrast gastrin, CCK, secretin, GIP, motilin, and glucagon-like peptide 1. In your

Answer, include the following details for each term: secreted by, target(s), effects, and stimulus for release. (:0)Answer: See Table 21-1 in the CHAPTER.

15) Increased parasympathetic stimulation A) increases heart rate. B) increases gastric motility. C) causes sweat glands to release sweat. D) causes blood vessels in the skin to dilate. E) causes the pupils to dilate.

Answer: B

135) sensorineural hearing loss

Answer: B

45) This particle has a neutral charge and a mass of 1.

Answer: B

74) also called the mitral valve

Answer: B

39) storage

Answer: C

46) the total amount of air that can be exchanged at will

Answer: C

86) H2CO3

Answer: C

138) Light shines on center of on-center cell.

Answer:A

22) tissue

Answer:A

50) a single muscle cell

Answer:A

86) present in breast milk

Answer:A

15) The primary organ where erythropoietin is produced is the A) liver. B) kidney. C) spleen. D) bone marrow. E) endothelial cells throughout the body.

Answer: B

66) ventricular depolarization

Answer: B

69) disulfide bond

Answer: B

41) large molecules synthesized from smaller ones

Answer: C

51) easily observable symptom of hypercortisolism

Answer: C

70) can be cell-matrix junctions

Answer: C

57) These result when two atoms share a pair of electrons.

Answer: D

111) transporter for cholesterol absorption

Answer: E

134) central hearing loss

Answer:A

45) shaft of penis

Answer:A

53) What can you CONCLUDE based on the new figure?

Answer:An increase in epinephrine concentration increases the average heart rate of Sprague- Dawley rats.

137) Transcription of the DNA triplet sequence ATC yields A) UAG. B) TAG. C) UTC. D) ATC.

Answer:A

12) As blood vessel length increases, A) resistance increases. B) flow decreases. C) friction decreases. D) A and B E) A, B, and C

Answer: D

C. Once cells are fully differentiated, they can no longer divide to produce more cells. Disease conditions or injuries that result in cell loss may be remedied by treatment with appropriate stem cells. Examples are neurological injuries and diseases marked by irreversible damage to cells that cannot (yet) be replaced by new cells.

...

13) Osmoreceptors depolarize after they __________ in response to __________ plasma osmolarity. A) shrink, decreased B) shrink, increased C) swell, decreased D) swell, increased

Answer: B

13) Type II alveolar cells A) allow rapid diffusion of gases through their thin membranes. B) secrete a chemical known as surfactant. C) are phagocytic. D) all of the above E) none of the above

Answer: B

13) When a quantity is expressed as "4 cm/sec," what is being described is the A) flow rate. B) velocity of flow. C) pressure. D) pressure gradient. E) volume.

Answer: B

130) Greater visual acuity results from convergence of __________ numbers of cells onto retinal ganglion cells. A) higher B) lower

Answer: B

2) The literal meaning of the term physiology is knowledge of A) organs. B) nature. C) science. D) chemistry. E) math.

Answer: B

2) The primary route for ion loss from the body is the __________ system. A) respiratory B) urinary C) digestive D) integumentary E) cardiovascular

Answer: B

128) In the light-adapted state, A) visual receptors are much more sensitive to stimulation. B) visual receptors are much less sensitive to stimulation. C) we can only see black and white. D) we can see better in the dark. E) colors are very dull.

Answer: B

69) "Water, water, every where, nor any drop to drink" is a phrase from the Rime of the Ancient Mariner by Samuel Taylor Coleridge. This poem describes an ocean ship unable to sail to land and running out of its supply of fresh water; sailors have long known that drinking sea water cannot prevent death from dehydration. What would result if the sailors attempted to alleviate their dehydration by drinking sea water? Justify your

Answer by describing kidney physiology. Why wouldn't reflexes in response to dehydration fully compensate? What does this illustrate about the force allowing kidneys to retain water under more normal conditions? (:0)Answer: The osmolarity of sea water is higher than that of the kidney medulla, thus the osmotic gradient that normally allows net reabsorption of water does not exist. Sea water osmolarity is largely a result of permeant ions such as sodium and chloride, so ingested sea water raises plasma osmolarity when it is absorbed. When the kidneys form a filtrate of this high-osmolarity solution, the descending limb fails to reabsorb water because the kidney medulla has a lower osmolarity than the filtrate (opposite of normal), along its entire length, and in fact the medulla would lose water as it moved into the descending limb. While ions would be reabsorbed by the ascending limb as usual, it is not enough to drive sufficient water reabsorption. Reflexes such as secretion of water-preserving hormones are ineffective because, ultimately, water reabsorption depends only on osmosis, and the normal osmotic gradient has been disrupted.

160) Name four questions related to heart function that you could

Answer by examining an ECG and one abnormal condition that may be indicated by each function. (:0)Answer: 1. What is the heart rate? Abnormal values could indicate tachycardia or bradycardia. 2. Is the rhythm of the heartbeat regular or irregular? Abnormal rhythm could indicate fibrillation. 3. Are all normal waves present in recognizable form? Electrical disorders in atria or ventricles could be indicated. 4. Does a QRS complex follow each P wave, and is the PR segment constant in length? Heart block may be indicated.

77) Gossip, an undergraduate, has normal PCO2 levels, high H+ levels, low pH and bicarbonate levels. What type of disturbance is Gossip suffering from and what might cause this? If his PCO2 were elevated, would your

Answer change? Explain. (:0)Answer: Gossip is likely suffering from metabolic acidosis, since his PCO2 is normal. If his PCO2 was elevated, then he would likely be suffering from respiratory acidosis. Metabolic acidosis may be caused by ingestion of methanol, aspirin, and ethylene glycol. Respiratory acidosis can be caused by drugs or alcohol depression, increased resistance in asthma, impaired gas exchange in fibrosis or severe pneumonia, and muscle weakness in muscular dystrophy and other muscle diseases. The most common cause is chronic obstructive pulmonary disease.

11) Mitotic divisions in germ cells A) are complete before birth. B) begin at puberty. C) continue throughout life. D) end at andropause or menopause. E) The

Answer depends on gender. (:0)Answer: E

166) Describe the action of aspirin and opiates in pain relief. What does your

Answer suggest about opiates? (:0)Answer:A spirin inhibits prostaglandins and presumably slows the transmission of pain signals from the site of injury to the brain. Opiates like morphine act on a special class of receptor molecules in the CNS called opioid receptors. Opiates decrease the secretion of neurotransmitter from the primary sensory neurons. The discovery of the mode of action of opiates, that is, the discovery of specific receptors for opiate drugs, led scientists to search for and discover naturally occurring opiates in the CNS—enkephalins and endorphins.

Use the table and graph below to

Answer the following questions. Heart rates (bpm) of Sprague-Dawley rats after administration of various concentrations of epinephrine. Heart Rates Epinephrine (mg) Animal 1 Animal 2 Animal 3 50 48 62 55 100 58 67 63 150 67 70 79 200 80 85 93 150 67 70 79 Table 1-1 Figure 1-1 50) List all of the errors in Fig. 1-1. (:0)Answer: 1. The units of concentration are labeled as M when they should be mg. 2. The x-axis is in decreasing order of concentration. 3. The graph needs a legend. 4. The data for each concentration of epinephrine should be averaged and plotted as a single trend.

129) Which stimulus opens more sodium channels in photoreceptors? A) brighter light B) dimmer light

Answer: B

136) Describe the anatomical and fluid compartments of the body. How do the lumens of hollow organs fit into these classifications? Which fluid-containing cavities are considered to be internal and which are external? Of those that are external, why are they external (give examples)? You may wish to design a flow chart to help

Answer this. (:0)Answer: The anatomical compartments are the cranial cavity, containing the brain; the thoracic cavity, containing the heart and lungs; and the abdominopelvic cavity, containing organs of the digestive, urinary, and reproductive systems. The fluid compartments are the intracellular fluid (inside the cells) and the extracellular fluid (outside the cells). The extracellular fluids are found in the interstitial fluid between cells and the plasma of the blood, which is in the lumen of the circulatory system. Lumens of hollow organs such as the digestive and urinary tracts are part of the external environment, whereas the intracellular and interstitial fluids are internal. The lumen of the circulatory system is also internal. External lumens are those that open to the outside environment. These openings include the mouth, nostrils, anus, urethral and vaginal orifices.

102) You and your lab partner use a living nerve and muscle preparation (removed from the animal, but cells function normally for several hours), to study muscle contraction. When you stimulate the nerve electrically, the muscle contracts; why? When you turn off the stimulator but add a high concentration of caffeine to the fluid surrounding the tissues, the muscle contracts; why (speculate on possible sites/mechanisms of action)? When curare (which blocks neuromuscular transmission) is added with caffeine, contraction occurs; why (speculate, and how does this alter your previous

Answer)? Can you identify a single site of action from this information? Explain your (:0)Answer. (:0)Answer: The applied electrical current opens voltage-gated ion channels in the nerve, causing an action potential, which leads to a normal muscle response to neurotransmitter. One could speculate that caffeine stimulates an action potential in the nerve by opening ion channels, triggers neurotransmitter release, stimulates a muscle action potential directly, or triggers contraction directly. It is clear from the final treatment that caffeine can affect the muscle directly, but it is possible that it also affects the nerve. It is not clear what the precise mechanism of stimulation is, from the information provided.

78) Do the factors affecting affinity of hemoglobin for oxygen have the same impact on loading and unloading in lungs and tissues? Explain your

Answer, and indicate if this presents a conflict in remedying hypoxia. Assuming a medical team has treatments available for changing all the factors affecting hemoglobin's oxygen affinity, how should they manipulate pH, temperature, PCO2, and 2,3-DPG content in a hypoxic patient? (:0)Answer: Hemoglobin affinity changes in the same way in the lungs and tissues, thus increasing unloading in the tissues does decrease loading in the lungs, which seems as if one cancels out the other. But from the oxygen dissociation curves it is evident that the impact is greater in the tissues; thus increased unloading can remedy hypoxia. The medical team should decrease pH, increase temperature, increase PCO2, and increase 2,3-DPG.

116) What are the characteristics that determine gender (consider genetic, anatomical, hormonal, and other factors)? Is it always the case that genetic, psychological, and physical gender cues indicate the same gender? Explain your

Answer, citing specific examples of conditions in which there is gender conflict. (:0)Answer: Gender is determined by sex chromosomes, by internal and external genitals, by sex hormones, and by psychological factors. Pseudohermaphrodites are XY with male internal genitals, but are deficient in one of the male hormones that trigger formation of external genitals. Thus they are born looking like girls. At puberty, however, more male characteristics begin to develop. Testicular-feminized males lack androgen receptors so their testes fail to descend and their brains are not masculinized, and their outward appearance is female, though they are sterile. Adrenal tumors can secrete hormones that masculinize or feminize the opposite genetic sex. Sometimes psychological gender identity fails to match genetic sex, for unknown reasons.

96) A diagnosis of thyroid cancer may be treated with removal of the thyroid gland. Which hormone or hormones may now be deficient? Considering each hormone individually, which ones should be restored and why? Explain any age-related considerations. Which nonthyroid hormone may be affected by careless thyroidectomy? Explain your

Answer, indicating whether or not it is critical to maintain this hormone. (:0)Answer: The thyroid gland is the source of thyroid hormones and calcitonin. Thyroid hormones should be replaced in children to maintain normal growth and in adults to maintain quality of life. Calcitonin replacement is not necessary in adults, because parathyroid hormone and calcitriol should be unaffected, and they are sufficient to maintain normal calcium balance. It is unclear how important calcitonin replacement may be in children. Careless thyroidectomy could result in removal of parathyroid glands. If parathyroid hormone is not restored, death results.

112) How does altitude affect blood composition and why? How would athletes' performance be affected if they live, train, and compete at different altitudes? Explain your

Answer, using different scenarios. Is such training a type of doping? Explain. (:0)Answer: Spending several days at higher than normal altitude will stimulate erythropoiesis, due to the lower oxygen concentration in the air, which when breathed would result in hypoxemia. Athletes would be less competitive if they competed at a higher altitude than that in which they lived or trained, and more competitive if they lived and trained at a higher altitude than that in which they competed, at least in theory. While students may not know this, experimental results support the best regimen for maximizing performance at a low-altitude competition to be living at high altitude but training at low altitude. Neither living and training high nor living and training low achieved the same results. High-altitude training is necessary, however, for high- altitude competition. This is a matter of opinion, but most people would probably argue that these training regimens are natural, and therefore not a type of doping.

125) True or False? Lipids contain substantially more oxygen than carbohydrate molecules. Based on your

Answer, what does that suggest about lipid solubility in water? (:0)Answer: False. Lipids contain much less oxygen than carbohydrates. Oxygen often participates in hydrogen bonding. With less oxygen, lipids are not able to hydrogen bond with water molecules and therefore are relatively insoluble in aqueous environments.

106) Examination of a sample of glandular cells reveals an extensive network of smooth endoplasmic reticulum. Which of the following would be a likely product of these cells? A) digestive enzymes B) steroid hormones C) protein (peptide) hormones D) transport proteins E) antibodies

Answer: B

77) Your studies of the neuromuscular junction reveal that the excitatory neurotransmitter opens a channel permeable to all three cations. Calculate the equilibrium potential for each cation. In which direction will each cation move when the channel is open? Does movement of ions promote muscle depolarization? Explain your

Answer. (:0)Answer: ENa+ = 61 × log 450/50 = 58 mV. EK+ = 61 × log 20/420 = -81 mV. EMg++ = 61 × log 60/10 = 47 mV. Sodium and magnesium will enter the cell, potassium will exit. Exit of potassium opposes depolarization, but presumably there will be a net gain in intracellular positive charge sufficient to depolarize the cell.

52) Are relaxation of a muscle and inhibition of a muscle the same thing? Explain your

Answer. (:0)Answer: No. Relaxation is a passive process that normally follows contraction and occurs in the absence of further activity in the motor neurons. Inhibition involves prevention of muscle stimulation and consequent contraction by activity in interneurons that inhibit motor neurons. While in both cases the motor neuron is not producing signals, the reason is lack of stimulation in one case, inhibition in the other.

159) What are ryanodine receptors? What type of tissue are they present in? Are all ryanodine receptors regulated in the same way? Explain your

Answer. (:0)Answer: Ryanodine receptors are calcium channels, present in cardiac and skeletal muscle tissue. In cardiac muscle they are regulated by calcium, resulting in calcium-induced calcium release from the SR. In skeletal muscle, they are regulated by mechanical linkage to voltage- sensing receptors in SR, thus they are indirectly voltage-regulated.

144) What type of epithelium would one expect to compose the alveoli (air sacs) in the lungs? Defend your

Answer. (:0)Answer: Since gases must diffuse across the alveoli and associated capillaries you would expect to find exchange epithelia, composed of very thin cells (simple squamous epithelium). Thicker types of epithelial cells would slow the process of gas diffusion to and from the blood.

103) A certain drug is known to block monovalent cation channels. This drug is used as a "muscle relaxer." Would it affect both smooth and skeletal muscle, or only skeletal muscles? Defend your

Answer. (:0)Answer: Skeletal muscle relies on monovalent cation channels for the generation of action potentials to trigger contraction, whereas smooth muscle has more "routes" available to trigger contraction. Blocking one route would not be sufficient to cause relaxation, hence this drug would be more likely to be effective on skeletal muscle.

167) Grandma's house is a half hour drive from yours. Every Sunday your family has lunch with Grandma, and you always fall asleep in the back seat on the way home, yet you always wake up as the car turns into the driveway at home. Which receptors were monitoring the drive, even though the conscious part of your brain was asleep? Explain your

Answer. (:0)Answer: The vestibular apparatus, including the utricle, saccule, and semicircular canal's receptors, the cristae, were monitoring your progress, keeping track of the turns, the accelerations and decelerations, and so on along the familiar route. When the appropriate number of changes of direction and speed were reached, your cerebral cortex was notified, and you "regained consciousness" at the appropriate time.

181) Sonja is pregnant. Over the next 40 weeks, her uterus will enlarge as the placenta develops and the fetus grows, her abdomen will add skin to accommodate distension, her breasts will grow, her blood volume will increase, and many other changes will occur. Using what you have learned about basic cardiovascular physiology, predict how parameters such as blood pressure and cardiac output will change. (Note that the fetus has a separate cardiovascular system, which does not need to be considered.) Explain your

Answer. (:0)Answer: [Note to instructor: These predictions are logical, but not all actually happen.see next question.] The growth of maternal tissues will require addition of blood vessels, thus total vessel length will increase. This would increase total peripheral resistance. Blood pressure may increase with increasing blood volume, and also to overcome the greater resistance and return the blood to the heart. Stroke volume should increase; cardiac output may increase as a result, or may remain the same if heart rate decreases.

97) Define, compare, and contrast each of the pathologies listed below. Are the risk factors for each the same? Explain your

Answer. A. coronary artery disease B. atherosclerosis C. myocardial infarction D. hypertension E. congestive heart failure (:0)Answer: A. Coronary blood vessels become blocked by cholesterol or blood clots, causing reduced blood flow. B. Atherosclerosis is hardening of the arteries, which results from accumulation of cholesterol and other lipids, proliferation of vascular smooth muscle, and hardening due to calcification and fibrosis. The bulge that forms protrudes into the vessel lumen, reducing blood flow. C. Myocardial infarction is the death of heart cells as a result of blocked blood flow. D. Hypertension is high systemic blood pressure that develops as a result of increased peripheral resistance. E. Congestive heart failure occurs after prolonged hypertension, when the cardiac muscle can no longer generate the necessary cardiac output against the high peripheral resistance. As left cardiac output decreases, fluid backs up into the lungs. Oxygenation of the blood decreases, the heart becomes even weaker, leading to more fluid accumulation, and eventually death.

69) Both alpha-bungarotoxin and curare bind to the same neurotransmitter receptor, but only curare binds reversibly. Which receptor is involved? List some locations for this receptor. Would either toxin be appropriate to use as a paralytic during surgery? Explain your

Answer. Are all such receptors necessarily affected by a given toxin in the same way? Explain the significance of your (:0)Answer. (:0)Answer: Both of these toxins bind to the nicotinic acetylcholine receptor. This type of receptor is located on skeletal muscle and in autonomic ganglia. For a paralytic during surgery, curare would be appropriate but bungarotoxin would not, because the irreversibility of bungarotoxin binding results in permanent paralysis of skeletal muscles. Bungarotoxin binds to the receptors in skeletal muscle but not to those in autonomic ganglia, illustrating a structural difference in the receptors in these two sites.

156) Veins are said to carry deoxygenated blood. Does this mean that there is no oxygen in venous blood? Explain your

Answer. Are there veins that carry highly oxygenated blood? Again, explain your (:0)Answer. What color is most venous blood compared to most arterial blood? (:0)Answer: Venous blood has some oxygen, but usually less that arterial blood, so it is only relatively deoxygenated. Pulmonary veins are an example of veins carrying highly oxygenated blood. Venous blood is usually dark red, arterial blood is usually bright red.

80) Define hyperventilation and explain what may cause it. Is the increased ventilation that occurs while exercising an example of hyperventilation? Explain your

Answer. How are PO2 and PCO2 affected by hyperventilation? Can breathing into a paper bag remedy hyperventilation or is this just unfounded folk medicine? Explain. (:0)Answer: Hyperventilation is an increase in alveolar ventilation that exceeds metabolic demand. A person can deliberately hyperventilate, or it may occur as a result of emotional stress or high altitude. Strictly speaking, the increase in ventilation during exercise is necessary to meet increased metabolism and is therefore not hyperventilation. PO2 is increased and PCO2 is decreased during hyperventilation, because more O2 is inhaled and more CO2 is exhaled; this produces abnormally low PCO2. Hyperventilation can be remedied by a paper bag treatment, because rebreathing exhaled air will increase PCO2 in the body back to normal.

102) The usual treatment for neonatal jaundice is exposing the infant to certain wavelengths of light. The infant wears only a diaper and undergoes the light treatment continuously for several days, even at night. Would you expect that this treatment affects organ function? If so, explain your

Answer. If not, what does the light affect? Why doesn't the infant have to be on this treatment indefinitely? (:0)Answer:A s the blood circulates through the skin, the energy in the light penetrates into the blood and breaks down the bilirubin. Liver function improves on its own as the liver matures and as production of fetal hemoglobin stops; and hyperbilirubinemia doesn't appear again.

68) Diagram the cerebral motor cortex and descending input to spinal motor neurons. Voluntary suppression of the knee-jerk reflex originates in the motor cortex, which can send inhibitory signals to the motor neurons. If the left motor cortex is active, is the left or right knee-jerk reflex suppressed or are both suppressed? Explain your

Answer. If the left side of the spinal cord at the level of leg muscle control is damaged, is it the left or right knee-jerk reflex that is suppressed or are both suppressed? (:0)Answer: The diagram should resemble Fig. 13-12 in the CHAPTER. The left motor cortex controls the right spinal motor neurons and therefore the right knee reflex. The left side of the spinal cord controls the reflex on the left side.

103) How would the process of red blood cell production be affected by constricting the arteries that supply blood to the kidney? Explain your

Answer. Name three other situations that would similarly affect the process of red blood cell production. (:0)Answer: Red blood cell production would be increased. Constricting arteries would reduce blood flow; thus, the kidney would experience hypoxia. The kidney produces the hormone erythropoietin, which stimulates red blood cell production, in response to hypoxia. Increased red blood cell production would also result from anemia, extended presence at high altitude, and following hemorrhage.

75) Though you are an attentive parent, you lost track of your inquisitive toddler for a few minutes, just long enough for him to wander into the garage and open a package of insecticide that you use to control insects in your garden. You have no idea if any of the poison has been ingested, inhaled, or absorbed through his skin. You take the package away from your child and read under the "caution" section that it is an anticholinesterase. Given that this poison was made for insects, not humans, should you be worried? Explain your

Answer. What would happen to the child if he has indeed been affected? Explain which type of synapses could be affected. Propose an antidote (it's OK if you don't remember a specific compound, just describe what type of effect may reverse the effects of the insecticide). (:0)Answer: Insects have acetycholine receptors that are similar enough to ours so that yes, you should be worried. Anticholinesterases block the degradation of acetylcholine, thereby prolonging its effects in both muscarinic and nicotinic synapses. If the dose is high enough, spastic paralysis could result, in which muscles contract uncontrollably. If this happens with the respiratory muscles, the victim may die of suffocation. Autonomic effects would also be expected, including sweating and nausea. A chemical such as atropine, which blocks muscarinic receptors, will alleviate some of the symptoms. Oximes (not mentioned in text) block nicotinic receptors and can be administered to counteract nicotinic effects of the poison.

99) Define anemia, and explain how it is the same or different from hypoxemia. Give examples of three different types of anemia and a brief synopsis of each kind. Is sickle cell disease a true anemia? Explain your

Answer. When sickle cell disease is fatal, what is the cause of death? (:0)Answer:A nemia is a condition in which the blood's hemoglobin content is too low. Hypoxemia means low oxygen in blood, which could be due to an impaired respiratory system and can therefore occur independent of anemia. Because oxygen binds to hemoglobin, individuals with anemia cannot transport as much oxygen to the tissues, resulting in fatigue and weakness. This may be especially noticeable with exercise. Some major causes of anemia are summarized in Table 16-3 in the CHAPTER. Sickle cell disease is considered an anemia because the amount of normal hemoglobin is low. Death is related to hypoxia-caused tissue damage.

Match the following terms to the correct

Answer: A. tricuspid valve B. bicuspid valve C. aortic valve D. pulmonary valve 72) an AV valve that has three flaps (:0)Answer:A

90) Summarize the key steps of smooth muscle contractions.

Answer: 1. Increase in cytosolic calcium initiates contraction; calcium is released from the sarcoplasmic recticulum and also enters the extracellular fluid. 2. Calcium binds to calmodulin. 3. Calcium binding to calmodulin is the first step in a cascade that ends with contraction. 4. Phosphorylation of proteins is an essential step in smooth muscle contractions. 5. The primary control of smooth muscle contraction resides in the regulation of myosin ATPase activity. For more information, see the "Phosphorylation of Proteins Plays a Key Role in Smooth Muscle Contraction" section and Fig. 12-28 in the CHAPTER.

For these questions, the data were separated and analyzed by gender. 59) A. Write a hypothesis regarding gender and weight. B. What is the dependent variable? What is the independent variable? C. Based on the data in the graph above, what is your conclusion? D. Why is a bar graph a good choice for presentation of these data? Would another type of chart be as effective?

Answer: A. Males weigh more than females. B. Weight depends on gender; thus weight is dependent, gender is independent. C. Males weigh more than females. D. Bar graph allows comparison of the average of two groups. No.

127) Which of the following is a key intermediate of glucose catabolism? A) ATP B) pyruvate C) oxygen D) lactate E) glucokinase

Answer: B

107) Name the three portal systems in the body.

Answer: 1. hepatic portal system 2. renal portal system 3. hypothalamic-hypophyseal portal system 108) The pressure created in the ventricles is called the __________ pressure because __________. (:0)Answer: Driving, it is the force that drives blood through the blood vessels 109) The heart is encased in a tough membranous sac called the __________ and is composed mostly of __________ muscle tissue, called the __________. (:0)Answer: pericardium, cardiac, myocardium 110) The __________ directs blood from the __________ of the heart to the lungs. Backflow of blood to the heart is prevented by __________. Blood from the lungs returns blood to the heart via __________. (:0)Answer: pulmonary trunk, right ventricle; pulmonary valve; pulmonary veins 111) The opening between each atrium and its ventricle is guarded by the __________, which connect the ventricular side to collagenous tendons, called __________. __________ muscles provide stability for these tendons. (:0)Answer:A trioventricular valve, chordae tendineae; Papillary 112) Cell junctions in contractile cells are called __________, which consist of two components: __________ and __________. (:0)Answer: intercalated disks, desmosomes, gap junctions 113) The __________ are calcium channels in cardiac contractile cells. Opening them causes __________. (:0)Answer: ryanodine receptors; calcium-induced calcium release 114) The AV node delays the transmission of action potentials slightly to allow __________. (:0)Answer: the atria to complete their contraction before ventricular contraction begins

70) A man's leg was crushed between a car bumper and a wall. His physicians believe their patient has suffered kidney damage from myoglobin blocking glomerular pores. Tests showed the following results: plasma creatinine: 30 mg/100 mL plasma 24 hour urine specimen: volume = 1 liter urine creatinine 30 mg/mL urine A. How many mg of creatinine are in the urine specimen? How much creatinine appears in the urine per hour? B. What is the patient's creatinine clearance in mL/min? C. What is the patient's GFR? D. Is this a normal GFR? Did the patient sustain kidney damage?

Answer: A. 1000 mL urine × 30 mg creatinine/mL urine = 30,000 mg creatinine in the specimen. 30,000 mg/24 hr = 1250 mg creatinine. B. Creatinine clearance = excretion rate of creatinine/plasma concentration creatinine. clearance = 1250 mg creatinine/hr × 1 hr/60 min/30 mg creatinine/100 mL plasma = 20.8 mg creatinine/min/0.3 mg creatinine/mL plasma = 69.4 mL plasma/min C. GFR approximately equal to clearance rate for creatinine, or approximately 70 mL/min. D. A more typical value for GFR is 125 mL/min, so 70 mL/min represents a severe decline. Thus, kidney damage has most likely occurred.

114) Sonja is 28 years old and weighs 132 lb (2.2 lb = 1 kg). Calculate the following values for her, assuming she is an average healthy adult. A. What is her approximate blood volume? B. What is her approximate plasma volume? C. Assuming a normal hematocrit, what is her volume of cellular elements? D. What is her typical RBC count? E. WBC? F. Differential count? G. Platelets? H. What hemoglobin level would you expect to find?

Answer: A. 132 lbs = 60 kg. Blood volume is approximately 7% of body weight: 60 kg × 0.07 = 4.2 kg = 4.2 L. B. If her hematocrit were 40%, then her plasma volume = 60% of her blood volume = 0.6 × 4.2 = 2.5 L. Normal hematocrits may range from 37% to 47% in females. C. With a hematocrit of 40%, her packed cell volume would be 1.6 L. D. Values are found in Fig. 16-3: RBC count = 3.9 to 5.6 × 106. E. WBC = 4,000 - 11,000. F. Ranges of normal are presented in Fig. 16-3. G. Platelets = 200,000 - 500,000. H. Hemoglobin = 12 to 16 g/dL blood.

111) Cindy is taking 14 breaths per minute. Her vital capacity is 3000 mL, her total lung capacity is 4000 mL, and her tidal volume is 450 mL per breath. Calculate the following: A. Cindy's total pulmonary ventilation (minute volume) B. Cindy's alveolar ventilation rate

Answer: A. 14 breaths/min × 450 mL/breath = 6300 mL/min B. Alveolar vent = 14 breaths/min × (450 mL/ breath - 150 mL dead space) = 14 breaths/min × 300 mL/breath = 4200 mL/min

186) Miguel is a 25-year-old jogger with a target heart rate of 125 bpm. His resting pulse is 70 bpm. His blood volume is approximately 6.8 liters. At rest, his cardiac output is 6.3 liters/min. A. What must his stroke volume be? B. What might his EDV and ESV be? C. During his workout, his heart rate increases to 125 bpm and his cardiac output to 12,500 mL/min. Do his SV, EDV, and ESV change with exercise?

Answer: A. 90 mL CO = SV * HR; SV=CO/HR = 6.3L/min/ 70 bpm. B. Any combination where EDV minus ESV = 90 mL. Average values are ESV = 44-60 mL, EDV 134-150 mL. C. Yes, the SV changes to 100 mL; EDV probably increases proportionately, to 149-167 mL, thus the ESV would be 49-67 mL. One would assume an increase in venous return would lead to atrial stretching and an increased EDV.

62) A. Disregarding gender, write a hypothesis that expresses the relationship between weight and height. B. What is the dependent variable? What is the independent variable? C. From the data in Table 1-3, construct a graph that examines this relationship.

Answer: A. A prediction such as "As height increases weight increases" would be appropriate. B. The dependent variable would be weight, the independent variable is height. C.

61) A. Write a hypothesis regarding the effects of breakfast consumption on reaction time. B. What is the dependent variable? What is the independent variable?

Answer: A. A prediction such as "Eating breakfast prior to testing improves reaction time of subjects (compared to subjects who did not eat breakfast)" is appropriate. B. The independent variable is breakfast consumption; the dependent variable is reaction time.

60) A. Write a hypothesis regarding gender and recovery time. B. What is the dependent variable? What is the independent variable? C. Create a graph using the averages from the data table. Based on these data, what do you conclude?

Answer: A. A prediction such as "Males recover from exercise more quickly than females" would be appropriate. B. The independent variable is gender; the dependent variable is recovery time. C. A bar graph such as the one below is appropriate. In this study, men recovered from exercise more quickly than women.

88) List the factors that influence arteriolar resistance, and give an example of each.

Answer: 1. Sympathetic reflexes. Example: temperature regulation. 2. Local control of arteriolar resistance matches tissue blood flow to the metabolic needs of the tissue—Example: paracrines that respond to low oxygen, high carbon dioxide. 3. Hormones such as those that regulate salt and water excretion by the kidney.

61) What are the two possible causes of lower alveolar PO2? Give examples of each.

Answer: 1. The composition of the inspired air is abnormal. Altitude affects oxygen content of air. 2. Alveolar ventilation is inadequate. Pathological factors include increased airway resistance (asthma), decreased lung compliance (fibrosis), and overdoses of drugs or alcohol.

189) Using the appropriate equations, predict the effects in each scenario. A. Vessels X and Y have the same diameter. X is 5 cm long, Y is 10 cm long. Which has the faster velocity of flow for the same fluid? B. A blood vessel has a radius of 4 cm. Vasoconstriction reduces the radius to 1 cm. In order to maintain the same flow, by what factor must pressure increase? C. The blood vessel in b. dilates from 1 cm to 4 cm. Does the flow rate increase or decrease? D. A new subdivision is built between your house and the water storage tower. If the water company makes no adjustments, will the water pressure at your house change? If so, in which direction? Why?

Answer: A. Assuming the same driving pressure, the velocity is the same. Length is not a factor (v = Q/A). B. 256. Q a DP/R, so DP a R. As R a 1/r4, DP a 1/r4. As Q does not change, DP1 × r14 = DP2 × r24. r14 = 44 = 256. C. increase D. Your water pressure would decline, owing to its diversion to the new subdivision houses.

120) Endocrine disruptors are environmental chemicals, which, if people and other animals are exposed, can have hormone-mimicking or hormone-blocking effects. These chemicals are used in industries and in homes and are associated with such commonly used products as pesticides and herbicides, plastic food containers and baby bottles. Those that mimic or block sex steroids are of special concern, because they are present in household plastics and are fat-soluble, thus easily stored in fatty tissue of the body and can affect a developing fetus. Research is ongoing and results are controversial, but these chemicals have been associated with: A. delayed sexual maturity in fish; B. poor masculinization of genitals in male alligators with decreased ability to produce offspring; C. feminized behavior in male rats, and in humans; D. decreased lactation; E. diminished male genitals and semen quality; and F. early puberty. For each condition A.-F., specify which hormones may have been blocked and which mimicked (endocrine regulation in fish, alligators, and rats is nearly identical to that in humans).

Answer: A. Blocked estrogen and androgen; B. blocked testosterone or mimicked estrogen; C. blocked testosterone or mimicked estrogen; D. blocked estrogen or prolactin; E. blocked testosterone; and F. mimicked estrogen.

124) One way to determine the total energy in calories of a food is to burn it in a device called a bomb calorimeter. The heat produced during combustion minus the heat added to begin the combustion is the total calories in the food. The typical human digestive tract does not absorb all of the calories in food; thus it is not 100% efficient. Furthermore, calories in undigestible organic compounds may not be absorbed at all, and thus can be considered to be zero calorie relative to human nutrition. Artificial sweeteners and fats are designed to produce desired sensation in the mouth without being absorbed. These are organic compounds that would release heat in a bomb calorimeter, thus they technically have calories. A. What does this suggest about the accuracy of the bomb calorimeter in determining available food calories for humans? What would be the results of bomb calorimetry of human feces? B. What is it about a particular food substance that may prevent absorption of its calories? C. What happens to the "zero-calorie" molecules as they pass through the digestive tract? What are the potential effects on defecation?

Answer: A. Bomb calorimetry is not 100% accurate, because it does not take into account that some calories are not absorbed. Human feces would be determined to contain some calories. B. In order for nutrients in a food to be usable by the body, they must be chemically digested and absorbed. Digestive enzymes exhibit specificity, so they do not digest molecules that are different in structure from their substrate. If an artificial food additive is undigestible, this simply means that the body lacks the appropriate enzyme. C. Gut bacteria may be able to digest and absorb these compounds. Extra organic solute present in the gut may cause retention of water and production of bacterial gas as the bacteria digest the substance in question. This can lead to distention, discomfort, and diarrhea.

190) Michi is on the cross-country team at her college. While she's running, her pulse reaches 150 bpm and her cardiac output reaches 25 liters/min but at rest it's only 65 bpm and 5 liters/min. A. Calculate her stroke volume, possible EDVs and ESVs at rest and exercising. Show your calculations. B Michi has a diastolic blood pressure of 78 mm Hg and a systolic pressure of 118 mm Hg; what is her pulse pressure, when pulse pressure = systolic pressure - diastolic pressure? C. If her carotid pulse rate is 80 bpm, what is her radial pulse rate?

Answer: A. CO = HR × SV; SV = CO/HR; SV = 254 L/min/150 bpm (exercise) or 5 L/min/65 bpm (rest); SV exercising = 167 mL, rest = 77 mL. Any combination of EDV and ESV where SV = EDV minus ESV. B. Pulse pressure = 40 mm Hg C. Radial pulse = 80 bpm

84) Refer to the graph below. A. At a PO2 of 80 mm Hg, which type of hemoglobin binds more oxygen? B. At a PO2 of 40 mm Hg, which type of hemoglobin has released more oxygen to the cell? C. Explain the significance of the differences in fetal and maternal hemoglobin affinity. D. If a worm lived in low oxygen mud flats where the PO2 is 60 mm Hg, which type of hemoglobin would be better for it to have? Explain.

Answer: A. Fetal hemoglobin has a higher affinity for oxygen than maternal hemoglobin at that oxygen pressure. B. Maternal hemoglobin releases oxygen more readily than fetal hemoglobin at that oxygen pressure. C. The higher affinity of fetal hemoglobin for oxygen ensures a net transfer of oxygen from the maternal blood to the fetal blood in the placenta. While unloading from fetal hemoglobin to fetal tissues is less favorable due to the higher affinity, the high rate of oxygen consumption in fetal tissues and consequent low oxygen pressure ensures unloading will occur. D. The fetal form of hemoglobin would be better in this situation, because it has a higher affinity for the oxygen at this pressure. Thus, the worm's blood would become more saturated, and more oxygen would then be available for the worm's cells.

106) You were the top student in your physiology class last semester. For this reason, your professor has asked that you prepare some lectures for her physiology class this semester. Specifically, she would like you to discuss the similarities and differences between the cardiovascular and respiratory systems, in terms of pressure gradients and fluid flow. A. Explain how will you describe the role of the following structures: the pump and the tubes. B. Explain how you will describe the importance of an open vs. closed system, and the relevant differences between liquids and gases. C. What is fundamentally different about the respiratory pump compared to the cardiac pump, and why does this difference exist?

Answer: A. In both systems, fluids flow down a pressure gradient, within different-sized tubes of variable resistance; the pressure gradient is generated by the action of a muscular pump. In the cardiovascular system, the tubes vary from elastic arteries that minimize pressure extremes and fluctuations, muscular arteries and arterioles whose diameter is changeable to regulate flow and that account for most of the resistance, finally to the large number of tiny capillaries in parallel, with veins to conduct the blood back to the pump. In the respiratory system, the larger tubes are rigid and account for most of the resistance, and lead to a large number of tiny tubes in parallel whose diameter is adjusted to regulate flow. B. In the closed cardiovascular system, the highest pressure is generated within the left ventricle, and an incompressible liquid flows in a continuous fashion. High pressure is created when the ventricle contracts, squeezing the blood it contains; low pressure results when the ventricle relaxes. In the open respiratory system, the highest pressure during normal breathing is generated when the respiratory muscles relax, decreasing the volume of the alveoli as air flows out; low pressure results when the muscles contract. Air is compressible, thus Boyle's law of inverse relationship between volume and pressure is important. The flow of air is intermittent, flowing into the lungs then stopping, and flowing out of the lungs then stopping again. C. The driving pressure for fluid flow in the cardiovascular system is created directly by contraction of the heart chamber, and depends upon the total peripheral resistance. The driving pressure for fluid flow in the respiratory system is created indirectly by contraction of respiratory muscles, which produces a volume increase that creates a pressure decrease relative to atmospheric pressure. The compressibility of air and the incompressibility of blood, as well as the fact that the respiratory system is open and thus dependent on atmospheric pressure, while the cardiovascular system is closed and thus wholly responsible for pressure, explains these differences.

106) Calcium atoms have two electrons in the outermost shell. As a result, you would expect calcium to form ions with a charge of A) +1. B) +2. C) -1. D) -2. E) none of the above

Answer: B

13) Blood flow to a tissue will increase if the A) level of oxygen at the tissue increases. B) level of carbon dioxide at the tissue increases. C) pH rises. D) vessels constrict. E) all of the above

Answer: B

58) You are doing an experiment to determine if caffeine consumption affects reaction time. A. Which is the dependent variable? B. Which is the independent variable? C. Briefly describe some ways you might manipulate the independent variable. D. Name three stimuli you could use, and how you might measure reaction time for each. E. Write an appropriate hypothesis for this study. F. You compute the following average values from your experiment. What would be a logical conclusion for these data? Average caffeine consumer's reaction time: 400 ms Average noncaffeine consumer's reaction time: 650 ms G. Sketch a simple graph to convey these results to your classmates. What kind of graph did you choose? Why? Which variable did you plot on the x-axis? Which one did you plot on the y-axis? Why? H. Do the results of this experiment support the hypothesis you chose?

Answer: A. Reaction time B. Caffeine consumption C. Vary the amounts of caffeine consumed; vary the source, for example, use coffee, pills, cola drinks, and/or chocolate; vary both the amounts and sources. D. (:0)Answers will vary. Example: a computer-based timer could measure the time elapsed between the subject's detecting the appearance of an object on the computer monitor and depressing a key on the keyboard. Auditory or touch stimuli could be used, too. E. Depending on the (:0)Answer to C, could choose: "Consumption of caffeine decreases reaction time" or similar statement. F. Consumption of caffeine improves reaction time by 250 ms, on average. G. Bar graph; allows comparison of the average of two groups. The x-axis: group, caffeine or none. The y-axis: reaction time in milliseconds. H. Yes, in case of hypothesis written in D.

184) A person has a total blood volume of 5 L. Of this total, 4 liters is contained in the systemic circulation and 1 L in the pulmonary circulation. If the person has a cardiac output of 5 L/min: A. how long will it take for a drop of blood leaving the left ventricle to return to the heart? B. how long will it take a drop of blood to go from the right ventricle to the left ventricle?

Answer: A. SV = 5 L/min, volume = 4 L, which would be distributed in 4/5 min = 48 seconds. B. Volume in the pulmonary circuit = 1 L, which would require 1/5 min, or 12 seconds to return to the left atrium from the right ventricle.

93) Define each term listed, and explain its significance to blood flow or pressure. A. HDL/LDL/cholesterol B. vascular smooth muscle C. baroreceptors D. edema E. angiogenesis

Answer: A. See Fig. 15-24. B-D. See the "Emerging Concepts" section of the CHAPTER. E. See "The Blood Vessels" section of the CHAPTER.

152) A. Define stem cells and differentiation, and describe the different types and the extent to which they are present during the life of an individual (include the fertilized egg as well as the adult that eventually results). B. Which of the four tissue types contain populations of stem cells of known function, and what is that function? Which types of tissue have stem cells of unknown function? Do any types of tissue lack stem cells? C. How do fully differentiated cells differ from stem cells? How can stem cells be used in medical treatment? Give examples.

Answer: A. Stem cells are cells that are mitotic and not fully differentiated. Differentiation is the process cells undergo as they become more and more specialized in structure and function as is typical of the specific tissues they compose. The fertilized egg and the cells resulting from the first few cleavage divisions after fertilization are totipotent, which means they can ultimately produce all the types of cells in an adult. Within the first week after conception, the cells begin differentiation and become capable of producing many types of cells but not all. These cells are pluripotent. By adulthood, stem cells can be described as multipotent, which are stem cells that can produce many of the cell types in a specific tissue, and committed stem cells, which can only become one specific cell type within that tissue. B. Epithelial and connective tissues have active populations of multipotent stems cells, which replace cells lost to damage or normal turnover. Muscle and nervous tissues contain stem cells but appear to be unable to replace lost cells. They were once thought to lack stem cells altogether.

168) Carol has severe bouts of dizziness that her doctor says are related to her allergies. A. Explain how this could happen. She has been taking medications for some time and is showing improvement, but still has occasional problems. One event that always brings back her dizziness is having to run the windshield wipers in her car. B. Explain how seeing them could upset Carol's equilibrium. What areas of the brain are involved?

Answer: A. The allergies must be upsetting the fluid balance in the inner ear, through the same mechanisms that allergies more typically cause runny nose or watery eyes. The receptors are not able to adapt to the additional fluid, causing overstimulation of the hair cells, and the resultant perception of dizziness. B. The vestibular nuclei have collateral pathways that coordinate the cerebellum, the medulla and eye movement. She is probably compensating for the fluid imbalance in the inner ear by relying more heavily on visual cues to help maintain her equilibrium. When her eyes are "distracted" by the repetitive movement of the windshield wipers, her brain interprets this as a loss of balance, and tries to compensate further, resulting in dizziness and "confusion" of the brain as to what to correct next!

115) Hydatid mole is a type of pregnancy that, without medical intervention, frequently results in the death of the mother. The hydatid mole is thought to result most often from an abnormality in fertilization, such that the egg has two copies of paternal chromosomes, and no maternal DNA. A fetus fails to develop; instead there is rapidly growing chorionic tissue in the placenta, which secretes hCG and thus leads to a positive pregnancy test and outward signs of pregnancy. The first clue to the obstetrician that the pregnancy is abnormal is often the lack of a heartbeat in the first trimester. A. What is the genotype of the hydatid mole tissue? From the genotype alone, would most science-literate students predict that a normal embryo would form? Explain, indicating what this suggests about maternal and paternal contributions to the embryo. B. The most common cause of molar pregnancy-related maternal death is a type of cancer called choriocarcinoma, with the hydatid mole being the source and thus resembling a tumor. What characteristic of tumors leads to cancer? C. While these pregnancies often spontaneously abort, some women may elect to terminate a hydatid molar pregnancy. Is this a special case of elective abortion or is it tumor-removal? Explain.

Answer: A. The genotype would be 46XX or 46XY; 46YY is not viable at all because the essential genes on the X chromosome are missing. These genotypes are normal, so it is unclear why a fetus does not form. This suggests that there is some contribution of the maternal nucleus to development apart from just the chromosome number. B. It is metastasis, or spread of abnormal cells due to loss of cell adhesion, that causes cancer. C. (:0)Answers are based in part on personal opinion and will vary; if students consider primarily that this pregnancy does not produce a fetus at all, they may argue that removal of this tissue is more like tumor removal than aborting a person.

151) A. Distinguish between the mass of a molecule and the mass of a mole, using NaCl in your example. B. Calculate the mass of a mole of NaCl in g, using the mass of one Da (amu). C. Calculate the mass of a dozen NaCl molecules, a dozen donuts, and a mole of donuts, assuming a 30 gram donut.

Answer: A. The mass of a molecule is determined by the mass of its component atoms. From the periodic table, the mass of Na is 23 amu and of Cl is nearly 36 amu, so the mass of one molecule of NaCl is 59 Da. A mole is like a dozen, i.e., it is a particular number of items, specifically 6.02 × 1023. B. A mole of NaCl = 59 Da × 6.02 × 1023 = 3.55 × 1025 Da. 1 Da = 1.66 × 10-27 kg, so 3.55 × 1025 Da × 1.66 × 10-27 kg/Da × 1000 g/kg = 59 g. C. A dozen NaCl molecules: 12 × 59 Da × 1.66 × 10-27 kg/Da × 1000 g/kg = 1.2 × 10-21 g. A dozen donuts: 12 × 30 g = 360 g. A mole of donuts: 6.02 × 1023 × 30 g = 1.8 × 1025 g.

13) During the fasting state, the energy stores of the __________ become the major source of glucose for the whole body. A) brain B) liver C) muscles D) pancreas E) adipose tissues

Answer: B

115) A. Patrick is snorkeling (surface swimming with a tube in his mouth to maintain breathing while his face is in the water). The snorkel tube is 40 cm in length and 1.2 cm in radius. How does this affect Patrick's breathing? Calculate the increase in relative resistance contributed by the snorkel tube. B. What is dead space and how does that affect snorkeling? Answer, assuming the snorkel tube is a round cylinder and cylinder volume is pr2L. How does the snorkel tube affect alveolar ventilation (assume a ventilation rate of 12 breaths/min, tidal volume of 500 mL, and normal dead space volume of 150 mL)? If normal alveolar ventilation is 4200 mL/min, how must Patrick breathe during snorkeling compared to normal? C. Patrick loves snorkeling, but wishes he could go a little deeper. He decides to create a longer snorkel tube to try to achieve this desire. He isn't sure how radius figures in, so he makes two tubes of 60 cm length, one with a 1.2 cm radius and the other with a 2 cm radius. What are the relative resistances of these two new tubes? Do you think he will be able to snorkel well with either of these tubes? Explain. Explain your

Answer: A. The snorkel tube effectively increases the total length of the respiratory passage, which increases the resistance to air flow (RaL./r4); this makes Patrick work harder to breathe. Resistance increases proportionally to L/r4: 40/1.24 = 19. B. Dead space is the volume of air that enters the respiratory structures but does not reach the alveoli and cannot contribute to gas exchange. The snorkel tube increases the amount of dead space by p1.22 × 40 = 181 mL, which more than doubles the dead space to 331 mL. Alveolar ventilation = ventilation rate × (tidal volume - dead space) = 12/min × (500 mL - 331 mL) = 2028 mL/min; this is less than half of normal alveolar ventilation. To maintain 4200 mL/min, Patrick could increase his ventilation rate to 25 breaths/min (4200/(500-331)). Alternatively he could increase his ventilation depth to 681 mL/breath (4200/12 + 331). Or he could alter both rate and depth. C. Tube 1: 60/1.24 = 29. Tube 2: 60/24 = 3.75. The first tube would require significantly more effort to use than the tube in part A, so he would not be able to snorkel as long, if at all. The second tube sounds desirable, but dead space must be considered. The snorkel tube adds an additional p 2cm2 × 60 cm = 754 mL of dead space, which is greater than normal tidal volume and would reduce alveolar ventilation. Neither tube is practical.

149) Consider the two chemical reactions. Identify each statement following as true or false, and clarify as needed. A + B . C C . A + B A. The enzyme catalyzing the formation of C must be different from that catalyzing the breakdown of C. B. The enzyme catalyzing the formation of C must be the same as that catalyzing the breakdown of C. C. Only one of the reactions could occur in a given container or cell or system. D. Both reactions must be occurring in an equilibrium situation. E. Equilibrium is always achieved in a chemical reaction. F. Equilibrium can be prevented from occurring by constantly removing the product of either reaction. G. An equilibrium, once established, cannot be disturbed.

Answer: A. false. There may be more than one enzyme for the reactions listed, but a given enzyme is often capable of performing a reaction in both directions. B. false. The enzyme for both reactions could be the same, but often a particular enzyme is more likely to perform the forward reaction while a different enzyme is more likely to perform the reverse. C. false. If the reactions are at equilibrium, both are occurring in the same location. If they are not at equilibrium, it is possible that the reaction is occurring only in one direction. D. true. By definition, a reaction is at equilibrium when both directions are occurring such that there is no net change in amounts of reactants and products. E. false. Equilibrium is not necessarily established, especially in a cell where a product is immediately removed from the system for use elsewhere, such as the product ATP, or if a given enzyme does not perform the reverse reaction. F. true. Any change in amounts of enzyme, substrate, or reactant will disturb an equilibrium, and if these things constantly change, equilibrium will not be established. G. false. Equilibrium can be disturbed by changing the amount of enzyme, substrate, or product, or by altering the enzyme's efficacy by changing temperature or pH.

113) Below are results of a blood analysis for a patient undergoing chemotherapy. The left two sets of bars, labeled "Min" and "Max," represent the target values for each cell type. The other sets of bars depict the results of blood counts done on the corresponding days after chemotherapy was administered. Please Answer the following questions. A. According to the graph, which type of formed elements reached normal values first? B. Which kind came closest to its maximum value? C. On Day 10, which of the following sets of symptoms/characteristics would the patient most likely Exhibit: 1. pale and easily fatigued 2. likely to bruise easily or bleed excessively 3. be at high risk for infections Explain your choice.

Answer: A. platelets B. RBCs C. 3, because the WBC count is the most below minimum values, and it is the WBCs that fight infection.

83) Because the suffix "itis" means __________, swelling of the lower extremities due to blockage of lymph flow is NOT called ________-itis; rather it is called __________.

Answer: "inflammation of", elephant, elephantiasis

149) The resting membrane potential is __________ for skeletal muscle and __________ for contractile myocardium. It is __________ for autorhythmic myocardium.

Answer: -70 mV, - 90 mV; an unstable pacemaker potential (usually starts at -60 mV)

81) Describe differences in effects of hormones of growth and metabolism in children compared to effects in adults. Consider effects of normal amounts, as well as hyper- and hyposecretion.

Answer: 1. Cortisol provides a catabolic protective effect against hypoglycemia, enhancing the effectiveness of glucagon, important in response to stress. Hypocortisolism in newborn girls can masculinize the external genitals. In adult females, excess body hair may result. 2. Thyroid hormone provides substrates for oxidative metabolism in adults and is necessary for full expression of growth hormone in children. Thyroid hormone deficiency in children will be developmentally delayed and short in stature and may have mental retardation. In adults, hypothyroidism may cause cold-intolerance, slow reflexes, fatigue, slow heart rate, and many other conditions. 3. Growth hormone is anabolic, essential for tissue growth and maintenance. Growth hormone deficiency causes dwarfism and excess causes giantism in children. Acromegaly characterizes GH excess in adults. 4. Calcitonin is essentially for normal skeletal growth in children, and is important in adults when net bone deposition is needed, such as during pregnancy.

114) List the four basic processes of the digestive system, and describe each.

Answer: 1. Digestion is the chemical and mechanical breakdown of foods into smaller units. 2. Absorption is the active or passive transfer of substances from the lumen of the GI tract to the extracellular fluid. 3. Motility is the movement of material in the GI tract as a result of muscle contraction. 4. Secretion refers to both transepithelial transfer of water and ions from the ECF to digestive tract lumen and the release of substances synthesized by GI epithelial cells.

98) Air flow in the respiratory system and blood flow in the cardiovascular system have many similarities—name them. How are they different?

Answer: 1. Flow takes place from regions of higher pressure to regions of lower pressure. 2. A muscular pump creates pressure gradients. 3. Resistance to fluid flow is influenced primarily by the diameter of the tubes. The primary difference is that air is a compressible mixture of gases while blood is a noncompressible liquid.

99) What are the functions of pleural fluid?

Answer: 1. It creates a moist, slippery surface so opposing membranes can slide across one another as the lungs move within the thorax. 2. It holds the lungs tight against the thoracic wall.

13) Most reflex movements are integrated by A) the brain. B) the spinal cord. C) central pattern generators. D) proprioceptors. E) effectors.

Answer: B

63) Describe GFR autoregulation, and explain why it is important.

Answer: 1. The myogenic response is the ability of the vascular smooth muscle to constrict or dilate to keep pressure and flow constant, thereby keeping GFR constant. Vasoconstriction is more effective at this than vasodilation. 2. Tubuloglomerular feedback involves production of a paracrine, in response to increased GFR, which constricts the afferent arteriole to decrease GFR. Autoregulation helps protect the filtration barriers from high blood pressures that could damage them.

100) Summarize the four gas laws.

Answer: 1. The total pressure of a mixture of gases is the sum of the pressures of the individual gases (Dalton's law). 2. Gases, singly or in a mixture, move from areas of higher pressure to areas of lower pressure. 3. If the volume of a container of gas changes, the pressure of the gas will change in an inverse manner (Boyle's law). 4. The pressure and volume of a container of gas are directly related to the temperature of the gas and the number of molecules in the container: PV = nRT (the ideal gas law).

76) What are the three common causes of hypercortisolism?

Answer: 1. adrenal tumor autonomously secretes cortisol (primary hypercortisolism) 2. pituitary tumor autonomously secretes ACTH (secondary hypercortisolism) 3. iatrogenic hypercortisolism (physician-caused)

159) What are the five ways that cells regulate flow of molecules through their metabolic pathways?

Answer: 1. by controlling the enzyme concentration 2. by producing allosteric and covalent modulators 3. by using two different enzymes to catalyze reversible reaction 4. by isolating enzymes within intracellular organelles 5. by maintaining an optimum ratio of ATP to ADP

68) What are the two ways of bicarbonate reabsorption in the proximal tubule?

Answer: 1. convert HCO3- into carbon dioxide, then back into HCO3- 2. through the metabolism of glutamine (see Fig. 20-21)

61) List and summarize the forces that influence filtration across the walls of glomerular capillaries.

Answer: 1. hydrostatic pressure 2. colloid osmotic pressure 3. hydrostatic fluid pressure See the "Filtration Occurs Because of Hydrostatic Pressure in Capillaries" section of the CHAPTER.

87) Name the two mechanisms and the factors involved that limit the extent of blood clotting within a vessel.

Answer: 1. inhibition of platelet adhesion: protacyclin 2. inhibition of coagulation cascade: plasminogen and plasmin and anticoagulants

88) Name the five types of white blood cells. Which type develops into macrophages? Which white blood cells are collectively known as phagocytes and why?

Answer: 1. lymphocytes 2. monocytes 3. neutrophils 4. eosinophils 5. basophils; monocytes develop into macrophages; neutrophils, monocytes, and macrophages are collectively known as phagocytes because they engulf and ingest foreign particles such as bacteria.

79) Diabetes mellitus produces many homeostatic imbalances, including acidosis. The pH imbalance is due to ketoacidosis, which results from excessive accumulation of by-products of fat metabolism, as the body cannot meet energy needs from carbohydrate metabolism. Sally is a teenaged diabetic who sometimes rebels by not taking her insulin. Her body is beginning to develop ketoacidosis as a result. Create a chart to indicate and explain how her blood pH, HCO3-, and PCO2 react, by indicating "increase," "decrease," or "no change." 1. ketoacidosis has just developed 2. respiratory compensation occurs 3. renal compensation occurs

Answer: 1. pH decreases by definition, HCO3- decreases as the carbonic acid reaction shifts to the right, PCO2 is unchanged because the excess is expelled by the lungs due to the higher gradient. 2. pH increases and HCO3- decreases as exhaled CO2 drives the carbonic acid reaction to the right, and PCO2 decreases as it is exhaled. 3. pH increases as H+ is excreted, HCO3- increases as it is reabsorbed, and increases as respiration returns to normal.

66) What three stimuli control vasopressin secretion? The most potent stimulus for vasopressin release is __________.

Answer: 1. plasma osmolarity 2. blood volume 3. blood pressure; an increase in plasma osmolarity

60) List and describe the three filtration barriers that substances leaving the plasma must pass through before entering the tubule lumen.

Answer: 1. the glomerular capillary endothelium 2. a basal lamina 3. the epithelium of Bowman's capsule See the "The Renal Corpuscle Contains Three Filtration Barriers" section of the CHAPTER.

62) Reabsorption involves what two methods of transport? Describe each. What determines which route a solute will take?

Answer: 1. transepithelial transport: substances cross both apical and basolateral membranes of the tubule epithelial cell. 2. paracellular pathway: substances pass through the junction between two adjacent cells. The permeability of the epithelial junction and the electrochemical gradient for the solute determines which route it will take.

10) Each of the following changes will result in increased blood flow to a tissue except one. Identify the exception. A) increased blood volume B) decreased vessel diameter C) increased blood pressure D) decreased peripheral resistance E) relaxation of precapillary sphincters

Answer: B

100) A drug that blocks the action of carbonic anhydrase in parietal cells would result in A) a lower pH during gastric digestion. B) a higher pH during gastric digestion. C) decreased production of pepsinogen by chief cells. D) increased protein digestion in the stomach. E) decreased gastrin production.

Answer: B

100) Looking at elements 1-20 in the periodic table, what do you notice about the relationship between the atomic number and the atomic mass? A) The atomic mass is roughly the same as the atomic number. B) The atomic mass is roughly double the atomic number. C) The atomic mass is roughly half the atomic number. D) There is no predictable relationship between these values.

Answer: B

101) A drug that blocks the action of the hormone cholecystokinin would affect A) the amount of bile produced by the liver. B) the composition of pancreatic secretions. C) the level of intestinal gastrin. D) secretions of the duodenal glands. E) all of the above

Answer: B

102) A double covalent bond is formed when atoms A) share one pair of electrons (a total of two). B) share two pairs of electrons (a total of four). C) swap two pairs of electrons. D) transfer a pair of electrons from one atom to the other. E) transfer two pairs of electrons from one atom to the other.

Answer: B

107) Magnesium atoms have two electrons in the outermost shell and chlorine atoms have seven. The compound magnesium chloride would contain A) one magnesium and one chlorine. B) one magnesium and two chlorine. C) two magnesium and one chlorine. D) two magnesium and seven chlorine. E) impossible to tell without more information

Answer: B

108) transports iron to the blood

Answer: B

11) A glomerulus is A) the expanded end of a nephron. B) a "knot" of capillaries that lies within the Bowman's capsule. C) the portion of the nephron closest to the renal corpuscle. D) the portion of the nephron that attaches to the collecting duct. E) the hairpin-shaped segment of the nephron.

Answer: B

11) Most of the oxygen transported by the blood is A) dissolved in plasma. B) bound to hemoglobin. C) in ionic form as solute in the plasma. D) bound to a plasma protein. E) carried by white blood cells.

Answer: B

11) The motor activity of the muscularis externa is controlled by the A) submucosal plexus. B) myenteric plexus. C) migrating motor complex. D) extrinsic neurons. E) motilin.

Answer: B

11) The values obtained when measuring blood pressure, such as 120/80, A) exactly match the pressures inside the ventricle during systole and diastole. B) reflect the pressure in the major arteries during ventricular systole and diastole. C) are the same on both the pulmonary and systemic circuits. D) A and B E) A, B, and C

Answer: B

11) Which of the following has its cell body in the ganglion? A) preganglionic neuron B) postganglionic neuron C) somatic motor neuron D) A and B E) all of the above

Answer: B

110) A layer of glycoproteins and a network of fine protein filaments that prevents the movement of proteins and other large molecules from the connective tissue to epithelium describes A) interfacial canals. B) the basal lamina. C) the reticular lamina. D) areolar tissue. E) endothelium.

Answer: B

111) Put these structures in the correct order in which sound waves are transmitted for hearing. 1. malleus 2. oval window 3. tympanic membrane 4. stapes 5. endolymph 6. perilymph 7. incus A) 1, 7, 4, 3, 2, 5, 6 B) 3, 1, 7, 4, 2, 6, 5 C) 2, 1, 7, 4, 3, 6, 5 D) 3, 2, 1, 7, 4, 6, 5 E) 3, 1, 4, 7, 2, 5, 6

Answer: B

113) Low frequency sound waves create the maximum displacement __________ of the basilar membrane. A) near the stiff and narrow proximal end B) near the flexible and wide distal end C) in the middle

Answer: B

116) Put these autorhythmic cells into the correct order for conveying electrical signals through a normal heart. 1. bundle of His 2. internodal pathway 3. Purkinje fibers 4. atrioventricular node 5. sinoatrial nodes 6. left and right bundle branches A) 5, 2, 1, 6, 4, 3 B) 5, 2, 4, 1, 6, 3 C) 4, 2, 5, 1, 6, 3 D) 3, 6, 1, 4, 2, 5 E) 5, 4, 1, 6, 2, 3

Answer: B

117) Left ventricular pressure is higher than pressure in the aorta during A) atrial systole. B) ventricular diastole. C) ventricular systole. D) A and C E) all of the above

Answer: B

117) Only totipotent stem cells are capable of producing new cells in an adult. A) true B) false

Answer: B

119) A certain drug decreases heart rate by producing hyperpolarization in the pacemaker cells of the heart. This drug probably binds to A) nicotinic receptors. B) muscarinic receptors. C) alpha adrenergic receptors. D) beta receptors.

Answer: B

119) A device that monitors the activity of the photoreceptor cells of the eye indicates that there is a constant flow of neurotransmitter being released by the photoreceptor cells. This information implies that the subject is A) reading. B) in a dark room. C) outside in sunlight. D) in a brightly lit auditorium. E) focusing on a distant object.

Answer: B

12) The induced-fit model of enzyme activity states that A) either the substrate or the products can bend to bind with the enzyme, allowing interaction. B) neither the substrate nor the products can bend to bind with the enzyme; the enzyme must bend to allow interaction. C) only the active site of the substrate interacts with the enzyme. D) None of these describes the induced-fit model.

Answer: B

12) The portion of the nephron closest to the renal corpuscle is the A) loop of Henle. B) proximal tubule. C) distal tubule. D) collecting duct. E) minor calyx.

Answer: B

120) Under which set of circumstances would the diameter of peripheral blood vessels be the greatest? A) increased sympathetic stimulation B) decreased sympathetic stimulation C) increased parasympathetic stimulation D) decreased parasympathetic stimulation E) both increased parasympathetic and increased sympathetic stimulation

Answer: B

122) The following steps occur in rods when they are excited by photons of light. Place them in the correct sequence. 1. Membrane sodium channels close. 2. cGMP is broken down. 3. The membrane hyperpolarizes and the rate of neurotransmitter release declines. 4. Transducin is activated. 5. Opsin is activated. A) 1, 5, 2, 4, 3 B) 5, 4, 2, 1, 3 C) 3, 5, 2, 1, 4 D) 4, 5, 2, 1, 3 E) 1, 3, 4, 5, 2

Answer: B

124) This tissue actively and selectively regulates the exchange of nongaseous material, such as ions and nutrients, and can be regulated in response to various stimuli.

Answer: B

126) During feedback inhibition, in order to slow or stop a cell from using a particular biochemical pathway, the end product of that pathway A) is removed from the cell more quickly. B) accumulates inside the cell. C) turns into a sticky mucus. D) becomes toxic to the cell.

Answer: B

127) If the membranes of the cardiac muscle cells in the SA node become more permeable to potassium ions, A) the heart rate will increase. B) the heart rate will decrease. C) the membrane will depolarize. D) the stroke volume will increase. E) the intracellular concentration of calcium ion will increase.

Answer: B

127) Put these structures of the visual system in the order that they transmit light or light-related information. 1. optic disk 2. lens 3. cornea 4. visual cortex of the occipital lobe 5. rods 6. optic chiasm 7. lateral geniculate body of the thalamus A) 2, 3, 1, 5, 7, 6, 4 B) 3, 2, 5, 1, 6, 7, 4 C) 1, 3, 2, 5, 6, 7, 4 D) 5, 3, 1, 2, 6, 7, 4 E) 3, 2, 1, 5, 7, 6, 4

Answer: B

130) Which of the following conditions would have the greatest effect on peripheral resistance? A) doubling the length of a vessel B) doubling the diameter of a vessel C) doubling the viscosity of the blood D) doubling the turbulence of the blood E) doubling the number of white cells in the blood

Answer: B

131) If the connection between the AV node and bundle of His becomes blocked, A) the ventricles will beat faster. B) the ventricles will beat more slowly. C) the ventricular rate of contraction will not be affected. D) the stroke volume will increase. E) tachycardia will occur.

Answer: B

135) In which of the following situations would the end-systolic volume (ESV) be the greatest? A) when sympathetic stimulation of the heart is increased B) when parasympathetic stimulation of the heart is increased C) when the force of myocardial contraction is increased D) when the intracellular stores of calcium are increased E) when stroke volume is increased

Answer: B

136) In completing the phases of glucose catabolism leading to ATP, the carbons that start the reaction as glucose actually end up A) as ATP. B) as CO2. C) as NADH. D) as oxaloacetate. E) as pyruvate.

Answer: B

139) Light shines on surround of off-surround cell.

Answer: B

139) The sense strand of DNA A) is not involved in mRNA synthesis. B) serves as a guide for mRNA synthesis. C) is not a strand of DNA. D) none of the above

Answer: B

14) Iodine accumulation in thyroid cells involves symport with A) hydrogen. B) sodium. C) potassium. D) chloride.

Answer: B

140) Joey develops a medical condition that decreases his blood viscosity. Assuming no other change (e.g., no compensatory reflex), what happens to his blood pressure?

Answer: B

141) When a blood vessel dilates, resistance through that vessel is __________.

Answer: B

142) If the sense strand of DNA's triplet is ATC, its complementary antisense strand is A) UAG. B) TAG. C) UTC. D) ATC.

Answer: B

143) When a blood vessel dilates, velocity of blood is __________.

Answer: B

147) If total cross-sectional area of blood vessels in an organ increases, what happens to velocity of blood through that organ?

Answer: B

15) The alpha cells of the pancreas produce A) insulin. B) glucagon. C) renin. D) cortisol. E) digestive enzymes.

Answer: B

15) The most important chemical regulator of respiration is A) oxygen. B) carbon dioxide. C) bicarbonate ion. D) sodium ion. E) hemoglobin.

Answer: B

15) When venous return is increased, stretch receptors in the atria of the heart are activated. This results in A) secretion of ADH. B) inhibition of ADH secretion. C) increased glomerular filtration. D) decreased urine production. E) increased thirst.

Answer: B

16) Chemicals such as histamine, prostaglandins, serotonin, and substance P A) increase the inflammatory response. B) activate nociceptors by lowering their activation threshold. C) can cause tissue damage. D) A and B E) all of the above

Answer: B

16) The amount of plasma that filters into the nephrons is approximately __________ of the total volume. A) 4/5 B) 1/5 C) 3/4 D) 1/2 E) 90%

Answer: B

16) The subepithelial connective tissue of the GI tract, immediately beneath the epithelium, is the A) muscularis mucosae. B) lamina propria. C) submucosa. D) serosa. E) submucosal plexus.

Answer: B

16) When baroreceptors in the carotid and aortic bodies register increased blood pressure, this results in A) secretion of ADH. B) inhibition of ADH secretion. C) increased glomerular filtration. D) decreased urine production. E) increased thirst.

Answer: B

17) The initial change in heart rate at the onset of exercise is due to A) sympathetic stimulation. B) the withdrawal of parasympathetic inhibition. C) stretching of the cardiac muscle fibers in response to increased venous return. D) overfilling of the ventricles.

Answer: B

17) The structure whose abnormal function is associated with Parkinson's disease is the A) spinal cord. B) basal ganglia. C) cerebellum. D) primary motor cortex. E) skeletal muscle.

Answer: B

18) A competitive inhibitor binds to A) a region of the enzyme other than the active site. B) the active site. C) the substrate. D) the product. E) the surrounding tissue.

Answer: B

18) The expiratory neurons control the __________ muscles, whereas the inspiratory neurons control the __________ muscles. A) diaphragm and external intercostal, abdominal and internal intercostal B) abdominal and internal intercostal, diaphragm and external intercostal C) diaphragm and internal intercostal, abdominal and external intercostal D) abdominal and external intercostal, diaphragm and internal intercostal E) diaphragm and abdominal, intercostal internal and external

Answer: B

19) Sperm production occurs in A) the ductus deferens. B) the seminiferous tubules. C) the epididymis. D) the seminal vesicles. E) the rete testis.

Answer: B

19) When blood glucose levels fall, A) insulin is released. B) glucagon is released. C) peripheral cells take up less glucose. D) protein synthesis decreases. E) all of the above

Answer: B

2) An angstrom is A) 6.02 × 10-23 meters. B) 1 × 10-10 meters. C) 1.66 × 10-27 meters. D) 1 × 10-10 kg. E) 1.66 × 10-27 kg.

Answer: B

2) Egg and sperm cells have A) 46 chromosomes. B) 23 chromosomes. C) 22 pairs of autosomes plus one pair of sex chromosomes. D) A and B E) A, B, and C

Answer: B

2) Glycolytic metabolism is also known as A) aerobic metabolism. B) anaerobic metabolism. C) gluconeogenesis. D) glycogenesis. E) beta-oxidation.

Answer: B

2) If the partial pressure of oxygen in both air and water is 100 mm Hg, then the concentration of the oxygen is the same in the air and water. A) true B) false

Answer: B

2) The space that is surrounded by the tissue wall of hollow organs is known as A) the peritoneal cavity. B) the lumen. C) the extracellular space. D) the epidural space. E) the tract.

Answer: B

2) The two divisions of the efferent side of the peripheral nervous system are A) somatic motor neurons and voluntary neurons. B) somatic motor neurons and autonomic neurons. C) the sympathetic and parasympathetic divisions. D) voluntary nervous system and somatic motor neurons.

Answer: B

20) Air moves out of the lungs because A) the gas pressure in the lungs is less than outside pressure. B) the volume of the lungs decreases with expiration. C) the thorax is muscular. D) contraction of the diaphragm increases the volume of the pleural cavity. E) all of the above

Answer: B

20) All of the following are characteristics of acquired immunity EXCEPT A) cell-mediated immunity. B) nonspecific immunity. C) humoral immunity. D) adaptive immunity.

Answer: B

20) The primary function of the proximal tubule is A) filtration. B) reabsorption of ions, organic molecules, and water. C) secretion of acids and ammonia. D) secretion of drugs. E) adjusting the urine volume.

Answer: B

20) You are interested in learning more about Parkinson's disease, a neurological disorder that primarily affects motor function. What is the best source to begin your investigation? A) Google B) PubMed C) public library D) physiology textbook E) a physician

Answer: B

21) An enzyme that digests fats is A) hydrolase. B) lipase. C) lyase. D) ligase. E) kinase.

Answer: B

22) Aldosterone A) is secreted in response to elevated levels of sodium in the blood. B) promotes sodium retention in the kidneys. C) helps decrease blood volume. D) increases the concentration of sodium in urine. E) functions in pH regulation.

Answer: B

22) Chemotaxins function in A) digesting invading cells. B) attracting additional immune cells. C) poisoning and killing invading cells. D) A and B E) A, B, and C

Answer: B

22) During increased exercise, A) vasoconstriction occurs at the active skeletal muscles. B) venous return increases. C) cardiac output decreases. D) stroke volume decreases. E) all of the above

Answer: B

22) During starvation, A) carbohydrate utilization increases. B) gluconeogenesis accelerates. C) there is a decline in circulating ketone bodies. D) structural proteins cannot be used as a potential energy source. E) all of the above

Answer: B

22) In regard to lipids, the term unsaturated refers to A) the lack of double bonds between adjacent carbon atoms in a fatty acid. B) the presence of double bonds between adjacent carbon atoms in a fatty acid. C) the ring structure of steroids. D) glycerol, which acts as an anchor for joined fatty acids. E) fats, such as butter and lard, which come from animal sources.

Answer: B

22) Slow waves are A) cycles of smooth muscle contraction and relaxation. B) spontaneous cycles of depolarization and repolarization. C) reflexes that originate and are integrated in the enteric nervous system. D) peristaltic contractions. E) segmental contractions.

Answer: B

23) Calcium reabsorption at the kidneys is promoted by the hormone A) calcitonin. B) calcitriol. C) aldosterone. D) cortisol. E) ADH.

Answer: B

23) Centrioles A) function as pipelines to move fluid through the cell. B) direct the movement of DNA during cell division. C) hold the cell's ribosomes in place. D) are white blood cells out of vessels. E) provide shape and stability to a cell.

Answer: B

23) Reactive hyperemia is A) lack of blood flow due to an allergic reaction. B) increased blood flow following a period of reduced blood flow. C) increased blood pressure after stress. D) reflex contraction of smooth muscle in response to stress. E) none of the above

Answer: B

23) The cervix is A) homologous to the penis. B) the neck of the uterus. C) the sensory tissue of the vagina. D) A and B E) A, B, and C

Answer: B

23) The term that means "a substance that can be tasted" is A) gustatant. B) tastant. C) umami. D) gustducin. E) analgesic.

Answer: B

23) an example would be slowing or speeding the heart and other internal organs that are not consciously controlled

Answer: B

24) Opsonins function by A) poisoning and killing invading cells. B) marking or tagging pathogens so phagocytes can find and ingest them. C) digesting invading cells directly. D) A and B E) A, B, and C

Answer: B

24) The alpha-helix and pleated sheet are examples of the __________ structure of a protein. A) primary B) secondary C) tertiary D) quaternary E) pentanary

Answer: B

24) The brief period of time between the end of the action potential in the muscle and the beginning of contraction is referred to as A) the refractory period. B) the latent period. C) the relaxation phase. D) the depolarization period. E) the repolarization period.

Answer: B

25) Which does NOT accurately complete the sentence? One of the major functions of both types of endoplasmic reticulum (ER) is the A) synthesis of biomolecules. B) storage of genetic material for the cell. C) transport of biomolecules. D) storage of biomolecules.

Answer: B

25) movement from the glomerulus to the nephron lumen

Answer: B

26) A primary target tissue for insulin is the A) brain. B) liver. C) intestine. D) A and B E) A, B, and C

Answer: B

26) As the environmental pH or temperature of a particular enzymatic reaction is changed, the activity of the enzyme may decrease due to A) destruction of the amino acid sequence. B) change of the three-dimensional structure of the enzyme. C) change in the shape of the substrate. D) locking of the enzyme-substrate complex.

Answer: B

26) Metabolism of __________ is the fastest way for cells to increase their supply of ATP in order to meet the ATP demands of heavy exercise. A) proteins B) glucose C) fatty acids D) A and B E) A, B, and C

Answer: B

27) Adaptive significance is an important concept in physiology because it describes A) the importance of a highly variable external environment. B) the physiological functions that promote an organism's survival. C) the ability of an organism to monitor and restore its internal state to normal conditions when necessary. D) the similarities between ancient and modern marine organisms. E) the parameters necessary to maintain a constant internal environment.

Answer: B

27) Muscle fatigue is classified into these types: A) alpha and beta. B) central and peripheral. C) consonant and dissonant. D) striated and unstriated. E) eccentric and concentric.

Answer: B

27) The active sites of enzymes are A) products of an enzymatic reaction. B) regions of an enzyme that are involved in bringing reactants together. C) bonds between the substrate and certain amino acids. D) not necessarily involved in a reaction.

Answer: B

27) The neurotransmitter acetylcholine is released by neurons at the neuroeffector synapse.

Answer: B

28) There are __________ primary taste sensations. A) 2 B) 5 C) 12 D) 20 E) more than 50

Answer: B

28) found at the junction of tendons and muscle fibers

Answer: B

28) increased PCO2

Answer: B

29) Red blood cell production increases when A) oxygen levels in the blood increase. B) oxygen levels in the blood decrease. C) carbon dioxide levels in the blood increase. D) carbon dioxide levels in the blood decrease. E) protein levels in the blood increase.

Answer: B

29) increased 2,3-DPG

Answer: B

3) All of the hormones secreted by the adrenal gland play a major role in growth and metabolism. A) true B) false

Answer: B

3) Antibiotics are drugs that destroy or inhibit the growth of A) viruses. B) bacteria. C) multicellular pathogens such as hookworms. D) A and B E) A, B, and C

Answer: B

3) Because anatomy and physiology have different definitions, they are usually considered separately in studies of the body. A) true B) false

Answer: B

3) Identify the FALSE statement. A) Adipose tissue is controlled by autonomic efferents. B) All reflexes require input from the brain. C) Some reflexes are genetically determined. D) Muscle spindles are stretch receptors. E) Proprioceptors detect limb position and movement.

Answer: B

3) The lumen of a hollow organ such as the stomach is considered to be part of the __________ environment. A) internal B) external

Answer: B

3) The upper respiratory tract includes all EXCEPT which of the following? A) nasal cavity B) lungs C) trachea D) larynx E) mouth

Answer: B

3) With the exception of olfaction, all sensory pathways first travel to the __________, which acts as a relay and processing station. A) cerebrum B) thalamus C) cerebellum D) hypothalamus E) medulla oblongata

Answer: B

30) Emission is defined as A) engorgement of the penis. B) movement of sperm into the urethra. C) movement of sperm out of the penis. D) loss of erection following orgasm. E) failure to achieve ejaculation.

Answer: B

30) Hyperthyroidism A) decreases oxygen consumption. B) increases protein catabolism. C) decreases protein synthesis. D) slows heart rate. E) causes cold intolerance.

Answer: B

30) Metabolism is a term that describes A) all work done by a living organism. B) all chemical reactions that take place within an organism. C) only chemical reactions that release ATP from living cells. D) the energy released from chemical bonds in living cells. E) the extraction of nutrients from biomolecules.

Answer: B

30) The proteins synthesized in the rough endoplasmic reticulum are then sent to A) the smooth endoplasmic reticulum for storage. B) the Golgi complex for packaging. C) the lysosome for modification. D) the cell membrane for secretion. E) the nucleus for cellular use.

Answer: B

31) Due to the differences in opposing forces, there is net __________ occurring at the arteriolar end of most capillaries, coupled with net __________ at the venous end. A) absorption, filtration B) filtration, absorption

Answer: B

31) The primary purpose of the auditory ossicles is A) to equalize pressure in the middle ear. B) to amplify vibration. C) to dampen vibration. D) to transmit otitis media. E) none of the above

Answer: B

32) The corpus luteum in a nonpregnant woman usually lasts about A) 7 days. B) 12 days. C) 3 days. D) 28 days. E) 3 months.

Answer: B

32) Within a single fiber, the tension developed during a twitch depends on A) the magnitude of the (at or above threshold) stimulus. B) the length of the sarcomeres prior to contraction. C) the force required to move the relevant joint. D) the amount of neurotransmitter in the synapse. E) the duration of the stimulus.

Answer: B

33) Convective heat loss occurs when A) water evaporates from the skin's surface. B) warm air rises from the body's surface. C) a cooler object rests on the body's surface. D) one swims in water below body temperature. E) all of the above

Answer: B

33) The carrier protein that transports absorbed iron through the blood is A) hemoglobin. B) transferrin. C) erythropoietin. D) thrombopoietin. E) intrinsic factor.

Answer: B

34) In the condition known as complete heart block, what happens? A) Coronary arteries are blocked by plaques, preventing blood and oxygen from reaching the myocardial contractile cells. B) Electrical signals from the SA node never reach the ventricles, so the contraction of the atria is not coordinated with the contraction of the ventricles. C) The fibrous skeleton of the heart breaks down, interfering with the passage of blood from the atria to the ventricles. D) The mitral valve leaflets calcify and close, preventing blood from being pumped efficiently by the left side of the heart. E) Blood flow through the foramen ovale is blocked.

Answer: B

12) The primary osmoreceptors are located in the A) pons. B) kidney. C) stomach. D) hypothalamus. E) medulla.

Answer: D

36) The gastric phase of gastric secretion is triggered by A) the sight, thought, or smell of food. B) the entry of food into the stomach. C) the entry of chyme into the small intestine. D) the entry of chyme into the large intestine. E) the release of cholecystokinin and secretin by the small intestine.

Answer: B

37) Some symptoms mimic diabetes mellitus.

Answer: B

38) A heart rate of 125 beats per minute could be correctly termed A) bradycardia. B) tachycardia. C) an arrhythmia. D) fibrillation. E) a normal resting heart rate.

Answer: B

38) In the aerobic metabolism of glucose, __________ acts as the final electron acceptor. A) water B) oxygen C) carbon dioxide D) lactate E) ATP

Answer: B

38) When blood pressure is normal, the receptors in arterial walls fire action potentials A) very seldom. B) continuously.

Answer: B

38) Which of the following is a common feature of soluble proteins? A) structural support B) noncovalent interaction C) receptor binding D) chemical modulation E) all of the above

Answer: B

38) synthesis of tissues

Answer: B

39) When blood pressure receptors sense a loss of blood pressure, they __________ their firing rate. A) increase B) decrease

Answer: B

39) Which event happens at the start of a cardiac cycle? A) Blood is ejected from the atrium. B) The SA node fires. C) The P wave develops. D) Ventricular systole occurs. E) Atrial systole occurs.

Answer: B

4) A receptor potential is A) an action potential. B) a graded potential. C) the resting membrane potential of a receptor cell. D) A or B E) none of the above

Answer: B

4) Mast cells in tissues are considered to be a type of A) eosinophil. B) basophil. C) lymphocyte. D) monocyte. E) neutrophil.

Answer: B

4) The advantage of anaerobic metabolism is A) high yield of ATP. B) rapid ATP production. C) fewer pesky by-products. D) greater use of fat stores. E) greater endurance.

Answer: B

4) The basic structural and functional unit of the human body is A) protein. B) the cell. C) tissue. D) the organ. E) the organ system.

Answer: B

4) The two subatomic particles that contribute almost all of the mass of any particular element are: 1. electrons 2. protons 3. neutrons A) 1 and 2 B) 2 and 3 C) 1 and 3 D) electrons only E) protons only

Answer: B

4) Which of the following is an accessory organ of digestion? A) stomach B) pancreas C) spleen D) colon E) esophagus

Answer: B

40) The structure that separates the cochlear duct from the tympanic duct is the A) tectorial membrane. B) basilar membrane. C) membranous labyrinth. D) bony labyrinth. E) stapedius.

Answer: B

40) Which of the following events result in the first heart sound? A) The AV valves open. B) The AV valves close. C) The semilunar valves close. D) The semilunar valves open. E) The atria contract.

Answer: B

41) Coding sound for pitch is a function of the A) tectorial membrane. B) basilar membrane. C) vestibular membrane. D) endolymph. E) ossicles.

Answer: B

41) During the isovolumic phase of ventricular systole, A) the atria contract. B) the atrioventricular valves and semilunar valves are closed. C) blood is ejected into the great vessels. D) the ventricles are relaxing. E) the ventricles are filling with blood.

Answer: B

41) Isotopes of the same element differ by having different numbers of __________.

Answer: B

41) The hormone primarily responsible for the milk let-down reflex is A) progesterone. B) oxytocin. C) prolactin. D) estrogen. E) growth hormone.

Answer: B

42) During the cardiac cycle, A) the P wave of the ECG occurs between the first and second heart sounds. B) the QRS complex of the ECG precedes the increase in ventricular pressure. C) the third heart sound occurs during atrial systole. D) the second heart sound coincides with the QRS complex of the ECG. E) the greatest increase in ventricular pressure occurs during the ejection phase.

Answer: B

42) Louder noises result in __________ action potentials in the sensory neuron. A) taller B) more frequent C) less frequent D) shorter E) broader

Answer: B

42) large molecules broken into smaller ones

Answer: B

42) may result from a pituitary tumor

Answer: B

43) Before converting amino acids into intermediates for energy metabolism, they must first undergo A) denaturation. B) deamination. C) detoxification. D) depeptidization. E) delousing.

Answer: B

43) Which of these does NOT increase the risk for cardiovascular disease? A) being a male over 30 but under 55 B) being a female over 30 but under 55 C) having a sister with coronary artery disease D) menopause without estrogen replacement therapy E) having diabetes mellitus

Answer: B

44) The cardiac output is equal to A) the difference between the end-diastolic volume and the end-systolic volume. B) the product of heart rate and stroke volume. C) the difference between the stroke volume at rest and the stroke volume during exercise. D) the stroke volume less the end-systolic volume. E) the product of heart rate and blood pressure.

Answer: B

44) section where chyme is processed to remove water and electrolytes, leaving waste products of digestion

Answer: B

45) the amount of air inhaled during an active (forced) inspiration

Answer: B

46) Place the following blood vessels that carry blood to and within the kidney in the order in which blood passes through them. 1. afferent arteriole 2. efferent arteriole 3. glomerulus 4. peritubular capillary A) 4, 2, 3, 1 B) 1, 3, 2, 4 C) 4, 3, 2, 1 D) 1, 2, 3, 4 E) 3, 4, 2, 1

Answer: B

47) Drugs known as beta-blockers will A) increase heart rate. B) decrease heart rate. C) increase stroke volume. D) increase cardiac output. E) decrease the end-systolic volume.

Answer: B

47) This condition results from excess androgens in females.

Answer: B

47) a category that includes clotting factors and enzymes

Answer: B

48) The __________ provides a reference point for the direction of bending of stereocilia on hair cells. A) tip link B) kinocilium C) otolith D) macula E) cupula

Answer: B

48) spermatogonium or oogonium

Answer: B

49) An obstruction in a glomerulus would affect the flow of blood into the A) renal artery. B) efferent arteriole. C) afferent arteriole. D) renal vein.

Answer: B

49) The brain relies solely on __________ as its energy source. If it is deprived of this substrate, the cells soon die. A) fatty acids B) glucose C) amino acids D) insulin

Answer: B

49) hunchback appearance associated with osteoporosis

Answer: B

5) The purpose of having valves in the cardiovascular system is to A) provide sounds so that heart health can be monitored. B) ensure that blood flows in one direction. C) prevent blood from flowing too quickly. D) regulate blood pressure. E) provide the force for circulation.

Answer: B

5) The purpose of transverse tubules is to A) ensure a supply of Ca2+ ions through the muscle fiber. B) rapidly conduct action potentials to the interior of the muscle fiber. C) ensure a supply of glycogen throughout the muscle sarcoplasm. D) conduct ATP molecules out of the mitochondria throughout the sarcoplasm. E) All of the above are true.

Answer: B

50) primary spermatocyte or oocyte

Answer: B

51) salivation

Answer: B

52) The sensory receptors of the semicircular canals are located in the A) saccules. B) ampullae. C) cristae. D) utricles. E) cupulas.

Answer: B

52) a bundle of adjacent muscle cells

Answer: B

53) Protein digestion begins here.

Answer: B

53) The simple squamous epithelial lining of blood vessels is called A) basolateral epithelium. B) endothelium. C) luteal cells. D) the vasa recta. E) none of the above

Answer: B

53) The typical pattern for molecules absorbed by Na+-dependent transport involves this: an apical __________ and a basolateral __________. A) facilitated diffusion carrier, osmotic gradient B) symport protein, facilitated diffusion carrier C) symport protein, osmotic gradient D) osmotic gradient, symport protein E) facilitated diffusion carrier, symport protein

Answer: B

53) __________ is the process where fatty acids are slowly disassembled into two-carbon units. A) Kreb's cycle B) Beta oxidation C) Oxidation phosphorylation D) Glycolysis E) Lipogenesis

Answer: B

54) The RAAS pathway begins with A) secretion of angiotensin converting enzyme. B) secretion of the enzyme renin. C) secretion of angiotensinogen. D) secretion of aldosterone. E) secretion of antidiuretic hormone.

Answer: B

54) estrogen in nonpregnant woman

Answer: B

55) For maximum efficiency in loading oxygen at the lungs, A) the pH should be slightly acidic. B) the temperature should be slightly lower than normal body temperature. C) the should be about 70 mm. D) DPG levels in the red blood cells should be high. E) all of the above

Answer: B

55) Measurements in a nephron reveal a glomerular hydraulic pressure of 69 mm Hg, and a fluid pressure in the Bowman's capsule of 15 mm Hg. Assuming that the plasma osmotic pressure is 30 mm Hg, and that essentially no plasma proteins are filtered by the glomerulus, what is the net glomerular filtration pressure in this case? A) -6 mm Hg B) 24 mm Hg C) 54 mm Hg D) 84 mm Hg E) 114 mm Hg

Answer: B

55) Secreted lysozyme kills bacteria.

Answer: B

55) The vitreous chamber of the eye A) contains the lens. B) helps to stabilize it and gives physical support to the retina. C) contains blood vessels that nourish the retina. D) is located between the lens and the iris. E) all of the above

Answer: B

56) A student in your lab volunteers to enter a hypoxic breathing chamber for 10 minutes, and his alveolar PO2 drops to 50 mm Hg. What other change would occur? A) decrease in arterial pH B) decrease in arterial PCO2 C) decrease in pH of cerebrospinal fluid D) increase in alveolar PCO2 E) hypoventilation

Answer: B

56) NK cells cause apoptosis of viral-infected cells.

Answer: B

56) Which of the following will increase flow in a vessel the most? A) Decrease length by 1 unit. B) Increase radius by 1 unit. C) Decrease viscosity by 1 unit. D) All of the above have the same effect on flow.

Answer: B

56) a stem-cell dysfunction that produces too many blood cells

Answer: B

56) blocks urination

Answer: B

57) The ciliary muscle helps to A) control the amount of light reaching the retina. B) control the shape of the lens. C) control the production of aqueous humor. D) move the eyeball. E) none of the above

Answer: B

58) A molecule that blocks the activity of carbonic anhydrase would A) interfere with oxygen binding to hemoglobin. B) cause an increase in blood pH. C) increase the amount of bicarbonate formed in the blood. D) decrease the amount of carbon dioxide dissolved in the plasma. E) all of the above

Answer: B

58) The process of forming mRNA is called A) replication. B) transcription. C) translation. D) ribolation. E) protein synthesis.

Answer: B

Match the cells/tissue with the hormone it secretes. A. Leydig cells B. granulosa cells C. corpus luteum D. placenta 52) progesterone, early in pregnancy

Answer: C

59) If the neural connections between the pons and medulla are severed, A) breathing will stop. B) pulmonary ventilation will decrease. C) the respiratory minute volume will increase. D) tidal volumes will decrease. E) alveolar ventilation will increase.

Answer: B

59) The term meaning programmed cell death is A) necrosis. B) apoptosis. C) oncogenesis. D) diuresis. E) cytocide.

Answer: B

6) Alveolar ventilation refers to the A) movement of air into and out of the lungs. B) movement of air into and out of the alveoli. C) movement of dissolved gases from the alveoli to the blood. D) movement of dissolved gases from the blood to the alveoli. E) utilization of oxygen by alveolar cells to support metabolism.

Answer: B

6) The precursor chemical for ACTH synthesis is A) CRH. B) POMC. C) beta-endorphin. D) MSH. E) TRH.

Answer: B

60) The QRS complex of an ECG corresponds to A) the depolarization of the atria. B) the progressive wave of ventricular depolarization. C) the repolarization of the ventricles. D) atrial repolarization. E) none of the above

Answer: B

60) What effect would a decrease in pH have on the amount of potassium ion in the urine? A) an increase in the amount of potassium in the urine B) a decrease in the amount of potassium in the urine C) no effect on the amount of potassium in the urine

Answer: B

61) All of the triplet codes needed to produce exactly one specific polypeptide chain are found in one A) chromosome. B) gene. C) codon. D) anticodon. E) none of the above

Answer: B

61) Groups of tissues that carry out related functions may form structures known as A) cells. B) organs. C) organelles. D) organisms. E) impossible to tell from the information given

Answer: B

62) The amount of light reaching the retina is controlled by the A) lens. B) iris. C) optic disk. D) ciliary muscle. E) more than one of the above

Answer: B

62) Triglycerides and steroids are members of this group. As a class they are hydrophobic.

Answer: B

63) If your vision is 20/20, this means that you can clearly A) see 20 point type at 20 feet. B) see objects at 20 feet that individuals with normal eyesight can see at 20 feet. C) see objects at 20 feet that individuals with eye problems see at 20 feet. D) see 20 letters on an eye chart from 20 feet. E) only see objects that are not more distant than 20 feet.

Answer: B

63) Translation occurs in/on the __________ of the cell. A) nucleus B) ribosome C) Golgi apparatus D) smooth endoplasmic reticulum E) plasma membrane

Answer: B

66) the spatial arrangement of amino acids; can be a helix or a pleated sheet

Answer: B

67) Modifies proteins and packages them into secretory vesicles for export from the cell.

Answer: B

69) Which of the following conditions would have the greatest effect on peripheral resistance? A) doubling the length of a vessel B) doubling the diameter of a vessel C) doubling the viscosity of the blood D) doubling the turbulence of the blood E) doubling the number of white cells in the blood

Answer: B

69) contributes to the blood-brain barrier

Answer: B

7) Fat stores are used for energy only after muscles have depleted their carbohydrate energy stores. A) true B) false

Answer: B

7) Most of the time, the parallel thick and thin filaments of the myofibrils are connected by __________ that span the space between the parallel thick and thin filaments. A) tropomyosin molecules B) crossbridges C) nebulin molecules D) sarcomeres E) calcium ions

Answer: B

7) The reflex that prevents a muscle from exerting too much tension is the A) stretch reflex. B) tendon reflex. C) flexor reflex. D) crossed extensor reflex. E) reciprocal reflex.

Answer: B

70) lipid carriers

Answer: B

72) dehydratase

Answer: B

72) pressure

Answer: B

73) A sample of tissue from an injury shows a large number of basophils. This would indicate that the tissue was A) abscessed. B) inflamed. C) being rejected. D) infected by viruses. E) infected by multicellular parasites.

Answer: B

74) In type 2 diabetes, insulin levels are frequently normal, yet the target cells are less sensitive to the effects of insulin. This suggests that the target cells A) are impermeable to insulin. B) may have a defect in their signal transduction pathway. C) cannot convert insulin to an active form. D) have adequate internal supplies of glucose. E) none of the above

Answer: B

75) Put these events in the correct chronological sequence: 1. End-plate potentials trigger action potentials. 2. Transverse tubules convey potentials into the interior of the cell. 3. Acetylcholine binds to receptors on the motor end plate. 4. Binding sites on actin are uncovered, allowing myosin to bind and carry out power strokes. 5. Ca2+ is released from the sarcoplasmic reticulum. 6. Chemically regulated ion channels open, causing depolarization. 7. Ca2+ ions bind to troponin-C, pulling on tropomyosin. A) 5, 3, 2, 1, 4, 7, 6 B) 3, 6, 1, 2, 5, 7, 4 C) 4, 1, 3, 7, 2, 6, 5 D) 2, 4, 7, 6, 3, 1, 5 E) 3, 6, 1, 5, 7, 2, 4

Answer: B

76) In an experimental situation focused on the development of a new vaccine, mice were injected with a viral antigen. Their plasma was removed in order to detect the presence of antibodies. This is an example of A) innate immunity. B) active immunization. C) passive immunization. D) natural immunity. E) autoimmunity.

Answer: B

77) Breathing that involves active inspiratory and expiratory movements is called A) eupnea. B) hyperpnea. C) diaphragmatic breathing. D) costal breathing. E) shallow breathing.

Answer: B

78) A decrease in which population of lymphocytes would impair all aspects of an immune response? A) cytotoxic T cells B) helper T cells C) suppressor T cells D) B lymphocytes E) plasma cells

Answer: B

78) Both insulin and glucagon are peptide hormones that target liver cells. The responses of the target cells to these two hormones are exactly the opposite of each other. This information implies that A) one hormone binds to a receptor on the cell membrane and the other to an intracellular receptor. B) each of the two hormones uses a different second messenger. C) both hormones interact with receptors at the cell nucleus. D) one of the hormones does not interact with a membrane receptor.

Answer: B

78) One of the major differences among skeletal muscle fiber types is in their resistance to fatigue. The fibers with the most endurance rely on __________ for energy. A) anaerobic glycolysis B) oxidative phosphorylation C) lactic acid accumulation D) ketone body degradation

Answer: B

8) A tactile receptor that is composed of dendritic processes that lie within a series of concentric connective tissue layers is a A) Ruffini corpuscle. B) Pacinian corpuscle. C) Meissner's corpuscle. D) Merkel disc. E) root hair plexus.

Answer: B

8) The medical term for heart attack is A) heart failure. B) myocardial infarction. C) heart murmur. D) fibrillation. E) heart block.

Answer: B

8) The partial pressure of carbon dioxide in the cells of peripheral tissues is approximately __________ mm Hg. A) 40 B) 45 C) 50 D) 70 E) 100

Answer: B

8) Which structure is NOT part of the blood circulation through the kidney? A) vasa recta B) loop of Henle C) glomerulus D) renal corpuscle

Answer: B

80) In the lungs, an example of the body's reserve capacity is that A) some areas of the lung can be closed off during rest and opened again when needed during exercise. B) capillary beds in the lungs are reversibly collapsible, allowing blood to be shunted to additional areas during exercise. C) pulmonary blood flow is completely under the control of the autonomic nervous system, dilating arteries and arterioles to adjust blood flow. D) All of the above are true. E) None of the above are true.

Answer: B

80) Which of the following is greater? A) blood pressure when the peripheral vessels dilate B) blood pressure when the peripheral vessels constrict

Answer: B

81) Put these neurons and myofibers in order of recruitment from first to last: 1. highest threshold neurons 2. lowest threshold neurons 3. higher threshold neurons 4. fatigue-resistant slow-twitch fibers 5. fatigue-resistant oxidative fast-twitch fibers 6. glycolytic fast-twitch fibers A) 2, 4, 6, 1, 3, 5 B) 2, 4, 3, 5, 1, 6 C) 3, 4, 1, 6, 2, 5 D) 1, 6, 3, 5, 2, 4 E) 1, 4, 3, 5, 2, 6

Answer: B

82) In a condition known as pleurisy, there is excess fluid in the pleural space. How would you expect this to affect the process of pulmonary ventilation? A) Ventilation would require less energy. B) Breathing would be labored and difficult. C) It would be easier to expand the lungs on inspiration. D) More air would be forced out during an expiration. E) Tidal volume would increase.

Answer: B

82) Neil accidentally ingests a substance that activates the complement cascade in the absence of bound antibodies. This substance would A) produce a fever. B) cause extensive cellular damage. C) increase circulation of lymph. D) activate blood clotting. E) prevent inflammation.

Answer: B

82) bringing glucose molecules inside brain cells so one can think

Answer: B

83) In a routine examination, some blood is taken and analyzed. The results show a high IgM level for the mumps antigen. This would indicate A) the person has just recovered from mumps. B) the person is just coming down with mumps. C) the person is allergic to mumps. D) the person is immune to mumps. E) none of the above

Answer: B

84) increased respiratory rate and/or volume without increased metabolism

Answer: B

84) lymphocytes

Answer: B

88) A boy who has not passed through puberty sustains an injury to his anterior pituitary such that FSH is no longer released, but LH is normal. After the individual grows to maturity, one would expect that he would A) not develop secondary sex characteristics. B) be sterile. C) be impotent. D) have impaired function of the interstitial cells. E) produce large amounts of inhibin.

Answer: B

89) Bronchioles and systemic arterioles dilate.

Answer: B

89) reacts to blood group antigens

Answer: B

9) The I band contains A) thick filaments. B) thin filaments. C) an area of overlapping filaments. D) all of the above E) none of the above

Answer: B

9) Which is NOT produced in the body for the purpose of storing extra calories? A) fat B) protein C) carbohydrate D) All of the above are produced in the body for that purpose.

Answer: B

92) During defecation, A) stretch receptors in the rectal wall activate sympathetic centers in the spinal cord. B) stretch receptors in the rectal wall activate parasympathetic centers in the spinal cord. C) stretch receptors in the rectal wall activate both sympathetic and parasympathetic centers in the spinal cord. D) the internal anal sphincter is consciously relaxed. E) the external anal sphincter contracts.

Answer: B

92) If adenylyl cyclase (the enzyme that converts ATP to cAMP) is blocked, A) endometrial cells would not respond to estradiol. B) the corpus luteum would not respond to hCG. C) the mammary glands would not respond to progesterone. D) the level of progesterone produced by the ovary during the luteal phase would increase. E) the activity of progesterone at the endometrium would increase.

Answer: B

93) Decreased levels of bile salts in the bile would interfere with digestion of A) protein. B) fat. C) carbohydrates. D) nucleic acids. E) vitamins and minerals.

Answer: B

96) In response to the hormone secretin, the pancreas secretes a fluid A) rich in enzymes. B) rich in bicarbonate. C) rich in bile. D) that contains only proteases. E) that contains only amylase.

Answer: B

18) The chemical bonding behavior of an atom is directly determined by A) the number of protons. B) the number of neutrons. C) the number and arrangement of electrons. D) the size of the atom. E) the mass of the atom.

Answer: C

99) Diarrhea often accompanies intestinal infections. Why? A) The immune system increases the production of enzymes for added protection. B) Bacterial toxins enhance the secretion of KCl from the cells, which pulls fluid into the intestine. C) Dehydration of the body will kill the bacteria causing the infection. D) Loss of electrolytes will kill bacteria.

Answer: B

A horse runs 10 races, each a mile long, during a 6-month period, and you are interested in determining if the horse's race time changes with experience. You set up a graph to display the race finish times of this horse. 33) What type of graph would be best to display the race finish times of this horse? A) bar graph B) line graph C) scatter plot

Answer: B

For the following questions, match the level of organization with its definition below. A. a collection of similar cells that carry out similar functions B. the smallest living unit C. a collection of different tissues that carry out related functions D. groups of organs functioning in a coordinated manner 21) cell

Answer: B

For the following questions, match the type of modulator with the best description below. A. involved in phosphorylation B. cannot be displaced by competition C. bind to proteins away from the active site D. reversible antagonist 72) irreversible antagonist

Answer: B

Human Physiology, 5e (Silverthorn) CHAPTER 15 Blood Flow and the Control of Blood Pressure 1) Fainting is also known as A) eclampsia. B) vasovagal syncope. C) infarction. D) reactive hyperemia. E) orthostatic hypotension.

Answer: B

Human Physiology, 5e (Silverthorn) CHAPTER 20 Integrative Physiology II: Fluid and Electrolyte Balance 1) The primary route for water loss from the body is the __________ system. A) respiratory B) urinary C) digestive D) integumentary E) cardiovascular

Answer: B

Human Physiology, 5e (Silverthorn) CHAPTER 22 Metabolism and Energy Balance 1) Adipocytes secrete the hormone A) ghrelin. B) leptin. C) neuropeptide Y. D) orexin. E) insulin.

Answer: B

Human Physiology, 5e (Silverthorn) CHAPTER 4 Energy and Cellular Metabolism 1) Energy is defined as A) doing tasks that make you tired. B) the capacity to do work. C) the light and heat from the sun. D) using glucose to synthesize ATP.

Answer: B

Identify each of the following as an element (A) or a compound (B). 52) CO2

Answer: B

In the reaction shown, identify the role of each of the participants. CO2 + H2O H2CO3 A. enzyme B. substrate(s) C. product(s) 84) CO2 + H2O

Answer: B

Match each bond type with its description. A. van der Waals B. ionic C. hydrogen D. covalent 55) This results when an atom has such a strong attraction for electrons that it pulls one or more electrons completely away from another atom.

Answer: B

Match each product with the cell or region that secretes or contains it. A. enzymes B. HCl C. HCO3- D. mucus E. more than one of these 57) parietal cells

Answer: B

Match each substance with its primary mode of transport across the kidney epithelium. A. transcytosis B. active transport C. symport with a cation D. passive reabsorption/diffusion 27) sodium

Answer: B

Match the bipotential structure with its gender-specific result. A. urethral folds and groove B. gonad cortex C. gonad medulla D. Wolffian duct E. Mullerian duct 42) ovary

Answer: B

Match the cell type with its description. A. lymphocytes B. neutrophils C. monocytes D. eosinophils E. basophils 49) These are the most abundant leukocyte; they are formed in the bone marrow.

Answer: B

Match the descriptions to the correct protein category. A. fibrous B. globular 67) soluble in water

Answer: B

Match the disease with the description. A. Addison's disease B. Cushing's disease C. Graves' disease D. Paget's disease 34) hypercortisolism

Answer: B

Match the factor with its effect on the affinity of hemoglobin for oxygen. A. increase B. decrease 26) increased temperature

Answer: B

Match the following terms with the correct descriptions. A. lymphocytes B. erythrocytes C. eosinophils D. platelets E. neutrophils 48) These cells contain hemoglobin and iron.

Answer: B

Match the ganglion cell response to the location of the light stimulus. A. fully excited B. fully inhibited C. weakly excited D. weakly inhibited 137) Light shines on center of off-center cell.

Answer: B

Match the lung disease to its description. A. fibrotic lung disease B. emphysema C. asthma D. pulmonary edema 20) destruction of alveoli

Answer: B

Match the lung volume with its description. A. tidal volume B. inspiratory reserve volume C. expiratory reserve volume D. residual volume 39) the additional air inhaled after a normal inspiration

Answer: B

Match the name of the wave with the correlated event. A. P wave B. QRS complex C. T wave D. PR segment E. ST segment 63) immediately followed by ventricular contraction

Answer: B

Match the ploidy with a cell stage. A. haploid B. diploid 47) germ cell

Answer: B

Match the type of receptor with the appropriate description. A. chemoreceptors B. touch receptors C. temperature receptors D. nociceptors 65) The most studied examples of this type are phasic receptors called Pacinian corpuscles.

Answer: B

On the row of cells indicated below, match A-E to these terms: Figure 3-1 128) apical surface

Answer: B

113) List the key components of bile. Is bile action similar to the action of lipases? Explain.

Answer: Bile salts, bile pigments, and cholesterol are the key components. Bile does not contain enzymes or have enzymatic action; rather bile emulsifies lipids so that lipases have sufficient access to their lipid substrates.

18) The myenteric plexus is A) a layer of circular smooth muscle. B) a layer of longitudinal smooth muscle. C) a network of neurons. D) the mucus secreting layer of the digestive tract. E) primarily composed of connective tissue.

Answer: C

148) Explain how these statements differ, and identify which one(s) is/are correct. Explain what is wrong with each incorrect statement. Explain when chemical reactions continue and when they stop. A. A chemical reaction proceeds until the enzyme molecules are all consumed. B. A chemical reaction proceeds until the substrate molecules are all consumed. C. A chemical reaction proceeds until equilibrium is established. D. A chemical reaction proceeds until all the product molecules are removed.

Answer: B and C are correct as written, but B requires some clarification. A is incorrect because enzyme molecules are not consumed in chemical reactions and will continue to perform until something external to the reaction causes it to stop. B is correct if the product is removed from the system, but if the product remains, some of it will break down into substrates; therefore they would not be consumed. In D, if product molecules are removed, the reaction would continue to proceed to form new product (assuming enzymes and substrates are still available). Chemical reactions continue if conditions are favorable, including presence of substrate and enzyme and fairly constant temperature and pH. In living systems, most chemical reactions do not have a steady supply of substrate; for example, the chemical reactions of digestion will stop when the small intestine is empty.

84) Vitamin __________ must be complexed with a protein called __________ to be absorbed from the small intestine.

Answer: B12, intrinsic factor 85) Defects in __________ structure or function lead to the disease cystic fibrosis. What ion does this channel transport? (:0)Answer: Cystic fibrosis transmembrane regulator (or CFTR chloride channel); it transports chloride. 86) What is the ENS, and when was it discovered? What is its significance, what is its nickname, and what are the characteristics that justify that nickname? (:0)Answer: ENS is the enteric nervous system, discovered over 100 years ago. The ENS controls reflexive peristalsis in the intestine, independent of the CNS. The ENS is nicknamed the "little brain" because it shares these characteristics with the actual brain: secretion of neurotransmitters and neuromodulators, and the presence of glial cells, a diffusion barrier, and integrating centers. 87) What are short reflexes? What types of behaviors do they regulate? What are long reflexes? Give examples of each. (:0)Answer: Reflexes controlled by the enteric nervous system are short reflexes. They regulate motility, secretion, and growth. Long reflexes are digestive reflexes that are integrated in the CNS. Long reflexes include feedforward reflexes and emotional reflexes. 88) List and briefly describe the action of peptides in the GI tract. (:0)Answer: Generally, GI peptides stimulate or inhibit motility and secretion. The peptides classified as hormones include gastrin, cholecystokinin, secretin, GIP, motilin, and glucagon-like peptide I. 89) List the six types of epithelial cells associated with gastric glands. Briefly describe each one's role in the stomach. (:0)Answer: Parietal cells secrete HCl and intrinsic factor; chief cells secrete pepsinogen; D cells secrete somatostatin; ECL cells secrete histamine; G cells secrete gastrin; mucous cells secrete mucus and bicarbonate. 90) Name two functions of cholecystokinin. (:0)Answer: Functions include stimulation of pancreatic enzyme secretion, bicarbonate secretion, gall bladder contraction, sense of satiety, and inhibition of gastric emptying. See Table 21-1 in the CHAPTER.

102) Calculate the body mass index of a woman who is 5'5" tall and weighs 180 lbs. Now calculate the BMI of a man with those dimensions. 1 kg = 2.2 lbs, 1 m = 39.24 in. Is either individual considered obese? Explain. Could BMI be misleading as a means of indicating obesity? Explain.

Answer: BMI is kg/m2. 1 kg/2.2 lb x 180 lb = 81.8 kg. 5 ft x 12 in/ft = 60 in. 1 m/39.24 in x 65 in = 1.66 m. 81.8 kg/(1.66m)2 = 30. BMI calculation is the same for both genders, but the classification for obesity is not. For men, a BMI greater than 25 is obese; for women a BMI greater than 30 is obese. Based on this, only the man is considered obese, but the woman is on the border. BMI does not take into account the amount of lean weight versus fat weight, so a very fit and muscular person could be classified as obese based on BMI but will not have obesity-related health problems.

88) Define basal metabolic rate (BMR) and describe the six factors affecting BMR.

Answer: BMR is an individual's lowest metabolic rate, which occurs when a person is sleeping. Factors affecting BMR are age and gender, amount of lean muscle mass, activity level, diet, hormones, and genetics.

91) Compare and contrast the nature of the pathogens in bacterial and viral infection.

Answer: Bacteria are independent cells with DNA that can survive and reproduce outside a host. Viruses are not cells, but rather are small pieces of DNA or RNA enclosed in a protein coat. Viruses can only survive and reproduce inside a host cell. Bacteria are susceptible to antibiotics. Viruses are treated with antiviral drugs. See Table 24-1 in the CHAPTER.

103) List and discuss three to five different methods of contraception, including at least one barrier method and one hormonal treatment.

Answer: Barrier methods include diaphragms, cervical caps, contraceptive sponges, and condoms. Barriers prevent the sperm from reaching the egg. Hormonal methods include pills containing estrogen and progesterone, which prevent ovulation.

94) Charlie has a blood test that shows a normal level of LDLs but an elevated level of HDLs in his blood. Since his family has a history of cardiovascular disease, he wonders if he should modify his lifestyle. What would you tell him?

Answer: Based just on the information given, Charlie would appear to be in good health, at least relative to his diet and exercise. Problems are associated with elevated levels of LDLs, which carry cholesterol to peripheral tissues and make it available for the formation of atherosclerotic plaques in blood vessels. High levels of HDLs indicate that a considerable amount of cholesterol is being removed from the peripheral tissues and carried to the liver for disposal. You should encourage Charlie not to change, and keep up the good work.

135) Your swimming buddy Mark jumped into a pool parallel to the water surface. When he stood up, he yelled "ouch" and you noticed that the skin on his chest and belly looked red and irritated. How would you describe the properties of water to explain to Mark why this happened? Why doesn't it hurt when pool water is penetrated perpendicular to the surface, as with a hands- first or feet-first dive?

Answer: Because Mark was parallel to the water surface, the force of his mass was spread out over a relatively large area of the water, making the force per unit water lower than in a typical dive. The surface tension of water, while not strong enough to keep Mark from penetrating the water surface, was strong enough to momentarily resist him. The force of the water pushing back on Mark, however briefly, was enough to cause pain. In a typical dive position, the force of Mark's entire mass is spread only over a tiny area of the water, and thus the force per unit water surface is greater. This higher force is sufficient to immediately break the hydrogen bonds and overcome the surface tension.

48) List and discuss the benefits of exercise, especially in regard to decreased risk of cardiovascular disease.

Answer: Benefits include decreased systemic blood pressure and resting heart rate, decreased plasma triglycerides, and increased plasma HDLs.

162) Why might a cell that lacks the ability to regulate its internal environment be described as a "ghost town"?

Answer: Building, maintaining, and removing worn-out parts is not possible without energy.

101) By binding to the Ca2+ channel proteins, drugs known as calcium channel blockers have two main effects. Name them. Why are some cells unaffected by these drugs?

Answer: Binding to calcium channels makes them less likely to open, effectively blocking the normal inward calcium current that would result from neural or endocrine stimulation. This causes vasodilation and a decrease in heart rate. Insensitive tissues have a different subtype of calcium channel.

160) Explain how vision is affected by the loss of one eye. Explain how hearing is affected by the loss of one ear. In reality, how disadvantaged are people who are blind in one eye or deaf in one ear? Would loss of half the tongue or half the olfactory epithelium have any comparable effects on those systems? Explain.

Answer: Binocular vision provides for accurate depth perception. In people who have lost vision in one eye, their prior experiences with relative sizes of familiar near and distant objects allows them to compensate for this loss and function relatively normally. Binaural hearing provides for accurate sound localization. People who have lost hearing in one ear are poorer at sound localization, but can cope well based on past experiences. Comparable losses of sensory tissues for taste or smell would have far less impact on those sensations because there is no similar central comparison of inputs from the two sides.

100) Explain how modern birth control pills work as contraceptives.

Answer: Birth control pills contain hormones such as estrogen that act through negative feedback to inhibit the release of GnRH, which in turn prevents release of FSH and LH. Thus primordial follicles do not develop and ovulation is prevented.

97) Kathy and Sally have been best friends since high school. Each is 5'5" tall and weighs about 160 lbs., which is considered to be overweight. Kathy's blood pressure is 125/82 and her blood analysis shows healthy levels of glucose and lipids. Sally's blood pressure is 135/87 and her blood analysis shows elevated glucose and triglycerides. Given that they are the same height and weight, would you expect their blood analyses to be more similar? Explain. Would you expect their body shapes to be similar? Explain. Which one is more likely to be diagnosed with metabolic syndrome? Explain.

Answer: Blood amounts of glucose and lipids are more related to the location than to the amount of extra fat, so height and weight alone do not predict blood analysis results. Sally is more likely to have upper-body fat (apple shape) and Kathy is more likely to have lower-body fat (pear shape). Sally is at greater risk for metabolic syndrome, and appears to have it, based on her weight, BP, and blood glucose and lipids.

83) Describe the endocrine regulation of bone growth.

Answer: Bone growth is regulated by many hormones, including growth hormone, insulin-like growth factor, sex steroids, and hormones that regulate calcium metabolism.

112) What do virus-infected cells and cancer cells have in common, related to the type of immune response and its long-term effectiveness?

Answer: Both are treated as altered self cells by the immune system, and both can undergo frequent mutation and thus can fail to be eliminated by the immune system.

103) Compare and contrast the roles of cytotoxic T cells and NK cells.

Answer: Both are types of lymphocytes, and both are more effective against viruses than bacteria. Cytotoxic T cells bind to and destroy specific virus-infected self cells and sometimes cancer cells, as part of the immune system. NK cells are part of the innate defenses, secreting antiviral cytokines and attacking host cells that lack MHC-I complexes as a result of a viral infection or cancer.

92) Compare and contrast the effectiveness of the immune response to bacterial and viral infection.

Answer: Both pathogens are foreign and thus have antigens, therefore they each stimulate both branches of the immune response. The humoral response is more effective at eliminating bacterial infections than viral infections because bacteria typically do not have self antigens. The cell-mediated immune response is more effective at eliminating viral infections than bacterial, because viruses infect and therefore alter self cells, and it is altered self cells that stimulate cytotoxic T cells.

174) Explain what is meant by the terms respiratory pump and skeletal muscle pump. Why are these important to the cardiovascular system?

Answer: Both refer to pressure exerted on the blood in veins that assist in venous return. During the pressure changes in the thoracic cavity associated with normal breathing, pressure in the thoracic portion of the inferior vena cava fluctuates, resulting in blood being pulled upward in the vena cava during each inhalation. Contractile activity of normal skeletal muscle momentarily squeezes the veins within and near the muscles, also helping to propel the blood. These and other slight pressure changes are essential because the overall pressure gradient of the venous system is low.

64) Describe the general rules for the identity of neurotransmitters secreted by pre- and postganglionic neurons in the autonomic division. Be sure to specify the types of receptors, where relevant, and describe the exceptions.

Answer: Both sympathetic and parasympathetic neurons secrete acetylcholine onto nicotinic receptors within the autonomic ganglion. Postganglionic sympathetic neurons secrete norepinephrine onto adrenergic receptors. Postganglionic parasympathetic neurons secrete acetylcholine onto muscarinic receptors. Exceptions include sympathetic postganglionic neurons that secrete acetylcholine, and nonadrenergic, noncholinergic neurons that secrete substance P, somatostatin, vasoactive intestinal peptide, adenosine, nitric oxide, or ATP.

82) Compare and contrast carbon monoxide (CO) poisoning, in which CO displaces oxygen from hemoglobin, and metabolic poisoning such as by cyanide.

Answer: Both types of poisoning interfere with oxygen-dependent metabolism and thus can be fatal. CO prevents oxygen loading at the lungs and thus produces hypoxia. Metabolic poisons exert their effects on the chemical reactions that consume oxygen to produce ATP, and thus have no effect on PO2.

162) Explain brain plasticity, and give an advantage and a disadvantage of this process.

Answer: Brain plasticity is the reorganization of synapses according to how much different parts of the brain are used. An advantage is that people who lose a sensation, such as vision, may develop greater sensation in another area, such as the fingertips of a blind person who uses Braille to read. A disadvantage is that the process is imperfect, and sensations may persist from a body part that is no longer there, such as phantom pain from an amputated limb.

37) The hormone that is the basis for a pregnancy test is A) LH. B) progesterone. C) human chorionic gonadotropin (hCG). D) human placental lactogen (hPL). E) relaxin.

Answer: C

85) __________ refers to the mass movement of fluid between the blood and the interstitial fluid as the result of __________ or __________ pressure gradients. If the direction of bulk flow is into the capillary, the fluid movement is called __________. If the direction of flow is out of the capillary, the fluid movement is known as __________.

Answer: Bulk flow, hydrostatic, osmotic; absorption; filtration

73) In a laboratory experiment, adding curare, which binds to acetylcholine receptors, to the solution around a muscle decreases the size of the end plate potential. Adding prostigmine, an acetylcholinesterase blocker, increases the size of the end plate potential. Explain why.

Answer: By binding to acetylcholine receptors, curare prevents the binding of acetylcholine. Curare does not open the receptor channels. An end plate potential results from the opening of several ACh receptor channels. Thus, if the receptor is blocked, any end plate potential that is produced would be smaller. Prostigmine blocks the enzyme that stops ACh action by degrading the neurotransmitter. Interfering with this enzyme prolongs the action of ACh, thus more receptor channels open and the end plate potential is larger.

10) Homeostasis is the ability of the body to A) prevent the external environment from changing. B) prevent the internal environment from changing. C) quickly restore changed conditions to normal. D) ignore external stimuli to remain in a state of rest. E) prevent excessive blood loss.

Answer: C

10) Most of the carbon dioxide in the blood is transported as A) solute dissolved in the plasma. B) carbaminohemoglobin. C) bicarbonate ions. D) solute dissolved in the cytoplasm of red blood cells. E) carbonic acid.

Answer: C

10) Rugae, plicae, and villi are all features that accomplish A) secretion of enzymes. B) secretion of hormones. C) an increase in surface area. D) immune function. E) mechanical digestion.

Answer: C

10) The Bowman's capsule and glomerulus make up the A) renal pyramid. B) loop of Henle. C) renal corpuscle. D) renal papilla. E) collecting system.

Answer: C

10) The C cells of the thyroid gland produce A) thyroxine. B) TSH. C) calcitonin. D) parathyroid hormone. E) all of the above

Answer: C

10) The lungs are enclosed in __________ membranes. A) pericardial B) pulmonary C) pleural D) thoracic E) costal

Answer: C

10) The nutrients that yield the most energy per gram when metabolized are A) carbohydrates. B) proteins. C) fats. D) nucleic acids. E) vitamins.

Answer: C

10) __________ are clusters of lymphatic nodules located beneath the epithelial lining of the small intestine. A) Tonsils B) Adenoids C) Peyer's patches D) Immune complexes E) Lymph nodes

Answer: C

103) A flagellum moves a cell through a fluid medium. What moves the fluid medium across the surface of cells that are not free to move? A) centrioles B) thick filaments C) cilia D) ribosomes E) endoplasmic reticulum

Answer: C

103) The term polar is used to describe molecules because A) polar covalent molecules are found in colder climates. B) polar covalent molecules were first discovered in polar bears. C) there are at least two distinct ends of the molecule regarding electron position. D) there are at least two distinct ends of the molecule regarding hydrogen placement. E) such molecules are always linear in shape.

Answer: C

11) Diet-induced thermogenesis is highest after ingestion of A) spicy foods, like jalapeno peppers. B) fats. C) proteins. D) carbohydrates.

Answer: C

11) Which of the following is NOT considered an essential element for a living organism? A) carbon B) hydrogen C) mercury D) oxygen E) nitrogen

Answer: C

11) __________ regulates the growth and maturation of megakaryocytes. A) Erythropoietin B) Interleukin C) Thrombopoietin D) Colony-stimulating factor E) none of the above

Answer: C

111) A fatty acid that contains three double bonds in its carbon chain is said to be A) saturated. B) monounsaturated. C) polyunsaturated. D) hydrogenated. E) carboxylated.

Answer: C

111) The distinguishing characteristic of connective tissue is A) that it is arranged in sheets of tissue that lie on body surfaces. B) always dividing, constantly being replaced throughout the body. C) the presence of extensive extracellular matrix containing widely scattered cells. D) the collagen fibers that offer support. E) All of these characteristics help make connective tissue unique.

Answer: C

112) Most of the fat found in the human body is in the form of A) steroids. B) phospholipids. C) triglycerides. D) prostaglandins. E) monoglycerides.

Answer: C

112) Which type of connective tissue does NOT fit with the typical characteristic regarding relative cell density? A) cartilage B) bone C) adipose D) blood E) dense

Answer: C

114) Examination of a tissue sample reveals groups of cells united by junctional complexes and interlocking membranes. The cells have one exposed surface and lack blood vessels between them. The tissue is most likely A) muscle tissue. B) neural tissue. C) epithelial tissue. D) connective tissue. E) adipose tissue.

Answer: C

114) Information about sound must travel through each of these areas of the brain. Put them in the correct order. 1. thalamus 2. medulla 3. auditory cortex of cerebrum 4. midbrain A) 1, 2, 3, 4 B) 2, 1, 4, 3 C) 2, 4, 1, 3 D) 3, 2, 1, 4 E) 3, 4, 1, 2

Answer: C

115) Place these structures in the order that blood returning to the heart from the body would pass through them. 1. right ventricle 2. left atrium 3. right atrium 4. pulmonary artery 5. left ventricle 6. pulmonary vein A) 4, 2, 5, 6, 3, 1 B) 2, 5, 6, 4, 3, 1 C) 3, 1, 4, 6, 2, 5 D) 1, 3, 6, 4, 5, 2 E) 3, 2, 4, 6, 1, 5

Answer: C

116) Mature nerve and muscle cells are expected to lack which organelle? A) nucleus B) endoplasmic reticulum C) centrioles D) ribosomes E) Golgi bodies

Answer: C

18) Redistribution of blood flow during exercise results from A) vasodilation. B) vasoconstriction. C) vasodilation in skeletal muscle and vasoconstriction in viscera. D) vasoconstriction in skeletal muscle and vasodilation in viscera.

Answer: C

116) Movement of the cupula in the ampullae of the semicircular canals A) produces sound. B) allows us to hear sounds. C) stimulates hair cells alerting us to rotational movements. D) stimulates hair cells alerting us to a change in body position with respect to gravity. E) allows us to perceive linear acceleration.

Answer: C

117) Damage to the cupula of the lateral semicircular canal would interfere with our perception of A) the direction of gravitational pull. B) linear acceleration. C) horizontal rotation of the head. D) vertical rotation of the head. E) angular rotation of the head.

Answer: C

118) A viral infection involving the vestibular nuclei may result in A) loss of hearing. B) loss of sight. C) a sense of dizziness. D) local paralysis. E) high blood pressure.

Answer: C

118) Abnormally slow conduction through the ventricles would change the __________ in an ECG tracing. A) P wave B) T wave C) QRS complex D) PR interval E) RT interval

Answer: C

12) An immunoliposome is a liposome that A) suppresses the immune system. B) stimulates the immune system. C) can recognize cancer cells. D) none of the above

Answer: C

12) EPOC is A) excess physiological oxygen contribution. B) extra phosphorylated oxygenated creatine. C) excess postexercise oxygen consumption. D) exercising physiological oxygen consumption. E) extraneous preoxygen contribution.

Answer: C

12) How genetics influences the body's response to drugs is called A) pharmacokinetics. B) pharmacogenetics. C) pharmacogenomics. D) pharmacodynamics. E) pharmageddon.

Answer: C

12) Meiosis of one cell results in production of A) four gametes. B) one gamete. C) four gametes if male, one if female. D) four gametes if female, one if male.

Answer: C

12) Pain from one area of the body, such as in cardiac ischemia, can be felt in another area, such as the neck and left shoulder; this is called A) analgesia. B) adaptation. C) referred pain. D) tonic reception. E) latency.

Answer: C

12) Red blood cell production is regulated by the hormone A) thymosin. B) angiotensin I. C) erythropoietin. D) M-CSF. E) cobalamin.

Answer: C

12) The effects of thyroid hormone on peripheral tissue include A) decreased oxygen consumption. B) decreased heart rate. C) increased sensitivity to sympathetic stimulation. D) increased activity of osteoclasts. E) all of the above

Answer: C

12) Which area(s) of the brain exert(s) control over the autonomic nervous system? 1. cerebrum 2. cerebellum 3. hypothalamus 4. pons 5. medulla 6. thalamus A) 1, 2, 3, 4, 5, 6 B) 1, 3, 5 C) 1, 3, 4, 5 D) 1, 2, 3, 5 E) 2, 3, 4, 5

Answer: C

120) If the sodium pump in the membranes of the photoreceptors were blocked, A) we would constantly see in color. B) our visual acuity would increase. C) blindness would result. D) we would only be able to see in dim light. E) the cells would not be able to produce neurotransmitter molecules.

Answer: C

121) If a person's cornea is flatter than normal, this person is A) presbyopic. B) myopic. C) hyperopic. D) hypopic. E) astigmatic.

Answer: C

124) Enzymes are often useful as diagnostic tools. How? A) Enzymes destroy pathogenic bacteria so blood levels of bacteria decline. B) Enzymes destroy damaged cells so X-rays reveal smaller body organs. C) Damaged cells release enzymes into the blood so that plasma levels of the enzyme are noticeably elevated. D) The liver releases enzymes that accumulate in the urine. E) None of the above offers a rational explanation.

Answer: C

124) Put these phases of the cardiac cycle in the correct order. 1. opening of the semilunar valves 2. isovolumic contraction 3. beginning of atrial systole 4. closure of the AV valves 5. completion of ventricular filling 6. beginning of ventricular systole 7. ventricular relaxation 8. ventricular ejection A) 4, 5, 1, 2, 7, 8, 3, 6 B) 3, 2, 6, 1, 4, 5, 8, 7 C) 3, 5, 6, 4, 2, 1, 8, 7 D) 3, 5, 6, 1, 8, 4, 2, 7 E) 3, 2, 6, 4, 5, 8, 7, 1

Answer: C

124) Why do objects viewed underwater while scuba diving or snorkeling appear closer and larger than they really are? A) The water magnifies the image because it is denser than the lens of the eye. B) Light moves more slowly through the water, causing images to appear larger and closer. C) The dive mask refracts light before it reaches the cornea and the extra refraction causes the image to appear closer and larger. D) The cornea does not function in water and the light is not refracted as much, causing the image to appear larger and closer. E) Light waves are scattered by the water before reaching the eye, causing the image to appear larger and closer.

Answer: C

125) As a result of the long refractory period, cardiac muscle cannot exhibit A) tonus. B) treppe. C) tetany. D) recruitment. E) fatigue.

Answer: C

127) This tissue is composed of cells with membrane extensions that beat in a coordinated fashion to move fluid and particles across the tissue.

Answer: C

129) The net energy yield for the anaerobic metabolism of one glucose molecule is A) two molecules of ATP and two NADH. B) two molecules of NADH and 0 ATP. C) two molecules of ATP and 0 NADH. D) 36 molecules of ATP and 0 NADH. E) two molecules of ATP and three NADH converted to ATP later.

Answer: C

13) Aromatase A) is an enzyme that inactivates sexual signals after they are smelled. B) makes one more sensitive to the smell of others. C) is an enzyme that converts testosterone to estradiol. D) enhances one's response to all signals.

Answer: C

13) The portion of the nephron that attaches to the collecting duct is the A) loop of Henle. B) proximal tubule. C) distal tubule. D) collecting duct. E) minor calyx.

Answer: C

13) The tension generated in a muscle fiber is directly proportional to the A) number of thick and thin filaments present. B) number of nebulin and titin molecules present. C) number of crossbridges formed. D) change in length of the thick filaments. E) change in length of the thin filaments.

Answer: C

18) Sensations of burning or aching pain A) are carried by type A fibers. B) reach the CNS quickly. C) cause a generalized activation of the reticular formation and the thalamus. D) are quite localized. E) all of the above

Answer: C

132) Oxidative phosphorylation occurs __________; the beneficial end product is __________. A) in the cytoplasm just inside the plasma membrane, H2O B) in the cytoplasm just inside the plasma membrane, ATP C) in the mitochondria due to the electron transport system, ATP D) in the mitochondria due to the electron transport system, CO2 E) twice during the citric acid cycle, ATP

Answer: C

133) Manganese ions block the calcium channels in the cardiac muscle membrane. How would the presence of manganese in the extracellular fluid affect the contraction of the heart muscle? A) The plateau phase of contraction would be longer. B) The refractory period would be shorter. C) The heart would beat less forcefully. D) The heart rate would increase. E) The contraction phase would be prolonged.

Answer: C

134) If there is a blockage between the AV node and the AV bundle, how will this affect the appearance of the electrocardiogram? A) The PR interval will be smaller. B) The QRS interval will be longer. C) There will be more P waves than QRS complexes. D) There will be more QRS complexes than P waves. E) The T wave will disappear.

Answer: C

135) What is the result of alternative splicing? A) production of new genes B) replication of the DNA molecule C) production of similar yet distinct proteins from a single gene D) genetic mutations E) conversion of a protein product into an isomer

Answer: C

138) Transcription of the DNA triplet sequence ATC summons a tRNA bearing A) UAG. B) TAG. C) AUC. D) ATC.

Answer: C

139) The blood pressure in a vessel is 20 units at point A and 10 units at point B. One minute later, the pressure is 15 units at point A and five units at point B. Flow between those points is __________.

Answer: C

14) Blood pressure is determined by A) measuring the size of the pulse pressure. B) measuring the pressure in the left ventricle. C) measuring the force exerted by blood in a vessel. D) measuring the degree of turbulence in a closed vessel. E) all of the above

Answer: C

14) G cells of the stomach secrete A) cholecystokinin. B) secretin. C) gastrin. D) enterokinase. E) pepsin.

Answer: C

14) In normal adults, red blood cells are formed in A) the liver. B) the spleen. C) red bone marrow. D) yellow bone marrow. E) lymph nodes.

Answer: C

14) In the medulla oblongata, the nucleus tractus solitarius contains the __________ of neurons. A) pontine respiratory group B) ventral respiratory group C) dorsal respiratory group D) pre-Botzinger complex

Answer: C

14) The activity of an enzyme is changed by environmental factors such as temperature or pH; these factors are referred to as A) specificity. B) common bond. C) modulators. D) isozyme. E) product.

Answer: C

14) The hormone ADH A) is secreted by the anterior pituitary gland in response to changes in blood osmolarity. B) stimulates the kidneys to retain sodium ion. C) stimulates water conservation at the kidneys. D) causes the kidneys to produce a large volume of urine. E) all of the above

Answer: C

14) The sac around the heart is the A) peritoneum. B) pleural sac. C) pericardium. D) myocardium. E) epicardium.

Answer: C

14) When a muscle cell contracts, A) the myosin heads produce a single power stroke. B) the position of each myosin head is identical at any given moment. C) some myosin heads form crossbridges as others release them. D) the intracellular calcium concentration decreases.

Answer: C

14) Which class of movement can be considered a combination of the other two? A) reflex B) voluntary C) rhythmic

Answer: C

14) With endurance training, muscle mitochondria increase in A) size. B) number. C) A and B D) none of the above

Answer: C

142) When a blood vessel dilates, blood viscosity is __________.

Answer: C

144) A 5 M solution of 100 mL of glucose contains how many grams of glucose, m.w. 180 daltons? A) 180 B) 360 C) 90 D) 6.02 × 1023 E) 1.0

Answer: C

146) How many grams of NaCl, m.w. 58.5 daltons, are the molar equivalent to 90 g of glucose (m.w. 180 daltons)? A) 0.25 B) 0.5 C) 29.25 D) 117 E) 14.6

Answer: C

147) How many grams of NaCl, m.w. 58.5 daltons, are necessary to make 1 liter of 5% saline? A) 58.5 B) 1 C) 50 D) 6.02 × 1023 E) 2.9

Answer: C

15) A placebo is A) any drug being tested in a clinical trial. B) any drug in a class of drugs commonly used as pain relievers. C) a drug or treatment that is expected to have no pharmacological effect. D) a nutritive and respiratory organ in fetal development. E) a hole in a cavity wall through which an organ protrudes.

Answer: C

15) The difference between the systolic and diastolic pressures is called the A) systemic pressure. B) mean arterial pressure. C) pulse pressure. D) blood pressure. E) circulatory pressure.

Answer: C

15) The process of filtration in the kidney is most accurately described as A) highly specific. B) completely nonspecific. C) relatively nonspecific.

Answer: C

16) "Dual innervation" refers to an organ receiving A) two nerves from the spinal cord. B) both autonomic and somatomotor nerves. C) both sympathetic and parasympathetic nerves. D) nerves from both the brain and the spinal cord. E) none of the above

Answer: C

16) An antioxidant is A) the active ingredient in bleach. B) a compound that absorbs oxygen. C) a molecule that alters free radicals. D) a free radical. E) a vitamin.

Answer: C

16) The vessels that are the main site of variable resistance in the circulatory system, and that contribute more than 60% of the total resistance, are the A) muscular arteries. B) elastic arteries. C) arterioles. D) venules. E) veins.

Answer: C

17) A scientifically logical guess is a A) model. B) theory. C) hypothesis. D) law. E) variable.

Answer: C

17) In normal kidneys, blood cells and plasma proteins are A) filtered then reabsorbed. B) secreted then reabsorbed. C) not filtered. D) filtered and secreted.

Answer: C

18) The power stroke begins when A) the actin is released by the myosin head. B) the ATP is hydrolyzed by the myosin head. C) the inorganic phosphate and ADP are released from the myosin. D) the actin is attached to the myosin head. E) A and B

Answer: C

18) The term myogenic indicates that the heart muscle is the source of A) the contractile force for pumping. B) a hormone that indirectly regulates blood volume. C) the electrical signal that triggers heart contraction. D) receptors that trigger blood pressure reflexes. E) stem cells that repair damaged heart tissue.

Answer: C

19) C-reactive protein is a type of A) cytokine. B) chemotaxin. C) acute-phase protein. D) pyrogen. E) granzyme.

Answer: C

19) The average life span of a red blood cell is A) 1 week. B) 1 month. C) 4 months. D) 6 months. E) 1 year.

Answer: C

2) Autoimmune diseases are an example of a/an __________ response by the immune system. A) lack of B) overactive C) incorrect D) none of the above

Answer: C

2) Identify the FALSE statement. A) Most of the special senses use receptors that are NOT neurons. B) Sensory transduction results in conversion of stimulus energy into a change in membrane potential. C) Each type of sensory receptor responds to only one form of energy. D) The receptive field of a sensory receptor may overlap with the field of another receptor. E) A person is not necessarily aware of a threshold stimulus to a visual receptor.

Answer: C

2) The most accurate definition of artery is a vessel that A) carries highly oxygenated blood. B) contains smooth muscle in its wall. C) transports blood away from the heart. D) transports blood toward the heart. E) contains internal valves.

Answer: C

20) An enzyme that transfers chemical groups among substrates is A) hydrolase. B) lipase. C) lyase. D) ligase. E) kinase.

Answer: C

20) Tay-Sachs is a disease caused by having ineffective A) mitochondria. B) Golgi bodies. C) lysosomes. D) rough endoplasmic reticulum. E) smooth endoplasmic reticulum.

Answer: C

20) The feeding and satiety centers are located in the A) medulla oblongata. B) pons. C) hypothalamus. D) cerebrum. E) cerebellum.

Answer: C

20) The function of red blood cells is to A) remove carbon dioxide from the lungs. B) remove nitrogenous wastes from active tissues. C) carry oxygen from the lungs to the body's cells. D) carry nutrients from the digestive system to the body's cells. E) defend the body against infectious organisms.

Answer: C

20) The matching of blood flow to the changing metabolic needs of a tissue is due to A) neural control. B) hormonal control. C) local control.

Answer: C

21) Excitation-contraction coupling refers to A) acetylcholine triggering the opening ion channels. B) the arrival of acetylcholine at the neuromuscular junction due to its exocytosis from the axon terminal. C) the chemical and electrical events that trigger the mechanical events in a muscle fiber. D) the enzymatic removal of acetylcholine from the synapse, which can then allow relaxation to occur. E) B and D

Answer: C

21) In quiet breathing, A) inspiration and expiration involve muscular contractions. B) inspiration is passive and expiration involves muscular contractions. C) inspiration involves muscular contractions and expiration is passive. D) inspiration and expiration are both passive processes. E) none of the above

Answer: C

21) The __________ are double sheets of peritoneal membrane that hold some of the visceral organs in their proper position. A) serosa B) adventitia C) mesenteries D) fibrosa E) lamina propria

Answer: C

21) The rapid depolarization phase of the action potentials of myocardial contractile cells is due to which ion(s)? A) Ca2+ B) K+ C) Na+ D) A and B E) A and C

Answer: C

22) Boyle's law states that gas volume is A) directly proportional to pressure. B) directly proportional to temperature. C) inversely proportional to pressure. D) inversely proportional to temperature. E) none of the above

Answer: C

22) Each of the following paracrines may cause vasodilation EXCEPT A) nitric oxide. B) H+ ions from metabolic acids. C) Ca2+. D) K+. E) CO2.

Answer: C

22) The porphyrin ring of heme contains an atom of A) magnesium. B) calcium. C) iron. D) sodium. E) copper.

Answer: C

22) The role of the pituitary hormone FSH in males is to A) stimulate the interstitial cells to produce testosterone. B) stimulate the Sertoli cells to produce inhibin. C) initiate sperm production in the testes. D) develop and maintain secondary sex characteristics. E) influence sexual behaviors and sex drive.

Answer: C

22) integrated in the spinal cord, often modulated by the brain

Answer: C

23) The contraction cycle is triggered by the rise in __________ released from the SR. A) Na+ B) K+ C) Ca2+ D) A and B E) A, B, and C

Answer: C

25) Benefits of regular exercise include all of the following, EXCEPT A) lowered blood pressure. B) increased plasma levels of HDL (high-density lipoproteins). C) increased levels of LDL (low-density lipoproteins). D) decreased risk of heart attack. E) decreased plasma levels of triglycerides.

Answer: C

25) Insulin secretion A) is stimulated by sympathetic neurons. B) decreases in response to elevated amino acid concentrations. C) is stimulated by parasympathetic neurons. D) is inhibited by GLP-1. E) none of the above

Answer: C

25) The period of time during which the sarcomeres return to their resting length is referred to as A) the refractory period. B) the latent period. C) the relaxation phase. D) the depolarization period. E) the repolarization period.

Answer: C

26) Inside the ganglia are interneurons, which modulate messages.

Answer: C

26) The surge in LH that occurs during the middle of the ovarian cycle triggers A) follicle maturation. B) menstruation. C) ovulation. D) menopause. E) atresia.

Answer: C

26) have afferent neurons that bring information to the CNS

Answer: C

27) Several blood-borne chemicals affect the lumen size of arterioles. Which are NOT paired correctly? A) kinins - vasodilation B) histamine - vasodilation C) serotonin - vasodilation D) sumatriptan - vasoconstriction E) epinephrine - vasoconstriction with alpha receptors

Answer: C

27) The importance of the plateau phase of the action potential of myocardial cells is in A) preventing overstretching of the cells. B) enhancing the efficiency of oxygen use by the cells. C) preventing tetanus. D) preventing fibrillation. E) regulating Ca2+ availability to the cells.

Answer: C

27) Vanilloid receptors A) are found on the taste buds. B) are also called transient receptor potential vanilloids (TRPV). C) respond to damaging heat as well as to capsaicin. D) A and B E) all of the above

Answer: C

27) Which of the following is NOT a molecule synthesized in the smooth endoplasmic reticulum (SER)? A) fatty acids B) steroids C) proteins D) lipids E) All of the above are synthesized in the SER.

Answer: C

28) Muscle fatigue arises primarily from failure in A) neuromuscular transmission. B) initiation by the motor cortex. C) excitation-contraction cycle. D) signal relay from brain to spinal cord. E) calcium pumps.

Answer: C

28) The cells responsible for cell-mediated immunity are the A) B lymphocytes. B) plasma cells. C) T lymphocytes. D) suppressor T lymphocytes.

Answer: C

28) The principal hormone secreted by the corpus luteum is A) LH. B) FSH. C) progesterone. D) estradiol. E) estrogen.

Answer: C

28) When an enzyme is working as fast as it can because its active site is continually refilled with substrate, the condition is referred to as A) allosteric modulation. B) equilibrium. C) saturation. D) the Ricardo-Mertz phenomenon. E) specificity.

Answer: C

28) When the diaphragm and external intercostal muscles contract, A) expiration occurs. B) intrapulmonary pressure increases. C) intrapleural pressure decreases. D) the volume of the lungs decreases. E) all of the above

Answer: C

28) You conduct an experiment on twenty 18-year-old male subjects to see how various intensities of exercise influence heart rate. Which of the following is/are considered an independent variable? A) age of subjects B) sex of subjects C) intensity of exercise D) heart rate E) more than one of these

Answer: C

29) Nutrient absorption occurs primarily in the A) stomach. B) liver. C) small intestine. D) large intestine. E) A and C

Answer: C

29) Sinusoids are modified vessels that replace __________ in some tissues. A) arteries B) veins C) capillaries

Answer: C

3) The process by which dissolved gases are exchanged between the blood and interstitial fluids is A) pulmonary ventilation. B) external respiration. C) diffusion. D) cellular respiration. E) breathing.

Answer: C

30) Contains cholinergic neurons.

Answer: C

31) An inflammatory response is triggered when A) red blood cells release pyrogens. B) T lymphocytes release interferon. C) mast cells release granules containing histamine and heparin. D) neutrophils phagocytize bacteria. E) blood flow to an area increases.

Answer: C

31) If a cell lacked lysosomes, it would not be able to A) synthesize lipids. B) produce enzymes. C) digest cellular wastes and bacteria. D) destroy H2O2. E) transport water-soluble molecules.

Answer: C

32) Homeothermic refers to A) natural treatments or remedies. B) shivering. C) regulating body temperature within a narrow range. D) warm relationships with others of the same gender.

Answer: C

32) Restoring lost fluid from the capillaries back to the circulatory system is one of the major functions of the __________ system. A) urinary B) thirst quenching C) lymphatic D) immune E) digestive

Answer: C

32) Which substance is responsible for activating the digestive enzymes inside lysosomes? A) air B) water C) acid D) enzymes E) base

Answer: C

33) Cells that help to regulate the immune response are A) B lymphocytes. B) plasma cells. C) helper T cells. D) cytotoxic T cells. E) NK cells.

Answer: C

33) Functions of the large intestine include A) chemical digestion of chyme. B) temporary food storage. C) resorption of water and compaction of feces. D) absorption of the products of digestion. E) all of the above

Answer: C

35) During the cephalic phase of gastric secretion, A) the stomach responds to distention. B) secretin inhibits parietal and chief cells. C) there is an increased flow of action potentials along the vagus nerve to the stomach. D) the intestine reflexively inhibits gastric emptying. E) production of gastric juice slows down.

Answer: C

35) The embryo forms from the A) placenta. B) morula. C) inner cell mass of the blastocyst. D) outer layer of the blastocyst. E) zona pellucida.

Answer: C

35) The integrating center for neural control of blood pressure resides in the A) cerebrum. B) cerebellum. C) medulla oblongata. D) pons variolli. E) hypothalamus.

Answer: C

36) When the heart is in fibrillation, A) the myocardial cells may become damaged from contracting too fast. B) the myocardial cells deplete their oxygen supply because they are contracting too fast, and the lactic acid produced damages the myocardial cells. C) effective pumping of the ventricles ceases because the myocardial cells fail to work as a team, and the brain cannot get adequate oxygen. D) the myocardial cells are contracting together as they should; fibrillation indicates a normal sinus rhythm of 75 beats per minute. E) there is no contraction of the myocardium.

Answer: C

37) Protein specificity is A) the activation of a specific protein that is needed to perform a particular function. B) the degree to which a protein is attracted to a ligand. C) the ability of a protein to bind a certain ligand or a group of related ligands. D) the degree to which a protein-ligand complex initiates a response. E) B and C

Answer: C

37) The reactions of glycolysis occur in the cell's __________; the reactions of the citric acid cycle occur in the __________. A) cytoplasm, cytoplasm B) mitochondria, mitochondria C) cytoplasm, mitochondria D) mitochondria, cytoplasm

Answer: C

37) The senses of equilibrium and hearing are provided by receptors of the A) outer ear. B) middle ear. C) inner ear. D) bony labyrinth. E) perilymph.

Answer: C

38) Arrange the following events in protein secretion in the proper sequence. 1. The polypeptide chain enters the lumen of the endoplasmic reticulum. 2. A secretory vesicle is formed. 3. A transport vesicle is formed. 4. The polypeptide chain enters the lumen of the Golgi complex. A) 1, 2, 3, 4 B) 1, 3, 2, 4 C) 1, 3, 4, 2 D) 4, 3, 1, 2 E) 3, 1, 4, 2

Answer: C

38) Histamine's primary role in the respiratory system is as a A) surfactant. B) bronchodilator. C) bronchoconstrictor. D) vasodilator. E) vasoconstrictor.

Answer: C

38) Immunoglobulins that attach to mast cells and are involved in allergic reactions are A) IgA. B) IgD. C) IgE. D) IgG. E) IgM.

Answer: C

38) Most products of fat digestion are absorbed by A) capillaries. B) veins. C) lymphatic vessels. D) arterioles.

Answer: C

38) The extrinsic pathway of coagulation is activated by the A) sticking of platelets to damaged tissue. B) activation of a proenzyme exposed to collagen. C) release of tissue factor by a damaged endothelium. D) release of heparin from the liver. E) conversion of prothrombin to thrombin.

Answer: C

38) The receptors of equilibrium and hearing are the A) utricles. B) saccules. C) hair cells. D) supporting cells. E) ampullae.

Answer: C

4) The following is a list of several levels of organization that make up the human body. 1. tissue 2. cell 3. organ 4. molecule 5. organism 6. organ system The correct order from the smallest to the largest is A) 2, 4, 1, 3, 6, 5. B) 4, 2, 1, 6, 3, 5. C) 4, 2, 1, 3, 6, 5. D) 4, 2, 3, 1, 6, 5. E) 6, 4, 5, 2, 3, 1.

Answer: C

4) The inner lining of blood vessels is called A) endocardium. B) endoangium. C) endothelium. D) basal lamina. E) endostatin.

Answer: C

4) Urine is produced by the A) gall bladder. B) urinary bladder. C) kidney. D) ureter. E) urethra.

Answer: C

40) A contraction that generates force and moves a load is known as __________, whereas one that generates force without movement is known as __________. A) isotropic, isometric B) isometric, eccentric C) isotonic, isometric D) isometric, isotonic E) isotonic, eccentric

Answer: C

40) For each NADH molecule that moves through the electron transport system, what is the potential yield of ATP molecules? A) 1 B) 2 C) 2.5 D) 3.5 E) 4

Answer: C

40) The hormone primarily responsible for milk synthesis is A) progesterone. B) oxytocin. C) prolactin. D) estrogen. E) growth hormone.

Answer: C

41) The components of muscle that allow force generation without a change in length are the A) series collagenous components. B) parallel collagenous components. C) series elastic elements. D) overlapping actin filaments with myosin crossbridges. E) none of the above

Answer: C

41) The process of fibrinolysis A) activates fibrinogen. B) draws torn edges of damaged tissue closer together. C) dissolves clots. D) forms emboli. E) forms thrombi.

Answer: C

41) the extra amount actively (forcibly) exhaled after a normal exhalation

Answer: C

42) After a sunburn, you can peel off your dead skin in relatively large sheets, because of the presence of this type of junction. A) gap B) tight C) anchoring

Answer: C

42) Dehydration causes A) fluid to shift from the ECF to the ICF. B) decreased secretion of ADH. C) increased thirst. D) increased levels of aldosterone. E) all of the above

Answer: C

42) If a person is bedridden for several days, the baroreceptor reflex may fail upon standing. Why? A) Blood pools in the feet and legs. B) There is reduced blood flow to the brain. C) The kidneys have reduced the blood volume. D) A and B E) A, B, and C

Answer: C

42) The specificity of an antibody is determined by A) the fixed segment. B) the antigenic determinants. C) the variable region. D) the size of the antibody. E) the antibody class.

Answer: C

43) The type of junction protein important in growth and development are A) claudins. B) occludins. C) CAMs. D) connexins. E) integrins.

Answer: C

43) The volume of blood ejected from each ventricle during a contraction is called the A) end-diastolic volume. B) end-systolic volume. C) stroke volume. D) cardiac output. E) cardiac reserve.

Answer: C

43) testis

Answer: C

44) Each of the following is a primary tissue type except one. Identify the exception. A) muscle tissue B) neural tissue C) osseous tissue D) connective tissue E) epithelial tissue

Answer: C

44) Smooth muscle cells lack which protein(s)? A) actin B) myosin C) troponin D) tropomyosin E) C and D

Answer: C

44) This particle has a charge of -1 and a negligible mass.

Answer: C

45) During ventricular systole, A) the atria are contracting. B) blood is entering the ventricles. C) the AV valves are closed. D) the pressure in the ventricles declines. E) the ventricles are relaxed.

Answer: C

45) Sound waves are transmitted directly into the inner ear by movements of the A) auditory ossicles. B) cochlea. C) oval window. D) round window. E) tympanic membrane.

Answer: C

45) essential to the process of blood clotting

Answer: C

46) According to Starling's law of the heart, the cardiac output is directly related to the A) size of the ventricle. B) heart rate. C) venous return. D) thickness of the myocardium. E) end-systolic volume.

Answer: C

46) Epinephrine and norepinephrine that are released from the adrenal glands affect target tissue for a longer period of time than the same substances released from neurons at their peripheral receptors. Why? A) The adrenal gland releases larger amounts of the neurotransmitters than the neurons. B) The hormones released from the adrenal glands bind to different receptors than those released from neurons. C) There are no enzymes to break down epinephrine and norepinephrine in the blood and very little in peripheral tissues. D) The effectors are less sensitive to epinephrine and norepinephrine released by the adrenal glands. E) The epinephrine and norepinephrine from the adrenal glands are released by sympathetic neurons, whereas parasympathetic neurons release these substances at the effector organs.

Answer: C

46) Epithelial cells that are adapted for membrane transport of materials, such as ions and nutrients, usually have __________ on their apical surface. A) mitochondria B) cilia or flagella C) microvilli D) junctional complexes E) vesicles

Answer: C

46) The frequency of a sound is indicated to the nervous system by the A) frequency of stereocilia vibration. B) number of rows of hair cells that are stimulated. C) region of the inner ear that is stimulated. D) movement of the perilymph in the cochlear duct. E) frequency of vibration of the tectorial membrane.

Answer: C

46) mental retardation associated with infantile hypothyroidism

Answer: C

47) A harmful substance that the liver produces when metabolizing fatty acids is A) ammonia. B) urea. C) ketone bodies. D) rancid fatty acids. E) beta units.

Answer: C

47) Information about the position of arms, legs, and peripheral joints of the body in space comes from A) maculae. B) cristae. C) proprioceptors. D) semicircular canals. E) the vestibular apparatus.

Answer: C

48) Damage to the renal medulla would interfere first with the functioning of the A) Bowman's capsule. B) distal tubule. C) collecting ducts. D) proximal tubule. E) glomerulus.

Answer: C

48) Inappropriate or excessive immune responses to antigens are A) immunodeficiency diseases. B) autoimmune diseases. C) allergies. D) the result of stress. E) common in the elderly.

Answer: C

48) The liver and skeletal muscles store glucose as __________ for a ready energy source. A) lipids B) glucose C) glycogen D) urea E) ketone bodies

Answer: C

49) The term used to describe the amount of blood in the ventricle available to be pumped out of the heart during the next contraction is A) cardiac output (CO). B) heart rate (HR). C) end-diastolic volume (EDV). D) stroke volume (SV). E) end-systolic volume (ESV).

Answer: C

5) "Glucose is transported from blood into cells because cells require glucose to meet their energy needs." This type of explanation is A) mechanistic. B) theological. C) teleological. D) metalogical. E) scatological.

Answer: C

5) The larger the receptive field, A) the larger the stimulus needed to stimulate a sensory receptor. B) the more sensory receptors it includes. C) the harder it is to discriminate the exact point of stimulation. D) the larger the area of the somatosensory cortex in the brain that deals with the area. E) all of the above

Answer: C

5) The lung pathology most likely to result from certain kinds of heart disease is A) emphysema. B) asthma. C) pulmonary edema. D) fibrotic lung disease. E) lung cancer.

Answer: C

5) The watery medium that surrounds a cell is known as A) cytosol. B) protoplasm. C) extracellular fluid. D) cytoplasm. E) plasma.

Answer: C

5) Urine is carried to the urinary bladder by A) blood vessels. B) lymphatics. C) the ureters. D) the urethra. E) all of the above

Answer: C

5) Which is NOT true regarding autonomic reflexes? A) Vomiting, sneezing, and coughing are all examples of autonomic reflexes. B) Integrating centers for autonomic reflexes in the brain include the hypothalamus, brain stem, and limbic system. C) Autonomic reflexes are all monosynaptic, with their synapse in the central nervous system. D) Many autonomic reflexes are characterized by tonic activity, a continuous stream of action potentials.

Answer: C

51) Gluconeogenesis refers to A) converting glucose to glycogen storage. B) removing glucose from storage as glycogen. C) creating glucose from glycerol, amino acids, or lactate. D) converting glucose to storage as adipose tissue. E) removing fatty acids from adipose storage.

Answer: C

51) Precursors of macrophages, these cells are relatively rare in blood because they are actually in transit to some other site.

Answer: C

51) Urea is A) actively secreted in the distal tubule. B) actively reabsorbed in the proximal tubule. C) passively reabsorbed in the proximal tubule. D) completely eliminated in the urine. E) actively transported into the filtrate by the cells of the collecting duct.

Answer: C

52) The function of microvilli, often seen on the apical membrane of transporting epithelia, is to A) increase the movement of extracellular fluid. B) increase the resistance of the cell to viruses. C) increase the cell's surface area. D) increase the toughness of the cell. E) allow the cell to move through a fluid medium.

Answer: C

53) At an intercalated disc A) the cell membranes of two cardiac muscle fibers are completely separated by a synapse. B) the myofibrils are loosely attached to the membrane of the disc. C) two cardiac muscle cells are connected by gap junctions. D) t-tubules unite the membranes of the adjoining cells. E) all of the above

Answer: C

54) One substance has no membrane transporters to move it but can diffuse freely through open leak channels if there is a concentration gradient. Initially, this substance's concentrations in the filtrate and extracellular fluid are equal. Later, however, the active transport of Na+ and other solutes creates a gradient by removing water from the lumen of the tubule where it is located. What substance is this? A) glucose B) calcium C) urea D) A and B E) A, B, and C

Answer: C

Match from the following list. A. A band B. I band C. Z disk D. H zone E. M line 53) the structures that serve as the attachment site for the thin filaments and mark the boundaries for one sarcomere

Answer: C

54) Our perception of the pull of gravity and linear acceleration is the result of A) changes in the pressure exerted by the cupula on hair cells. B) vibrations of the tectorial membrane striking hair cells. C) the pressure exerted by the otolithic organ on hair cells of the maculae. D) the movement of the otolithic organ within the ampullae of the semicircular canals. E) none of the above

Answer: C

56) Cells that store fat are called A) fibroblasts. B) liposomes. C) adipocytes. D) mast cells. E) melanocytes.

Answer: C

56) The central opening in the eye through which light passes is the A) conjunctiva. B) cornea. C) pupil. D) lens. E) fovea.

Answer: C

56) These are weak attractive forces between hydrogen and certain other atoms.

Answer: C

57) Caused by a dietary deficiency, this condition results in red blood cells that are small and pale.

Answer: C

57) When the pH of the extracellular fluid declines, A) the kidneys excrete more sodium ions. B) the kidneys excrete more bicarbonate ions. C) the kidneys reabsorb more potassium ions. D) the pH of the urine increases. E) the kidneys reabsorb less water.

Answer: C

58) An area of the retina that contains only cones and is the site of sharpest vision is the A) outer segment. B) inner segment. C) fovea. D) optic disc. E) tapetum lucidum.

Answer: C

6) It takes approximately __________ of aerobic exercise to begin mobilizing fats from adipose tissue. A) 30 seconds B) 10 minutes C) 30 minutes D) 45 minutes E) 60 minutes

Answer: C

6) Most body water is located A) in plasma. B) in interstitial fluid. C) inside cells. D) in lumens of organs open to the outside.

Answer: C

6) The sensory fibers of the muscle spindle organs synapse onto A) Golgi tendon organs. B) gamma motor neurons. C) alpha motor neurons. D) joint receptors. E) spinal interneurons.

Answer: C

6) Which area is NOT normally considered to be an autonomic control center? A) pons B) medulla C) amygdala D) hypothalamus

Answer: C

6) Which artery/arteries branch(es) most proximal to the beginning of the aorta at the heart? A) hepatic B) renal C) coronary D) carotid E) pulmonary

Answer: C

60) A blind spot in the retina occurs where A) the fovea is located. B) ganglion cells synapse with bipolar cells. C) photoreceptors are absent, as the optic nerve leaves the eye. D) rod cells are clustered to form the macula. E) amacrine cells are located.

Answer: C

61) Composed of units called amino acids, these can be linked into chains over 100 peptides long.

Answer: C

65) the three-dimensional shape of an amino acid chain; can be fibrous or globular

Answer: C

66) The digestive system of a cell, degrading and/or recycling bacterial or organic components.

Answer: C

66) These receptors slowly adapt, but only in a safe range; their receptive field is about 1 mm in diameter.

Answer: C

67) Fenestrated capillaries are present in A) skeletal muscles. B) cardiac muscle. C) the liver. D) the spleen. E) skin.

Answer: C

68) Premature closure of the epiphyseal plates could be caused by A) high levels of vitamin D3. B) too much calcium in the diet. C) elevated levels of sex hormones. D) too little thyroxine. E) an excess of growth hormone.

Answer: C

68) Protein sorting ensures that newly synthesized proteins arrive at the organelles where they are needed. To help this process, proteins are often assembled directly into the A) Golgi apparatus. B) lysosome. C) endoplasmic reticulum. D) nucleus. E) extracellular fluid.

Answer: C

70) A drug that blocks the action of lipoprotein lipase would A) decrease the emulsifying action of bile. B) interfere with fat digestion. C) raise blood levels of LDLs. D) increase the synthesis of cholesterol by the liver. E) provide more fatty acids for adipocytes.

Answer: C

70) Each of the following factors would increase peripheral resistance except one. Identify the exception. A) increased sympathetic stimulation B) elevated levels of epinephrine C) vasodilation D) irregularities in the vessel walls caused by plaques E) factors that cause increased hematocrit

Answer: C

71) If a person has a blood pressure of 120/80, her mean arterial pressure would be A) 200 mm Hg. B) 100 mm Hg. C) 93 mm Hg. D) 80 mm Hg. E) 40 mm Hg.

Answer: C

71) When blood levels of glucose, amino acids, and insulin are high, and glycogenesis is occurring in the liver, the body is in the A) fasting state. B) postabsorptive state. C) absorptive state. D) stress state. E) bulimic state.

Answer: C

71) the amount of blood in the ventricle available to be pumped out of the heart during one contraction

Answer: C

72) The lymphatic system A) has heart-like pumps called lymph nodes. B) stores blood when circulatory demand is low. C) empties the lymph vessels into the veins near the clavicles. D) can be removed without health consequences. E) all of the above

Answer: C

73) fibroblast

Answer: C

74) flutter

Answer: C

74) kinase

Answer: C

75) allosteric modulator

Answer: C

75) has three cuplike leaflets and has the aorta on one side

Answer: C

76) After death, when metabolism stops, in which step of the contractile cycle will skeletal muscles remain stuck, for most of the first day or so? A) It depends on what part of the contractile cycle they were in at the time of death. B) the power stoke phase C) the rigor state D) a weak binding state E) none of the above

Answer: C

57) Name the components of a reflex pathway, and describe generally how a reflex works.

Answer: See Figs. 13-1 and 13-2 in the CHAPTER.

76) Edema is likely to occur when A) the concentration of protein in the blood increases. B) hemorrhage occurs. C) the heart becomes an insufficient pump. D) blood hydraulic pressure at the capillary decreases. E) the blood hydraulic pressure in a capillary is equal to the blood osmotic pressure.

Answer: C

8) The periodic table of the elements is directly organized according to A) atomic diameter. B) atomic mass. C) number of protons. D) number of neutrons. E) number of electrons.

Answer: C

8) The presence of two peripheral efferent neurons in a pathway is typical of the __________ division. A) somatic B) sensory C) autonomic D) A and B E) A and C

Answer: C

8) When a body is dehydrated, water in the urinary bladder A) can be returned to the circulation directly. B) can be returned to the circulation after moving back into the kidneys. C) will still be expelled from the body in the urine.

Answer: C

80) Milly has just received a kidney transplant and is taking cyclosporin A, a drug that suppresses cytotoxic T cells. What does this medication do? A) prevents inflammation from destroying the transplanted kidney B) depresses hematopoiesis C) prevents rejection D) increases the number of antibodies in the blood E) none of the above

Answer: C

81) Joe is playing in an intramural football game when he is tackled so hard that he breaks a rib. He can actually feel a piece of the rib sticking through the skin, and he is having a difficult time breathing. Joe probably is suffering from A) a collapsed trachea. B) an obstruction in the bronchi. C) a pneumothorax. D) decreased surfactant production. E) a bruised diaphragm.

Answer: C

81) Leslie has a bad sore throat and the lymph nodes in her neck are swollen. This indicates A) the focus of the infection is the lymph nodes. B) lymph is not flowing through these lymph nodes. C) the affected lymph nodes contain an increased number of lymphocytes. D) the lymph gland has increased its secretion of thymosin.

Answer: C

83) a thoroughbred horse, or greyhound dog, running around the track

Answer: C

86) rapid breathing

Answer: C

9) The two-point discrimination test A) is used to determine clarity of vision. B) provides information about olfactory receptors. C) provides a detailed map for sensory receptors. D) is used to test for hearing disorders. E) monitors the activity of taste buds.

Answer: C

9) Which of the following statements about colony-stimulating factors (CSFs) is FALSE? A) CSFs are required to induce both cell division and cell maturation. B) CSFs regulate leukopoiesis. C) CSFs are made by epithelial cells. D) B and C

Answer: C

9) Which statement is NOT true? A) The blood supply to the kidneys comes through the renal arteries. B) As much as one fourth of the cardiac output may flow to the kidneys at any given moment. C) The kidneys do not reabsorb filtered waste products. D) The urinary bladder is filled by two ducts, called ureters, and emptied by the single urethra. E) Women are more likely to develop urinary tract infections than men.

Answer: C

93) Sally is an avid jogger and she trains incessantly. She has slimmed down so that she is now underweight for her height and has very little fat tissue. You would expect Sally to A) have heavy menstrual flows. B) double ovulate. C) be amenorrheic. D) have painful menstrual cramps. E) show elevated levels of FSH.

Answer: C

95) A blockage of the ducts from the parotid glands would A) result in the production of more viscous saliva. B) impair the lubricating properties of saliva. C) interfere with carbohydrate digestion in the mouth. D) eliminate the sense of taste. E) all of the above

Answer: C

98) Digestion of a carbohydrate results in large amounts of the monosaccharides glucose and galactose. This carbohydrate was probably A) sucrose. B) maltose. C) lactose. D) cellulose. E) glycogen.

Answer: C

An alien retrieved alive from a crashed UFO has had a thorough medical examination. A hormone with a distinct but similar structure to epinephrine has been isolated, and named ufo-epi. 47) If humans administered a physiological dose of ufo-epi responded to the chemical, which of the following would indicate ufo-epi is an epinephrine agonist? A) constriction of respiratory tubes B) hyperglycemia (high blood glucose) C) increase in fatty acids in the blood D) localized sweating E) decreased heart rate

Answer: C

Human Physiology, 5e (Silverthorn) CHAPTER 16 Blood 1) The total volume of blood in the body of a 70 kg man is approximately __________ liters. A) 25-30 B) 10-15 C) 5-6 D) 2-4 E) 1-2

Answer: C

Human Physiology, 5e (Silverthorn) CHAPTER 19 The Kidneys 1) Functions of the kidneys include all but one of the following. Identify the exception. A) regulation of extracellular fluid volume B) maintenance of ion balance in body fluids C) regulation of blood protein levels D) regulation of blood osmolarity E) homeostatic regulation of blood pH

Answer: C

Human Physiology, 5e (Silverthorn) CHAPTER 2 Molecular Interactions 1) The location of electrons in an atom is A) constant, within the nucleus. B) constant, within an energy shell. C) variable, within an energy shell. D) variable, within the nucleus. E) C and D

Answer: C

Human Physiology, 5e (Silverthorn) CHAPTER 21 The Digestive System 1) Mechanical digestion refers to A) the progressive dehydration of indigestible residue. B) the input of food into the digestive tract. C) the enzymatic breakdown of food. D) the absorption of nutrients in the gut. E) the effects of chewing, churning, and segmentation.

Answer: C

Human Physiology, 5e (Silverthorn) CHAPTER 26 Reproduction and Development 1) Gametes are produced by A) internal genitalia. B) external genitalia. C) gonads. D) autosomes. E) sex chromosomes.

Answer: C

Human Physiology, 5e (Silverthorn) CHAPTER 3 Compartmentation: Cells and Tissues 1) Since the widespread adoption of the Pap test, deaths from cervical cancer have dropped A) 25%. B) 50%. C) 70%. D) 95%.

Answer: C

Match each term to its definition. A. intermediate B. anabolism C. biochemical pathway D. key intermediate E. catabolism 87) a series of interconnected chemical reactions

Answer: C

Match the following structures with the appropriate description. A. appendix B. colon C. duodenum D. ileum E. jejunum 43) location of most peptic ulcers

Answer: C

Match the immunoglobulin to its description. A. IgA B. IgM C. IgG D. IgD E. IgE 85) crosses the placenta

Answer: C

Match the symbol with the correct element: A. P B. Na C. Ca D. C E. K F. Pb 46) calcium

Answer: C

There are 10 cloned horses, born on the same day, with identical chromosomes. They are each subjected to the same physical training regimen, but given daily injections of different concentrations of a particular vitamin. They all run the same race. You set up a graph to explore a relationship between race finish time and vitamin dose. 35) Which type of graph is best to explore a relationship between race finish time and vitamin dose? A) bar graph B) line graph C) scatter plot

Answer: C

65) Describe the two distinct endocrine cell types that comprise the thyroid gland.

Answer: C cells secrete calcitonin, and follicle cells secrete thyroid hormones. 66) Damage to cells of the zona fasciculata of the adrenal cortex would result in A) the disappearance of axillary and pubic hair. B) increased volume of urine formation. C) decreased levels of sodium ion in the blood. D) decreased ability to convert lipids to glucose. E) increased water retention. (:0)Answer: D

81) Hemoglobin binds to and has a much higher affinity for carbon monoxide (CO) than for oxygen. CO is colorless and odorless and can be produced in homes heated by natural gas; victims usually do not realize they are being poisoned and often die in their sleep. Describe the likely changes in a CO poisoning victim.

Answer: CO will gradually displace O2 on hemoglobin molecules. While this increases O2 unloading in tissues, it will significantly decrease O2 loading in lungs, therefore hypoxia will result. Victims gradually lose consciousness as the brain tissues become hypoxic.

156) CO2 can combine with water to form two different products, carbonic acid or glucose. Write the summary equation for each reaction; don't worry about balancing.

Answer: CO2 + H2O . C6H12O6 CO2 + H2O . H2CO3

69) Write the chemical equation catalyzed by the enzyme carbonic anhydrase. Suppose the concentration of H+ is increased by an outside force in a solution that had been at equilibrium. According to the law of mass action, what must happen to the CO2 concentration to reestablish equilibrium after this disturbance? What must happen to the concentration of bicarbonate?

Answer: CO2 + H2O . H2CO3 . H+ + HCO3- Carbon dioxide concentration will increase as the reaction is shifted to the left. Bicarbonate concentration will decrease.

72) Devising new food recipes is more successful when the chefs understand basic food chemistry. For example, to create a new cake recipe, one must understand the role of both acids and bases in leavening (rising, or trapping of tiny gas bubbles as dough is baking, to provide the characteristic texture). The principle is to incorporate baking soda (NaHCO3) into an acidic batter, causing a chemical reaction resulting in gas production. Looking at the chemical formula, what gas do you think is formed? Summarize the chemical reaction that occurs in a cake batter that contains NaHCO3 and vinegar or acetic acid (produces acetate when ionized: CH3COO- + H+). What would happen to your cake if you didn't use enough baking soda? What would happen if you forgot to acidify your batter with vinegar or a similar food acid?

Answer: CO2 gas is formed. NaHCO3 + CH3COO- + H+ . Na+ + CH3COO- + CO2 + H2O. Less gas would be produced, so the cake wouldn't rise normally. The chemical reaction would not occur, so the cake wouldn't rise.

104) The interaction between Ca2+ and calmodulin in smooth muscle fibers has been described as a second messenger system. Develop an argument to support this position, using another second messenger system you have studied for comparison.

Answer: Ca2+ forms a complex with calmodulin. This complex activates myosin light chain kinase, which in turn activates myosin ATPase via phosphorylation, triggering the contraction cycle. The second messengers cGMP and cAMP also phosphorylate proteins to affect cell activity.

92) What effect does a calcium channel blocker have on the heart, the blood pressure, and cardiac output? Why?

Answer: Calcium channel blockers inhibit the opening of calcium channels and ensuing calcium current. Vascular smooth muscle dilates, and heart rate decreases. Vasodilation will result in a decrease in systemic blood pressure, and the decrease in heart rate will decrease cardiac output.

162) Explain how Ca2+ levels inside myocardial cells are altered.

Answer: Calcium enters the cell through voltage-gated calcium channels in the cell membrane. Calcium ions then open ryanodine receptor channels in the sarcoplasmic reticulum, resulting in calcium-induced calcium release. Calcium ions are transported back into the SR by a calcium ATPase, or removed from the cell by a sodium-calcium antiport protein.

154) Nervous tissue consists of two general types of cells: nerve cells and glial cells. Glial cells are mitotic, whereas neural cells are not (excluding the small population of stem cells). Which type of cell is most likely to be involved in brain cancer, and why?

Answer: Cancer results from uncontrolled cell division in mitotic cells. Therefore, glial cell cancers (gliomas) are the most common type of nervous system cancers, as they have the most mitotic activity.

146) Cancer is not one disease but a group of related diseases, caused by abnormal genes, environmental factors, and/or viral infections. Describe the basic common characteristics of cancers, including the role of anchoring junctions and proteases. What is a tumor? What is metastasis? Is cancer usually considered to be contagious? Explain.

Answer: Cancer results when cells begin to divide and grow and do not respond to normal controls that would limit their growth. Anchoring junctions eventually fail to keep the cancer cells together, allowing them to spread to other organs where they continue to grow uncontrolled. Some cancer cells secrete proteases that improve their ability to spread or metastasize. A tumor is a lump of cancerous cells. Cancer is usually not contagious, being caused instead by genetic mutations in an individual or exposure to environmental factors by that individual; exceptions are cancers caused by viruses, which are believed to be the minority.

121) The technique known as __________ uses computers to noninvasively visualize sections through the body that can be used to reconstruct the three-dimensional structure of specific organs.

Answer: Computerized tomography

163) Candy bars are well known to supply large amounts of energy for metabolism. With all this energy trapped inside, how can the candy bar sit still on the shelf in the grocery store?

Answer: Candy bars contain large amounts of energy, but it is trapped as potential energy in carbohydrates (both simple and complex). The carbohydrates must be broken down and enter the cell's metabolic pathways in order to convert the potential energy to another form.

133) Cholesterol is absorbed without being digested into smaller pieces. How does this compare to absorption of carbohydrates and proteins? What characteristic of cholesterol suggests it would be transported by simple diffusion? What is the evidence that transport proteins are involved in cholesterol absorption? Does this discovery rule out transport by simple diffusion? Explain.

Answer: Carbohydrates are digested to monosaccharides before they are absorbed. Proteins are absorbed as amino acids, di- or tripeptides, or even oligopeptides. As a lipid, cholesterol can dissolve in the phospholipid bilayer and therefore cross membranes by simple diffusion. The drug ezetimibe inhibits cholesterol absorption, suggesting there must be a membrane transporter. This discovery does not rule out additional transport of cholesterol by simple diffusion, but does suggest there is a significant amount of transport dependent upon a transport molecule.

45) The enzyme that catalyzes the conversion of H2O and CO2 to H2CO3 is called __________.

Answer: Carbonic anhydrase 46) Removal of excess water in urine is known as __________. (:0)Answer: Diuresis 47) The high __________ osmolarity allows urine to be concentrated. (:0)Answer: medullary interstitial 48) AQP2 water pores are added to the cell membrane by __________ and withdrawn by __________ in a process known as __________. (:0)Answer: Exocytosis, endocytosis, membrane recycling 49) The anatomical arrangement of the kidney that allows transfer of solutes from one blood vessel to another is called the __________. (:0)Answer: Countercurrent exchange system 50) Paracrine feedback from the __________ in the distal tubule to the granular cells stimulates release of __________. (:0)Answer: macula densa, renin 51) The __________ cells of the distal nephron are interspersed among the principal cells and contribute to acid-base regulation. (:0)Answer: intercalated 52) How do kidneys alter urine concentration? (:0)Answer: Kidneys alter urine concentration by varying the amounts of water and sodium reabsorbed in the distal nephron. 53) A rise in angiotensin II levels would result in A) elevated blood pressure. B) increased retention of sodium ions at the kidney. C) increased water retention. D) increased blood volume. E) all of the above (:0)Answer: E

85) The period of time from the beginning of one heartbeat to the beginning of the next is termed the __________.

Answer: Cardiac cycle 86) The period when blood is pushed into the arteries is called __________. (:0)Answer: ventricular ejection 87) Narrowing of the opening of a heart valve is referred to as __________. (:0)Answer: stenosis 88) Listening to the heart through the chest wall is called __________. (:0)Answer:A uscultation 89) The rule that states "Within limits, the heart pumps all of the blood that returns to it" is known as the __________. (:0)Answer: Frank-Starling Law of the Heart 90) The __________ circuit carries blood to and from the alveoli of the lungs. (:0)Answer: pulmonary 91) The __________ circuit carries blood to and from all parts of the body except the alveoli of the lungs. (:0)Answer: systemic 92) __________ are blood vessels that carry blood away from the heart. (:0)Answer:A rteries 93) __________ are blood vessels that carry blood toward the heart. (:0)Answer: Veins 94) The heart is enclosed in the __________. (:0)Answer: pericardial membrane 95) The superior portion of the heart where the major blood vessels enter and exit is the __________. (:0)Answer: Base

163) Discuss the attachments between adjacent cardiac muscle cells. What features are important anatomically and physiologically? Is there a disadvantage to this arrangement?

Answer: Cardiac muscle cells are joined by structures called intercalated disks. Abutting cells both have desmosomes, to tightly hold the cells together to withstand the physical force of contraction, and gap junctions, which act as tiny tunnels for ions to cross between cells. In this way, action potentials flow across cardiac muscle cells without disruption, as if the cells were one. One disadvantage of gap junctions is that they can be shut down, promoting fibrillation, which occurs when the cardiac muscle cells contract independently. Rather than producing useful pumping, the heart is only quivering as the teamwork of these millions of cells ceases.

58) Plasma consists of blood minus __________ and __________.

Answer: Cells, platelets 59) Active bone marrow is red because of the presence of __________ whereas inactive marrow is __________ because of an abundance of adipocytes. (:0)Answer: hemoglobin, yellow 60) Blood is approximately __________% plasma by volume. (:0)Answer: 55 61) Hormones called __________ are involved in regulation of white blood cell populations. (:0)Answer: Colony-stimulating factors 62) In healthy adults, blood cell production occurs only in the __________. (:0)Answer: red bone marrow 63) The term used for a cytokine that is released by one white blood cell that acts on another is _________. (:0)Answer: interleukin 64) The growth and maturation of megakaryocytes is regulated by the glycoprotein __________. (:0)Answer: thrombopoietin (TPO) 65) The hormone that regulates the production of red blood cells is __________. (:0)Answer: Erythropoietin 66) The carrier protein that transports absorbed iron through the blood is __________. (:0)Answer: transferrin

93) Distinguish between cellular and external respiration.

Answer: Cellular respiration refers to the intracellular reactions that consume oxygen and produce ATP. External respiration refers to the exchange of gases between the environment and the lungs, lungs and blood, blood and cells, and the transport of those gases.

86) Factors playing a role in fatigue can be classified into __________ and __________ mechanisms. Define each mechanism and give an example for each mechanism.

Answer: Central fatigue, peripheral fatigue. Central fatigue arises in the central nervous system. Peripheral fatigue arises anywhere between the neuromuscular junction and the contractile elements of the muscle. Examples of central fatigue: subjective feeling of tiredness and a desire to cease activity. Example of peripheral fatigue is depletion of muscle glycogen stores. See Fig. 12-10 in the CHAPTER.

147) Identify the virus associated with some types of cervical cancer. How is this virus acquired? Is it accurate to consider cervical cancer to be contagious? Explain.

Answer: Cervical cancer can be caused by the human papilloma virus, or HPV. This virus is typically acquired from sexual contact with an infected person. Given that the virus is contagious, some consider cervical cancer to be a sexually transmitted cancer.

119) Draw a flow chart illustrating the development of bipotential reproductive structures in an XY individual and an XX individual, indicating the stimuli for development or regression of the structures. Add to this chart the changes that occur at puberty and their stimuli. Finally, indicate the points on the chart at which gender anomalies associated with the following conditions occur: A. testicular feminization or androgen insensitivity (XY genotype, permanent female external morphology with internal testes and androgen production but faulty androgen receptors); B. male pseudohermaphrodites (XY genotype and external female morphology with internal testes, and masculinization occurring at puberty); C. congenital adrenal hyperplasia (XX genotype with normal female internal organs, masculinized external organs observed at birth); D. sexual development is similar in nonhuman mammals, but there are some interesting pathologies not seen in humans. A freemartin occurs in cattle twins, with one male and one female. The male is normal but the XX female is sterile, born masculinized, and has testicular tissue. Explain the most likely cause of this masculinized phenotype, and indicate on your chart the point or points at which something abnormal occurred.

Answer: Chart of normal male development should resemble Fig. 26-4 in the CHAPTER, with a similar chart in parallel for female development, including details from Table 26-1. Appearance of secondary sexual characteristics in both genders results from increases in gonadal steroids. A.-B. Both testicular feminization and male pseudohermaphroditism begin with lack of response of target tissues to androgens produced in fetal development; testicular feminization allows development of female secondary sexual characteristics, whereas pseudohermaphroditism allows masculinization at puberty. C. Congenital adrenal hyperplasia allows normal development of female internal structures, but masculinizes external organs before birth as an effect of adrenal androgens. If untreated, puberty- related changes would also be masculine. D. The freemartin cow must receive anti-Mullerian hormone through shared placental circulation; therefore testicular tissue develops and androgens are secreted to produce further masculinization, in the fetus and at puberty.

147) List and define the four major receptor groups.

Answer: Chemoreceptors, bind chemical ligands mechanoreceptors, are stimulated by a form of mechanical energy (gravity, vibration) thermoreceptors, respond to temperature photoreceptors, respond to light

61) Describe the different types of cholinergic receptors in the nervous system. Which neurotransmitter binds to each type?

Answer: Cholinergic receptors are either nicotinic or muscarinic; they are distinguishable by the binding of either nicotine or muscarine to the same receptors. Both receptors bind to acetylcholine.

83) Explain the relationship among chylomicrons, cholesterol, and the various forms of lipoproteins (LDL, HDL, etc.). What is the role of each one in the body? Why are some considered "bad" and others "good"? Where does each come from? Where could each end up?

Answer: Chylomicrons are complexes composed of triglycerides, cholesterol, and lipoprotein, assembled in the intestinal epithelium. Most lipoprotein in blood is low-density, or LDL; other forms include very low-density (VLDL) and high-density lipoprotein (HDL). The triglyceride portion of chylomicrons is a source of free fatty acids and monoglycerides for cells, which use them for energy or convert them back to triglycerides for energy storage. Cholesterol is a precursor for steroid hormones and bile components and, along with phospholipids synthesized from fatty acids, a structural component of cell membranes. LDLs are a means of transporting cholesterol to most cells, whereas HDLs transport cholesterol primarily to the liver. LDL is "bad" or "lethal" because elevated LDL is associated with atherosclerosis. HDL is "good" or "healthy" because it takes cholesterol to the liver, where it is metabolized or excreted.

53) Cortisol secretion shows a __________ pattern of secretion, normally peaking in the __________.

Answer: Circadian, morning 54) The most important metabolic effect of __________ is protection against __________, crucial to surviving significant stress. (:0)Answer: Cortisol, hypoglycemia 55) Structurally similar steroids sometimes bind to the same receptor, producing __________ effects. (:0)Answer: Crossover

106) In some cultures, infant boys are circumcised. Explain what circumcision is and why there is support for as well as opposition to the procedure.

Answer: Circumcision is removal of the foreskin of the penis. Supporters of the procedure believe it may reduce chances of acquiring and/or possibly infecting a partner with certain diseases, including urinary tract infections, sexually transmitted diseases, and penile cancer. Opponents suggest the incidence of these is too low to warrant inflicting pain and mutilating the infant, and possibly decreasing sexual pleasure when the child becomes an adult.

96) Explain clonal expansion and the roles and interactions of effector cells, memory cells, naive lymphocytes,and plasma cells.

Answer: Clonal expansion is the increase in population of a lymphocyte of a particular antigen specificity, as a result of exposure to that antigen. Lymphocytes produced during clonal expansion differentiate either into effector cells, which produce the immune response and die within a few days, or memory cells, which persist for years and continue to reproduce. Naive lymphocytes are those present at birth, with antigen specificity already determined, but in small numbers because they have not yet been exposed to the antigen and undergone clonal expansion. Plasma cells are the effector B lymphocytes, which become specialized for producing and secreting large amounts of antibody.

90) Define, compare, and contrast "clot busters" and anticoagulants; describe how each works.

Answer: Clot busters are enzymes that destroy clots that have already formed, while anticoagulants prevent clots from forming in the first place.

56) The osmotic pressure created by the presence of proteins is known as __________, which is (higher/lower) in the plasma than in the interstitial fluid.

Answer: Colloid osmotic or oncotic pressure, higher 57) The decrease in blood pressure upon standing is known as __________. (:0)Answer: orthostatic hypotension 58) The term for chronically elevated blood pressure is __________. (:0)Answer: hypertension

129) Essential nutrients are those that our cells require but cannot make, and thus they must be present in the diet. Only some amino acids and some fatty acids are essential. What does that suggest about extreme diets that eliminate fats or proteins? What does that suggest about our carbohydrate intake? Describe the typical American diet, in terms of relative amounts of ingested carbohydrates, fats, proteins, and nucleic acids. Propose some explanations for why one of those is predominant.

Answer: Complete elimination of proteins or fats would eventually be fatal. Carbohydrates need not be consumed at all, from the standpoint of essential nutrients. In reality, however, a carbohydrate-free diet would cause metabolic problems because of consequences of manufacturing glucose from lipids and proteins, as described in Ch. 22 in the text. About half of the typical dietary intake is carbohydrate, with most of the remainder in the form of fats and proteins. Carbohydrates are the primary molecule in plant products, which makes up most of what we eat because of abundance and price.

49) List and explain the factors that influence gas diffusion in the lungs.

Answer: Concentration gradient, surface area, membrane thickness, and diffusion distance are the four factors. This is discussed in the "Diffusion and Solubility of Gases" section of the CHAPTER. 50) List and explain the factors that influence the presence of gases in liquids. (:0)Answer: Pressure, solubility, and temperature are three factors. This is discussed in "The Solubility of Gases in Liquids Depends on Pressure, Solubility, and Temperature" section of the CHAPTER. 51) Name three pathological changes that adversely affect gas exchange. (:0)Answer: 1. a decrease in the amount of alveolar surface area available for gas exchange 2. an increase in the thickness of the alveolar membrane 3. an increase in the diffusion distance between the alveoli and the blood 52) What are the three ways CO2 is transported in blood? Approximately what percentage is transported by each way? (:0)Answer: 1. attached to hemoglobin, 23% 2. dissolved, 7% 3. as bicarbonate, 70% 53) Carbon dioxide is more soluble in water than oxygen. To get the same amount of oxygen to dissolve in plasma as carbon dioxide, you would have to A) decrease the temperature of the plasma. B) increase the partial pressure of oxygen. C) decrease the partial pressure of nitrogen. D) increase the rate of plasma flow through the lungs. E) decrease the alveolar ventilation rate. (:0)Answer: B

93) __________ results from conditions in the middle ear that block the normal transfer of vibration from the tympanic membrane to the oval window.

Answer: Conduction deafness 94) __________ deafness occurs because of a problem in the cochlea or somewhere along the auditory pathway. (:0)Answer: Sensorineural 95) __________ cells connect photoreceptors to ganglion cells. (:0)Answer: Bipolar 96) The bending of light that occurs as it passes between media of different densities is called __________. (:0)Answer: refraction 97) Our eyes are sensitive to light in the visible spectrum between the wavelengths of __________. (:0)Answer: 400 to 750 nm 98) A __________ is a single energy packet of visible light. (:0)Answer: photon 99) __________, or __________ ganglion cells, transmit information about texture, while __________, or __________ ganglion cells, transmit information about movement. (:0)Answer: Parvocellular, p, magnocellular, m 100) A person suffering from __________ can see objects that are close, but distant objects appear blurred. (:0)Answer: myopia 101) __________ is the loss of accommodation that occurs with age and is associated with a loss of lens elasticity. (:0)Answer: Presbyopia 102) __________ is a condition that causes blurred vision due to the cornea and/or lens not being smoothly curved and symmetrical. (:0)Answer:A stigmatism

175) Explain what is meant by the contractility of the heart. How is contractility controlled?

Answer: Contractility is the intrinsic ability of a cardiac muscle fiber to contract at any given fiber length. Contractile force increases with ventricular end diastolic volume, which is determined by venous return. Both the endocrine and nervous systems regulate contractility.

65) __________ is the transfer of heat energy to air.

Answer: Convection 66) After ingestion of carbohydrates in an insulin-deficient diabetic, __________ would happen. A) protein degradation B) fat synthesis C) ketone production D) glycolysis E) A and C (:0)Answer: E

79) Which hormones of growth and metabolism have a permissive effect on other hormones of growth and metabolism?

Answer: Cortisol is required for glucagon to be fully effective against a hypoglycemic challenge. In children, thyroid hormone is necessary for the full expression of growth hormone.

73) Explain how cortisol suppresses the immune system, and why this is sometimes a useful therapy. What are some negative side effects of cortisol therapy? Why do doctors prescribe cortisol in doses that taper off at the end of the therapy period? Why do doctors sometimes prefer to prescribe nonsteroidal anti-inflammatory drugs?

Answer: Cortisol prevents cytokine release and antibody production by white blood cells and decreases leukocyte mobility and migration; cortisol is thus an effective anti-inflammatory and antirejection drug for transplant patients. Cortisol weakens the skeleton and alters brain function, in addition to suppressing the immune system. Because cortisol suppresses ACTH production by negative feedback, the adrenal cortex atrophies during cortisol therapy. Tapering off of dosing allows adrenal cortex function to return gradually. Nonsteroidal anti-inflammatories avoid the negative side effects of cortisol therapy and are therefore more appropriate for minor inflammation.

87) In an effort to impress her physiology instructor, your roommate did an Internet search on cortisol. In her exuberance, she failed to read the book CHAPTER, the instructor has not yet lectured on this CHAPTER, and frankly she knows next to nothing about this hormone. She discovers such seemingly unrelated therapeutic effects as treatment for bee stings and rejection of transplanted organs. You, however, completed your physiology course last semester, and she turns to you for enlightenment. What can you tell her about some other therapeutic uses for cortisol, and what functional connections do these illustrate? What are some of the negative side effects?

Answer: Cortisol suppresses the immune system by preventing cytokine release, inhibiting antibody production, and inhibiting the inflammatory response. Allergic reactions to bee stings, poison ivy, and pollen involve the immune system; thus cortisol can be used to suppress these responses. Rejection of a transplanted organ is also an immune response. Negative feedback from the exogenous hormone inhibits ACTH production, which can result in atrophy of the adrenal cortex. Hypercortisolism can also result, with symptoms similar to diabetes mellitus, and extra fat deposition leading to "moon face," and changes in mood and cognition.

80) What are the symptoms and causes of Cushing's disease? How do they compare and contrast with symptoms and causes of Addison's disease?

Answer: Cushing's disease is hypercortisolism, which can arise from adrenal or pituitary tumors or exogenous administration of cortisol. Symptoms include gluconeogenesis and consequent hyperglycemia, tissue wasting (especially in the limbs) from breakdown of protein and fat, excess fat deposition in trunk and face, mood elevation followed by depression, and difficulty with learning and memory. Addison's disease, far less common, is hyposecretion of cortisol as well as other adrenal steroids. It is usually caused by autoimmune destruction. Symptoms include darkening of the skin due to effects of ACTH on melanocytes.

10) Each of these statements is true except one. Identify the exception. A) Monoamine oxidase is the main enzyme responsible for the degradation of catecholamines. B) ß1 receptors respond equally well to both epinephrine and norepinephrine. C) ß2 receptors are not innervated by sympathetic neurons, so are more sensitive to epinephrine, delivered via the blood. D) Activation of a receptors opens Na+ channels in the membrane.

Answer: D

10) Spinal interneurons prevent muscle antagonists from interfering with an intended movement by A) initiating a crossed extensor reflex. B) initiating a stretch reflex. C) initiating a tendon reflex. D) the process of reciprocal inhibition. E) activating reverberating circuits.

Answer: D

10) The atomic mass of an atom can be estimated by counting the number of A) protons. B) electrons. C) neutrons. D) protons and neutrons. E) protons and electrons.

Answer: D

10) Which of the following is NOT a membrane lipid? A) sphingolipids B) cholesterol C) phospholipids D) All of the above are membrane lipids.

Answer: D

10) __________ are a group of diseases characterized by the abnormal growth and development of white blood cells, and __________ are diseases where patients have too few white blood cells. A) Neutropenias, leukemias B) Anemias, leukemias C) Neutropenias, anemias D) Leukemias, neutropenias E) Leukemias, anemias

Answer: D

102) Which of the following cytoskeleton components are responsible for the movement of chromosomes during cell division? A) microfilaments B) intermediate filaments C) thick filaments D) microtubules E) all of the above

Answer: D

104) Which of the following statements is NOT true regarding mitochondria? A) The outer mitochondrial membrane is responsible for its shape. B) The intermembrane space is used in the production of ATP. C) Mitochondria contain their own DNA and RNA. D) Mitochondria can replicate themselves only when directed by the cell's nuclear DNA. E) Mitochondria are responsible for providing energy to the cell.

Answer: D

105) Plasma is to blood as __________ is to cytoplasm. A) inclusion B) organelle C) protein D) cytosol E) serum

Answer: D

106) smooth muscle of duodenum is a target

Answer: D

108) Chondrocytes are to cartilage as osteocytes are to A) blood. B) epithelium. C) fat. D) bone. E) neural tissue.

Answer: D

108) When a small amount of hydrochloric acid is added to a solution of Na2HPO4, the pH of the solution does not change. The pH does not change when a small amount of NaOH is added, either. Based on these observations, the compound Na2HPO4 is A) able to accept extra hydrogen ions from the HCl. B) able to donate hydrogen ions to the OH- from NaOH. C) acting as a buffer. D) all of the above

Answer: D

109) How would a molecule that blocks the action of adenyl cyclase affect the sense of olfaction? A) It would increase the level of sensitivity. B) It would expand the range of sensitivity. C) It would convert the olfactory receptors from phasic receptors to tonic receptors. D) It would interfere with the ability to smell. E) It would have no effect on olfaction.

Answer: D

109) The tissue(s) that is/are considered excitable because of the ability to generate electrical signals is/are called A) muscle tissue. B) neural tissue. C) epithelial tissue. D) A and B E) A, B, and C

Answer: D

11) Because of involvement of the __________, pain may be accompanied by emotional distress and nausea or sweating. A) hypothalamus B) limbic system C) medulla D) A and B E) A, B, and C

Answer: D

11) Cortisol, the catecholamines, and growth hormone do all of the following, EXCEPT A) mobilize glycogen from the liver. B) raise plasma glucose levels. C) promote the conversion of triglycerides to glycerol and fatty acids. D) enhance deposition of fat in adipose tissue.

Answer: D

11) The hormone that regulates water excretion by the kidneys A) decreases water permeability throughout the kidney tubules. B) increases water permeability throughout the kidney tubules. C) decreases water permeability in only certain portions of the kidney tubules. D) increases water permeability in only certain portions of the kidney tubules.

Answer: D

11) The lungs are located in the __________ cavity. A) pericardial B) pulmonary C) pleural D) thoracic E) costal

Answer: D

11) Which of the following statements is TRUE? A) Titin returns stretched muscles to their resting length. B) Titin stabilizes the position of the contractile filaments. C) Titin is helped by actin. D) A and B E) all of the above

Answer: D

110) After an injury, Paul finds that he has a difficult time recognizing and interpreting certain sounds. He is still able to hear and his acoustic reflexes are normal. These symptoms imply damage to the A) cochlear nerve. B) cochlear nucleus. C) inferior colliculus. D) auditory cortex. E) all of the above

Answer: D

112) The quantity of neurotransmitter that is released by a hair cell into the synapse with a sensory neuron increases A) in response to any mechanical stimulation of the hair cell. B) when the stereocilia of the hair cell are displaced away from the kinocilium. C) when the stereocilia are equally displaced to either side of the kinocilium. D) when the stereocilia of the hair cell are displaced toward the kinocilium. E) none of the above

Answer: D

113) Each of the following is a function of proteins except one. Identify the exception. A) support and structure B) transport C) catalyst D) storage of genetic information E) carrying of messages

Answer: D

115) Following are steps that occur in the production of an auditory sensation. Put them in the correct sequence. 1. The pressure wave distorts the basilar membrane on its way to the round window. 2. Movement of the tympanic membrane causes displacement of the malleus. 3. Displacement of the stereocilia stimulates sensory neurons of the cochlear nerve. 4. Movement of the malleus causes movement of the incus and stapes. 5. Distortion of the basilar membrane forces the hair cells of the organ of Corti toward or away from the tectorial membrane. 6. Movement of the oval window establishes pressure waves in the perilymph of the vestibular duct. A) 2, 4, 1, 6, 5, 3 B) 2, 4, 6, 3, 5, 1 C) 2, 1, 4, 6, 5, 3 D) 2, 4, 6, 1, 5, 3 E) 4, 2, 1, 5, 3, 6

Answer: D

115) Microscopic examination of a tissue reveals an open framework of fibers with a large volume of fluid ground substance and elastic fibers. This tissue would most likely have come from A) the inner wall of a blood vessel. B) the lungs. C) the spleen. D) the tissue that separates skin from underlying muscle. E) the bony socket of the eye.

Answer: D

12) The synthesis of glucose from a noncarbohydrate precursor is referred to as A) glycogenesis. B) glycogenolysis. C) glycolysis. D) gluconeogenesis. E) glycogen.

Answer: D

12) Which statement is TRUE? A) As the sarcomere shortens, the two Z disks at each end move closer together, and the I band and H zone virtually disappear. B) The thin actin filaments slide along the thick myosin filaments as they move toward the M line in the center of the sarcomere. C) When a muscle contracts, myosin molecules coil up like springs to shorten the length. D) A and B E) A, B, and C

Answer: D

122) An exergonic chemical reaction A) converts molecules into molecules that have less free energy. B) releases energy as the reaction progresses. C) requires the input of energy. D) A and B E) A, B, and C

Answer: D

123) How would the lack of a required cofactor for an enzyme affect that enzyme's function? A) The enzyme's function would not be altered. B) The enzyme would function more slowly. C) The enzyme would function more quickly. D) The enzyme would not be able to function. E) none of the above

Answer: D

125) One way a cell can safely control physiological processes is by A) heating up to 110° F. B) letting an enzyme accumulate in order to increase its cellular concentration. C) breaking an enzyme down faster in order to decrease its cellular concentration. D) B and C E) All of the above are safe mechanisms for cells to use.

Answer: D

126) When a rod cell in the eye is stimulated by light, A) the cell membrane becomes depolarized. B) the inactive form of retinal associates with bleached opsin. C) transducin activity decreases. D) the intracellular level of cGMP decreases. E) more neurotransmitter is released.

Answer: D

13) Chronic hypoxia A) increases 2,3-DPG production in blood. B) shifts the HbO2 dissociation curve to the left. C) can be caused by anemia. D) A and C E) all of the above

Answer: D

13) The number of electrons in an energy shell A) is always eight. B) is always eight except for the outermost shell. C) is always eight except for the innermost shell. D) can be up to eight except for the innermost shell. E) can be up to eight except for the outermost shell.

Answer: D

13) The only known example of neurons with a high rate of replacement in the adult human are the A) gustatory receptors. B) nociceptors. C) Pacinian corpuscles. D) olfactory receptors. E) retinal cells.

Answer: D

13) Thyroid hormone contains the mineral A) sodium. B) thallium. C) iron. D) iodine. E) zinc.

Answer: D

13) __________ have been found to be the precursor cells of macrophages. A) Eosinophils B) Basophils C) Neutrophils D) Monocytes E) Lymphocytes

Answer: D

131) NADH is produced from each reaction described except one. Identify the exception. A) during aerobic glycolysis B) during the conversion of pyruvate to acetyl CoA C) during the citric acid cycle D) in the electron transport system E) NADH is produced in each reaction listed.

Answer: D

131) Which two sensory systems contain structures called maculae? A) gustatory and olfactory B) olfactory and auditory C) auditory and vestibular D) vestibular and visual E) visual and somatosensory

Answer: D

131) plasma membrane

Answer: D

14) A positively charge ion is called a(n) A) electron. B) proton. C) neutron. D) cation. E) anion.

Answer: D

14) Lipoproteins that carry mostly cholesterol and phospholipids from peripheral tissues to the liver are called A) very low-density lipoproteins (VLDLs). B) low-density lipoproteins (LDLs). C) intermediate-density lipoproteins (IDLs). D) high-density lipoproteins (HDLs). E) very high-density lipoproteins (VHDLs).

Answer: D

14) The Internet database for molecular, cellular, and physiological information is called the __________ Project. A) Human Genome B) Physiognomy C) Physiosome D) Physiome E) Manhattan

Answer: D

14) The intensity of a stimulus can be determined by A) population coding. B) labeled line coding. C) frequency coding. D) A and C E) all of the above

Answer: D

14) Which of the following are NOT antigen-presenting cells (APCs)? A) lymphocytes B) dendritic cells C) macrophages D) neutrophils E) monocytes

Answer: D

140) During starvation, steroid hormones trigger the transcription of genes for lipid metabolism in their target cells. This would be an example of control by A) negative feedback. B) positive feedback. C) repressors. D) inducers. E) none of the above

Answer: D

15) Exercise hyperventilation results from signals from A) muscle and joint mechanoreceptors and proprioceptors. B) the carotid body chemoreceptors. C) activating pulmonary stretch receptors. D) A and B E) A, B, and C

Answer: D

15) Surfactant A) protects the surface of the lungs. B) phagocytizes small particulate matter. C) replaces mucus in the alveoli. D) helps prevent the alveoli from collapsing. E) is not found in healthy lung tissue.

Answer: D

15) The addition of a phosphate group to a substrate is called __________. The enzyme that catalyzes this reaction is referred to as a __________. A) proteolysis; peptidase B) phosphorylation; phosphatase C) proteolysis; kinase D) phosphorylation; kinase E) None of the above represents the correct terminology.

Answer: D

16) Excess secretion of growth hormone during adulthood will cause A) goiter. B) exophthalmus. C) giantism. D) acromegaly. E) diabetes.

Answer: D

16) In the heart, valves are located A) between the atria and the ventricles. B) between the ventricles and the arteries. C) between the great veins and the atria. D) A and B E) A, B, and C

Answer: D

17) The primary stimulus for the release of erythropoietin is A) hypoxemia. B) low oxygen levels in the tissues. C) low blood pressure. D) A and B E) A, B, and C

Answer: D

17) Which valves have chordae tendineae? A) aortic and pulmonary valves B) bicuspid and tricuspid valves C) mitral valve D) B and C E) A, B, and C

Answer: D

18) Dalton's law states that A) gas volume and temperature are directly proportional. B) gas volume and pressure are inversely proportional. C) the volume of gas that will dissolve in a solvent is proportional to the solubility of the gas and the gas pressure. D) in a mixture of gases like air, the total pressure is the sum of the individual partial pressures of the gases in the mixture. E) none of the above

Answer: D

18) Each of the following statements concerning mitochondria is TRUE except one. Identify the exception. A) The mitochondrial cristae increase the inner surface area of the organelle. B) The matrix of the mitochondria contains metabolic enzymes involved in energy production. C) Respiratory enzymes are attached to the surface of the cristae. D) The mitochondria contain no DNA or RNA. E) The mitochondria produce most of a cell's ATP.

Answer: D

18) If a scientific opinion has been verified repeatedly it becomes a A) model. B) theory. C) hypothesis. D) law. E) variable.

Answer: D

21) During exercise, the most significant factor affecting the pH of body tissues is the production of A) lactic acid. B) ketone bodies. C) organic acids. D) carbon dioxide. E) hydrochloric acid.

Answer: D

21) Excess growth hormone prior to puberty would result in A) osteoporosis. B) cretinism. C) rickets. D) giantism. E) dwarfism.

Answer: D

21) Microvilli are found A) mostly in muscle cells. B) on the inside of cell membranes. C) in large numbers on cells that secrete hormones. D) in cells that are actively engaged in absorption. E) only on cells lining the reproductive tract.

Answer: D

22) During the plateau phase of the action potentials of myocardial contractile cells, which ion(s) is/are crossing the membrane? A) Ca2+ B) K+ C) Na+ D) A and B E) A and C

Answer: D

22) When Golf is activated, A) a second messenger cascade is activated. B) olfactory epithelial cells depolarize. C) an odorant is allowed to combine with olfactory binding protein. D) A and B E) A, B, and C

Answer: D

23) A substance that induces fever is known as a(n) A) opsonin. B) chemotaxin. C) acute phase protein. D) pyrogen. E) cytokine.

Answer: D

23) Atrial natriuretic peptide A) increases the GFR. B) inhibits the release of renin. C) stimulates the release of renin. D) A and B E) A and C

Answer: D

24) A typical value for intrapleural pressure is __________ mm Hg. A) +6 B) +3 C) 0 D) -3 E) -6

Answer: D

24) An increase in plasma potassium levels is properly called A) hypernatremia. B) hyperpotassemia. C) hyperpotasseplasmia. D) hyperkalemia. E) hypercalcemia.

Answer: D

24) Gustatory neurons are stimulated by taste cells for A) salty. B) sour. C) bitter. D) A and B E) B and C

Answer: D

24) Most of the ATP required to power cellular operations is produced in the A) ribosomes. B) endoplasmic reticulum. C) nucleus. D) mitochondria. E) Golgi apparatus.

Answer: D

25) A hematocrit is used to indicate A) the ratio of red blood cells to the total blood volume. B) the packed cell volume. C) coagulation time. D) A and B E) all of the above

Answer: D

25) Interactions between different globular or fibrous polypeptide chains result in which type of structure? A) primary B) secondary C) tertiary D) quaternary E) pentagonal

Answer: D

26) A normal adult hematocrit would be approximately __________%. A) 100 B) 75 C) 66 D) 45 E) 10

Answer: D

26) All of the following are synthesized along various sites of the endoplasmic reticulum except one. Identify the exception. A) proteins B) fatty acids C) steroids D) RNA E) phospholipids

Answer: D

26) Thirst is A) controlled by centers in the hypothalamus. B) triggered by increased osmolarity. C) relieved only when plasma osmolarity is decreased. D) A and B E) A, B, and C

Answer: D

26) movement from the peritubular capillaries to the nephron lumen

Answer: D

27) Mucus functions in A) protection. B) lubrication. C) enzyme activation. D) A and B E) A, B, and C

Answer: D

28) A nucleotide consists of A) a five-carbon sugar and phosphate group. B) a five-carbon sugar and a nitrogenous base. C) a phosphate group and a nitrogenous base. D) a five-carbon sugar, a nitrogenous base, and a phosphate group. E) a five-carbon sugar and an amino acid.

Answer: D

28) The percentage of whole blood occupied by red blood cells is the A) viscosity. B) specific gravity. C) pH. D) hematocrit. E) differential cell count.

Answer: D

28) With heat acclimatization, A) sweating become more productive. B) aldosterone increases. C) sweating becomes more concentrated. D) A and B E) B and C

Answer: D

28) __________ capillaries are very porous and allow high volumes of fluids to pass through them, whereas __________ capillaries consist of more tightly joined cells that allow a high degree of selective materials to pass. A) Fenestrated, transcytotic B) Continuous, fenestrated C) Transcytotic, continuous D) Fenestrated, continuous E) Transcytotic, fenestrated

Answer: D

29) Glucagon A) stimulates gluconeogenesis. B) primarily targets the liver. C) primarily targets skeletal muscle. D) A and B E) A and C

Answer: D

29) Menstruation is triggered by a drop in the level(s) of A) FSH. B) LH. C) relaxin. D) estrogen and progesterone. E) human chorionic gonadotropin.

Answer: D

29) Thyroid hormones A) are lipophilic. B) are hydrophilic. C) must bind to plasma proteins for transport to target cells. D) A and C E) B and C

Answer: D

29) You conduct an experiment on twenty 18-year-old male subjects to see how various intensities of exercise influence heart rate. Which of the following is/are considered a dependent variable? A) age of subjects B) sex of subjects C) intensity of exercise D) heart rate E) more than one of these

Answer: D

29) urea

Answer: D

30) Compared to arteries, the velocity of flow of the blood through the capillaries is A) at least 10 times faster. B) at least twice as fast. C) about the same. D) much slower. E) impossible to predict without more information.

Answer: D

30) The first line of cellular defense against pathogens are the A) T lymphocytes. B) B lymphocytes. C) NK cells. D) phagocytes. E) plasma cells.

Answer: D

30) The most important energy-transferring compound in cells is A) glucose. B) fructose. C) protein. D) adenosine triphosphate. E) deoxyribonucleic acid.

Answer: D

30) The process of red blood cell production is called A) erythrocytosis. B) erythropenia. C) hemocytosis. D) erythropoiesis. E) hematopenia.

Answer: D

30) Which of the following is involved in the pathway that causes a jump in ventilation rate at the beginning of exercise? A) Proprioceptors in muscles send information to the motor cortex of the brain. B) Output from the limbic system and cardiovascular control center triggers parasympathetic discharge. C) Widespread vasoconstriction causes increase in blood pressure. D) A and C E) B and C

Answer: D

31) Central fatigue A) includes feeling tired. B) may precede physiological muscle fatigue. C) may be related to changes in the brain related to changes in the pH of the blood. D) A and B E) A, B, and C

Answer: D

31) Each red blood cell is shaped as a biconcave disk. This allows it to A) change its shape to squeeze through narrow openings. B) shrink or swell slightly in response to osmotic conditions. C) synthesize new proteins and membrane components readily. D) A and B E) A, B, and C

Answer: D

31) GH A) is secreted by the anterior pituitary. B) is secreted by the posterior pituitary. C) binds to a plasma protein. D) A and C E) B and C

Answer: D

31) Which bases below are purines? 1. adenine 2. cytosine 3. guanine 4. thymine 5. uracil A) 1 and 2 B) 2 and 3 C) 1, 3, and 5 D) 1 and 3 E) 2, 4, and 5

Answer: D

32) Lymphocytes that attack foreign cells or body cells infected with viruses are A) B lymphocytes. B) plasma cells. C) helper T cells. D) cytotoxic T cells. E) suppressor T cells.

Answer: D

33) Cholesterol is A) a precursor to steroid hormones. B) a structural component of cell membranes. C) a dangerous fat that is absent from a healthy body. D) A and B E) A, B, and C

Answer: D

33) One kilocalorie (kcal) is the amount of energy needed to raise one __________ of water by 1 degree Celsius. A) milliliter B) tablespoon C) cup D) liter E) gallon

Answer: D

33) Osmotic pressure resulting from presence of plasma proteins in blood is called __________ pressure. A) oncotic B) colloid osmotic C) hydrostatic D) A and B E) B and C

Answer: D

33) Symptoms of low plasma pH may include A) CNS depression. B) confusion and disorientation. C) numbness, tingling, or muscle twitches. D) A and B E) A, B, and C

Answer: D

33) The AV node is important because it A) directs electrical impulses from the atria to the ventricles. B) delays the transmission of the electrical impulses to the ventricles in order for the atria to finish contracting. C) serves as the pacemaker in a normal heart. D) A and B E) A, B, and C

Answer: D

33) The external auditory canal ends at the A) vestibule. B) cochlea. C) ossicles. D) tympanic membrane. E) pinna.

Answer: D

34) The most important factor affecting the pH of plasma is the concentration of A) lactic acid. B) ketone bodies. C) organic acids. D) carbon dioxide. E) hydrochloric acid.

Answer: D

34) The release of many GI tract hormones is stimulated by a particular food or substance. Which hormone is INCORRECTLY paired with its stimulus? A) CCK - fatty foods B) secretin - acid in the small intestine C) GIP - glucose in the small intestine D) motilin - acid in the stomach E) gastrin - peptides, increased sympathetic activity

Answer: D

36) A structure that allows the middle ear to communicate with the nasopharynx is the A) pinna. B) membranous labyrinth. C) bony labyrinth. D) eustachian tube. E) auditory meatus.

Answer: D

36) Aerobic metabolism of glucose A) requires oxygen. B) produces more ATP per glucose than anaerobic pathways. C) is the fastest way to produce glucose. D) A and B E) A, B, and C

Answer: D

36) Each of these statements is true, except one. Identify the exception. A) A power muscle, such as in the calf of the leg, may have 2000 muscle fibers in one motor unit. B) All muscle fibers in a single motor unit are of the same fiber type. C) With endurance training, the metabolic characteristics of muscle fibers can be changed and enhanced. D) The entire skeletal muscle contracts in an all-or-none manner. E) Muscles generate a pushing force.

Answer: D

36) Heat loss is promoted by A) sweating. B) dilation of cutaneous blood vessels. C) nonshivering thermogenesis. D) A and B E) A, B, and C

Answer: D

36) The chorionic villi A) form the umbilical cord. B) form the umbilical vein. C) form the umbilical arteries. D) increase the surface area available for exchange between the placenta and the maternal blood. E) form the portion of the placenta called the decidua capsularis.

Answer: D

36) ________ is the immunoglobulin class that comprises 75% of antibodies in adult blood. A) IgA B) IgD C) IgE D) IgG E) IgM

Answer: D

37) Immunoglobulins that are mainly responsible for resistance against viruses, bacteria, and bacterial toxins are A) IgA. B) IgD. C) IgE. D) IgG. E) IgM.

Answer: D

37) Prolonged vomiting of the stomach's contents can result in A) respiratory acidosis. B) respiratory alkalosis. C) metabolic acidosis. D) metabolic alkalosis. E) none of the above

Answer: D

38) Which of the following hormones is NOT produced by the placenta? A) human chorionic gonadotropin (hCG) B) human placental lactogen (hPL) C) relaxin D) luteinizing hormone E) progesterone

Answer: D

39) The explanation for how ATP bonds are actually formed during oxidative phosphorylation is called A) the Krebs hypothesis. B) the second law of thermodynamics. C) the proton principle. D) the chemiosmotic theory. E) Murphy's law.

Answer: D

4) In the arm, the biceps brachii is a flexor whereas triceps brachii is an extensor. Which of the following statements is TRUE? A) The biceps brachii is the primary muscle responsible for movement of the hand and forearm towards the shoulder. B) The triceps brachii is the primary muscle responsible for movement of the hand and forearm towards the shoulder. C) Theses muscles are an example of antagonistic muscles. D) A and C E) B and C

Answer: D

4) The lower respiratory tract includes A) all of the bronchial branches. B) the lungs. C) the trachea. D) A and B E) A, B, and C

Answer: D

4) The two organ systems that work together to regulate most aspects of the body's water balance are A) digestive and respiratory. B) urinary and respiratory. C) cardiovascular and respiratory. D) urinary and cardiovascular. E) digestive and cardiovascular.

Answer: D

4) Viruses A) must reproduce inside cells of the host. B) are considered to be parasitic. C) easily survive in the environment outside the body. D) A and B E) A, B, and C

Answer: D

40) The common pathway of coagulation begins with the A) sticking of platelets to damaged tissue. B) activation of a proenzyme exposed to collagen. C) release of tissue factor by a damaged endothelium. D) activating of a clotting factor that converts prothrombin to thrombin. E) activation of a clotting factor that converts fibrinogen to fibrin.

Answer: D

41) The type of protein found in gap junctions is the A) claudin. B) occludin. C) cadherin. D) connexin. E) integrin.

Answer: D

43) Collateral pathways take information about sound from the medulla to the A) reticular formation. B) cerebellum. C) thalamus. D) A and B E) A, B, and C

Answer: D

43) Compared to skeletal muscle, smooth muscle A) is slower to contract in response to a stimulus. B) sustains contractions longer without fatigue. C) develops tension more rapidly and relaxes more rapidly. D) A and B E) A, B, and C

Answer: D

43) In order for a lymphocyte to respond to an antigen, the antigen must A) be phagocytized by the lymphocyte. B) enter the cytoplasm of the lymphocyte. C) bind to the DNA of the lymphocyte. D) bind to specific receptors on the lymphocyte membrane. E) depolarize the lymphocyte.

Answer: D

44) When an antigen complex is bound to a Class I MHC molecule, it can stimulate a A) B cell. B) plasma cell. C) helper T cell. D) cytotoxic T cell. E) NK cell.

Answer: D

45) Class II MHC molecules are found A) on all cells with a nucleus. B) only on red blood cells. C) only on granulocytes and macrophages. D) only on lymphocytes and macrophages. E) only on liver cells and macrophages in the spleen.

Answer: D

45) Functions of epithelia include all of the following EXCEPT A) providing physical protection. B) controlling permeability. C) producing specialized secretions. D) storing energy reserves. E) movement.

Answer: D

45) The reactions where fats are broken down into glycerol and fatty acids are called A) glycerolysis. B) glycolysis. C) gluconeogenesis. D) lipolysis. E) liposuction.

Answer: D

46) A person's blood type is determined by A) the size of the red blood cells. B) the shape of the red blood cells. C) the chemical character of the hemoglobin. D) the presence or absence of specific glycoprotein molecules on their cell membrane. E) the number of specific molecules on the cell membrane.

Answer: D

46) In smooth muscle, what is the small protein chain that regulates contraction and relaxation? A) actin B) myosin heavy chain C) calcium D) myosin light chain E) none of the above

Answer: D

46) bind(s) with iron in the blood

Answer: D

46) distal-most section of small intestine

Answer: D

47) A person with type A blood has A) A antibodies on his red blood cells. B) A antibodies in his plasma. C) B antibodies on his red blood cells. D) B antibodies in his plasma. E) the ability to receive AB blood cells.

Answer: D

47) When the plasma concentration of a substance exceeds its renal concentration, more of the substance will be A) filtered. B) reabsorbed. C) secreted. D) excreted. E) none of the above

Answer: D

47) carbon

Answer: D

48) If the results of ufo-epi treatment of humans included pupil dilation, localized sweating, high blood pressure, and high blood glucose, which conclusion would be indicated? A) It acts on AMPA receptors. B) It acts at the neuromuscular junction. C) It acts on muscarinic receptors. D) It acts on alpha adrenergic receptors. E) It acts on beta adrenergic receptors.

Answer: D

48) In order for blood to enter the heart, A) the atria must be in diastole. B) the pressure in the atria must be lower than in the veins. C) the AV valves must be open. D) A and B E) A, B, and C

Answer: D

49) These are fragments of a megakaryocyte.

Answer: D

5) If oxygen is NOT available for aerobic ATP production, __________ builds up on muscles. A) glucose B) ATP C) pyruvic acid D) lactic acid

Answer: D

5) Monocytes leave the circulation to go to the tissues, where they are called A) eosinophils. B) basophils. C) lymphocytes. D) macrophages. E) neutrophils.

Answer: D

5) Smooth muscle is present in the walls of A) veins only. B) arteries only. C) muscular arteries only. D) all vessel types except capillaries. E) all vessel types.

Answer: D

5) The __________ is a significant site of absorption of water and electrolytes, but NOT of nutrients. A) mouth B) stomach C) small intestine D) large intestine E) none of the above

Answer: D

14) Sweat glands contain A) cholinergic receptors. B) alpha receptors. C) beta receptors. D) all of the above E) none of the above

Answer:A

5) Which of these is matched incorrectly? A) innate immunity - nonspecific responses B) acquired immunity - responses directed toward specific invaders C) active immunity - an immune response started and developed by the cells D) actively acquired immunity - immunity passed via the placenta or milk to offspring E) Each of these is correctly described.

Answer: D

50) Cytotoxic, these cells are associated with allergic reactions and parasitic infestations. Usually these cells are less than 3% of the leukocyte population, and only live up to 12 hours.

Answer: D

50) puffy appearance associated with hypothyroidism

Answer: D

51) Sensations of gravity and linear acceleration are registered in the A) semicircular canals. B) cochlea. C) ossicles. D) saccule and utricle. E) organ of Corti.

Answer: D

52) ATP synthase tranfers the __________ energy of the H+ ions to the high-energy phosphate bond of __________. A) potential, NADH B) potential, ATP C) kinetic, NADH D) kinetic, ATP E) none of the above

Answer: D

52) Carbohydrate digestion is completed here.

Answer: D

52) Which of these will increase the heart rate? A) sympathetic stimulation to the SA node B) the application of epinephrine to the SA node C) the application of acetylcholine to the SA node D) A and B E) A, B, and C

Answer: D

54) The chloride shift occurs when A) hydrogen ions leave the red blood cells. B) hydrogen ions enter the red blood cells. C) bicarbonate ions enter the red blood cells. D) bicarbonate ions leave the red blood cells. E) carbonic acid is formed.

Answer: D

55) Which of the following statements about hydrostatic pressure is NOT true? A) If a fluid is not moving, the pressure that it exerts is called hydrostatic pressure. B) Force is not equal in all directions. C) The lateral component of moving fluid represents the hydrostatic pressure. D) A and C E) B and C

Answer: D

55) genetic condition resulting in crescent moon-shaped red blood cells

Answer: D

56) Fat digestion is completed here.

Answer: D

56) In a normal kidney, which of the following conditions would cause an increase in the glomerular filtration rate (GFR)? A) constriction of the afferent arteriole B) a decrease in the hydraulic pressure of the glomerulus C) an increase in the capsular hydraulic pressure D) a decrease in the concentration of plasma proteins in the blood E) a decrease in the net glomerular filtration pressure

Answer: D

56) When the pH of body fluids begins to fall, proteins will A) release a hydrogen from the carboxyl group. B) release a hydrogen from the amino group. C) bind a hydrogen at the carboxyl group. D) bind a hydrogen at the amino group. E) none of the above

Answer: D

57) Jill lives in St. Louis, which is close to sea level. She decides to spend a month of her summer vacation working in the mountains outside of Denver. After a week in the mountains, what kinds of changes would you expect to see as Jill adapts to the higher altitude? A) decreased hematocrit B) decreased blood pressure C) decreased alveolar ventilation rate D) decreased PO2 in the alveoli E) all of the above

Answer: D

57) Loose connective tissue functions in A) supporting small glands. B) supporting epithelia. C) anchoring blood vessels and nerves. D) all of the above

Answer: D

58) goblet cells

Answer: D

59) After interacting with the ribosome, each codon of mRNA interacts with a third molecule. This molecule is called A) DNA. B) an amino acid. C) rRNA. D) tRNA. E) rough ER.

Answer: D

59) When all three cone populations are stimulated equally, we see A) purple. B) orange. C) brown. D) white. E) black.

Answer: D

6) The highly branched contractile cells that regulate capillary permeability are called A) podocytes. B) vascular smooth muscle. C) endothelial cells. D) pericytes. E) epitheliocytes.

Answer: D

6) Which of the following can be measured to determine a person's metabolic rate? A) heat released from body in sealed chamber B) oxygen consumed by body in sealed chamber C) carbon dioxide produced by body in sealed chamber D) all of the above E) none of the above

Answer: D

60) ATP and DNA are examples; they transfer energy and encode genetic information.

Answer: D

60) Blocking afferent action potentials from the chemoreceptors in the carotid and aortic bodies would interfere with the brain's ability to regulate breathing in response to all EXCEPT which of the following? A) changes in PCO2 B) changes in PO2 C) changes in pH due to carbon dioxide levels D) changes in blood pressure E) all of the above

Answer: D

62) Atrial contraction A) begins during the first part of the P wave. B) begins just after the T wave. C) begins just after the Q wave. D) begins during the latter part of the P wave. E) none of the above

Answer: D

64) atrial contraction

Answer: D

66) Each of the following is an acceptable definition of a gene except one. Identify the exception. A) a region of DNA that contains all the information needed to produce a molecule of mRNA B) a segment of a chromosome that contains all the information needed to produce a particular polypeptide C) a chain of as many as 5000 nucleotides specifying a protein necessary to exhibit a given trait D) a strand of DNA that contains all the information needed to produce all of the proteins necessary to exhibit a given trait E) All of the above are acceptable definitions.

Answer: D

69) An exon is A) a piece of DNA that is excised. B) a piece of mRNA that is able to exit the nucleus. C) a segment of mRNA that is excised. D) a segment of mRNA that remains after editing. E) a noncoding region of mRNA.

Answer: D

69) the amount of blood pumped out of the heart during one contraction

Answer: D

7) Indifferent gonads develop into testes under the influence of A) androgens. B) estrogens. C) peptide or protein hormones. D) none of the above

Answer: D

7) The actual sites of gas exchange within the lungs are A) bronchioles. B) alveolar ducts. C) pleural spaces. D) alveoli. E) terminal sacs.

Answer: D

7) The lamina propria and mucous epithelium are components of the A) serosa. B) adventitia. C) muscularis mucosa. D) mucosa. E) submucosa.

Answer: D

7) Which of the following is a buffer zone between the outside world and most of the cells of the body? A) blood B) lumen C) lymph D) extracellular fluid E) all of the above

Answer: D

72) If you were in a desert without water, which nutrient would you like to have stored in your body in a large amount? A) protein B) sugars C) complex carbohydrates D) fat E) amino acids

Answer: D

73) The cardiovascular control center in the brain can directly cause A) arterioles to dilate or constrict. B) the heart rate to increase or decrease. C) the contractility of the heart to increase or decrease. D) A and B E) A, B, and C

Answer: D

73) a semilunar valve that has the right ventricle on one side

Answer: D

75) Bill wants to determine his blood type, so he takes a few drops of blood from a puncture wound in his finger and mixes it with various antisera. His blood cells agglutinate when mixed with the anti-A serum but not with the anti-B. Therefore this is true: A) Bill could receive type B blood in a transfusion. B) Bill could donate blood to an individual with type B blood. C) Bill is Rh positive. D) Bill's plasma contains B antibodies. E) Bill's plasma would cross-react with type O red blood cells.

Answer: D

77) One of the factors determining the speed and duration of a twitch in various types of fibers is the speed of the A) removal of Ca2+ ions from the cytosol. B) hydrolysis of ATP. C) movement of K+ across their membranes. D) A and B E) A, B, and C

Answer: D

8) Place the following structures of the respiratory tree in the order in which air passes through them. 1. secondary bronchi 2. bronchioles 3. primary bronchi 4. alveoli 5. terminal bronchioles A) 4, 1, 2, 3, 5 B) 1, 3, 5, 2, 4 C) 3, 1, 5, 2, 4 D) 3, 1, 2, 5, 4 E) 1, 3, 2, 5, 4

Answer: D

8) The adrenal medulla produces A) androgens. B) glucocorticoids. C) mineralocorticoids. D) catecholamines. E) corticosteroids.

Answer: D

8) The largest metabolic reserves for the average adult are stored as A) carbohydrates. B) proteins. C) amino acids. D) triglycerides. E) fatty acids.

Answer: D

80) The force generated by a single muscle fiber A) is always the same. B) can be increased by increasing the frequency of action potentials. C) can be increased due to summation. D) B and C E) all of the above

Answer: D

82) The following is a list of structures of the male reproductive tract. 1. ductus deferens 2. urethra 3. ejaculatory duct 4. epididymis Idnetify the order in which sperm pass through these structures from the testes to the penis. A) 1, 3, 4, 2 B) 4, 3, 1, 2 C) 4, 1, 2, 3 D) 4, 1, 3, 2 E) 1, 4, 3, 2

Answer: D

9) Contraction of the __________ alters the shape of the intestinal lumen and moves epithelial pleats and folds. A) mucosa B) submucosa C) submucosal plexus D) muscularis mucosa E) adventitia

Answer: D

9) Enzymes A) may be synthesized as inactive molecules. B) may require the binding of cofactors such as calcium. C) do not require coenzymes for metabolic reactions. D) A and B E) all of the above

Answer: D

9) Hypoxia resulting from fluid accumulation in the alveoli that cannot be corrected by oxygen therapy can lead to A) emphysema. B) fibrotic lung disease. C) asthma. D) adult respiratory distress syndrome. E) sudden infant death syndrome.

Answer: D

9) Sister chromatids A) are found in cells called primary gametes. B) contain twice the normal amount of DNA. C) occur only during ovum formation. D) A and B E) A, B, and C

Answer: D

9) The study of body function in a disease state is A) necrology. B) physiology. C) microbiology. D) pathophysiology. E) histology.

Answer: D

Human Physiology, 5e (Silverthorn) CHAPTER 23 Endocrine Control of Growth and Metabolism 1) The action of a hormone on a target cell involves effects on target cell A) receptor proteins. B) nonreceptor proteins. C) lipids. D) both A and B E) both A and C

Answer: D

Human Physiology, 5e (Silverthorn) CHAPTER 25 Integrative Physiology III: Exercise 1) The role of phosphocreatine is to A) store energy. B) store oxygen. C) release energy. D) transfer energy. E) provide a substrate for gluconeogenesis.

Answer: D

Match each level of protein structure with its description. A. primary B. secondary C. tertiary D. quaternary 63) applies to proteins containing more than one peptide chain

Answer: D

Match the cell type to its tissue or function. A. cartilage B. destroys bone matrix C. loose connective tissue D. fat E. maintains bone matrix 72) adipocyte

Answer: D

Match the change with the cardiovascular event described. A. increased B. decreased C. unchanged D. stopped 138) The blood pressure in a vessel is 10 units at point A and 10 units at point B. Flow between those points is __________.

Answer: D

Match the lung capacity with its description. A. total lung capacity B. inspiratory capacity C. vital capacity D. functional residual capacity 43) the amount of air remaining in the lungs after a normal breath

Answer: D

Match the organelle with its function. A. endoplasmic reticulum B. Golgi apparatus C. lysosomes D. mitochondria E. peroxisomes 64) The "powerhouse" of the cell, ATP is produced here.

Answer: D

Match the specific enzyme to its class. A. oxidoreductase B. hydrolase C. transferase D. ligase E. lyase 70) synthetase

Answer: D

149) If in an acid-base reaction donates two H+, one mole of would equal how many equivalents? 2HSO24- 2HSO24- A) 0.75 B) 1 C) 4 D) 0.5 E) 2

Answer: E

164) A certain reaction pathway consists of four steps. How would decreasing the amount of enzyme that catalyzes the second step affect the process?

Answer: Decreasing the amount of enzyme at the second step would slow down the whole series of reactions because less substrate would be available for the next two steps. The net result would be a decrease in the rate of production of the product and therefore, within a specified time, a decrease in the amount of the product.

59) Loss of pericytes around retinal capillaries is a hallmark of the disease __________.

Answer: Diabetic retinopathy 60) Capillary exchange involving movement between the endothelial cells is called the __________ pathway. (:0)Answer: paracellular 61) Capillary exchange involving movement through the cells is called __________ transport. (:0)Answer: transendothelial 62) The turbulent flow of blood causes a noise called a __________ that can be heard through the stethoscope when taking blood pressure. (:0)Answer: Korotkoff sound 63) The structures that regulate blood flow into single capillaries within a tissue are __________. (:0)Answer: precapillary sphincters 64) The accumulation of fluid in the interstitial space is called __________. (:0)Answer: Edema 65) Receptors that monitor blood pressure are called __________. (:0)Answer: Baroreceptors 66) Compensation for decreased blood volume includes increases in A) sympathetic stimulation to blood vessels. B) sympathetic stimulation of the heart. C) water conservation by the kidneys. D) A and C E) A, B, and C (:0)Answer: E

54) __________ is the amount of heat generated during the digestion of a meal.

Answer: Diet-induced thermogenesis 55) __________ is a condition of insulin deficiency from beta cell destruction. (:0)Answer: Type 1 diabetes mellitus 56) __________ is known as insulin-resistant diabetes. (:0)Answer: Type 2 diabetes 57) The loss of water in the urine due to unreabsorbed solutes is known as __________. (:0)Answer: osmotic diuresis 58) __________ is a test used to measure insulin response. (:0)Answer: Glucose tolerance test 59) The combination of type 2 diabetes, atherosclerosis, and high blood pressure is called __________. (:0)Answer: metabolic syndrome 60) __________ monitor skin temperature and core body temperature. (:0)Answer: Thermoreceptors 61) The production of heat from rhythmic tremors of skeletal muscle is referred to as __________. (:0)Answer: shivering thermogenesis 62) Chemicals known as __________ are fever-producing cytokines that are part of the normal immune response. (:0)Answer: pyrogens 63) A genetic condition in which body temperature becomes abnormally elevated is __________. (:0)Answer: malignant hyperthermia 64) __________ is the direct transfer of heat energy from one object to another. (:0)Answer: Conduction

76) What is a differential white cell count and when is it used?

Answer: Differential white cell count estimates the relative numbers of the five types of white blood cells. It is used by clinicians for diagnosis. See discussion under "Colony-Stimulating Factors Regulate Leukopoiesis." 77) What are the three major steps of hemostasis? (:0)Answer: 1. vasoconstriction 2. temporary blockage of a break by a platelet plug 3. blood coagulation, or formation of a clot that seals the hole until tissues are repaired 78) How would a decrease in the concentration of iron in the blood affect the process of hemostasis? A) Platelet plugs would fail to form. B) Coagulation would proceed more rapidly. C) Coagulation would proceed more slowly. D) Retraction would occur prematurely. E) There would be no effect. (:0)Answer: E

43) The energy content of a food or similar substance is measured by a process called __________.

Answer: Direct calorimetry 44) The __________ represents the minimum resting energy expenditures of an awake, alert individual. (:0)Answer: Basal metabolic rate (or BMR) 45) Measuring oxygen consumption as a way of estimating a person's metabolic rate is a type of __________. (:0)Answer: indirect calorimetry 46) A diet of pure carbohydrate would produce an RQ of __________. (:0)Answer: 1.0 47) The enzyme __________ converts triglycerides into free fatty acids and monoglycerides. (:0)Answer: lipoprotein lipase 48) The lipoprotein most likely to be called "good cholesterol" is __________. (:0)Answer: HDL 49) __________ are lipoproteins that are formed in the intestine to carry lipids into circulation. (:0)Answer: Chylomicrons 50) Lipoproteins that contain large amounts of cholesterol for transport to peripheral tissues are called __________. (:0)Answer: low-density lipoproteins (LDLs) 51) The pancreatic hormone that raises blood glucose concentration is __________. (:0)Answer: glucagon 52) The process of synthesizing glucose from lipids, amino acids, or other carbohydrates is called __________. (:0)Answer: gluconeogenesis 53) The process of glycogen formation is known as __________. (:0)Answer: glycogenesis

63) The three sections of the small intestine, in order according to movement of its contents, are __________, __________, and __________.

Answer: Duodenum, jejunum, ileum 64) Most absorbed nutrients first enter the blood of the __________ system. (:0)Answer: hepatic portal 65) The primary complex carbohydrate ingested and digested by people is __________. (:0)Answer: starch 66) After processing in the stomach, the gastric contents are referred to as __________. (:0)Answer: Chyme 67) Bile salts aid in the digestion of fats by __________ large fat droplets. (:0)Answer: Emulsifying 68) __________ is intestinal gas produced by bacteria in the colon during the metabolism of undigestible carbohydrates. (:0)Answer: flatus 69) The enzyme that digests starch into disaccharides and trisaccharides is __________. (:0)Answer:A mylase 70) Maltose is broken down by maltase into two molecules of __________. (:0)Answer: glucose 71) __________ are tiny droplets of fatty acids, monoglycerides, and bile salts. (:0)Answer: micelles 72) Slow waves originate in modified smooth muscle cells called __________. (:0)Answer: interstitial cells of Cajal 73) The __________ is a "housekeeping function" that sweeps food remnants and bacteria out of the upper GI tract and into the large intestine. (:0)Answer: migrating motor complex

93) Explain the role of exposure to antigens in the development of the immune system in the fetus and in the primary and secondary immune responses in children and adults. How does this compare to the role of innate immunity?

Answer: During development, any lymphocytes bearing receptors that bind to molecules in the fetus are eliminated. Thus the only lymphocytes that persist at birth are those that failed to bind to anything, that is, those that have never encountered an antigen. Once the immune system has matured (early in infancy), primary immune responses develop in response to never-before encountered antigens, and secondary immune responses develop with subsequent exposures to the same antigens. Innate immunity is, at all times, independent of prior antigen exposure.

15) Which statement is TRUE? A) Myosin is a motor protein that converts the chemical bond energy of ATP into the mechanical energy of motion. B) Each myosin molecule acts as an enzyme that both binds ATP and releases its energy. C) The energy released by ATP produces the power stroke by swiveling the myosin head. D) A and B E) A, B, and C

Answer: E

105) Diagram a table explaining the feedback of estrogen on the anterior pituitary during the ovarian cycle.

Answer: During early/mid follicular phase, low levels of estrogen inhibit FSH and LH release from the anterior pituitary. In late follicular phase, rising levels of estrogen stimulate the release of FSH and LH. High levels of estrogen, however, along with progesterone in the luteal phase negatively feed back to the anterior pituitary preventing further release of FSH and LH.

180) Vern, suffering from cardiac arrhythmias, is brought into the emergency room of a hospital. He begins to exhibit tachycardia and as a result loses consciousness. His anxious wife asks you why he has lost consciousness. What would you tell her?

Answer: During tachycardia, the heart beats at an abnormally fast rate. The faster the heart beats, the less time there is between contractions for it to fill with blood again. As a result, over a period of time the heart fills with less and less blood and thus pumps less blood out. The stroke volume decreases, as does the cardiac output. When the cardiac output decreases to the point where not enough blood reaches the central nervous system, loss of consciousness occurs.

101) Most of the volume of an atom is occupied by A) electrons. B) protons. C) neutrons. D) the nucleus. E) empty space.

Answer: E

11) A liposome is A) an additive to creams and lotions. B) a drug-delivery vehicle. C) a type of lipid. D) a structural component of cell membranes. E) A and B

Answer: E

11) In the metabolic reactions of cells, molecules such as vitamin C and biotin act as A) vitamins. B) cofactors. C) coenzymes D) modulators. E) B and C

Answer: E

11) Pendrin is an anion transporter important in the production of A) parathyroid hormone. B) calcitonin. C) TRH. D) TSH. E) thyroid hormone.

Answer: E

110) The fuel molecule all cells in the body can use is A) sucrose. B) starch. C) protein. D) vitamins. E) glucose.

Answer: E

115) Glycoprotein molecules A) act as receptors on the surface of cell membranes. B) function as cell markers. C) are present in the secretions coating the respiratory tract. D) function as hormones from the pancreas. E) A and B

Answer: E

12) Intestinal crypts A) increase the surface area of the mucosa of the small intestine. B) carry products of digestion that will not pass through the walls of blood capillaries. C) produce new cells for the mucosa of the small intestine. D) function in the absorption of nutrients. E) A and C

Answer: E

12) Radioisotopes A) are unstable and emit energy called radiation. B) exist only artificially, using particle accelerators to produce them. C) can be used medically for both diagnosis and treatment of diseases. D) are only harmful and are known to cause different diseases. E) A and C

Answer: E

12) Which cell is incorrectly matched with the characteristic? A) eosinophils - numbers increase with allergies and parasites B) eosinophils - cytotoxic cells may live only 6-12 hours C) basophils - have granules containing histamine and heparin D) neutrophils - most abundant white cell E) neutrophils - most difficult to identify

Answer: E

121) Acetylcholine slows the heart rate by A) increasing ion influx thus increasing the rate of depolarization. B) increasing the permeability to Ca2+. C) increasing the permeability to K+. D) decreasing the permeability to Ca2+. E) C and D

Answer: E

125) Damage to the fovea of the eye would interfere with the ability to A) focus an image. B) regulate the amount of light striking the retina. C) bleach visual pigments. D) see black and white. E) see color.

Answer: E

126) These cells produce a substance and release it, either onto a surface or directly into the blood.

Answer: E

128) Lactate dehydrogenase A) catalyzes the reaction pyruvate . lactate. B) catalyzes the reaction lactate . pyruvate. C) uses NADH to produce NAD+. D) uses NAD+ to produce NADH. E) All of the above statements are true.

Answer: E

128) The ECG of a person suffering from complete heart block would show A) an increased PR interval. B) an inverted P wave. C) no visible T wave. D) a smaller QRS complex. E) more P waves than QRS complexes per minute.

Answer: E

129) If blood pressure doubled at the same time that the peripheral resistance doubled, the blood flow through a vessel would be A) doubled. B) halved. C) 16 times greater. D) 1/16 as much. E) unchanged.

Answer: E

13) Intermediate filaments A) provide the cell with strength. B) stabilize the position of organelles. C) transport materials within the cytoplasm. D) form the neurofilaments in nerve cells. E) all of the above

Answer: E

13) Thrombopoietin is produced in the A) liver. B) kidney. C) spleen. D) bone marrow. E) A and B

Answer: E

13) Which functions are controlled through the autonomic nervous system? 1. blood pressure 2. heart rate 3. water balance 4. temperature regulation A) 1 and 2 B) 1 and 3 C) 1, 2, 3 D) 2, 3, 4 E) 1, 2, 3, 4

Answer: E

130) Inside the matrix of the mitochondria, pyruvate A) is converted to acetyl coenzyme A. B) loses a CO2 to become a two-carbon molecule. C) interacts with a derivative of the vitamin pantothenic acid. D) A and B only E) A, B, and C

Answer: E

134) What happens to the oxygen that we breathe? A) It is exhaled. B) It is incorporated into organic molecules during cellular metabolism. C) It is combined with hydrogen to form water. D) It acts as a coenzyme in the production of ATP. E) A and C

Answer: E

137) If the EDV is 140 mL, which other values are most likely to occur in a healthy, normal person? A) The ESV could be 70 mL and the SV could be 70 mL. B) The ESV could be 90 mL and the SV could be 50 mL. C) The ESV could be 50 mL and the SV could be 90 mL. D) A and B E) A and C

Answer: E

148) 100 mg/dL is a typical blood concentration of glucose. The molecular weight of glucose is approximately 180 daltons. What is the molarity of this solution in millimoles? A) 100 B) 10 C) 0.56 D) 18 E) 5.6

Answer: E

14) The hairpin-shaped segment of the nephron is the A) loop of Henle. B) proximal tubule. C) distal tubule. D) vasa recta. E) minor calyx.

Answer:A

16) Each myosin head has A) a binding site for an ATP molecule. B) a binding site for an actin molecule. C) the ability to swivel when powered by ATP. D) A and B E) A, B, and C

Answer: E

16) Molecules that attract immune cells are known as A) interferons. B) lysozymes. C) antigens. D) opsonins. E) chemotaxins.

Answer: E

16) Phosphate groups may be transferred from one molecule to another during A) hydrolysis reaction. B) addition reaction. C) dehydration reaction. D) exchange reaction. E) B and D

Answer: E

17) A free radical is a A) charged particle. B) molecule with an extra electron. C) molecule with an extra neutron. D) molecule with an extra proton. E) molecule with an unpaired electron.

Answer: E

17) Sensations of prickling pain A) are carried by type A fibers. B) reach the CNS quickly. C) often trigger somatic reflexes. D) result in conscious attention. E) all of the above

Answer: E

17) The components of ribosomes are formed by A) the endoplasmic reticulum. B) Golgi complexes. C) lysosomes. D) mitochondria. E) nucleoli.

Answer: E

17) When an enzyme's activity is destroyed by heat or a change in pH, the enzyme is said to be A) broken. B) toxic. C) conjugated. D) tertiary. E) denatured.

Answer: E

17) Which of the following immune cells are also known as Langerhans cells? A) basophils B) eosinophils C) mast cells D) lymphocytes E) dendritic cells

Answer: E

18) The small paired glands at the base of the penis that produce a lubricating secretion are the A) seminal vesicles. B) prostate glands. C) preputial glands. D) Bartholin's glands. E) bulbourethral glands.

Answer: E

19) At the conclusion of the power stroke, A) inorganic phosphate has been released from the myosin. B) actin has been moved toward the M line. C) ADP is released from the myosin head. D) the myosin head is tightly bound to actin. E) all of the above

Answer: E

19) Atoms in a covalent molecule A) share electrons in single pairs. B) share electrons in double pairs. C) share electrons in triple pairs. D) share electrons singly, never in pairs. E) A, B, or C can occur.

Answer: E

19) Place these terms in the typical sequence in which they appear in the scientific process: experimental data, theory, model, observation, hypothesis, replication. A) experimental data, theory, model, observation, hypothesis, replication B) replication, hypothesis, experimental data, theory, model, observation C) theory, observation, experimental data, hypothesis, replication, model D) observation, replication, model, experimental data, hypothesis, theory E) observation, hypothesis, experimental data, replication, model, theory

Answer: E

2) A polysynaptic reflex has at least __________ in the reflex pathway. A) one synapse B) two synapses C) two neurons D) three neurons E) B and D

Answer: E

2) Ions directly regulated by the kidney include all EXCEPT which of the following? A) Na+ B) K+ C) Ca2+ D) HCO3- E) OH-

Answer: E

2) The endocrine gland that is a modified sympathetic ganglion is the A) thyroid. B) anterior pituitary. C) posterior pituitary. D) adrenal cortex. E) adrenal medulla.

Answer: E

20) Regulation of cross-bridge formation is accomplished by A) tropomyosin. B) troponin. C) calcium ions. D) A and B E) A, B, and C

Answer: E

20) Stretching a myocardial cell A) decreases the force of a contraction. B) allows more Ca2+ to enter. C) increases the force of contraction. D) A and B E) B and C

Answer: E

21) All of these statements about carbohydrates are true except one. Identify the exception. A) Simple sugars include lactose, glucose, and ribose. B) Cellulose is the most abundant carbohydrate on earth. C) Glycogen is a storage polysaccharide made by animal cells. D) Polysaccharides are important both for energy storage and to provide structure to cells. E) Glycogen is important both for energy storage and to provide structure for cells.

Answer: E

22) Microtubules A) are composed of tubulin. B) are hollow, filamentous structures. C) form cilia that aid in cell movement. D) are the largest cytoplasmic fibers. E) all of the above

Answer: E

22) When acetylcholine is released into the neuromuscular junction, A) it binds to receptors that allow Na+ and K+ ions to cross the sarcolemma. B) the muscle membrane depolarizes, creating an end-plate potential (EPP). C) action potentials are triggered, which lead to activation of the contraction cycle. D) A and B E) A, B, and C

Answer: E

23) The brain can use only __________ for energy. A) glucose B) fats C) ketones D) lactate E) A and C

Answer: E

24) An enzyme that transfer phosphates is A) hydrolase. B) lipase. C) lyase. D) ligase. E) kinase.

Answer: E

24) Powerful contractions that occur a few times each day in the colon are called A) segmentation. B) tonic contractions. C) phasic contractions. D) peristalsis. E) mass movements.

Answer: E

24) Which of the following factors increases basal metabolic rate? A) thyroid hormones B) acetylcholine C) insulin D) epinephrine E) A and D

Answer: E

25) Coenzymes are A) derived from vitamins. B) organic molecules. C) organic cofactors. D) A and B only E) A, B, and C

Answer: E

26) In order to cause vasodilation of most vascular smooth muscle, A) acetylcholine combines with nicotinic receptors. B) acetylcholine combines with muscarinic receptors. C) norepinephrine combines with alpha receptors. D) norepinephrine combines with ß1 receptors. E) sympathetic stimulation is removed.

Answer: E

26) Umami A) is another name for salty taste. B) describes the way glutamate and some nucleotides taste. C) signals nutritious, desirable foods. D) A and C E) B and C

Answer: E

26) Which of the following projects aims to catalog the structure and function of all proteins in the body? A) Human Physiome Initiative B) Human Genome Project C) Human Proteome Project D) Human Physiology Initiative E) Human Proteome Initiative

Answer: E

27) A hormone that helps to regulate the sodium ion concentration of the blood is A) cortisol. B) parathormone. C) thymosin. D) somatotropin. E) aldosterone.

Answer: E

27) GLUT4 transporters are A) stored in cytoplasmic vesicles. B) inserted in response to glucagon. C) inserted into the plasma membrane by endocytosis. D) found in adipose and skeletal muscles. E) A and D

Answer: E

27) Nucleic acids are polymers of units called A) amino acids. B) fatty acids. C) bases. D) ribose. E) nucleotides.

Answer: E

27) ________ decreases ovarian follicle sensitivity to FSH. A) Progesterone B) Estrogen C) GnRH D) LH E) Anti-Mullerian hormone

Answer: E

28) Insulin A) stimulates glycolysis. B) stimulates lipogenolysis. C) inhibits gluconeogenesis. D) A and B E) A and C

Answer: E

28) Which of the following statements concerning the hormone atrial natriuretic peptide is FALSE? A) Atrial natriuretic peptide is produced by cells in the heart. B) Atrial natriuretic peptide promotes sodium loss at the kidneys. C) Atrial natriuretic peptide reduces the sensation of thirst. D) Atrial natriuretic peptide suppresses ADH secretion. E) Atrial natriuretic peptide increases aldosterone secretion.

Answer: E

29) According to the rules of complementary base pairing, a nucleotide containing the base cytosine would only pair with a nucleotide containing the base A) thymine. B) adenine. C) uracil. D) cytosine. E) guanine.

Answer: E

29) All of the structures listed below are involved in storage, EXCEPT A) lysosomes. B) peroxisomes. C) secretory vesicles. D) storage vesicles. E) All of the above are involved in cellular storage.

Answer: E

29) If a student inhales as deeply as possible and then blows the air out until he cannot exhale any more, the amount of air that he expelled is his A) tidal volume. B) inspiratory reserve volume. C) expiratory reserve volume. D) minimal volume. E) vital capacity.

Answer: E

29) Which of these results in a REDUCED molecule? A) loss of H+ B) gain of H+ C) loss of electrons D) gain of electrons E) A and D

Answer: E

3) The atomic number of an element directly or indirectly indicates A) the number of protons in one atom. B) the placement of the atom in the periodic table of the elements. C) the number of electrons in one atom. D) the number of neutrons in one atom. E) A, B, and C

Answer: E

3) ________ are also known as the pressure reservoir of the cardiovascular system. A) Veins B) Venules C) Capillaries D) Arterioles E) Arteries

Answer: E

30) Amylases, the enzymes used to digest carbohydrates, are secreted by A) salivary glands into the mouth. B) gastric glands into the stomach. C) the pancreas into the intestine. D) A and B E) A and C

Answer: E

31) Anabolism is a term that describes A) reactions that release energy. B) reactions that require a net input of energy. C) reactions that result in the synthesis of large biomolecules. D) A and C E) B and C

Answer: E

37) Platelets are A) large cells that lack a nucleus. B) small cells that lack a nucleus. C) large cells with a prominent, indented nucleus. D) small cells with a many-shaped nucleus. E) fragments of large cells.

Answer: E

39) Asynchronous recruitment A) helps avoid fatigue during sustained muscle contractions. B) refers to different motor units taking turns maintaining muscle tension. C) makes an entire muscle seem to contract smoothly. D) A and B E) A, B, and C

Answer: E

4) Cholesterol is a precursor for the synthesis of A) progesterone. B) aldosterone. C) estradiol. D) two of the above E) all of the above

Answer: E

4) High carbon dioxide concentration in body fluids is called A) carbonation. B) hyperdioxia. C) hyperoxia. D) hypercarbia. E) hypercapnia.

Answer: E

40) Blood pressure and cardiac output can be altered according to A) body temperature. B) emotional responses. C) blood oxygen levels. D) A and B E) A, B, and C

Answer: E

40) Immunoglobulins that are the first antibodies to be produced in response to infection are A) IgA. B) IgD. C) IgE. D) IgG. E) IgM.

Answer: E

44) For a diagnosis of hypertension, a patient must have A) a systolic pressure above 120 mm Hg. B) a systolic pressure above 140 mm Hg. C) a diastolic pressure above 90 mm Hg. D) a diastolic pressure below 80 mm Hg. E) B or C

Answer: E

45) Caveolae are important because they contain A) voltage-regulated Ca2+ gates. B) chemically regulated Ca2+ gates. C) voltage-regulated Na+ gates. D) enzymes to resist atherosclerosis. E) A and B

Answer: E

47) Epithelia are connected to underlying connective tissues by A) a basement membrane. B) protein filaments embedded in glycoprotein. C) a basal lamina. D) A and B E) A, B, and C

Answer: E

48) When there is a resistance to the movement produced by a given limb muscle, A) gamma motor neurons are inhibited by higher brain centers. B) there is a decrease in the amount of force generated by extrafusal muscle fibers. C) the muscle involved in the movement immediately shortens when stimulated. D) intrafusal fibers receive fewer action potentials from the gamma motor neurons. E) muscle tone is increased and more motor units are recruited to the contraction.

Answer: E

49) Exocrine glands A) may make either mucous or serous secretions. B) release their secretions into the external environment. C) release their secretions through open tubes, called ducts. D) may work as single cells or as a multicellular organ. E) All of the above statements are true.

Answer: E

49) Ten-year-old Tina falls while climbing a tree and lands on her back. Her frightened parents take her to the local emergency room where she is examined. Her knee-jerk reflex is normal and she exhibits a negative Babinski reflex. These results suggest that A) Tina has injured one of her descending nerve tracts. B) Tina has injured one of her ascending nerve tracts. C) Tina has a spinal injury in the lumbar region. D) Tina has a spinal injury in the cervical region. E) Tina suffered no damage to her spinal cord.

Answer: E

52) These cells are related to mast cells, and both release chemical mediators such as histamine and heparin, which aid in the immune response.

Answer: E

54) Most lipids are synthesized in the A) smooth endoplasmic reticulum. B) rough endoplasmic reticulum. C) mitochondria. D) cytosol. E) A and D

Answer: E

54) The letter assigned to this structure stands for the German word for middle; it is the attachment site for the thick filaments.

Answer: E

54) Which type of tissue below has minimal extracellular matrix? A) epithelial B) connective C) neural D) muscle E) A, C, and D

Answer: E

55) Functions of connective tissue include A) establishing a structural framework for the body. B) transporting fluids and dissolved materials. C) providing protection for delicate organs. D) storing energy reserves. E) all of the above

Answer: E

57) Which of these is an inherited disease related to altered protein functions? A) hemophilia B) sickle cell anemia C) cystic fibrosis D) phenylketonuria E) All of the above are related.

Answer: E

58) ECGs A) provide indirect information about the heart function. B) are most useful in diagnosing heart murmurs. C) show the summed electrical potentials generated by all cells of the heart. D) have two major components: waves and nodes. E) A and C

Answer: E

58) Plasma is A) within the circulatory system. B) the fluid portion of blood. C) a subdivision of the ECF. D) sometimes donated instead of blood. E) all of the above

Answer: E

6) At a PO2 of 70 mm Hg and normal temperature and pH, hemoglobin is __________% saturated with oxygen. A) 10 B) 25 C) 50 D) 75 E) over 90

Answer: E

6) Lymphoid tissues include A) thymus. B) bone marrow. C) lymph nodes. D) spleen. E) all of the above

Answer: E

6) The product of the SRY gene is A) testosterone. B) anti-Mullerian hormone. C) dihydrotestosterone. D) 5-a reductase. E) testis-determining factor.

Answer: E

6) Tonic receptors A) are slowly adapting receptors. B) fire rapidly when first activated, then slow and stop firing even with a continuing stimulus. C) are activated by parameters that must be continuously monitored by the body. D) are proprioreceptors, for example. E) A, C, and D

Answer: E

6) Which group of elements makes up more than 90% of the body's mass? A) O, H, Na B) C, Na, K C) O, Ca, H D) Ca, C, O E) O, C, H

Answer: E

6) Which of the following is the sequence of layers from the lumen to the outer wall of the digestive tract? A) serosa, submucosa, mucosa, muscularis externa B) submucosa, mucosa, serosa, muscularis externa C) mucosa, submucosa, serosa, muscularis externa D) submucosa, muscularis externa, serosa, mucosa E) mucosa, submucosa, muscularis externa, serosa

Answer: E

63) Functions of the skin include A) maintaining body temperature. B) preventing dehydration. C) protection from bacteria or other pathogens. D) housing sensors to monitor environmental conditions. E) All of the above are functions of the skin.

Answer: E

67) Each of the following is a true statement, except one. Identify the exception. A) Noncoding segments of mRNA are referred to as introns. B) Alternative splicing allows the cell to construct different proteins from one segment of DNA. C) Segments of mRNA that are retained and used for protein synthesis are called exons. D) Initiation factors for gene activation must bind to the promoter region, allowing the RNA polymerase to bind to DNA, thus allowing transcription to proceed. E) Each of the above statements is true.

Answer: E

68) Regarding the cardiovascular system, the main role(s) of the kidneys is/are to A) restore lost fluid to the blood and therefore raise blood pressure. B) minimize fluid loss from the blood and therefore maintain blood pressure. C) reduce blood volume and therefore reduce blood pressure. D) A and B E) B and C

Answer: E

7) Enzymes act by A) lowering the activation energy of a reaction. B) increasing the likelihood that a reaction will start. C) binding to the reactants and putting them in favorable positions to react. D) A and B only E) A, B, and C

Answer: E

7) The partial pressure of oxygen in arterial blood is approximately __________ mm Hg. A) 40 B) 45 C) 50 D) 70 E) 100

Answer: E

7) The percentage of cells in blood-producing tissues that become white blood cells is about __________%. A) 90 B) 10 C) 50 D) 25 E) 75

Answer: E

7) Which of the following is NOT a function of membrane proteins? A) bind to ligands B) regulate the passage of ions C) act as transport molecules for various solutes D) anchor or stabilize the cell membrane E) produce energy

Answer: E

75) Which of the following drugs would be effective means for lowering plasma cholesterol levels? A) a drug that binds to bile acids and prevents them from being reabsorbed B) a drug that stimulates intestinal cholesterol transport C) a drug that stimulates the enzyme HMG coA reductase D) a drug that adds plant sterols and stanols to the diet E) A and D

Answer: E

75) osteocyte

Answer: E

76) deaminase

Answer: E

76) vibration

Answer: E

80) Brown fat A) is found in infants. B) functions in nonshivering thermogenesis. C) contains a rich vascular supply. D) is innervated by the sympathetic nervous system. E) all of the above

Answer: E

80) Tom suffers from a severe liver disease. Which of the following symptoms would you expect to see as a result of this condition? A) decreased clotting ability B) decreased blood osmotic pressure C) increased levels of bilirubin in the blood D) accumulation of fluid in the tissue spaces of the extremities E) all of the above

Answer: E

83) In the follicular phase of the ovarian cycle, the ovary is A) undergoing atresia. B) forming a corpus luteum. C) releasing a mature egg. D) secreting progesterone. E) maturing a follicle.

Answer: E

83) Relative to skeletal muscle, smooth muscle A) uses less energy to create the same force. B) resists fatigue due to anaerobic metabolism. C) has low oxygen consumption rates. D) A and B E) A, B, and C

Answer: E

85) Hemophilia, color blindness, and Duchenne muscular dystrophy occur more often in males than females. Why? A) These traits are carried only on the X chromosome. B) Since there is no matching gene on the Y chromosome, the traits are expressed more readily in males. C) Testis-determining factor (TDF) causes these diseases. D) These diseases develop due to the lack of estrogen and progesterone. E) A and B

Answer: E

88) binds to mast cells, triggering degranulation

Answer: E

91) If the ovaries were removed from an otherwise healthy 20-year-old female, which of the following would you expect to see? A) increased blood levels of GnRH B) increased blood levels of FSH C) increased blood levels of LH D) cessation of menstruation E) all of the above

Answer: E

Human Physiology, 5e (Silverthorn) CHAPTER 10 Sensory Physiology 1) This sensation involves a simple system with receptors that are neuron dendrites. A) smell B) taste C) balance D) hearing E) pain

Answer: E

Human Physiology, 5e (Silverthorn) CHAPTER 13 Integrative Physiology I: Control of Body Movement 1) The effector in a reflex is the A) control center. B) afferent neuron. C) efferent neuron. D) sensory receptor. E) muscle or gland.

Answer: E

Match each leukocyte to its normal percentage in a differential cell count. A. 50-70% B. 20-40% C. 2-8% D. 1-4% E. < 1% 82) basophils

Answer: E

Match the name of the pathological condition with its description. A. hereditary spherocytosis B. polycythemia vera C. iron-deficiency anemia D. sickle cell disease E. anemia 53) general term for the condition of low hemoglobin in the blood

Answer: E

Match the structures with the accurate description. A. muscle fiber B. fascicle C. sarcolemma D. myofibril E. t-tubules 48) inward extensions of the muscle cell membrane

Answer: E

Match the type of breathing with its description. A. hyperpnea B. hyperventilation C. tachypnea D. dyspnea E. apnea 83) cessation of breathing

Answer: E

118) Describe the development of reproductive organs in a fetus of each gender. Which genes, gene products, and hormones are involved in the sex differences? Which sex-related hormones are circulating in high concentrations in a pregnant woman compared to a nonpregnant woman? Do these hormones cross the placenta (if you are uncertain, consider hormone chemical class relative to membrane permeability to make an educated guess)? Assuming these hormones do cross the placenta, how do they affect the hormonal environment inside the fetus, and how does this relate to fetal reproductive development?

Answer: Each fetus initially has the same bipotential structures. These become distinct testes or ovaries and the internal and external genitals typical of each sex by 10 weeks. In males, the change in bipotential structures begins with expression of the SRY gene on the Y chromosome, which secretes testis-determining factor. Once the testes develop, they secrete androgens and anti-Mullerian hormone, which completes the masculinization of the fetus. The female structures develop in the absence of SRY and consequent hormones. Maternal estrogen and progesterone are very high, and as steroids they easily cross the placenta, creating a very female hormonal environment in a fetus of either gender. This underscores the importance of the male hormones in masculinizing male fetuses.

100) Define, compare, and contrast each term listed, and explain how the terms are related to each other: tendons, ligaments, cartilage, bone.

Answer: Each term is a type or subtype of connective tissue. Tendons attach skeletal muscle to bone, whereas ligaments connect bone to bone. Cartilage and bone together are structurally supportive tissues. See Table 3-5 in the CHAPTER. 101) If an animal cell lacked centrioles, it would not be able to A) maintain its balance. B) synthesize proteins. C) produce DNA. D) metabolize sugars. E) undergo nuclear division. (:0)Answer: E

107) How would a spirometer tracing be different in a person with a collapsed lung? Why? What must occur to restore the respiratory function to normal?

Answer: Each volume would be roughly reduced by half, because only one lung is functioning. The collapsed lung cannot be used for pulmonary ventilation until the damage to the pleura is repaired and the low intrapleural pressure and residual volume is reestablished.

78) List and explain the effects of cortisol.

Answer: Effects include promoting gluconeogenesis, breakdown of skeletal muscle proteins and lipolysis, immune suppression, negative calcium balance, alteration of cerebral function.

131) Sydney Student is trying to memorize chemical structures of every compound his professor has indicated is important to the human body. Explain to him that an easier way is to memorize a few rules of chemical bonding and then figure out the structure of the important compounds, especially the simpler compounds.

Answer: Elements combine to form molecules in predictable ways because of how the outer shell electrons combine between atoms. In most cases, the outer shell will be most stable with a total of eight electrons. This information is easily discernable from the periodic table. Therefore an atom with seven outer shell electrons, such as K, combines very readily with an atom with one outer shell electron, such as Cl; an atom with six outer shell electrons, such as Ca will combine with an atom with two outer shell electrons or with two atoms with one outer shell electron each, and so on. Examples: KCl, CaCl2, CH4.

64) Diagram the reactions and interactions of the renin-angiotensin-aldosterone pathway. What condition is the primary stimulus for its activation?

Answer: Fig. 20-13 diagrams these interactions. The primary stimulus is low blood pressure, detected in several ways.

100) Prior to the discovery of the role of insulin in metabolism and the development of insulin therapy to treat diabetes mellitus, a diagnosis of this disease was always fatal. What do untreated diabetics die from?

Answer: Fig. 22-16 in the CHAPTER provides a good summary of the effects of untreated diabetes mellitus. Metabolic acidosis and circulatory failure from dehydration are the primary triggers of death.

12) Pressure and volume of gas in a container are related to temperature and number of gas molecules. This is known as __________ law. A) the ideal gas B) Boyle's C) Dalton's D) Henry's E) Ohm's

Answer:A

82) The lining of the heart and blood vessels is called __________.

Answer: Endothelium 83) __________ is the extracellular component of connective tissues. (:0)Answer: Ground substance / Matrix 84) The combination of fibers and ground substance in supporting connective tissues is known as __________. (:0)Answer: (extracellular) matrix 85) The fluid substance of blood is called __________. (:0)Answer: plasma 86) The study of tissue structure and function is called __________. (:0)Answer: histology 87) Structures composed of epithelial cells that produce secretions are called __________. (:0)Answer: glands 88) __________ is a tissue that is modified to transmit electrical signals from one cell to another. (:0)Answer: Neural tissue / Nervous tissue 89) __________ secretions are discharged onto an epithelial surface. (:0)Answer: Exocrine 90) __________ secretions are released into interstitial space to diffuse into the blood. (:0)Answer: Endocrine 91) __________ proteins extend all the way across the cell membrane. (:0)Answer: Integral / Transmembrane 92) __________ proteins attach loosely to other membrane proteins or polar regions of phospholipids. (:0)Answer: Peripheral

139) While every level of a protein's structure is important to the function of that protein, which level of structure is most important to the function of enzymes, and why?

Answer: Enzymes and other globular proteins depend upon the three-dimensional shape resulting from the globular folding. Under conditions in which this shape is altered by denaturing agents such as heat, the protein ceases to function, though the primary and secondary structure may be unchanged.

86) During the fight or flight reflex __________ is released from the adrenal medulla and binds to __________ receptors, found on the vascular smooth muscle of heart, liver, and skeletal muscle arterioles and results in (increased/decreased) blood flow to these organs; the same hormone binds to __________ receptors on other tissues, causing (increased/decreased) blood flow to these organs.

Answer: Epinephrine, ß2, increased, a, decreased

134) Cancer is abnormal, uncontrolled cell division. What property of epithelial tissues makes them more prone to develop this condition?

Answer: Epithelial tissues contain a population of dividing cells, which divide at a moderate rate.

158) Explain how the analogy of a ball and a hill is useful for describing the forms of energy to beginning physiology students. Use the synthesis of glucose by plants and the decomposition of glucose by humans as specific examples of the energy hill.

Answer: Every student can relate to the fact that it takes energy for a person to move a ball from a lower height to a higher one, and that the ball will automatically roll down the hill if allowed to, without further input of energy. The substrates CO2 and H2O require energy to be combined into C6H12O6; the energy of the sun provides this energy to "roll" the substrates up the energy hill as they combine. The glucose then contains the potential energy provided by the sun. If an appropriate enzyme is present, the molecule easily decomposes to its components and this energy is transferred into work.

43) How does exercise hyperventilation maintain nearly normal PO2 and PCO2?

Answer: Exercise hyperventilation involves an increase in rate and depth of pulmonary ventilation. The result is an increase in alveolar ventilation in proportion to the level of exercise.

46) List and briefly discuss advantages or enhancements to metabolism brought about through aerobic exercise.

Answer: Exercise lowers the risk of cardiovascular disease, by lowering blood pressure, decreasing plasma triglycerides, and raising plasma HDLs. Exercise can partly reverse type 2 diabetes mellitus, by up-regulating glucose transporters and insulin receptors in skeletal muscle. Moderate, but not strenuous, exercise enhances the immune system. Exercise is inversely correlated with psychological depression.

151) The duration of the action potential is __________ in contractile myocardium, __________ in autorhythmic contractile, and __________ in skeletal muscle.

Answer: Extended: 200+ msec, variable: generally 150+ msec, short: 1-2 msec

51) How can you determine the control value using Fig. 1-1?

Answer: Extrapolation. The control value is the heart rate that corresponds to an epinephrine dose equal to zero.

24) our experiences tell us how to behave; bracing yourself in anticipation would be an example

Answer: F Match the structure to its description. A. muscle spindles B. Golgi tendon organs C. both 25) maintain muscle tone at rest through tonic activity (:0)Answer:A

115) Proteins must first be enzymatically degraded to single amino acids before entering the capillaries of the hepatic portal system. Is this true or false? What is the significance of this? Is absorption of carbohydrates and lipids restricted to monomers? Explain.

Answer: False. Absorption of small peptides allows peptides to provoke food allergies. The transporters for peptides are also responsible for absorption of some drugs. Carbohydrates are digested to monosaccharides because there are no transporters for larger carbohydrates. Lipids are absorbed as cholesterol, fatty acids, and monoglycerides, primarily by simple diffusion because they are lipophilic.

44) __________ allows the body to anticipate a stimulus and begin the response, whereas negative feedback results in the __________ of a response.

Answer: Feedforward, cessation 45) The control of voluntary movement can be divided into what three steps? (:0)Answer: 1. decision making and planning 2. initiating the movement 3. executing the movement 46) A muscle at rest exerts no tension. Is this statement true or false? Explain your (:0)Answer. (:0)Answer: False. Normal muscles maintain a resting tension known as muscle tone. 47) After stretching an intrafusal fiber, the next event is A) an increase in action potentials along the associated sensory neuron. B) a decrease in muscle tension. C) a decrease in muscle tone. D) decreased sensitivity to stretching. E) all of the above (:0)Answer:A

111) Explain the role of experience in the immune system. Given the importance of experience, is a person's immune response determined by the genes or the environment? Explain.

Answer: Fetal development of the immune system involves elimination of lymphocytes that can bind to self cells. Secondary exposures to a given antigen result in a faster and more vigorous immune response due the persistence of memory cells that were created following the first exposure. But the immune response is ultimately genetic because lymphocytes recognize antigens as a result of having specific complementary membrane receptors, and these receptors are proteins whose amino acid sequence and thus 3-D shape are determined by the DNA.

59) __________ are phagocytic cells that are permanent residents of specific tissues and organs.

Answer: Fixed macrophages 60) __________ are fixed macrophages that are found in the central nervous system. (:0)Answer: Microglia 61) __________ are fixed macrophages found in the liver. (:0)Answer: Kupffer cells

118) Using the appropriate formula, what effect does changing the following vessel diameters have on blood flow? A. diameter = 8 mm, decreasing to 6 mm B. diameter = 8 mm, increasing to 12 mm C. diameter = 10 mm, decreasing to 5 mm D. diameter = 10 mm, increasing to 20 mm

Answer: Flow a DP/R and R a 1/r4. A. Radius changes from 4 to 3, 44 = 256, 34 = 81, 256/81 = 3.16, so flow decreases by a factor of 3.16. B. Flow increases by a factor of 5.06. C. Flow decreases by a factor of 16. D. Flow increases by a factor of 16.

157) Distinguish between blood flow rate and blood flow velocity. When an expert in the field uses the term blood flow, does that term usually mean rate or velocity?

Answer: Flow rate is the volume of blood passing by a point per unit time. Velocity is how quickly a given amount of blood passes a point per unit time. Flow rate is usually what the expert is discussing.

58) Explain what is meant by antagonistic control.

Answer: For fine-tuned control over the body's internal state, one autonomic branch may have an excitatory effect on a particular organ while the other branch is inhibitory.

71) A. Ernie, an experienced batter, steps up to the plate, looks off into the distance, and plans where to hit the baseball. He then swings the bat, connects with the ball, sends it into the air, and follows through the swing with his arms, hips, and knees, and shifts his weight from one leg to the other. He watches the ball fly away, and takes off for first base. Explain what areas of the central nervous system are working during each phase of the action outlined, and which step is controlled by each part. B. Continue to integrate this with information from other topics as the action continues: Ernie hears the umpire yell "Infield fly! Batter's out!" He remembers the rules of the game, slows his pace, and leaves the baseline before arriving at first base.

Answer: For part A, see "The Integrated Control of Body Movement" section of the CHAPTER. For part B, students can include information learned in Ch. 9 and Ch. 10.

85) List and explain the functions of the regulatory molecules in a sarcomere, specifying how the on and off positions are controlled.

Answer: For the on position or during contraction, see Fig. 12-9b in the CHAPTER. For the off position or after contraction, see the "Myofibrils Are the Contractile Structures of a Muscle Fiber" section of the CHAPTER.

68) Fred weighs 100 kg. Assume his total blood volume is 7%, his heart pumps his total blood volume within 90 seconds, and his renal blood flow is 20% of his cardiac output. Calculate the volume of blood that flows through Fred's kidneys each minute.

Answer: Fred's blood volume = 100 kg × 0.07 = 7 L = 7000 mL. At least two thirds of this volume must circulate per minute = 7000 mL × 0.67 = 4690 mL/min minimum cardiac output (CO); values up to 5000 mL/min would still be within a normal range; 4690 × 20% = 938 mL/min; 5000 × 20% = 1000 mL/min, thus blood flow to Fred's kidneys approximates 938-1000 mL/min.

71) To perform surgery with a minimum of pain for the patient and hassle for the surgeon, a patient may be administered a general anesthetic to prevent sensation and consciousness, as well as a paralytic to prevent reflexive muscle spasms. From what you have learned of motor control, suggest mechanisms by which a paralytic can prevent muscle contraction. With paralytics, what extra measure must be taken to keep the patient alive? Name such a paralytic agent.

Answer: From Ch. 9, students should remember that somatic motor control involves the motor areas of the cerebral cortex and the basal nuclei, as well as the cerebellum. A drug that affects activity selectively in those areas could act as a paralytic; however, it is difficult to imagine the nature of the selectivity that would be required, given the use of similar neurotransmitters and receptors throughout the brain. These motor control areas of the brain act upon the spinal motor neurons, which release acetylcholine onto skeletal muscle fibers. These fibers have a type of receptor not found on autonomic targets: the nicotinic acetylcholine receptor. The neuromuscular junction is the single point of control of a skeletal muscle by its motor neuron. A drug that interferes with synaptic transmission at this junction could be a paralytic. Because the respiratory muscles are also controlled by nicotinic neuromuscular junctions, artificial respiration must be provided by the surgical team until the paralytic is cleared from the system. Curare is a drug that works this way.curare binds to the nicotinic ACh receptor, preventing the muscles from being activated by ACh.

56) At approximately what age do men begin to show higher systolic blood pressures than women? At what age does this trend reverse?

Answer: From age 10 to 40, male pressures are higher; after age 40, female pressures are higher.

19) several neurons involved in a pathway; can be complex

Answer: G 20) involves skeletal muscles (:0)Answer:A

82) Explain why glucose or other simple sugars should NOT be the sole source of energy in our diets. What is the best approach to meeting energy needs?

Answer: Given that glucose only meets energy rather than other essential nutrient needs, a person consuming only glucose would die of malnutrition. A mixture of fats, proteins, simple sugars, and complex carbohydrates optimizes the storage, nutrient, and caloric needs of the various organ systems. Glucose requires a relatively large amount of water for hydration and a large amount of space for storage. Glycogen is more compact, and fat is the most compact and efficient means of energy storage.

158) Explain why a heart can keep beating after it has been removed from a living body.

Answer: Heart tissue is autorhythmic; thus it does not require stimulation by nerves.

109) An individual who becomes dehydrated will have an elevated hematocrit. Explain how dehydration leads to an elevated hematocrit although blood cell production is normal.

Answer: Hematocrit = packed red blood cell volume/total blood volume. A dehydrated person will have a lower plasma volume, but red blood cell volume will remain constant. This results in an elevated hematocrit and the condition known as relative polycythemia.

98) What is the difference between hemostasis and homeostasis, or are they the same?

Answer: Hemostasis is the process that minimizes blood loss from the body. Homeostasis is the process that maintains normal physiology. Hemostasis is a type of homeostasis.

71) "Glucose, glucose, every where, nor any speck to utilize" is a phrase made up by one of the authors working on this test question. Similar to the irony of not being able to prevent dehydration by drinking sea water, people with untreated diabetes mellitus are unable to prevent starvation despite the large amount of glucose surrounding their cells; as if that isn't bad enough, dehydration is also a problem. Explain why there is glucose in the urine of such people, why glucose is not present in the urine of normal people, and why diabetics become dehydrated.

Answer: Glucose is in the urine simply because the plasma concentration exceeds the number of glucose transporters in the kidney tubules, because plasma glucose is unusually high. Normal individuals are able to reabsorb all the glucose in the filtrate because plasma glucose concentrations are normally low (normal body cells absorb glucose, diabetic cells do not). Glucose in the filtrate raises the osmolarity of the filtrate, which decreases the relative osmotic gradient for reabsorption of water; thus more water is lost in the urine.leading to dehydration.

58) Patrick's urine sample reveals a high concentration of glucose. Is glucose normally present in urine? Suggest two possible mechanisms to explain why the kidney would excrete excess glucose, and what abnormality may underlie those conditions.

Answer: Glucose transporters may be saturated, preventing complete reabsorption of glucose (filtration exceeds reabsorption). This can happen with extremely high glucose ingestion or with untreated diabetes mellitus. A genetic defect resulting in an insufficient number of glucose transporters is another possibility.

93) __________ is a protective layer made up of mostly membrane carbohydrates.

Answer: Glycocalyx 94) Explain the relationship between microtubules, cilia, flagella, centrioles, spindle fibers, and the centrosome. (:0)Answer: Microtubules are a cytoskeletal protein made of tubulin. Microtubules form a major component of the internal scaffolding of the cell. Microtubules can also be assembled into cilia and flagella, which are organelles that produce cell-generated movements, and into centrioles and spindle fibers. Centrioles produce the spindle fibers, which are responsible for changing the position of chromosomes during nuclear division. Centrioles are part of a larger structure known as the centrosome, which also includes a darkly staining material and acts as the cell's microtubule organizing center. 95) List the four major tissue types. Give an example and location of each. (:0)Answer: See Table 3-6 and the "Issues of the Body" section in the CHAPTER. 96) Describe the structure of the cytoskeleton, and list its functions. (:0)Answer: The cytoskeleton consists of actin microfilaments, intermediate filaments, and microtubules and forms a scaffolding throughout the cytoplasm. An interesting feature is that some of the proteins are relatively fixed in position, whereas others can be rapidly assembled or disassembled as necessary. The functions include providing mechanical strength and shape, stabilizing position of organelles, intracellular transport system, functional linkage to other cells and to extracellular space, and cell-generated movements. 97) Define, compare, and contrast each term listed and explain how the terms are related to each other: rough endoplasmic reticulum, smooth endoplasmic reticulum, ribosomes, and Golgi body. (:0)Answer:A ll are structures involved in synthesis of biomolecules. All but ribosomes are membranous structures. Ribosomes may be free or attached to ER, making it rough. See Fig. 3- 12 in the CHAPTER. 98) Sketch a cell membrane. Label at least three components and briefly explain what each one does. (:0)Answer: See Fig. 3-6 and the "Tissues of the Body" section in the CHAPTER. 99) Describe the composition and function of the extracellular matrix. (:0)Answer: In any tissue, the extracellular matrix consists of two basic components.proteoglycans and insoluble proteins. The matrix plays an important role in processes ranging from growth and development to cell death. The matrix aids in cell communication with its environment by attaching to the cell membrane or cytoskeleton.

129) Do you know what kinds of cell markers your red blood cells have? Do you know your own blood type? How are these two pieces of information related?

Answer: Glycoproteins and glycolipids can act as cell surface markers. On blood cells, some of these markers are designated as the blood type. The most common blood typing system is the A- B-O system, usually paired with the rh system, so your blood type may be, for example, B+ or O-. Some of this information is in Ch. 16.

101) What seems to be the most likely hormone that we could use as a male contraceptive pill? Explain.

Answer: GnRH agonists would reduce the secretion of GnRH thus reducing the level of the gonadotropins, FSH and LH, and therefore allowing the testes to stop producing sperm.

91) Therapeutic surgical removal of the thyroid gland was documented more than 1000 years ago. In the 19th century, a surgeon performing that procedure had only about 50% patient survival, and patient death was often unrelated to nonspecific complications such as infection or bleeding. What condition of the thyroid is so easily diagnosed that ancient surgeons would think to remove it? What hormonal problem unrelated to thyroid hormones may account for the 50% mortality of thyroidectomy patients in the 19th century (and probably throughout history)? Why didn't the surgeons anticipate this problem? Why wasn't the mortality closer to 100%?

Answer: Goiters develop from lack of iodine in the diet or from Graves' disease, producing an obvious enlargement in the neck. Loss of parathyroid hormone following thyroidectomy with inadvertent parathyroidectomy caused death in patients due to lack of calcium for excitable cells such as heart and nerve cells. The role of the parathyroid glands was not fully understood in the 19th century. Mortality was not 100% because the amount of parathyroid tissue varies in individuals, and some had parathyroid tissue that was not removed by thyroidectomy.

106) Draw and label a graph showing temporal summation, tetanus, and fatigue in a skeletal muscle.

Answer: Graph should resemble those on Fig. 12-17 in the CHAPTER.

52) Use Table 1-1 to graph the data appropriately.

Answer: Graphs should address the errors in Fig. 1-1.

114) Sketch the graphs of plasma antibody concentration in a primary immune response and a secondary immune response. Explain the significance of the graphs.

Answer: Graphs should resemble Fig. 24-11 in the CHAPTER. In a primary response, antibody is not detectable until several days after exposure to antigen, and peak antibody concentration is lower than it is in a secondary response. The secondary response shows a more immediate and steep increase in concentration. This more rapid and vigorous response indicates that the memory cells retained following the first exposure proliferate more readily.

175) Sketch graphs illustrating the electrical response of auditory hair cells and cochlear neurons to sound. You do not need to be precise on the amplitudes or durations; just show you understand the general trend.

Answer: Graphs should resemble those in Fig. 10-22 in the CHAPTER.

174) Briefly describe the difference between tonic and phasic receptors, and illustrate this difference by sketching graphs showing any type of stimulus (as a rectangular wave), the receptor potentials generated by each type of receptor, and the resulting action potentials in the neurons that respond to each type of receptor. You do not need to put numbers on your axes; rather you are to demonstrate you understand the relative differences in the responses.

Answer: Graphs should resemble those in Fig. 10-8 in the CHAPTER.

130) Sally Student does not understand the differences between ions, isotopes, and free radicals. Assuming she has learned and understood some basic chemistry, what is the likely source of her confusion? Review the definitions of these terms, then make a table or flowchart to help her sort this out.

Answer: Her confusion may arise from the fact that all of these terms describe a structure that has either gained or lost something. An ion is an atom that has gained or lost one or more electrons and thus bears an electrical charge. Ions form when salts dissolve in water and are required for normal cell function. An ion's charge affects both its behavior in solution and its chemical reactivity. An isotope is an atom that has gained or lost one or more neutrons; as neutrons lack a charge, isotopes remain neutral. Some isotopes emit radiation, a type of energy, rendering them both useful and dangerous; compared to ions, they are rare in nature. A free radical is an atom or molecule that has at least one unpaired electron (an electron is more stable if paired with another electron). Free radicals can be either electrically charged (e.g., superoxide) or neutral (e.g., hydroxy), depending upon the total number of protons and electrons present. Because free radicals are unstable, they are highly reactive and disruptive to cell function; compared to ions, they are rare in the body. Neither isotopes nor free radicals are known to be required for normal cell function.

67) A professor gave the following analogy of kidney function to his freshman biology class: "Imagine your desk is a disorganized mess, containing things worth keeping and things you should throw out. Instead of picking through each item one by one and considering if it is to keep or toss, you sweep your arm across your desktop in one smooth motion, knocking the entire contents of the desk into your large trash can. Then you pull the items you want to keep out of the trash can and place them back on your desk." In what ways is this analogy accurate? In what ways is it inaccurate? Some students in his class argue that the kidneys are inefficient and poorly designed. How should the professor respond? What modern medical device may he cite in his response?

Answer: His analogy is accurate in that the glomerulus does dump contents into the nephron that the body needs and will take back. His analogy is inaccurate in that the entire contents of the plasma are not dumped, rather about 1/5 of the volume enters the nephron. While one could easily imagine a more efficient way to remove waste from the blood, the kidneys can only use the tools they have, namely various membrane transport processes and the physical laws governing movement of water and solutes. Kidney dialysis machines rely on diffusion through semipermeable membranes, mimicking part of normal kidney function.

137) Stanley Student is confused on the similarities and differences between proteins and nucleic acids. Assuming he has learned and understood the basic chemistry, what is the likely source of his confusion? To help him sort this out, make a table or flowchart to explain the structure of these molecules and their relationship to each other.

Answer: His confusion probably stems from the fact that both proteins and nucleic acids are classified as macromolecules, and both are assembled by covalently bonding certain monomers in a particular order. Also, nucleic acids contain the information necessary for manufacturing proteins, the term acid is used in describing the structure of both nucleic acids and proteins, and both nucleic acids and proteins must contain nitrogen. The monomer of protein is the amino acid, which has a central carbon, a variable chain denoted as R, and a nitrogen-containing amino group. There are 20 naturally occurring amino acids. The monomer of the nucleic acid is the nucleotide, which has a sugar attached to a nitrogen-containing base, and a phosphate. There are five different bases and two different sugars. The sequence of bases in a DNA or RNA molecule determines the sequence of amino acids in the protein.

114) While a nurse takes a blood pressure measurement on a patient, what determines how much the pressure cuff should be inflated? How would the nurse determine if he inflated the cuff too much? Too little?

Answer: How much to inflate the cuff is determined from what systolic pressure is expected. Therefore, factors such as age and weight, BP history, and suspected cardiovascular pathology are relevant. The higher the cuff pressure, the greater the chance of causing a patient pain. The nurse would not know for certain he had inflated the cuff too much until he opened the valve to reduce pressure, and determined the systolic blood pressure, at which point he could conclude that he inflated the cuff more than necessary. If he had inflated the cuff insufficiently, he would hear sounds associated with flowing blood as soon as he begins to listen through the stethoscope.

153) You have spent the day on the beach, basking in the sun. Why were your cells unable to capture the energy of the sun to do work?

Answer: Human cells lack the appropriate enzymes for converting the light and heat energy of the sun into useful forms of energy.

69) You are a physiologist on a space flight to a distant planet. You discover intelligent humanoid aliens inhabiting the planet, and they willingly submit to your tests. You discover that the alien's kidney handles the sugar alcohol mannitol just like our kidneys handle inulin. You also test the alien for its renal handling of glucose. The data from that experiment are: Plasma level (constant over 24 hrs): 6 g/dL of mannitol and 2 g/dL of glucose; 24 hr. urine sample: volume of 2 L, containing 144 g mannitol and 52 g glucose. A. What is the alien's glomerular filtration rate? B. What is the alien's clearance rate for glucose? C. How does the alien kidney handle glucose?

Answer: Human kidneys filter inulin, a polysaccharide from dahlia roots, but do not reabsorb or secrete it. Thus, 100% of the inulin in a urine sample is filtered inulin. For this reason, the inulin excretion rate is the same as the glomerular filtration rate. A. The alien's GFR = excretion rate of mannitol/plasma concentration of mannitol = 144 g/day × 1 dL/6 g × 0.1 L/1 dL = 2.4 L/day. B. Using 2.4 L/day as GFR, the filtered load for glucose = plasma concentration of glucose × GFR = 2 g/dL ×2.4 L/day × 1 dL/0.1 L = 48 g glucose. C. There were 52 g of glucose produced in 24 hours. Since the clearance of glucose is greater than the filtered load, some glucose must have been secreted. Therefore the alien's kidney filters and secretes glucose.

149) A vaccine has recently been developed that can prevent infection by the human papilloma virus. This vaccine is available to young girls. What disease could this virus prevent? Why may some parents hesitate to have their daughters receive this vaccine?

Answer: Human papilloma virus (HPV) causes some types of cervical cancer. Because HPV is sexually transmitted, some parents may be reluctant to make it available to their young daughters, preferring instead that their daughters abstain from sexual activity until they are older and/or married.

127) Water striders are insects that literally walk on water. These insects are frequently found living on ponds. If hydrogen bonds did not exist, how would this affect the life of water striders?

Answer: Hydrogen bonds are responsible for the surface tension of water, the attractive force between water molecules that can make it difficult to separate them. The surface tension is strong enough to support the weight of water striders thus allowing them to walk on water. If water molecules could not form hydrogen bonds, the water striders would not be able to walk on water because there would be no surface tension to support their weight. Therefore, these insects would have to adapt to terrestrial conditions near ponds or lakes rather than living on them.

75) Define hypertrophy and hyperplasia, and describe their hormone regulation.

Answer: Hypertrophy is increase in cell size, and hyperplasia is increase in cell number. Both are regulated by growth hormone, thyroid hormone, and insulin.

72) Cary deliberately hyperventilates for several minutes before diving into a swimming pool. Shortly after he enters the water and begins swimming, he blacks out and almost drowns. What caused this to happen?

Answer: Hyperventilation causes a decrease in the alveolar PCO2, and more carbon dioxide is eliminated from the body than during normal breathing. The loss of large amounts of carbon dioxide upsets the body's normal drive for ventilation, and Cary does not feel the urge to breathe as he swims. As the exercising muscles use oxygen, a state of hypoxia develops. This results in insufficient amounts of oxygen reaching the brain, causing Cary to lose consciousness.

152) In the autorhythmic cells, the __________ channels open when the cell membrane potential is -60 mV. These channels are permeable to __________ and __________.

Answer: If ; K+, Na+

106) Tom has just been exposed to the measles virus, and he can't remember if he has had measles before. He asks you if there is any way he can tell if he has been previously exposed or if he is going to get sick. What would you tell him? Why won't he get sick from a second infection?

Answer: If Tom has previously had the measles, there should be a significant amount of IgG antibody in his blood shortly after the most recent exposure, as a result of a secondary humoral immune response. If he has not previously had the disease and is in the early stages of a primary response, his blood might show an elevated level of IgM type antibodies. These are the first to be produced in a primary response and are ultimately replaced by IgG antibodies. IgG antibodies are able to mount a more vigorous and immediate response to subsequent exposures, preventing illness.

126) A condition known as lactose intolerance is characterized by painful abdominal cramping, gas, and diarrhea. The cause of the problem is an inability to digest the milk sugar, lactose. How would this cause the observed symptoms? Why is yogurt less likely to trigger the symptoms?

Answer: If an individual cannot digest lactose, then the sugar will pass through to the large intestine in an undigested form. The presence of the extra sugar in the chyme increases the osmolarity of the chyme, resulting in less water being reabsorbed by the intestinal mucosa. The bacteria that inhabit the large intestine can metabolize the lactose, and in the process they produce large amounts of carbon dioxide. The gas overstretches the intestine, which stimulates local reflexes that increase peristalsis. The combination of more fluid contents and increased peristalsis produces the symptom of diarrhea. The overexpansion of the intestine by gas causes the severe pain and abdominal cramping, and of course, the increase in intestinal gas release is directly related to increased gas production by the bacteria. Yogurt contains bacteria that digest the lactose prior to the consumption of the yogurt.

161) Why is it necessary to direct the electrical signals in the heart through the AV node instead of allowing them to spread directly to the atria?

Answer: If electrical signals from the atria were conducted directly into the ventricles, the ventricles would start contraction at the top. The blood would be squeezed downward and would be trapped in the bottom of the ventricles.

66) Explain how an animal with a paralyzing spinal injury can be induced to walk, though it can't walk on its own. Diagram and label the structures involved (Hint: Keep it simple; for example, you could draw a rectangle and label it "right leg muscle" and another for the left, and don't worry about drawing all neurons involved, just enough to show you understand the basics. You may wish to start with a simple sketch of a muscle reflex then add onto it as necessary.)

Answer: If the animal is supported on a moving treadmill, the central pattern generators involved in walking will become active and will produce coordinated contraction and relaxation of limb muscles. Diagram should include a cross section of the spinal cord with an area labeled "central pattern generator," a sensory afferent, and a motor efferent as in any sketch of a reflex.

40) Why is it necessary to space grid marks on a graph proportionally to the quantity measured (example: each square represents one centimeter)?

Answer: If this is not done, a trend would be obscured or even misrepresented.

107) Mrs. Becker was trying to become pregnant. One of the Beckers' favorite ways of relaxation is to soak in the hot tub while drinking wine. A year went by with no pregnancy occurring, so they visit your clinic. You conclude that the hot tub is a possible factor in Juan's infertility. Angered and insulted, both assure you that they have no difficulty having intercourse, unless Mr. Becker has had too much wine, which happens infrequently. How will you explain to them the difference between impotence and infertility? Why should you suspect the hot tub as a factor in Juan's lowered sperm count? Is the wine a factor as well?

Answer: Impotence is the inability to sustain an erection; infertility in this instance is due to the inability to produce mature gametes. These are totally separate events. In this case, the elevated temperature of the hot tub may cause sperm cell production to decline or sperm maturation to cease. Alcohol can inhibit sexual performance; however, a small amount can enhance performance by decreasing inhibition.

119) Place these organs in order of descending percentage of blood flow received at rest: bones and other connective, brain, digestive organs and liver, heart, kidneys, lungs, skeletal muscle, skin.

Answer: Lungs (100%) > liver and digestive (27%) > skeletal muscle (21%) > kidneys (20%) > brain (14%) > bone and other (9%) > skin (5%) > heart (4%).

14) Type I alveolar cells A) allow rapid diffusion of gases through their thin membranes. B) secrete a chemical known as surfactant. C) are phagocytic. D) all of the above E) none of the above

Answer:A

108) Millie's grandfather suffers from congestive heart failure. Whenever she visits him, she notices that his ankles and feet appear to be quite swollen. She knows that you are an avid student of anatomy and physiology, and she asks you why this occurs. What would you tell her?

Answer: In heart failure, the heart is not able to produce enough force to circulate the blood properly. The blood tends to pool in the extremities and as more and more fluid accumulates in the capillaries, the blood hydraulic pressure increases. This results in a fluid shift from the blood to the interstitial space. The fluid accumulation exceeds the ability of the lymphatics to drain it and as a result, edema occurs and produces the obvious swelling.

107) Tom loves to soak in hot tubs and whirlpools. One day he decides to raise the temperature in his hot tub as high as it will go. After a few minutes in the very warm water, he feels faint, passes out, and nearly drowns. Luckily he is saved by an observant bystander. Explain what happened.

Answer: In response to the high temperature of the water, Tom's body shunted more blood to the superficial veins to decrease body temperature. The dilation of the superficial veins caused a shift in blood to the arms and legs and resulted in a decreased venous return. Because of the decreased venous return, the cardiac output decreased and less blood (and less oxygen) was delivered to the brain. This caused Tom to feel light-headed and faint, nearly causing his demise.

93) Blood clotting is considered to be an example of a positive feedback situation. Explain why this is so, and demonstrate the appropriate steps or areas of hemostasis as part of your explanation.

Answer: In the blood clotting process, each step triggers a new step(s), by activating enzymes or putting components together that were previously separated. Some of the products formed "feed back" to enhance earlier reactions, sustaining the cascade until some components are completely consumed.

149) Discuss pain modulation as it relates to the gate control theory.

Answer: In the gate control theory, pain is lessened because there is an integrated response that partially inhibits the stimuli sent up the pain pathway. Aß fibers help block the pain transmission when they synapse on inhibitory interneurons and enhance the inhibitory activity. If the inhibitory neuron is stimulated by both the Aß and the C fibers, the integrated response of decreased pain is sent up the ascending pathway.

157) Carbonic acid will form easily in a student physiology lab, simply by bubbling CO2 into a beaker of water. Suppose it took you and your lab partner 5 minutes to perform this reaction. How would your results differ if you added the enzyme carbonic anhydrase? Explain using the concept of the energy hill.

Answer: In the presence of the enzyme, the reaction will occur much more rapidly. This is because the enzyme lowers the activation energy for the reaction. This is analogous to the ball being rolled up a lower hill before it rolls down, when the enzyme is present.

95) Why is there a loss of blood volume when a patient is confined to bed for two or three days? What symptoms commonly follow this confinement?

Answer: Inactivity causes edema, because of the lack of skeletal muscle pump. The accumulated fluid of edema represents loss from the blood capillaries. This decreased blood volume causes decreased blood pressure, which may give the patient a sense of light-headedness upon standing.

77) Billy has been serving in a noncombat position in the army for two years and has just been informed that he is leaving on a combat mission tomorrow. Billy is feeling very stressed. How has his endocrine system responded to this bad news? Which hormones are involved, and what effects do they have?

Answer: Increased cortisol secretion by the adrenal cortex is stimulated via corticotropin- releasing hormone (CRH) and adrenocorticotropic hormone (ACTH). Cortisol generally has catabolic effects, to protect against hypoglycemia. These effects include gluconeogenesis in the liver, breakdown of skeletal muscle proteins, enhanced lipolysis, immune suppression, and bone loss. Additionally, this discouraging news may be accompanied by a fight-or-flight response, in which increased sympathetic activity will result in increased heart and respiratory rates, increased perfusion of skeletal musculature, and decreased digestive activity.

55) Based on what you learned in the CHAPTER on control of blood flow and pressure, what must happen to blood flow to meet the metabolic demands of exercising skeletal muscle? How is this accomplished? What is the predicted effect of this change on systemic blood pressure during exercise? What are the changes in heart function during exercise, and what effects should those changes have on systemic blood pressure? What actually happens to systemic blood pressure during exercise? Explain the apparent contradictions.

Answer: Increased metabolic demands of skeletal muscle require increased blood flow, which is accomplished through vasodilation. Given the relatively large amount of skeletal muscle in the body, this widespread vasodilation is expected to produce a decrease in systemic blood pressure. Cardiac output increases through an increase in heart rate and stroke volume, which is expected to produce an increase in systemic blood pressure. What actually happens is a slight increase in systemic blood pressure. Part of the predicted decrease from vasodilation is offset by the fact that many other vessels constrict to redistribute blood flow to the dilated vessels. Total peripheral resistance, however, does decrease; the expected decrease in systemic blood pressure is offset by the increase in blood pressure resulting from increased cardiac output. The slight increase seen in systemic blood pressure that actually occurs during exercise indicates a loss of the normal baroreceptor reflex. The reasons for this are not understood, but may involve a change in baroreceptor threshold, presynaptic inhibition of baroreceptor afferent neurons, or influence of an unidentified muscle chemoreceptor.

107) Compare and contrast inflammation and infection.

Answer: Infection implies the presence and active growth of microorganisms, thus an infection could be of a bacterial, viral or fungal origin, with the organisms often provoking an inflammation at the focus of the infection. Inflammation is characterized by redness, swelling, increased temperature, etc., but these symptoms are caused by one's own immune system, either fighting off the pathogen or repairing tissue damage, or both. It is also possible to have inflammation, such as with a sprained ankle, without infection.

164) You work in a paper mill. The smell is horrible every day when you get to work, but by the end of the day, you hardly notice it. Why?

Answer: Olfactory receptors are phasic. They fire rapidly when they are initially exposed to the smell, but they quickly adapt and cease firing when stimulus (smell) remains constant. They adapt to the new steady state and then turn off so that you don't notice the smell after a little while.

165) Some diseases, such as phenylketonuria, classical hemophilia, cystic fibrosis, and muscular dystrophy are a result of enzyme malfunction. Each of these diseases is inherited. Explain how there might be a relationship among these observations.

Answer: Inheritance is determined by the message coded into DNA; a mutation relays a faulty message, which disrupts protein synthesis, resulting in a complete absence of a protein product or production of a faulty protein. Enzymes are proteins. When the instructions for the synthesis of their structure is changed, they may not be able to perform their function, producing a disease condition. If the disease is not fatal prior to puberty and does not render its victim unable to reproduce, the faulty DNA will be copied during gametogenesis and will appear in a predictable percentage of offspring.

94) Compare and contrast innate and acquired immunity.

Answer: Innate immunity is a nonspecific response to foreign invaders or tissue damage and acts as a second line of defense. Acquired (adaptive) immunity has two major methods of response, cell-mediated immunity and humoral immunity, both of which are specific to the pathogen. Cells of the adaptive immune system are responsible for immunological memory.

176) A chronotropic agent is one that affects the heart rate. Compare and contrast this term with an inotropic agent. Give specific examples of each.

Answer: Inotropic agents affect contractility. Catecholamines (epinephrine, norepinephrine) and drugs such as digitalis are positive inotropic agents, meaning they increase contractility. Heart rate is increased by epinephrine (a positive chronotropic agent) and decreased by acetylcholine (a negative chronotropic agent); there is also a variety of drugs that affect heart rate.

86) Why does insulin deficiency lead to metabolic acidosis?

Answer: Insulin deficiency produces a fasted-state metabolism. The liver metabolizes fatty acids for its energy needs, producing ketones. The ketones enter the blood, then are absorbed by tissues and converted to acetyl CoA. Ketones themselves are strong acids, thus ketoacidosis results. Other tissues begin to produce lactic acid, due to their anaerobic state resulting from loss of access to glucose.

95) Describe the roles of interleukins, kinins, and complement in immunity. How does interleukin-1 mediate the immune response?

Answer: Interleukins mediate the inflammatory response, induce fever, stimulate cytokine and endocrine secretion, and promote other aspects of immune response. Kinins are inactive plasma proteins that are part of a cascade producing the end-product bradykinin. Bradykinin promotes inflammation and causes pain. Complement proteins are part of a cascade producing a membrane attack complex that can lyse pathogens. Complement proteins stimulate other aspects of the immune response as well.

114) The youngest female documented to have given birth to a healthy child was a 5-year-old girl in the 1930s; the father was never identified. She began having regular menstrual periods around eight months of age. She was diagnosed as pregnant after being taken to a doctor for suspicion of having an abdominal tumor; her periods had ceased several months earlier and her breasts were well-developed. When do the reproductive organs normally develop? Describe the maturation her body underwent to render her fertile, specifying at what ages these changes normally take place. Even without a positive pregnancy test, what clues suggest pregnancy rather than a tumor? What is the earliest age at which gamete development normally begins and ends in both genders?

Answer: Internal reproductive organs are present as early as six weeks after fertilization. The reproductive organs are gender-distinct by 10 weeks. To be fertile means to have mature reproductive organs and active hormonal control of a menstrual cycle. Final sexual maturation resumes at puberty as hormone production increases, normally between ages 8 and 14. These changes include the onset of menses, breast growth, skeletal changes, and development of axillary and pubic hair. The presence of breasts and history of menstrual cycling were major clues for suspecting pregnancy. Gamete development begins in fetuses of both genders with mitotic divisions in germ cells. Meiosis begins in female fetuses but halts in meiosis I, resuming at puberty and halting in meiosis II, and finishing meiosis only if fertilization occurs. Meiosis begins and is completed in males starting at puberty.

95) Distinguish between intrapulmonary and intrapleural pressure.

Answer: Intrapulmonary pressure is the pressure inside the alveoli. Intrapleural pressure is the pressure within the pleural cavity. Intrapleural pressure variation drives variation inside the alveoli, but is always lower than atmospheric pressure, where pressure inside the alveoli equilibrates with atmospheric pressure during the respiratory cycle.

98) Define and explain the difference between isotonic and isometric contraction and give an example of a specific movement for each. What is the apparent contradiction in the term isometric contraction? How is this type of contraction possible?

Answer: Isotonic means same tension, that is, a phase of muscle contraction that involves movement with a constant tension or force. An example is what the biceps brachii muscle does when flexing the elbow—once tension increases enough to move the weight of the forearm, the forearm moves steadily without further increase in tension. Isometric means same length, that is, a phase of muscle contraction that involves development of force without the muscle actually shortening and moving a load. An example is what the biceps brachii muscle does when pulling on a load too heavy to move—tension increases to the maximum that muscle can produce, but the muscle does not shorten because the load is too heavy to be moved. The contradiction is that contraction means shortening, yet shortening does not occur. Isometric contraction occurs because of series elastic elements that stretch slightly as the sarcomeres contract slightly, but overall muscle length does not change significantly.

90) Jill suffers from anorexia nervosa. One afternoon she is rushed to the emergency room because of cardiac arrhythmias. Her breath has the smell of an aromatic hydrocarbon, and blood and urine samples indicated ketonemia and ketonuria. Why do you think she is having the arrhythmias?

Answer: It appears that Jill is suffering from ketoacidosis as a consequence of her anorexia. Because she is literally starving herself, her body is metabolizing large amounts of fatty acids and amino acids to provide energy and in the process is producing large quantities of ketone bodies (normal metabolites from these catabolic processes). One of the ketones that is formed is acetone, which can be eliminated through the lungs. This accounts for the smell of aromatic hydrocarbons on Jill's breath. The ketones are also converted into keto acids such as acetic acid. In large amounts this lowers the body's pH. As the kidneys excrete excess H+, they reabsorb K+, leading to a state of hyperkalemia. This is probably the cause of her arrhythmias. See the Running Problem throughout the CHAPTER, including its conclusion.

161) "No brain, no pain" is a cute phrase, but is it true that a brain is required to feel pain? Explain. Brain surgery can be performed on awake patients with anesthesia only to the overlying tissues (i.e. brain, no pain). Does this fact indicate that "no brain, no pain" is just a rhyme with no scientific basis? Explain.

Answer: It is true that a brain is required to feel pain. Pain is a perception triggered by potentially damaging stimuli to peripheral receptors. Just as we are deaf or blind if auditory or visual brain areas do not function (or are not present), we feel no pain without brain processing of noxious stimuli. Cutting into the brain without anesthetizing it is possible because the brain lacks the receptors that respond to noxious stimuli, thus there are no painful inputs for the brain to process.

101) What is jaundice? What substance is present in abnormal levels? What can cause jaundice to occur? What difference may impede a doctor's ability to diagnose jaundice between people of European compared to African ancestry?

Answer: Jaundice simply means yellowing of skin and the whites of the eyes. It results when bilirubin levels in the blood become elevated (hyperbilirubinemia); superficial blood vessels allow the imparting of yellow color to skin and eyes. During erythrocyte breakdown, the heme is converted in the spleen and liver to a colored pigment called bilirubin. Bilirubin is carried by plasma albumin to the liver where it is metabolized and incorporated into bile (see Fig. 16-8 in the CHAPTER). Bile is secreted into the digestive tract, and the bilirubin metabolites leave the body in the feces. Small amounts of other bilirubin metabolites are filtered from the blood into the kidneys, where they contribute to the yellow color of the urine. The accumulation of bilirubin can occur from multiple causes. Newborns, whose fetal hemoglobin is degraded and replaced with adult hemoglobin, are particularly susceptible to jaundice, so doctors closely monitor bilirubin levels in the first weeks of life. Another common cause of jaundice is liver disease, where the liver is unable to process and/or excrete bilirubin. People with darker skin don't display the yellowing to the same extent as lighter-skinned people, but the yellowing in the eyes should be about the same.

94) Give three distinct characteristics of erythrocytes. Why is each advantageous to the cell?

Answer: Lack of a nucleus and the biconcave shape contribute to flexibility for moving through narrow capillaries. Thinness of the disk-shaped cell facilitates diffusion of oxygen from the erythrocyte to the cells outside the circulatory system.

62) __________ are fixed macrophages located in the epithelia of the skin.

Answer: Langerhans cells 63) __________ are free phagocytes found in the lungs. (:0)Answer:A lveolar macrophages 64) The ability of certain cells to mobilize in response to changes in their chemical environment is called __________. (:0)Answer: Chemotaxis 65) Substances that can trigger an immune response are known as __________. (:0)Answer:A ntigens 66) __________ are antibodies found in body fluids. (:0)Answer: Immunoglobulins 67) The process by which the surface of a microorganism is covered with antibodies and complement, rendering it more likely to be phagocytized, is called __________. (:0)Answer: opsonization 68) __________ cells enable the immune system to respond more quickly if the same antigen is encountered a second time. (:0)Answer: Memory 69) __________ exists when the immune system does NOT respond to a particular antigen. (:0)Answer: Tolerance 70) __________ are mediators released from white blood cells that regulate the activity of B cells and T cells and enhance nonspecific defense. (:0)Answer: Interleukins 71) __________ are cytokines that make cells resistant to viruses. (:0)Answer: Interferons 72) __________ are groups of lymphocytes with a specific ligand in common. (:0)Answer: Clones

145) Describe how lateral inhibition allows for localization of stimuli.

Answer: Lateral inhibition increases the contrast between the activated receptor field and the nearby inactive fields. If a stimulus activates four primary neurons, each of these will activate their secondary neurons. The secondary neuron that is closest to the stimulus suppresses the response of the other three activated secondary neurons while allowing its own signal to proceed on its pathway without interference. This allows for a greater contrast between the center and sides of the receptive field and a more precise location of stimuli.

95) Low leptin levels have been observed in athletes with chronic negative energy balance. What might be a possible explanation for this? Would you expect neuropeptide Y (NPY) levels to be high? Explain.

Answer: Leptin is believed to provide negative feedback to the hypothalamus and result in decreased food intake. Therefore, in a state where food intake is too low, it would make sense that leptin levels would be low. NPY, however, appears to be a stimulus for food intake. Therefore, in these athletes, we would expect NPY to be high.

98) In males, which cells are the primary targets for LH? What effect does the hormone have on these cells?

Answer: Leydig CELLS; LH causes these cells to secrete testosterone.

156) List and describe the two types of retinal bipolar cells.

Answer: Light-on bipolar cells are inhibited in the dark (due to photoreceptor release of glutamate) and released from this inhibition in the light. Light-off bipolar cells are the opposite. They are activated by the glutamate released in the dark and inhibited in the light. The difference between these cell types is the type of glutamate receptor (excitatory or inhibitory).

138) Proteins are described as having different levels of structure. List and distinguish between the level(s) that produce a linear shape vs. a globular shape, and explain why one of those levels can result in either a linear or globular shape.

Answer: Linear shapes: primary, secondary, quaternary. Primary structure is simply the sequence of covalently bonded amino acids in a peptide chain. Secondary structure is further bonding between nearby amino acids in a peptide chain, with the molecule still retaining a strand-like shape. Quaternary structure can involve separate linear polypeptide chains held together in a strand. Globular shapes: tertiary and quaternary. Tertiary structure involves bonding between distant amino acids, which causes the molecule to be wadded. Quaternary structure occurs when more than one globular peptide chain bonds together.

89) A person who goes from sea level to a city that is 5000 feet above sea level will show an increased hematocrit within 2 to 3 days. Draw the reflex pathway that links the hypoxia of high altitude to increased red blood cell production.

Answer: Low atmospheric at high altitude . low arterial . sensed by kidney . erythropoietin synthesized and released . acts on bone marrow to increase production of red blood cells.

74) Phenelzine is a common MAO inhibitor that has been used to treat depression. It is not, however, prescribed as often as other antidepressants due to its effects on the autonomic nervous system. Describe several side effects that may result from taking phenelzine and explain why they might occur.

Answer: MAO inhibitors prevent the enzymatic breakdown of catecholamines such as norepinephrine. As a result, a higher concentration of norepinephrine remains in the synapse and therefore is able to elicit a longer / stronger response on its target tissues. Consequently, norepinephrine can prolong its activity through sympathetic pathways to increase heart rate and blood pressure, decrease gastric motility causing constipation as well as prevent penile ejaculation leading to sexual dysfunction. This is discussed in "The Autonomic Division" section of the CHAPTER.

116) Calculate the mean arterial pressure (MAP) for each example listed. A. a person with a blood pressure of 120/80 mm Hg B. a person with a blood pressure of 130/95 mm Hg

Answer: MAP = diastolic P + 1/3 (systolic P - diastolic P) A. MAP = 80 + 1/3 (40) = 93 mm Hg B. MAP = 95 + 1/3 (35) = 107 mm Hg

117) Arthur's peripheral resistance has increased by 10%. To prevent his mean arterial pressure from changing, what factor must change to compensate, and by how much?

Answer: MAP a CO x R Arthur's cardiac output will have to decline by 10%.

48) You are designing a study to assess the effects of a new treatment for hypertension. What ethical considerations would you employ when monitoring your progress?

Answer: Major considerations should involve assessing the efficacy of the treatment such that the control group patients are not deprived as well as ensuring that the experimental treatment is not less effective than the standard treatments.

150) Explain how enzymes can be useful in the diagnosis of disease, giving specific examples. Is the change in enzyme concentration a direct or indirect result of the disease?

Answer: Many enzymes are normally present in the blood in a known range of typical concentrations. Alterations in these concentrations can be associated with abnormal conditions. For example, elevation in the digestive enzyme amylase, produced by the pancreas, may indicate pancreatic disease. Elevation of enzymes that normally operate inside cells can indicate tissue damage, such as the elevation of creatine kinase following myocardial infarction. Low amounts of hexosamidase A can indicate Tay-Sachs disease. In the examples given, the amylase and hexosamidase enzyme changes are direct results of the disease, whereas enzymes elevated because of tissue damage may simply result from cell rupture.

99) Draw a concept map illustrating the immune response to bacterial infection.

Answer: Map content should resemble Fig. 24-17 in the CHAPTER.

100) Draw a concept map illustrating the immune response to a viral infection.

Answer: Map content should resemble Fig. 24-18 in the CHAPTER.

95) Make a map using the following terms: bile, bilirubin, ferritin, heme, hemoglobin, intestine, iron in diet, kidney, plasma, spleen, transferrin. You may also add additional terms.

Answer: Maps will vary. See Fig. 16-7 in the CHAPTER.

66) Make a map of the following terms: blood, Bowman's capsule, collecting duct, distal tubule, excretion, external environment, filtration, loop of Henle, lumen, osmosis, peritubular capillaries, proximal tubule, urine, water. Add terms as needed.

Answer: Maps will vary. See Fig. 19-2 and the "Overview of Kidney Function" section of the CHAPTER.

118) Draw a map of the enteric nervous system response and include: stimuli, receptors, integrating centers, efferent path, and tissue response.

Answer: Maps will vary. See Fig. 21-11 in the CHAPTER.

120) Make a map of the following terms: amino acid chief cell D cell ECL cell enteric sensory neuron enteric plexus G Cell gastrin H+ histamine parietal cell pepsin pepsinogen somatostatin vagus nerve Where appropriate, indicate stimulatory or inhibitory effect. Terms may be added as needed.

Answer: Maps will vary. See Fig. 21-26 in the CHAPTER.

64) Ruby is 77 years old and can no longer control her urination. What changed in the reflex pathway that used to control her urination?

Answer: Micturition normally occurs when stretch receptors in the bladder wall are stimulated by distension due to the accumulation of urine. The stretch receptors project to an integration center in the spinal cord, which stimulates parasympathetic neurons of the bladder wall, which stimulates bladder contraction, forcing the internal urethral sphincter to open. At the same time, the spinal control center inhibits somatic motor neurons that control the external urethral sphincter, which relaxes in response, so that urine can flow out. The spinal reflex is normally overridden by descending control from the brainstem and cerebral cortex. The brain inhibits the parasympathetic output to the bladder and stimulates the somatic motor output to the external urethral sphincter, thus preventing urination until a convenient time. It is likely that Ruby's descending control is what is no longer working properly; thus she is not able to suppress the urge until she can get to a toilet.

134) Ionic bonds are considered to be strong chemical bonds. Yet, ions dissociate in water. Explain this apparent contradiction.

Answer: Molecules such as sodium chloride are bonded by ionic bonds. So much energy would be required to separate a molecule of NaCl into Na+ and Cl- that it is practically impossible. That is, if the sodium chloride is dry. Because water molecules have partial charges resulting from their polar covalent bonds, sodium chloride dissociates in water. This means that the ions separate and function relatively independently. An attraction between sodium and chloride still exists, however, and the dissociation can be described as an increase in bond length rather than a loss of the bond. Evaporate the water, though, and the sodium chloride crystals re-form.

123) Explain the general chemical structure for monosaccharides and amino acids.

Answer: Monosaccharides consist of carbon, hydrogen, and oxygen, in the ratio C:H:O of 1:2:1. Amino acids consist of a central carbon (CH), a carboxylic acid (COOH), an amine (NH2), and an organic side chain of variable structure (mainly a hydrocarbon chain, designated as R).

121) Name the proenzymes that the pancreas secretes. Which one is involved in the activation of the other proenzymes? What are their active forms?

Answer: Proenzymes include: trypsinogen, chymotrypsinogen, procarboxypeptidase, procolipase, and prophospholipase. Trypsinogen is converted to trypsin, which activates the other proenzymes. The active forms of the enzymes can be found in Fig. 21-29 in the CHAPTER.

89) Define and describe the mechanism of myogenic autoregulation.

Answer: Myogenic autoregulation is the self-regulation by the vascular smooth muscle in response to an increase in blood pressure. The mechanism responsible for the intrinsic response of vascular smooth muscle is stretch that opens mechanically gated Ca2+ channels in the muscle membrane. Calcium entering the smooth muscle cell combines with calmodulin and activates myosin light chain kinase, which in turn increases myosin ATPase activity and crossbridge activity.

150) The rising phase of the action potential is a result of __________ for skeletal muscle, __________ for contractile myocardium, and __________ for autorhythmic myocardium.

Answer: Na+ entry, Na+ entry, calcium entry

96) Draw the electron-dot shorthand for an atom of sodium. How many covalent bonds can sodium participate in?

Answer: Na; one 97) List and briefly describe the seven categories of soluble proteins. (:0)Answer: The seven categories: enzymes, membrane transporters, signal molecules, receptors, binding proteins, regulatory proteins, and immunoglobulins. See the "Protein Interactions" section of the CHAPTER. 98) Which of the following elements combine to form nonpolar covalent bonds? A) carbon and hydrogen B) nitrogen and hydrogen C) sodium and chlorine D) hydrogen and oxygen E) carbon and chlorine (:0)Answer:A

151) Apoptosis and necrosis are described as the two ways cells die. Which one is "messy"? Which is "tidy"? Explain. Why do these forms of cell death exist? What are some advantages and/or disadvantages of each? Use the lining cells of the digestive tract in an example of an advantageous process.

Answer: Necrosis is a result of damage to cells. The cells swell and rupture, and the enzymes that are released cause damage to other cells in the area.hence the descriptor "messy." Necrosis has the disadvantage of damaging cells that might otherwise have survived unscathed. Necrosis is not an advantageous process, but it is unavoidable because trauma to cells is unavoidable.we all get hurt sometimes. Apoptosis is "tidy" in that the cells do not rupture and release damaging enzymes; instead the cell breaks up into membrane-surrounded pieces that are consumed by immune cells. Apoptosis is advantageous in that it can shape a structure such as fingers and toes during development. In the case of the digestive tract, the harsh chemical environment would lead to frequent necrosis if it weren't for the programmed apoptosis occurring every day or so.

140) Define and distinguish between necrosis and apoptosis, and give specific examples of each.

Answer: Necrosis is cell death as a result of damage from toxins, physical trauma, or lack of oxygen; damaged cells release chemicals that may damage neighboring cells. An example is death of skin cells as a result of sunburn. Apoptosis is programmed cell death, which is an internally regulated process and does not involve neighboring cells unless they too are so programmed. An example is the loss of skin webbing between fingers and toes in a fetus.

99) Myasthenia gravis is a disease characterized by reduction in the number of acetylcholine receptors on skeletal muscles. Which physiological process would be affected? What would someone afflicted by this disease experience? How may this disease be treated?

Answer: Neuromuscular transmission would be affected, and therefore excitation-contraction coupling. Skeletal muscles would be less excitable, and muscle tone and strength would decrease. A treatment that could increase the amount of acetylcholine released or prolong its action, such as a treatment interfering with acetylcholine breakdown, would offer some improvement.

66) What steps are necessary to terminate neurotransmitter action? What would happen if these steps failed?

Answer: Neurotransmitter molecules are either enzymatically degraded or are taken back into the presynaptic terminal (reuptake). Calcium ions are removed from the axon terminal. Failure of these steps prolongs the action of the neurotransmitter. Consequences vary according to the identity of the effector organ. In a skeletal muscle, for example, this can lead to spastic paralysis.

72) At the molecular level, what are the effects of nicotine on the nervous system? How are these effects exerted? How are these effects similar to or different from those of curare? How do these effects explain some of the physiological consequences of smoking?

Answer: Nicotine binds to a type of acetylcholine receptor known as the nicotinic receptor; nicotine does not bind to the other type of acetylcholine receptor, the muscarinic receptor. Like acetylcholine, nicotine activates the receptor and produces a postsynaptic response. Nicotinic receptors are located on skeletal muscles and on the postganglionic neurons of both the sympathetic and parasympathetic nervous divisions. Curare also binds to the nicotinic acetylcholine receptor, but it does not activate the receptor; therefore the effect of curare is paralysis of skeletal muscles. Overall, nicotine is a CNS stimulant, suggesting greater stimulation of the sympathetic division than of the parasympathetic division.

53) Susie has hyperthyroidism. How might this affect ATP production during exercise?

Answer: Normally, only 20-25% of energy released during metabolism is converted to ATP. With hyperthyroidism, the oxidative phosphorylation is uncoupled, resulting in increased heat production. Therefore, this individual will be less efficient in ATP production. (Energy released during metabolism = Heat + ATP production). See the "Temperature Regulation During Exercise" section of the CHAPTER. For information on hyperthyroidism, refer to Ch. 23.

154) List and explain the three types of substances transported by the blood. What is the source of each type?

Answer: Nutrients, water, and gases from the external environment, substances moving from cell to cell, and waste products to be eliminated from the body. Nutrients and water are absorbed from the digestive tract, gases are absorbed from the lungs, cells secrete substances such as hormones to communicate with other cells, and cells produce waste products as a consequence of metabolism.

165) Why does there seem to be a link between smell and memory?

Answer: Olfactory pathways branch off from the olfactory cortex pathway and lead to parts of the limbic system (amygdala and hippocampus) involved with emotion and memory. It is not clearly understood, but because odors are processed through the limbic system, olfactory memories are created. When the smell is encountered again, it acts as a stimulus for both the olfactory pathway as well as the memory pathway (via the limbic system).

12) The mean arterial pressure (MAP) is important because A) it represents the driving pressure for blood flow. B) it reflects the difference in time that systole lasts compared to diastole. C) it forces the practitioner to do math, thus they must pay attention to the values obtained. D) A and B E) A, B, and C

Answer:A

65) Sylvia is suffering from severe edema in her arms and legs. Her physician prescribes a diuretic (a substance that will increase the volume of urine produced). Why might this help to alleviate Sylvia's problem?

Answer: One simple (:0)Answer is that a diuretic reduces the free fluid in the body. A more complex explanation is that increasing the volume of urine produced would decrease the total blood volume of the body. This in turn would lead to a decreased blood hydraulic pressure. Edema is frequently the result of hydraulic pressure of the blood exceeding the opposing forces at the capillaries in the affected area. Depending on the actual cause of the edema, decreasing the blood hydraulic pressure would decrease edema formation and possibly cause some of the fluid to move from the interstitial space back to the blood. Diuresis would also increase the concentration of the proteins in the plasma, contributing to the fluid's movement out of the tissues and into the blood.

151) Based on the facts about chemical equilibria in animal systems, indicate for each summary reaction below if it is reversible or irreversible (don't worry about balancing the reactions). Explain the lack of reversibility. In what kind of system would the reaction(s) you selected as irreversible be reversible? Explain. A. C6H12O6 . CO2 + H2O B. ADP + Pi . ATP C. H2O + CO2 . H2CO3

Answer: Only A is irreversible in animal systems. The products are rapidly removed from the system as soon as they are formed, so they are not recombined. This reaction is reversible in plants, which form and store glucose for food, but also need to use some of the glucose to make ATP.

147) Explain how these statements differ, and identify which one(s) is/are correct. Explain what is wrong with each incorrect statement. A. A chemical reaction is at equilibrium when there is no further change in substrates. B. A chemical reaction is at equilibrium when the amounts of substrate and product no longer change. C. A chemical reaction stops when it reaches equilibrium. D. A chemical reaction is at equilibrium when the amount of substrate equals the amount of product.

Answer: Only B is correct as written. A could be corrected by clarifying that it is concentration or amount of substrates that ceases to change; individual molecules can form or break at any given time, but the overall amount will not change because synthesis will be balanced by degradation. C is incorrect because the reactions continue as long as conditions such as presence of catalysts, favorable temperature, and presence of substrates are maintained. As a given enzyme can both synthesize and degrade, both processes will continue even at equilibrium. D is incorrect because at equilibrium there is no net change in amounts, but it is not necessary for concentrations of substrates and products to be the same as each other.

99) It has been reported that King Henry VIII of England divorced his wives because they were unable to produce a male heir to the throne. Some say the lack of a male heir is also used as justification for marrying additional wives in polygamous societies. From a genetics standpoint, what is wrong with these assumptions?

Answer: Only males can contribute the Y chromosome, thus the father determines the gender of the offspring.

107) Mike has classical hemophilia, a clotting disorder, but his case is considered "mild." Contrary to popular belief, Mike doesn't usually have problematic bleeding episodes related to cutting himself; a more severe problem for him would be brought on by a sprained ankle or by having a full body impact, like falling off a horse and landing hard on the ground, injuries considered "closed." With what you have learned about blood coagulation, can you explain how this could be so? Assuming Mike is lacking factor VIII, which pathways does he depend on?

Answer: Only the intrinsic pathway requires factor VIII for activation; the extrinsic pathway and the common pathway are unaffected by the lack of factor VIII. The extrinsic pathway is activated by tissue damage, so even in a person with hemophilia, these may function at close to normal levels.

104) Why is the scrotum, the sac that contains the testes, located outside of the body?

Answer: Optimum conditions for sperm development require that sperm be kept at 2 to 3 degrees below body temperature.

70) What are oropharynx receptors, which hormone do they suppress, and how is it known that they exist? If a person stranded on a desert island drank sea water to try to quench his thirst, how would this affect the oropharynx receptors?

Answer: Oropharynx receptors inhibit the sense of thirst when they come in contact with cool water. They also inhibit secretion of vasopressin (ADH). They have not been identified anatomically, but presence of even a small amount of cold water in the throat is known to satisfy thirst even if the water is not absorbed and thus does not actually alleviate dehydration. Presence of receptors that inhibit the thirst sensation are the best guess as to how this works. Presumably drinking sea water would temporarily reduce the sense of thirst by the same proposed mechanism.

100) List the steps and briefly explain what happens in orthostatic hypotension. What reflex counteracts it?

Answer: Orthostatic hypotension is the decrease in blood pressure upon moving rapidly from a stable reclining position to standing. While reclining flat, gravity is distributed equally and so is blood. When moving to a standing position, blood initially pools in the lower extremities and cardiac output drops from about 5 L/min to about 3 L/min. Arterial blood pressure then decreases. The baroreceptor reflex should counteract orthostatic hypotension.

146) Briefly define oxidation-reduction reactions. What does it mean for a molecule to be reduced or oxidized? Identify and explain the mnemonic device provided in the text for oxidation and reduction reactions. See if you can make one up yourself.

Answer: Oxidation-reduction reactions occur when either electrons or protons are transferred from one molecule to another. These reactions are important in energy extraction and transfer. The molecule that loses electrons (or gains protons) is oxidized, whereas the molecule that gains electrons (or loses protons) is reduced. OIL RIG (oxidation is loss, reduction is gain) is the device provided in the textbook.

145) Explain the different types of enzymatic reactions.

Answer: Oxidation-reduction reactions transfer electrons or protons between substrates. Hydrolysis-dehydration reactions form or break a chemical bond between organic substrates by removing or adding a water molecule. Addition-subtraction-exchange reactions add, remove, or exchange a functional group among the reactants. Ligation reactions involve synthetases and ATP.

12) Which of the following would make the oxygen-hemoglobin curve shift right? A) increased H+ concentration B) increased pH C) decreased temperature D) decreased CO2 E) none of the above

Answer:A

120) Explain the different types of enzymatic reactions.

Answer: Oxidation-reduction reactions transfer electrons or protons between substrates. Hydrolysis-dehydration reactions form or break a chemical bond between organic substrates by removing or adding a water molecule. Addition-subtraction-exchange reactions add, remove, or exchange a functional group among the reactants. Ligation reactions involve synthetases and ATP. 121) In an equilibrium reaction A) the rate of formation of products equals the rate of formation of reactants. B) increasing the amount of one of the reactants will increase the amount of product formed. C) increasing the amount of one of the products will increase the amount of reactants available. D) removing one of the products will increase the amount of product formed. E) all of the above (:0)Answer: E

66) Explain how oxygen and carbon dioxide are transported in the blood. How does the means of transport relate to the solubility and chemical reactivity of these gases in plasma?

Answer: Oxygen is not highly soluble in water, which is the main component of plasma. Less than 2% is dissolved in plasma, with the remainder bound to hemoglobin. The iron in the heme portion of the molecule can bind up to four oxygen atoms. Oxygen is not chemically reactive in the body. Carbon dioxide is more soluble than oxygen, at about 7% dissolved. Carbon dioxide is chemically reactive, combining with water to form carbonic acid, which then dissociates to bicarbonate and hydrogen ion. Most carbon dioxide is transported in the form of bicarbonate, about 70%, and the remaining 23% binds to amino acids on hemoglobin.

44) The maximal rate of consumption of __________ is used as a measurement of a person's ability to perform endurance exercise and is measured in what units? How is this molecule related to metabolism?

Answer: Oxygen, measured in liters consumed per minute; oxygen is required for aerobic ATP production, and ATP provides the energy for metabolism.

144) Explain why calling receptors for noxious stimuli "pain receptors" is inaccurate. What is the more accepted term for these receptors?

Answer: Pain is a perception, not a type of stimulus. Nociceptors (the correct term) respond to particular types of potentially damaging stimuli, such as abnormally high or low temperatures, or abnormally high pressure.

183) Atropine inhibits parasympathetic activity. Discuss the effects of atropine on the heart and describe common expected side effects.

Answer: Parasympathetic stimulation of the heart has an inhibitory effect, slowing the heart rate. This inhibition is constant under normal circumstances, and without it the heart beats at the rate set by the pacemaker cells, which is higher than normal. Thus atropine, by removing the inhibition, increases heart rate. Other organs affected by atropine include any with muscarinic receptors. For example, atropine would inhibit intestinal motility, resulting in constipation. Atropine also inhibits the pupillary reflex, resulting in dilated pupils.

84) Describe the sources, targets, and primary effects of the hormones of calcium metabolism.

Answer: Parathyroid hormone, calcitonin, and calcitriol are the primary hormones. See Tables 23-5, 23-6, and 23-7 in the CHAPTER.

112) There is a mixture of gases in dry air, with an atmospheric pressure of 760 mm Hg. Calculate the partial pressure of each gas for the examples below: A. 20.7% oxygen, 78.2% nitrogen, 0.4% carbon dioxide B. 45% oxygen, 41% nitrogen, 3% carbon dioxide, 11% hydrogen C. 0% oxygen, 15% nitrogen, 5% carbon dioxide, 1% argon

Answer: Partial pressure = Patm x % gas in atmosphere. A. 157.32 mm Hg oxygen, 594.32 mm Hg nitrogen, 3.04 mm Hg carbon dioxide B. 342 mm Hg oxygen, 311.6 mm Hg nitrogen, 22.8 mm Hg carbon dioxide, 83.6 mm Hg hydrogen C. 76 mm Hg oxygen, 114 mm Hg nitrogen, 38 mm Hg carbon dioxide, 7.6 mm Hg argon

132) Your roommate is not a science major but is interested in science and asks you to verify a rumor he has heard: it is theoretically possible for two people to walk through each other without causing harm. Confirm or refute what he has heard, and explain.

Answer: People are composed of molecules, which are in turn composed of atoms. Each atom is mostly empty space, because the protons, neutrons, and electrons are extremely tiny, and the electrons are relatively distant from each other and from the nucleus. A scientist acknowledges that there is often a finite probability, however small, that a very unlikely event could happen. If all of each person's subatomic particles were aligned just right, they could move through the other person's empty atomic space. This is so unlikely as to be practically impossible, however.

54) Athletes with chronic energy deficit (energy output > energy intake) tend to have high levels of cortisol. Do you expect their immune systems would be better than those who are sedentary? Explain.

Answer: People who exercise moderately have better immune systems than sedentary. However, people who exercise strenuously experience a decrease in immune function. Athletes in chronic energy deficit tend to be strenuous exercisers. In addition, cortisol suppresses the immune system (Ch. 23). Therefore, due to the elevated cortisol levels and the exercise stress, these individuals would likely exhibit a suppressed immune response.

62) Draw a flow chart that shows the components of the reflex in which an increase in blood PCO2 leads to increased ventilation.

Answer: Peripheral chemoreceptors in carotid and aortic bodies detect changes in PCO2 and pH. Central chemoreceptors in the medulla oblongata monitor CO2 in CSF. Either an increase in PCO2 or decrease in pH will stimulate the receptors, which project to a control center in the medulla oblongata. The control center stimulates somatic motor neurons that control the skeletal muscles involved in ventilation. The effect is increased ventilation, which lowers PCO2 by eliminating CO2, so blood pH increases because of this shift. See Fig. 18-19 in the CHAPTER.

68) List, compare, and contrast the locations and stimuli for respiratory chemoreceptors.

Answer: Peripheral chemoreceptors in the carotid and aortic bodies sense changes in oxygen concentration, pH, and PCO2 of the plasma. Central chemoreceptors monitor cerebrospinal fluid composition and respond to changes in the concentration of CO2 in the cerebrospinal fluid.

96) Name the process of red blood cell production, the hormone that controls it, the source of that hormone, and the stimulus for its production. Why was the hormone so difficult to isolate and identify?

Answer: Production of red blood cells is called erythropoeisis. It is stimulated by the kidney- produced hormone erythropoietin, in response to hypoxia. It was difficult to isolate and identify because it is made on demand rather than stored, and thus is not present all the time.

82) __________ are receptors that are normally inactive, but become active for a short time whenever there is a change in the condition they monitor.

Answer: Phasic receptors 83) The __________ detects pain and itch. (:0)Answer: nociceptor 84) Gustatory receptors are clustered in individual __________. (:0)Answer: taste buds 85) Each gustatory cell extends a(n) __________ into the surrounding fluids through a narrow taste pore. (:0)Answer: microvillus 86) Taste organs lie along the sides of epithelial projections called __________. (:0)Answer: papillae 87) Bitter, sweet, and umami ligands bind to __________ membrane receptors to begin taste transduction. Ions indicative of salty and sour tastes enter the taste cell through __________. (:0)Answer: G protein-coupled; ion channels 88) __________ are raised structures formed on the walls of the ampulla of semicircular canals by hair cells. (:0)Answer: Cristae 89) __________ is the fluid contained in the membranous labyrinth. (:0)Answer: Endolymph 90) The __________ is a region of dense bone that surrounds and protects the inner ear. (:0)Answer: Bony labyrinth 91) __________ is a fluid similar to cerebrospinal fluid that flows between the bony labyrinth and the membranous labyrinth. (:0)Answer: perilymph 92) The hair cells of the cochlear duct are located in the __________. (:0)Answer: organ of Corti

138) Give three examples of structures whose formation involves molecular interactions that either increase or decrease contact with water molecules, explaining how they are similar and how they differ from each other. (Hint: They all involve molecules that have both polar and nonpolar portions.)

Answer: Phospholipids have a polar portion that is attracted to water and a nonpolar portion that repels water. The molecules orient in water such that water is excluded from contacting the nonpolar portions. This is seen in: the bilayer arrangement of phospholipids in cell membranes, in which hydrophobic tails are in the middle of the layer; micelles, in which a single layer of phospholipids forms a sphere with the hydrophobic tails in the middle; and liposomes, which are hollow spheres made from phospholipid bilayers that can be filled with water-soluble molecules.

153) Describe pitch and loudness in scientific terms.

Answer: Pitch is a function of the frequency of sound waves, measured in waves per second (hertz). High frequency waves are interpreted as high pitched sounds. Loudness is the perception of sound intensity and varies within each individual. The intensity is a function of wave amplitude, measured on a logarithmic scale (units are decibels). High decibel noises can damage ear receptors resulting in hearing loss.

59) Trace a drop of water through the urinary system, beginning with the plasma in the renal artery.

Answer: Plasma in the renal artery flows into smaller branches, entering the arcuate arteries, which lead to the afferent arterioles, which in turn lead to the glomeruli. Water moves from glomeruli into the nephrons (PCT, loop of Henle, then DCT), into the collecting ducts, then renal calyces and into the renal pelvis, passes into a ureter, then to the urinary bladder; finally, it is expelled as urine through the urethra.

133) Define polar covalent, nonpolar covalent, ionic, and hydrogen bonding. Which of these bonds involves more than one molecule? Which of these bonds is/are important in determining the properties of water? Explain.

Answer: Polar covalent bonds occur within a single molecule that shares electrons unequally; i.e., the constantly orbiting electrons spend more time at some locations and less at others. Nonpolar covalent bonds occur within a single molecule that shares electrons equally; i.e., the probability of an electron occupying a particular location is the same at all locations. Ionic bonds occur within a single molecule in which one atom completely loses an electron to another, causing each to develop an opposite charge; it is this electrical attraction that holds the molecule together. Hydrogen bonding occurs between separate molecules that contain polar covalent bonds; where electrons spend more time the molecule is partially negative, and where electrons spend less time the molecule is partially positive. The ends of different molecules are thus electrically attracted to each other. Water is a polar covalent molecule, with the oxygen end being partially negative and attracted to the partially positive hydrogen portions of other water molecules. Hydrogen-bonding between water molecules is responsible for surface tension and the crystalline structure of ice.

67) The cellular mechanisms for renal handling of H+ and HCO3- involve several membrane transporters; name three.

Answer: Possible (:0)Answers include: 1. apical Na+-H+ exchanger 2. basolateral Na+- HCO3- symport 3. H+- ATPase 4. H+- K+- ATPase 5. Na+- NH4+ antiport

49) In your study of a drug's efficacy in treating hypertension, your subjects are white males, ages 40 to 60 years. Is your study applicable to all people? Explain.

Answer: Possibly but not necessarily. There are gender differences in appropriate therapies because of physiological effects of higher testosterone in males compared to females, for example. Drugs are often not tested in children, and children also have a different hormonal environment than adults (again, sex hormones are a good example, because their levels are low until just before the onset of puberty). There are also racial differences in effectiveness of therapies, and while it is a contentious issue as to whether these represent genetic or socioeconomic influences, they should be considered.

45) Why and how does intracellular K+ become depleted with exercise?

Answer: Potassium exits nerve and muscle cells with each action potential. Action potential rates increase in skeletal motor neurons, and in skeletal and cardiac muscle cells during exercise.

168) What are the primary components of a cardiovascular system? What diffusion-related problem is solved by having a cardiovascular system?

Answer: Primary components are the heart (a pump), the blood, and the vessels, which include arteries, capillaries, and veins. The cardiovascular system solves the problem of inefficient diffusion over large distances by providing a force for the blood and a system of vessels leading to capillaries, which are distributed at a high density throughout all the organs. No living cell is more than a short distance from a capillary.

137) Describe the two general meanings of the term membrane, as used in biology. Which usage indicates layers of cells and which indicates layers of molecules?

Answer: Prior to the use of microscopes, membranes were simply thin, flexible layers of cells that separated large compartments or lined large cavities. When microscopic study of cells allowed visualization of the cell envelope and organelles, the term membrane was additionally applied to thin layers of molecules.

34) The cells that are actively involved in immunological surveillance are the A) NK cells. B) plasma cells. C) B lymphocytes. D) helper T cells. E) suppressor T cells.

Answer:A

96) Children who suffer from protein starvation have bloated bellies of ascites (an accumulation of fluid in the tissue spaces of the abdomen). How would a shortage of protein in the diet cause this condition?

Answer: Protein is needed by the body for proper growth, maintenance, and reproduction. If there is a shortage of protein in the diet, the body must turn to itself for a source of amino acids. One of the first proteins to be dismantled for amino acids is plasma albumin. As the level of albumin in the plasma decreases, so does the osmotic pressure. Fluid that has leaked from the capillaries to the interstitial spaces does not return to the blood because of the decreased plasma osmotic pressure; because of the large vascular network in the abdominal viscera, the majority pools in this region.

120) Colloid osmotic pressure in plasma is fairly constant in a healthy person. How would protein starvation affect this pressure? How may a very high-protein diet affect this pressure? Suppose a person is suffering from an imbalance that causes an increase in colloid osmotic pressure from the typical 25 mm Hg to 30 mm Hg. Calculate the effect on net fluid flow across the capillary. What negative health consequences could result?

Answer: Protein starvation would reduce this pressure whereas excess protein would increase this pressure. Absorption would be affected in this way: pin = pIF - pcap. Assuming pIF = 0 mm Hg, pin = -30 mm Hg, favoring absorption. This is a 20% increase in absorption pressure. This would raise blood volume and blood pressure over normal.

115) Calculate the pulse pressure for each example listed. A. a person with a blood pressure of 120/80 mm Hg B. a person with a blood pressure of 130/95 mm Hg

Answer: Pulse pressure is systolic P minus diastolic P. A. 40 mm Hg B. 35 mm Hg

188) Compare the flow rates (Q) of Tubes one and two, assuming the following parameters. Tube one: DP = 30, r = 3, L = 20. Tube two: DP = 60, r = 4, L = 10.

Answer: Q a DP/(L/r4). Q1 a 30/(20/34) = 121. Q2 a 60/(10/44) = 1535. Flow rate in the second tube is about 12 times higher than that in the first tube.

90) Rani has read how important Ca2+ ions are to muscle contraction and neuron function. Rani, age 25, hates milk and most dairy products, and doesn't think she eats many foods that are sources of calcium, nor does she take supplements. Why don't Rani's muscles and nerves show problems related to calcium deficiency? What problems might Rani encounter later in life if she continues this lifestyle?

Answer: Rani's skeleton contains enough Ca2+ to, under the influence of PTH, release Ca2+ to maintain normal blood levels. The muscles, nerves, and other cells are thus able to obtain sufficient Ca2+. It is also likely that Rani consumes at least small amounts of Ca2+ in her diet. Rani will eventually lose enough bone density to be prone to fractures.

101) __________ is the disappearance rate of reactants or the appearance rate of the products of a reaction.

Answer: Reaction rate 102) The availability of __________ determines if pyruvate continues into the citric acid cycle. (:0)Answer: oxygen 103) Glucose is phosphorylated to __________ with the aid of __________. (:0)Answer: glucose-6-phosphate, ATP 104) Lipids are broken down into __________, which feeds into glycolysis, and __________, which are metabolized to acetyl CoA. (:0)Answer: glycerol, fatty acids 105) In the process of __________ a phosphate group is attached to a molecule. (:0)Answer: phosphorylation 106) Animals extract energy from biomolecules through the process of __________, which consumes oxygen and produces __________ and __________. (:0)Answer: respiration, carbon dioxide, water 107) Energy released by electrons moving through the electron transport system is stored as __________ by H+ ions concentrated in the __________. (:0)Answer: potential energy, intermembrane space 108) The electron transport system is located in the __________ and includes enzymes and iron- containing proteins known as __________. (:0)Answer: inner mitochondrial membrane, cytochromes 109) During the synthesis of proteins, amino acids are assembled in the proper sequence because the tRNA molecules that bring them to the ribosome have a(n) __________ that is complementary to a specific codon in the mRNA. (:0)Answer:A nticodon 110) mRNA is created in a process called __________. It then leaves the __________ and enters the cytosol to direct __________, the assembly of amino acids into protein. (:0)Answer: transcription; nucleus, translation

93) Describe the process of relaxation of a skeletal muscle fiber.

Answer: Relaxation is a passive process, resulting from loss of acetylcholine from the receptors as ACh is broken down and there is no further release. This allows the muscle cell to repolarize, which stops the release of calcium from the sarcoplasmic reticulum (SR). The pumping of calcium back into the SR causes the troponin to block the binding site on actin for myosin; this allows the crossbridges to detach and the cell to relax.

67) List, compare, and contrast the brain's centers for monitoring and controlling respiration.

Answer: Respiratory neurons in the medulla control inspiration and expiration. Neurons in the pons modulate ventilation. The rhythmic pattern of breathing arises from a network of spontaneously discharging neurons. Ventilation is subject to modulation by various chemical factors and by higher brain centers.

159) Create a concept map explaining the basic structure and function of sensory systems, starting with a receptor cell and ending with the primary sensory cortex. Be sure to include the general properties such as receptive fields, stimulus location, and so on. Consider each sensory system in turn and identify exceptions to your basic map, indicating how they are different.

Answer: Results will vary. This is discussed in "The Spinal Cord" section of the CHAPTER. Exceptions to the basic plan include: receptor structure is nonneural for most of the special senses, lack of receptive fields for all special senses except vision, lack of routing through the thalamus in the olfactory pathway, lack of stimulus localization by auditory receptors, hydrogen ion or calcium current depolarization in taste cells, oscillating response in auditory hair cells, cessation of ion influx in photoreceptors in response to light.

116) Draw a cell that secretes bicarbonate into the lumen of the pancreas, selecting transporters from the following list (you may not need them all): chloride channel CFTR channel GLUT4 H+-K+-ATPase Cl- - HCO3- Antiport Cl- - HCO3- Symport K+ leak channel leaky junctions Na+-K+-ATPase Label the cell showing the lumen, interstitial fluid, the basolateral membrane, and the apical membrane.

Answer: See Fig. 21-8 in the CHAPTER.

71) Draw a map of steroid hormone synthesis.

Answer: See Fig. 23-2 in the CHAPTER.

144) You suck milk through a 6-inch straw and through a 10-inch straw; the diameters are identical. Resistance is __________ in the 10-inch straw compared to the 6-inch straw.

Answer:A

78) Diabetes mellitus produces many homeostatic imbalances, including acidosis. The pH imbalance is due to ketoacidosis, which results from excessive accumulation of byproducts of fat metabolism, as the body cannot meet energy needs from carbohydrate metabolism. Sally is a teenaged diabetic who sometimes rebels by not taking her insulin. Her mother takes her to the hospital because her breathing has become deep and gasping. Explain Sally's breathing pattern. What other compensatory responses may occur and would they occur earlier or later than the respiratory response?

Answer: Sally's respiratory system is responding to her ketoacidosis. By hyperventilating, she removes more CO2 from her blood, which shifts the carbonic acid equation to the right, removing free H+ and raising pH. Chemical buffering in the blood occurs immediately after the initial pH disturbance, as this is an ongoing process. The kidneys buffer more slowly, so they are effective later.

93) Sandy has insulin-dependent diabetes mellitus. Sandy has an accident on his bike and breaks his arm. Would his insulin dosage change?

Answer: Sandy will need increased levels of insulin. The stress will lead to increased blood glucose levels, hence additional insulin will be needed to compensate. Insulin will also promote the necessary healing by supporting anabolism.

122) The enteric nervous system and the brain have several similarities. What are they?

Answer: See "The Enteric Nervous System Can Act Independently" section of the CHAPTER.

59) Map the divisions of the autonomic nervous system down to their receptors.

Answer: See Fig. 11-11 in the CHAPTER.

89) Outline and explain the events leading up to smooth muscle contraction, starting with the stimulus for contraction plus modulating conditions or molecules. Specify possible neurotransmitters and receptors. Be sure to include terms such as Ca2+-induced Ca2+ release, voltage-, chemically, and stretch-activated channels where appropriate, and pharmacomechanical coupling.

Answer: See Fig. 12-28 in the CHAPTER.

95) Define and explain the differences between slow wave potentials and pacemaker potentials in smooth muscle fibers.

Answer: See Fig. 12-31 in the CHAPTER.

92) Summarize the major steps of skeletal muscle contraction.

Answer: See Fig. 12-7 in the CHAPTER.

91) Explain the events leading up to a skeletal muscle twitch, starting with the arrival of neurotransmitter in the neuromuscular junction.

Answer: See Fig. 12-7, Fig. 12-9, and Fig. 12-11 in the CHAPTER.

74) Draw two graphs that represent the alpha-gamma coactivation: 1. muscle length as a function of time 2. action potentials of spindle sensory neuron as a function of time

Answer: See Fig. 13-5 in the CHAPTER.

58) Compare and contrast the structures and functions of the Golgi tendon organ and the muscle spindle.

Answer: See Fig. 13-6 in the CHAPTER.

62) Make a map of the integration of muscles reflexes. Include the following terms: brain stem, cerebellum, cerebral cortex, feedback, muscle contraction and movement, sensory receptors, signal, spinal cord, and thalamus.

Answer: See Fig. 13-9 in the CHAPTER.

170) Draw a diagram of the excitation-contraction coupling and relaxation in cardiac muscles.

Answer: See Fig. 14-11 in the CHAPTER.

192) Draw a graph of membrane potential as a function of time for an autorhythmic cell under normal conditions, under sympathetic stimulation, and under parasympathetic stimulation. Label depolarization and hyperpolarization on the graph.

Answer: See Fig. 14-16 in the CHAPTER.

173) List the four types of autorhythmic cells that drive the heartbeat. Draw a sketch of the heart and indicate the location of each type of cell.

Answer: See Fig. 14-18 in the CHAPTER.

90) Compare and contrast the structure and location of arteries, arterioles, capillaries, and veins. How do these differences contribute to the differences in function of each?

Answer: See Fig. 15-2 in the CHAPTER for a summary of structural differences in vessel walls. Arteries contain relatively more smooth muscle, for active regulation of vessel diameter. Large, elastic arteries have more elastic fibers, so that the vessel can stretch and recoil with ventricular contraction and relaxation, thereby minimizing fluctuation in blood pressure. Capillaries are thin and porous, reflecting their role in exchange of substances with the interstitial fluid. Veins contain valves to prevent backflow of blood, which is especially important in leg veins.

114) Draw three graphs depicting how the following change during normal inspiration and expiration: A. alveolar pressure B. intrapleural pressure C. volume of air moved

Answer: See Fig. 17.11 in the CHAPTER.

70) What are the PO2 and PCO2 in the alveoli, artery, peripheral tissue, and veins? Explain why the PO2 and PCO2 change.

Answer: See Fig. 18-3 and the "Gas Exchange in the Lungs and Tissues" section of the CHAPTER.

65) Compare and contrast the pulmonary pathologies that affect alveolar ventilation and gas exchange.

Answer: See Fig. 18-4 in the CHAPTER, which discusses emphysema, fibrotic lung disease, pulmonary edema, and asthma.

112) What are the end products of carbohydrate digestion? What are the enzymes involved; where are they located; and what reactions do they catalyze?

Answer: See Fig. 21-14 in the CHAPTER. Amylases are secreted into the mouth and small intestine, and disaccharidases are located on the intestinal brush border.

119) Diagram peptide absorption from the lumen into the portal vein. Be sure to include transporters and label the lumen, portal vein, apical membrane, and basolateral membrane.

Answer: See Fig. 21-17 in the CHAPTER.

123) Draw a cell that secretes HCl into the lumen of the stomach, selecting transporters from the following list (you may not need them all): chloride channel CFTR channel GLUT4 H+-K+-ATPase Cl- - HCO3- Antiport Cl- - HCO3- Symport Na+-K+-ATPase Label the cell showing the lumen, interstitial fluid, the basolateral membrane, and the apical membrane.

Answer: See Fig. 21-6 in the CHAPTER.

90) Draw and label a typical antibody molecule. Be sure to include the light chain, heavy chain, Fab, Fc, and the hinge region. Identify the antigen-binding site and disulfide bonds.

Answer: See Fig. 24-12 in the CHAPTER.

64) Draw a monosynaptic reflex and a polysynaptic reflex. Label each part of the reflex pathway. Briefly explain how the two reflexes differ.

Answer: See Figs. 13-1 and 13-2 in the CHAPTER. The (:0)Answer should include each component of the reflex pathway as in Fig. 13-1. Monosynaptic reflex has a single synapse between the afferent and efferent neurons; polysynaptic reflexes have two or more synapses.

172) Explain how Ca2+ levels inside myocardial cells are important to cell function.

Answer: See Figs. 14-11 and 14-30 in the CHAPTER.

94) Diagram the components and interactions of the baroreceptor reflex.

Answer: See Figs. 15-22 and 15-23 in the CHAPTER.

91) Draw a map of the baroreceptor reflex, starting with decrease in blood pressure. Indicate on the map the following: stimulus, receptor, afferent pathway, integrating center, efferent pathway, effector, tissue response, systemic response.

Answer: See Figs. 15-22 and 15-24 in the CHAPTER.

82) Describe the hypothalamic-pituitary-endocrine organ pathways for the adrenal cortex and thyroid gland. Do releasing and/or trophic hormones exert effects outside this pathway? Explain.

Answer: See Figs. 23-3 and 23-11 in the CHAPTER. CRH and ACTH receptors have been identified in cells of the immune system and are thought to mediate the association between stress and the immune system.

26) List the key concepts or themes in physiology.

Answer: See Table 1-2 in the CHAPTER.

141) Give three examples of sensory inputs that are processed subconsciously.

Answer: See Table 10-1 in the CHAPTER.

88) Briefly compare and contrast the structure and behavior of cardiac, smooth, and skeletal muscles.

Answer: See Table 12-3 in the CHAPTER.

94) List, compare, and contrast the types of skeletal muscle fibers based on their speed of contraction and resistance to fatigue. Give examples of where each type might be found, and why it is advantageous for each type to occur there.

Answer: See Table 12-3 in the CHAPTER.

55) Design a concept map that shows the types of reflexes, the basis for their classification, and their functions.

Answer: See Table 13-1 and Fig. 13-1 in the CHAPTER.

61) Compare and contrast the three categories of movement described in the text and give an example of each.

Answer: See Table 13-2 in the CHAPTER.

164) Compare and contrast the appearance of cardiac muscle cells and skeletal muscle cells under the microscope. Continue with an explanation of their similarities and differences in the initiation of their excitation-contraction coupling mechanisms. Finally, end your discussion by comparing their metabolic needs.

Answer: See Table 14-3 in the CHAPTER.

171) Compare and contrast the action potentials in the skeletal muscle, contractile myocardium, and autorhythmic myocardium.

Answer: See Table 14-3 in the CHAPTER.

64) Describe the different causes of hypoxia, and give specific examples of the associated conditions.

Answer: See Table 18-2 in the CHAPTER.

76) How does decreased blood pressure affect the following: granular cells, glomerulus, cardiovascular control center, hypothalamus? Indicate if decreased blood pressure directly affects the organ or tissue above or acts through a reflex pathway. If it acts through a reflex pathway, name the reflex.

Answer: See Table 20-1 in the CHAPTER.

102) List or provide a table of the male and female analogs that develop from the following bipotential structures: genital tubercle, urethral folds and groove, labioscrotal swellings, gonad (cortex), gonad (medulla), Wolffian duct, Mullerian duct.

Answer: See Table 26 -1 in the CHAPTER.

155) Explain these types of chemical reactions: combination, decomposition, single displacement, and double displacement.

Answer: See Table 4-2 in the CHAPTER.

169) Compare and contrast cardiac muscle and skeletal muscle.

Answer: See the "Cardiac Muscle and the Heart" section and Table 14-3 in the CHAPTER.

85) Explain how the mutant agouti mouse is proving useful as a model for obesity-related disease.

Answer: See the "Emerging Concepts: Melanocortins and the Agouti Mouse" section of the CHAPTER.

74) __________ are short segments of intestine that alternately contract and relax. They are responsible for __________.

Answer: Segmental contractions; mixing 75) The exocrine portion of the pancreas consists of lobules called __________, which secrete __________. (:0)Answer:A cini, digestive enzymes 76) __________ digests terminal peptide bonds to release amino acids. (:0)Answer: Endopeptidase 77) Bile is produced in the __________ and stored in the __________. (:0)Answer: liver, gallbladder 78) Digestive reflexes originating in the CNS are called __________. (:0)Answer: long reflexes 79) Long reflexes that originate completely outside the digestive system include __________ and __________, which are called __________ reflexes. (:0)Answer: feedforward reflexes, emotional reflexes, cephalic 80) Short reflexes of the digestive system are integrated in the __________. (:0)Answer: Enteric nervous system 81) The primary products of protein digestion are __________, __________, and __________. (:0)Answer: free amino acids, dipeptides, tripeptides 82) Fat digestion is carried out by the enzyme __________, which breaks down triglyceride into __________. (:0)Answer: lipase, two fatty acids and one monoglyceride 83) __________ is a protein cofactor that is secreted by the pancreas and that allows lipases to access fats inside the bile coating. (:0)Answer: Colipase

97) In males, which cells are the primary targets for FSH? What effect does the hormone have on these cells?

Answer: Sertoli CELLS; FSH stimulates secretion of paracrine factors that activate spermatogenesis in germ cells.

124) What are functional groups? List the most common functional groups found in biological molecules.

Answer: Several combinations of atoms that occur repeatedly in biological molecules. See Table 2-2 in the main text.

122) Sympathetic stimulation increases the heart rate by A) increasing ion influx thus increasing the rate of depolarization. B) increasing the permeability only to Ca2+. C) increasing the permeability only to K+. D) decreasing the permeability only to Ca2+. E) C and D

Answer:A

109) Sharon "gets the wind knocked out of her" during a skiing accident in which she attempted a jump and landed hard on her feet before tumbling over. She is conscious but breathing is labored, and she complains of pain and shortness of breath. The first aid-trained ski patrolman that comes to her aid determines that Sharon has no broken bones, nor is CNS injury likely. He places tiny tubes at her nostrils that blow air (with higher than normal percentage of oxygen) into her nose then places her on the snowmobile, and takes her to the first aid tent. Why is Sharon's breathing labored? How does the high-oxygen air help her condition?

Answer: Sharon suffered an impact that increased her intrapleural pressure above normal, possibly by pushing her diaphragm upward, forcing her to expel some of her residual volume. With her alveoli smaller than normal, they are very difficult to inflate. The lack of broken ribs suggests that she does not have a pneumothorax. The air being blown into her nostrils has a higher , which will increase the diffusion into her blood and compensate for her temporarily decreased tidal volume.

101) Misty is 16 and, with her family is visiting her grandmother for Christmas. At the end of a week surrounded by tempting holiday food, Misty weighs herself and sees that she has gained five pounds. She confides to her mother that she ate all of a 1-lb box of chocolates on Christmas day, but didn't overeat in any other way. There are 500 kilocalories per 100 g of chocolate. 1 lb = 454 g. There are 3500 kilocalories in a pound of fat. How many calories did she consume? Does this account for the five pounds she gained? Explain.

Answer: She consumed 454 g of chocolates. 454 g x 500 kcal/100g = 2270 kcal. The chocolates cannot account for more than about 2/3 of a pound. She must have consumed extra calories at her meals and other snacks as well, and she may have exercised less than normal.

125) Cody has always carefully controlled her caloric intake, and is lean. During her unplanned pregnancy, she continued to eat the same, hoping that taking prenatal vitamins and minerals would ensure that her fetus would grow normally. Her only complication during pregnancy was persistent constipation and consequent hemorrhoids. She gained only the minimum weight suggested by her doctor, and delivered a healthy, full-term infant; her pre-pregnancy weight was rapidly reestablished. Propose an explanation for how she managed to gain enough weight, did not violate the law of conservation of mass, and for her only complication. What does this suggest about "eating for two" that some pregnant women do?

Answer: She must have inefficiently absorbed calories prior to her pregnancy, losing many potential calories in her feces. When she became pregnant, one of her physiological changes included an increase in nutrient (and thus calorie) absorption. This would also contribute to her constipation, as her feces retained less water than normal and thus were more difficult to eliminate. It is not possible for her to have gained more weight than she ate.no one can violate the law of conservation of mass. Most obstetricians recommend only a slight increase in calories during pregnancy for this reason, and women who indulge in overeating too often are likely to gain unnecessary body fat and may produce a fetus that is larger than it should be.

171) Your roommate is taking a creative writing class. Worried neither about plagiarism nor organ rejection, she develops a story about a scientist constructing a living creature from the body parts of various human donors. She asks you for some technical details on the "wiring" of the sense organs to the brain, which has come complete with cranial nerves but from a different donor than the eyes, ears, nose, and mouth. You can't resist a practical joke, so you advise her that the scientist should connect cranial nerve VIII to the retina, and cranial nerve II to the cochlea. To accurately describe the creature's response to visual and auditory stimuli, what does your roommate need to know about sensory perception?

Answer: She needs to understand the concept of labeled line coding, as well as the functions of these two cranial nerves. Labeled line coding means that stimulation of any structure along the sensory pathway will result in the same sensation every time. From Table 9-1 in Ch. 9, cranial nerve II is the optic nerve and cranial nerve VIII is the vestibulocochlear. By misconnecting the vestibulocochlear nerve to the retina, the scientist has created a creature that will have auditory sensation whenever the nerve is stimulated by the retina in response to changes in light. Likewise, when the auditory receptors in the cochlea respond to sound and activate the optic nerve, the sensation will be visual. In other words, this poor creature will "see sounds" and "hear sights." While this is science fiction, there is a naturally occurring condition that resembles this mis-wiring, called synesthesia, where the person associates sensations in atypical ways, such as a particular word having a particular visual quality like color.

108) Cancer chemotherapy is designed to destroy cancer cells, but adversely affects healthy tissues as well; thus the oncologist has the difficult task of killing the cancer without killing the patient. What side effects are expected from destruction of blood cells?

Answer: Side effects include anemia from loss of red blood cells, impaired immune function from loss of white blood cells, and impaired coagulation from loss of megakaryocytes.

128) If the dissociation constant of a protein is less than one (Kd < 1), what can you conclude about the affinity of the protein for the ligand?

Answer: Since Kd < 1, you know that [P][L] < [PL]. Therefore, at equilibrium, there is a higher concentration of protein-ligand complex suggesting that the protein has a relatively high binding affinity for the ligand.

104) Nicotine stimulates cholinergic receptors of the autonomic nervous system. Based on this information, how would cigarette smoking affect the cardiovascular system?

Answer: Since cholinergic receptors are found in all of the ganglia of the autonomic nervous system, you would expect nicotine to stimulate both sympathetic and parasympathetic responses in the cardiovascular target tissues. In response to increased sympathetic stimulation the heart would beat faster and more forcefully. At the same time, however, increased parasympathetic stimulation would decrease the blood flow to the heart muscle. The increased heart rate and force of contraction would contribute to increased blood pressure as would vasoconstriction of peripheral blood vessels as a result of sympathetic stimulation.

95) List and briefly explain the function of each portion of the male reproductive tract, in order, from where the sperm cells are first produced to the point where they leave the male's body.

Answer: Sperm originate in the walls of the seminiferous tubules, which compose the bulk of the testes. The sperm mature in the epididymis, a coiled tubular structure on the surface of the testis. Upon ejaculation, the sperm enter the ductus deferens, flow through the prostate gland in the ejaculatory duct, then exit the body in the urethra, which is also a urine excretory structure.

101) During the winter, Brad sleeps in a dorm room that lacks a humidifier for the heated air. In the mornings he notices that his nose is "stuffy," similar to when he has a cold, but after showering and drinking some water, the stuffiness disappears until the next morning. What might be the cause of Brad's nasal condition?

Answer: Since the air that Brad is breathing is not humidified (thus dry), large amounts of moisture are leaving the mucus to humidify the air that is being respired. This makes the mucus tacky and the cilia have difficulty moving. As more mucus is produced, it builds up, forming the nasal congestion in the morning. As Brad showers and drinks fluid, the moisture is replaced and the mucus loosens up and is moved along the proper route as usual. The reason this happens mostly at night is because Brad is probably not getting up frequently to drink water to replace what is being lost to humidify the air.

87) Compare and contrast the structural and functional differences between single-unit and multi-unit smooth muscle fibers, particularly in regard to their ability to vary their force of contraction.

Answer: Single-unit characteristics: individual fibers contract as a unit, due to presence of gap junctions; found in walls of hollow organs; strength depends upon amount of calcium that enters the cell. Multi-unit characteristics: each fiber is controlled by an axon or varicosity; found in few locations, including iris and ciliary body; strength depends upon fiber recruitment.

63) Make a map of the following terms: alpha motor neurons, CNS, contractile fibers, extrafusual muscle fibers, Golgi tendon organs,integrating center, joint receptors, muscle spindles, proprioreceptors, sensory neurons, sensory receptors and somatic motor neurons.

Answer: Skeletal muscle reflexes have the following components: Sensory receptors known as proprioreceptors—the three types are: muscle spindles, Golgi tendon organs, and joint receptors. Sensory neurons carry input from proprioreceptors to the CNS, the integrating center. Somatic motor neurons carry the output signal from the CNS; alpha motor neurons are somatic motor neurons that innervate skeletal muscle contractile fibers, known as extrafusual muscle fibers.

145) During a lab practical, Kevin examines a tissue that is composed of densely packed protein fibers that are running parallel and form a cord. There are few nuclei and no striations, and there is no evidence of other cellular structures. Kevin identifies the tissue as skeletal muscle. Why is Kevin's choice wrong, and what tissue is he probably observing?

Answer: Skeletal muscle tissue is made up of densely packed fibers running in the same direction, but since muscle fibers are composed of cells, they would have many nuclei and mitochondria. Skeletal muscle also has an obvious banding pattern or striations due to the arrangement of the actin and myosin filaments within the cell. Kevin is probably looking at a slide of tendon (dense connective tissue).

107) Sketch a graph of the length-tension relationship in a contracting skeletal muscle, and explain its shape in terms of events in the sarcomere.

Answer: Sketch should resemble Fig. 12-16 in the CHAPTER. Explanation should describe optimal degree of overlap of thick and thin filaments at onset of contraction stimulus, and results of too little and too much overlap at stimulus onset.

110) Contraceptive pills containing estradiol or estradiol plus progesterone are given at programmed doses during the ovarian cycle to prevent follicle maturation and ovulation. Explain how these pills work.

Answer: Slightly elevated levels of estradiol or estradiol and progesterone inhibit both GnRH at the hypothalamus and the release of FSH and LH from the pituitary. Without FSH, primordial follicles do not initiate development, and the endogenous levels of estrogen remain low. An LH surge is necessary for ovulation to occur, and the LH surge is triggered by the peaking of estradiol. If the level of estradiol is not allowed to rise above the critical level, the LH surge will not occur and ovulation will not occur, even if a follicle managed to develop to a stage at which it could ovulate. Any mature follicles would ultimately degenerate, and no new follicles would mature to take their place. Although the ovarian cycle is interrupted, the level of hormones is still adequate to regulate a normal menstrual cycle.

105) Erythropoietin (EPO) was first isolated from the urine of anemic patients who had high circulating levels of the hormone. Despite the presence of the hormone that stimulates red cell production, however, these patients were unable to produce adequate amounts of hemoglobin or red cells. Give some possible reasons that the patients' own EPO was unable to correct their anemia.

Answer: Some other factor that is essential for red blood cell synthesis must be lacking. These would include iron for hemoglobin production, folic acid, and vitamin B12.

108) Zoonotic diseases are transmissable from animals to humans. These include ringworm, a fungus which infects pets and people, plague, a bacterial disease resulting from a flea bite to mammals including humans, and rabies, a viral disease of the central nervous system that affects all mammals and is almost always fatal. On the other hand, the feline immunodeficiency virus has no effect on humans and human immunodeficiency virus has no effect on cats, though each may be deadly to its normal host. Speculate on what may prevent cross-species infection in some cases and allow it in others.

Answer: Some pathogens require conditions that are species specific. In the case of viral infections, it may be that some species lack certain cell-surface markers that viruses bind to before they invade. In the case of a virus such as rabies, the virus may bind to a cell marker that is present in all mammalian neural tissue. Some pathogens may require specific chemical environments. The existence of zoonotic diseases underscores the common features of humans and nonhuman animals.

163) Describe how the specificity of sensory pathways is established.

Answer: Specificity occurs for many reasons. Receptors are sensitive to different and particular types of stimuli. The stimulus must be above a certain threshold or the sensory pathway will not be initiated. Stimuli have varying durations and intensities that are coded for in the frequency of action potentials and the number of receptors activated. The location of the stimulus and its modality are determined by which type of receptors are activated, where they are located, and, in certain cases, the timing of receptor activation. The brain can also tell the exact origin of sensory input because each sensory pathway reaches a region of the cerebral cortex that is dedicated solely to that particular receptive field.

133) Design a concept map for the types of cell junctions and the proteins that compose them.

Answer: This is discussed in the "Tissues of the Body" section of the CHAPTER.

122) endoplasmic reticulum

Answer:A

153) Stem cell research has become a political topic in the last few decades. Explain why the research is being done, why some cells are favored for this research over others, and what the factors are that cause some people not to support this type of research. Are any alternatives available that are less opposed?

Answer: Stem cell therapy may remedy previously untreatable diseases and conditions such as those involving brain and spinal injury. It has become political because one source of stem cells that are highly desirable for research is those removed from human embryos and fetuses. These cells are less differentiated and therefore more likely to be able to yield the specific types of mature cells desired. Many people opposed to abortions of human pregnancies are opposed to the use of human embryos and fetuses as a source of stem cells because these embryos and fetuses are destroyed in the process. Use of umbilical cord blood from live births is less opposed, though these cells are somewhat limited in the types of cells they can produce and therefore may not be valuable for treating as large a variety of diseases and injuries.

157) A single photoreceptor may respond to a single unit of light, a photon. A person, however, cannot see a single photon. Explain.

Answer: Stimulation of a receptor is not sufficient to produce sensation. The signals from a sensory organ are transmitted to different brain areas and processed, and not all signals make it to the conscious brain - that is, a single photon is below the perceptual threshold of the visual system.

105) A package insert for a medication states that "this medicine is a beta receptor stimulant used to treat symptoms of asthma, emphysema and other breathing conditions. Possible side effects include fast heart rate, nervousness, tremors or nausea." Why would these symptoms be expected?

Answer: Stimulation of beta receptors activates the sympathetic division, overriding the parasympathetic division. Increased heart rate, nervousness, nausea, and tremors all accompany sympathetic stimulation.

56) How does the stretch reflex protect a muscle?

Answer: Stretching activates the stretch reflex, which in turn contracts the muscle, thereby preventing overstretching.

104) Sherry is not able to produce surfactant. In order to inhale a normal tidal volume, does her intrapleural pressure have to be higher or lower than for a healthy individual? Explain.

Answer: Surfactant is necessary to reduce surface tension sufficient to prevent small or collapsed alveoli. Sherry needs to inhale more forcefully to get the same tidal volume, so lower intrapleural pressure would help.

96) What is surfactant? Why is it important?

Answer: Surfactant, made by the type II alveolar cells, reduces the surface tension in the fluid in the alveoli, thereby facilitating inflation and inhibiting collapse of the alveoli.

152) List and describe the five taste sensations, and explain the recent evidence that there may be a sixth sensation. What are these tastes related to in terms of keeping the body healthy?

Answer: Sweet (sugar), sour (acid), salty (certain chemical salts), bitter (alkaloid or potentially toxic), and umami (savory: glutamate and some nucleotides) are the five sensations. Studies in rats have identified a membrane receptor in taste pores that binds fats, suggesting a sixth sensation for "fatty." Sweet and umami are associated with nutritious foods (as is fatty), bitter with toxic compounds, salty with the presence of sodium, and sour with the presence of hydrogen ion; the latter two are closely regulated in the body.

63) Describe the major anatomical differences between the sympathetic and parasympathetic branches.

Answer: Sympathetic pathways exit the spinal cord in the thoracic and lumbar regions whereas parasympathetic pathways exit through the cranial and sacral regions. Also, sympathetic ganglia are located near the spinal cord; and therefore, sympathetic pathways typically have short preganglionic fibers and long postganglionic fibers. In contrast, parasympathetic ganglia are located on or near their target tissue; and therefore, parasympathetic pathways have long preganglionic fibers and short postganglionic fibers.

111) Compare and contrast the types of pressure and flow in systemic, pulmonary, glomerular, and peritubular capillaries using simple diagrams or tables. Be as complete and specific as possible. You do not need to provide values for the pressures, but you should indicate the relative strength of the different pressures.

Answer: Systemic and pulmonary blood flow is equal to the cardiac output, which is about 720 L/day, or 0.5 L/min. The glomerular and peritubular capillaries receive about 180 L/day, or 0.13 L/min, which represents about 25% of the cardiac output. Blood pressure (in mm Hg) is highest in the systemic circulation (120/80). Blood pressure in the pulmonary circulation is 25/8, in glomerular circulation is 55, and in peritubular capillaries is 10.

97) List five nutrients important for blood cell synthesis and briefly explain the role of each.

Answer: Table 16-3 in the CHAPTER lists B12, folic acid, and iron and a brief explanation of each one's role. The student savvy in metabolism will also note the need for the amino acids for the protein chain synthesis, and energy nutrients in order to supply adequate ATP for these synthetic reactions to occur, as well as the vitamins necessary for the TCA cycle and other aspects of metabolism, including niacin, riboflavin, and pantothenic acid.

46) Write a teleological explanation for why heart rate increases during exercise. Now write a mechanistic explanation for the same phenomenon.

Answer: Teleological: Heart rate increases because the increased activity of skeletal and cardiac muscles requires increased delivery of blood contents such as oxygen and glucose. Mechanistic: Heart rate increases in response to signals from the brain (pacemaker cells of the heart are stimulated by the nervous system).

97) Describe temporal summation, and explain the difference between temporal summation in a neuron and temporal summation in muscle.

Answer: Temporal summation is the addition of either voltages or contraction forces at one location that occur close to each other in time. In a neuron, temporal summation of PSPs results in a total potential that may or may not bring the membrane potential to the action potential threshold. In a muscle, temporal summation of twitches results in greater contraction force than a single twitch, and if the stimulus frequency is high enough, a sustained contraction with no relaxation is produced.

143) Sketch a short series of simple columnar epithelial cells. Label each of the three different borders. Briefly explain the different kinds of activities that may go on at each border, and tell how their structures and junctions support these functions.

Answer: This is discussed in the "Tissues of the Body" section of the CHAPTER.

105) Skeletal muscle can produce tetanic contraction. What is the function of this type of contraction? Can cardiac muscle produce tetanic contraction? What would be the advantage or disadvantage of this type of contraction to the function of the heart?

Answer: Tetanic contraction is a sustained muscle contraction without relaxation. It allows a muscle to maintain a steady force, as when holding or moving a load, or maintaining a particular body position. Cardiac muscle does not produce tetanic contraction. For the heart to function as a pump, it must relax after each contraction to fill with blood, which it will then pump during its next contraction.

101) Describe the muscle condition called tetanus. Is this a normal or a pathological event? If it is normal, what is the function? If it is pathological, what is the cause? The bacterium Clostridium tetani causes a disease called tetanus or lockjaw; you may have been vaccinated against tetanus, especially if you ever had hospital treatment for a skin wound. Speculate on whether or not the name of this disease is related to the muscle condition and why the disease can be fatal.

Answer: Tetanus is a state of maintained contraction that occurs as a result of increased frequency of stimulation by the nerve, whereby the muscle does not relax between stimuli. Tetanus is a normal event, which allows a muscle to maintain a steady force. The bacterial disease results in maintained contraction; thus it is logical to refer to it as tetanus also. The disease is fatal if respiratory muscles are unable to relax because then breathing would stop.

86) Juan has great hopes that his 12-year-old son will play professional basketball and wants to do anything that might be helpful. He has heard that the administration of testosterone or other androgens might increase his son's height. Alarmed, you tell him that this may be a treatment that would actually limit his future athletic potential. Why?

Answer: The administration of sex hormones may cause a quick increase in height, but could also contribute to the premature closure of the epiphyseal plates, limiting the final height this child may attain.

143) Explain labeled line coding and provide a specific example. What exactly is the "label" and the "line" in your example?

Answer: The area of the brain that is stimulated by activity in a particular receptor always perceives the same sensation, regardless of the true nature of the stimulus. For example, a primary auditory cortex neuron interprets as sound both a sound stimulus and an artificial electrical stimulus to a cochlear neuron. In this example, the label is "sound" and the line is all of the neurons involved in the pathway from cochlea to auditory cortex.

108) With what you have learned about pulmonary ventilation, how would you design an artificial respirator to keep patients with paralyzed respiratory muscles alive? How does artificial respiration by a machine compare to mouth-to-mouth respiration?

Answer: The artificial respirator needs to alternate the pressure it generates between slightly higher than atmospheric pressure and slightly lower than atmospheric pressure. These pressure fluctuations are what the respiratory system normally produces; thus they should be safe for the respiratory structures. The lungs of the patient will passively inflate and deflate in response, and gas exchange at the alveoli will occur. The principle is the same with mouth-to-mouth respiration.

150) Using NaCl to illustrate, explain atomic mass and molecular mass, and an estimated and true mass of one molecule. Your explanation should include the appropriate measurement unit for each, and why the estimated and true masses are not exactly identical.

Answer: The atomic mass is the actual mass of an atom, expressed in atomic mass units (amu) or daltons (Da), where 1 amu = 1.6 × 10-27 kg. Because most of the mass of an atom is contributed by the protons and neutrons, each of which is about 1 amu, the mass can be estimated by counting the number of these particles. The number of protons and neutrons is roughly equal, so the atomic number can simply be doubled to estimate the atomic mass. From the atomic number (see the periodic table) the estimated atomic mass of sodium is 22 amu and of chloride is 34 amu. The mass of a molecule (usually expressed in Da instead of amu) of NaCl is 22 + 34 = 56 Da. From the periodic table, the true weight of Cl is actually closer to 36 amu and of Na is 23 amu, so the true mass of NaCl is closer to 59 Da. There are two reasons why the estimated and actual are not the same: the mass of protons and neutrons is actually greater than one, and the number of neutrons may be more or less than double the number of protons.

152) How many neutrons are in the nucleus of the radioisotope Iodine-131?

Answer: The atomic number gives the number of protons. Therefore, subtract the atomic number of iodine from the radioisotope number and the result is the number of neutrons present in the nucleus. 131 - 53 = 78

102) While you study blood vessel structure in your dorm room, your roommate prepares to bike to his favorite fishing hole. You absent-mindedly watch as he puts on his bike shorts, which look absurdly small in his hands but stretch easily then snap back to fit tightly. His fishing shirt is a nonstretchy cotton weave that fits loosely, with open mesh panels across the back and armpits for ventilation. Considering only the basic structural differences in arteries, veins, and capillaries, which of the types of fabrics resemble which vessels, and why?

Answer: The bike shorts stretch and recoil like elastic arteries, the cotton weave is floppy and nonstretchy like veins, and the mesh is porous like capillaries.

87) Explain how and why abnormal temperatures, either too high or too low, can be dangerous.

Answer: The body's responses to high and low temperature are summarized in Figs. 22-19 and 22-20 in the CHAPTER and discussed in the section "Hyperthermia Includes Heat Exhaustion and Heat Stroke." Complications include severe dehydration, loss of enzyme function, and death. Hypothermia is associated with slowed metabolism and loss of consciousness, and death.

122) Compare/contrast the chemical bonds between adjacent monomers in DNA, and between two strands of DNA.

Answer: The bonds holding monomers together are covalent bonds, between sugar and phosphate molecules. The bonds holding neighboring strands together at the complementary bases are hydrogen bonds.

151) Describe the topographic organization of the somatosensory cortex.

Answer: The cells that respond to sensation from a particular body part are organized as a map of the body surface. Thus, cortex cells that respond to sensation from the elbow are located near cells that respond to sensation from the proximal forearm. Furthermore, the number of cortex cells devoted to a sensation is proportional to the sensitivity of that part—there are many more cells for sensation from fingers and lips than from the back or thighs.

75) You are a scientist who has been hired to write a screenplay for a popular TV series. The plot involves the discovery by a rogue botanist of a plant toxin that interferes with the function of the exchange pump responsible for the chloride shift. This scientist provides the toxin to bioterrorists. What should you write for the coroner to say about symptoms in the poisoning victims?

Answer: The chloride shift is the transport process that occurs in red blood cells and is necessary for normal carbon dioxide transport. In this process, a bicarbonate ion is exchanged for a chloride ion. As a result, the red blood cell loses bicarbonate, which carries a negative charge, but gains a chloride ion, thus maintaining its membrane potential. Losing the bicarbonate prevents the chemical reaction that produces bicarbonate from carbon dioxide and water from reaching equilibrium. Bicarbonate is the most important extracellular buffer in the body. Without the transporter function, bicarbonate will build up inside the red blood cell and will not be maintained in the plasma. With a reduction in plasma bicarbonate, acidosis will result. An increase in pulmonary ventilation is triggered by acidosis and can help reverse this process, but it may be unable to compensate fully for the lack of bicarbonate buffer. Ch. 20 covers some of the physiological consequences of acidosis.

108) This graph was generated by students in a physiology lab. The top trace shows contraction force, the bottom shows the electrical stimulus. On the graph, label a muscle twitch. What is happening to the muscle and why?

Answer: The clearest single twitches are the first few and last few peaks on the myogram. The muscle is being stimulated at an increasing rate, as evidenced by the decreasing time between subsequent stimulus pulses. This caused fusion of the twitches, producing partial then complete tetanus, as well as an increase in peak force produced. Though the stimulus is maintained, the peak force starts to fall as the muscle fatigues. The students then decreased the stimulus frequency, allowing recovery from fatigue, and the muscle was able to contract again.

154) Discuss the structure and function of the cochlea.

Answer: The cochlea is made up of the vestibular duct, cochlear duct, and tympanic duct. The vestibular and tympanic ducts are continuous at the helicotrema (found at the tip of the cochlea). Their fluid, perilymph, is similar to plasma. The cochlear duct is filled with endolymph, which is similar in composition to intracellular fluid and is secreted by epithelial cells in the duct. The cochlear duct contains the organ of Corti, which sits on the basilar membrane and is partially covered by the tectorial membrane. These membranes move in response to fluid waves in the vestibular duct and bend the hair cells found in the organ of Corti. When the hair cells move, the stereocilia bend as well and cause the opening of more ion channels. When the cell depolarizes, voltage gated Ca2+ channels open, neurotransmitters are released at a higher rate, and the primary sensory neuron increases its firing rate sending auditory information to the brain.

115) A study was done in 1998 by Dr. Benjamin Levine to explore the role of altitude training in athletic performance. First he trained athletes at low altitude, then measured their finish times in a 5 kilometer run. Then the athletes were placed into one of the following groups: I, living and training at high altitude; at high altitude and training at low; or III, living and training at low altitude. The athletes were then retested in another low-altitude 5 K race. In this study, what are the control and experimental data? What are the dependent and independent variables? The results are expressed as average change in speed of each group. I: -3.4 sec. II: +13.4 sec. III: -26.7 sec. Graph and interpret the results. Did the results surprise you? Why or why not? Propose some explanations for the lower performance by the live high-train high group. Why wasn't there a live low-train high group?

Answer: The control data are the running times of the athletes after baseline training at low altitude. The experimental data are the running times of the athletes after the additional training at various altitudes. The independent variable is altitude; the dependent variable is running time. A bar graph showing the baseline and postaltitude training results as a pair of bars for each group would be appropriate. The best training regimen was living high and training low, and the worst was living low and training low. Many would probably predict the best performance by the group that both lived and trained high, because they were constantly in a relatively hypoxic environment, which may have produced the greatest increase in hematocrit. Training at high altitude clearly did not allow for the best performance, possibly because the lower oxygen environment limited the ability to train at maximum potential. Living low and training high would likely show the worst performance, because even sedentary living at high altitudes produces an increase in hematocrit; only training at high altitude would be insufficient to produce an altitude-related increase.

80) The graph below has had its labels removed. One axis is plasma osmolarity. The other axis is EITHER plasma vasopressin concentration OR plasma aldosterone concentration. Write the correct label on each axis of the graph, and explain how you came to that conclusion.

Answer: The dependent variable is typically used for the y-axis, and the independent variable for the x-axis. Plasma osmolarity is the independent variable, and it regulates the secretion of hormones. As osmolarity increases, vasopressin secretion increases to promote water retention, but aldosterone secretion is inhibited to promote sodium loss. Therefore vasopressin concentration is the correct label for the y-axis. See Fig. 20-8 in the CHAPTER.

67) Diagram and label the knee-jerk reflex. What is the physiological function of this reflex? Explain how this reflex may be important during walking, if you didn't notice a dip in the sidewalk and stepped into it. What is the role of reciprocal inhibition? How would the reflex be affected if reciprocal inhibition failed? Describe the effects on the reflex of severing each structure involved in the reflex, considering one structure at a time. Describe the effects of damaging the opposite side of the spinal cord, or areas higher or lower on the spinal cord.

Answer: The diagram should resemble Fig. 13-7 in the CHAPTER. The function of the reflex is to control movement at the knee joint. If you stepped down farther than you expected, your opposite knee may bend more than it would have, activating the reflex and straightening that leg to prevent you from falling. Reciprocal inhibition allows muscles opposing extension of the leg to be inhibited. If this inhibition failed, leg extension would not occur; instead, the knee would be "locked." Severing the afferent or efferent nerve, the spinal cord at the level of the reflex, or the muscle would all prevent the reflex from occurring. Damage to the opposite side of the spinal cord or areas above or below the cells involved in the reflex should have no effect on the basic reflex. [Note to instructor: In reality, it is not so simple, but the point is to get the students to consider the basic mechanics of the reflex.]

63) Describe the problems that result in low arterial oxygen content.

Answer: Three categories of problems are inadequacies in oxygen reaching alveoli, oxygen exchange between the alveoli and blood, and transport of oxygen in the blood.

34) The number of mitochondria in skeletal muscle cells is __________ adipose (fat) cells. A) greater than B) less than C) equal to

Answer:A

106) Josie is in her last weeks of pregnancy. During a routine check-up, she asks why she is suddenly experiencing swollen feet when she wears knee-high stockings (with a tight elastic band just below the knee). She is confused because an elderly relative wears support stockings to prevent swollen feet, an apparent contradiction. Speculate on a logical explanation for her swelling feet and the difference between support hose and knee-high stockings.

Answer: The elastic band in one of Josie's stocking is so tight that it constantly compresses her veins just in the area under the elastic. This reduces venous return, increasing pressure below that area and causing fluid to exit her blood. Support stockings are medical devices that apply uniform pressure along their entire length, and so produce no constriction. They increase pressure slightly, continuously aiding venous return much like a muscle pump does intermittently.

109) In a condition known as endometriosis, endometrial cells are present in the uterine tubes and/or in the peritoneal cavity where they become established tissues. A major symptom of endometriosis is periodic pain. Why do you think this occurs?

Answer: The endometrial cells have receptors for the hormones estrogen and progesterone and respond to these hormones the same as they would if they were in the body of the uterus. Under the influence of estrogen, they proliferate at the beginning of the menstrual cycle and begin to develop glands and blood vessels, which then further develop under the control of progesterone. This dramatic change in tissue size and characteristics interferes with neighboring tissues by pressing on them or interrupting function in other ways. It is this interference that causes the periodic painful sensations.

152) What is the common ultimate source of energy for all life on earth? Which types of organisms capture this energy? How do humans acquire this energy?

Answer: The energy comes from the sun and is captured by plants. Humans eat plants and/or animals that have eaten plants; thus the energy is transferred to us.

148) Explain how sensory coding and processing allow the CNS to distinguish the four properties of a stimulus.

Answer: The four properties of a stimulus are modality, location, intensity, and duration. Modality, or nature of the stimulus, is indicated by which neurons are activated and where their associated sensory pathways terminate. According to the law of specific nerve energies, receptors respond most strongly to a particular type of stimulus, such as sound in the case of an auditory receptor. Labeled line coding means that if any cell along the pathway is stimulated, the perception will be the usual sensation. The cerebrum and other brain areas are highly organized into topographic maps of the receptive fields of their sensory receptors, ensuring that location of a stimulus is accurately detected. The number of receptors activated and the frequency of action potentials along the sensory pathway both increase with stimulus intensity, and stimulus duration is correlated with the duration of a volley of action potentials.

150) Suppose the brain of a lab animal (such as a frog) has been destroyed, but the animal's heart continues to beat and other organs to function for up to an hour or so. If its foot is placed in hot water or strong acid, noxious stimuli to an intact frog, what would happen? Does the frog feel pain under these circumstances? Explain.

Answer: The frog would withdraw its leg from the stimulus, as long as the spinal cord is intact. Without a brain, no pain is felt; the withdrawal of the leg is a simple spinal reflex.

112) Explain why observations of basal body temperature are useful for couples who want to get pregnant, but are only of limited use for couples who are trying to avoid getting pregnant.

Answer: The hormone, progesterone, is released after ovulation and is associated with a rise in body temperature. The slight rise in body temperature is evidence that a woman has already ovulated, and the elevation remains until the beginning of the next menstrual cycle. Couples who are trying to avoid getting pregnant cannot use the lower temperature as an indication that ovulation has not occurred because the temperature elevation is not immediate, and furthermore they must understand that sperm survive for several days in the female reproductive tract.

You and your fellow deep-sea scientists have discovered a new form of marine invertebrate, and are anxious to determine the similarities and differences to animals already characterized. Chemical analysis reveals the following concentrations of permeable cations: Ion [in] mM [out] mM Na+ 50 450 K+ 420 20 Mg++ 10 60 Cl- 50 550 Nernst equation: Eion = 61/z × log [ion]out/[ion]in 76) Control of the neuromuscular junction in this new invertebrate is found to involve dual innervation, where one motor neuron secretes an excitatory neurotransmitter and another secretes an inhibitory neurotransmitter onto the muscle. How does this compare to the human neuromuscular junction? Propose three different types of inhibitory receptors/channels that would produce inhibition of the invertebrate muscle, specifying ion and direction of ion flow. For each ion, calculate the equilibrium potential. What similarity do you notice in the equilibrium potentials of the ions involved, and how is that significant? How may inhibition be accomplished in human muscle?

Answer: The human neuromuscular junction is innervated by only an excitatory neuron. The inhibitory motor neuron in the invertebrate may open a channel to potassium, to chloride, or to both, resulting in hyperpolarization as potassium exits and/or chloride enters. EK+ = 61 × log 20/420 = -81 mV. ECl- = -61 × log 550/50 = -64 mV. Both of these equilibrium potentials are negative, indicating that these ions can be useful for inhibition. In humans, inhibition of muscle activity occurs within the CNS, rather than at the muscle.

89) The weather person gleefully tells us that the temperature is 95, but the heat index is 103 and a heat advisory is in effect. What does this mean? Is humidity high or low? Why does it matter? Who is most affected, and why? What are some steps one could take to adjust?

Answer: The humidity is high, thus slowing evaporative heat loss; the heat advisory means take it slow and suggests you plan your day around the heat and try to get out of it. Elderly people are most likely to be affected physiologically and possibly economically and sociologically as well. Use fans and/or air conditioning, swim, take advantage of a mall or movie theater's air conditioning, decrease activity, do outdoor work only in the early morning or late evening.

57) What are the three types of muscle tissues in the human body? What are the two common functions of muscles?

Answer: Three types of tissues are skeletal muscle, cardiac muscle, and smooth muscle. The two common functions are to generate motion and to generate force. 58) Groups of adjacent fibers bundled together into units are called __________. __________, elastic fibers, nerves, and __________ are found between these units. (:0)Answer: fascicles; Collagen, blood vessels (latter two in either order)

81) List and discuss two different theories about the regulation of food intake and the role of signal molecules and the brain.

Answer: The hypothalamus contains two centers that regulate food intake: the feeding center and the satiety center, which inhibits the feeding center. The glucostatic theory states that glucose utilization by hypothalamic centers regulates food intake. The lipostatic theory states that signals from the body's fat stores to the brain modulate eating behavior to maintain a particular weight. Leptin is a protein hormone synthesized in adipocytes and may signal the brain to halt feeding behavior. Neuropeptide Y is a brain neurotransmitter that appears to be the stimulus for food intake; this molecule is inhibited by leptin. Ghrelin is a peptide secreted by the stomach that stimulates hunger. Table 22-1 in the CHAPTER summarizes peptides involved in feeding.

97) Explain the roles and interactions of each of the following in immunity: B lymphocytes, antibodies, immunoglobulins, T lymphocytes, MHC, HLA.

Answer: The immune system consists of two main classes of cells: B lymphocytes and T lymphocytes. The effector B lymphocytes, plasma cells, are specialized for producing large amounts of a protein known as an antibody. Antibodies bind to specific antigens, leading to the destruction of the antigens and pathogens bearing the antigens. Immunoglobulins are just antibodies, categorized according to specific details of their structure and function. T lymphocytes are specialized for destroying cells that bear antigen within a self molecule known as MHC. MHC stands for major histocompatibility complex, which is a group of cell surface proteins essential for triggering the immune response. HLA stands for human leukocyte antigen, and is just an alternative name for MHC.

117) What is the induced-fit model? List the types of bonds involved and classify them as strong or weak.

Answer: The interaction between a protein binding site and a ligand that are in close proximity results in a conformational change of the protein to fit more closely to the ligand. The bonds involved are hydrogen (weak), ionic (strong), and van der Waals (weak).

110) Explain how anticoagulant drugs can be useful to heart attack or stroke patients.

Answer: The ischemia that accompanies a "heart attack" or stroke may be prevented by the administration of anticoagulants, which prevent clot formation.

142) What is the benefit of having some of the cellular organelles enclosed by a membrane similar to the cell (plasma) membrane?

Answer: The isolation of the internal contents of membrane-bound organelles allows them to manufacture or store secretions, enzymes, or toxins that could adversely affect the cytoplasm in general. Another benefit is the increased efficiency of having specialized enzyme systems concentrated in one place, for example, those necessary for energy production in the mitochondrion. In addition, the membranes themselves serve as "workspace," allowing the anchoring of enzymes or other proteins into a stabilized location.

161) What is the benefit of having some of the cellular organelles enclosed by a membrane similar to the cell membrane?

Answer: The isolation of the internal contents of membrane-bound organelles allows them to manufacture or store secretions, enzymes, or toxins that could adversely affect the cytoplasm in general. Another benefit is the increased efficiency of having specialized enzyme systems concentrated in one place. One example is the concentration of enzymes necessary for energy production in the mitochondrion.

111) Explain what might happen to a man taking sympathetic nervous system inhibitors to treat his hypertension, in terms of sexual function.

Answer: The man could potentially suffer from a form of sexual dysfunction that prevents ejaculation. Recall from Ch. 11, the sympathetic division of the autonomic nervous system is responsible for penile ejaculation whereas the parasympathetic division is responsible for penile erection.

100) A construction worker has been brought to the emergency room, after an accident in which his leg was crushed by falling beams. A urine sample is noticeably reddish brown. Assuming that color is related to the accident, what molecule could have been released from the damaged muscle cells that colored the urine? What is the function of this molecule?

Answer: The molecule is likely to be the pigment myoglobin. This molecule binds to oxygen in muscle cells, to supplement the oxygen supplied by the blood.

146) Describe the differences between slow and fast pain.

Answer: The most notable difference between the two is the timing. Fast pain is sharp and localized. It is rapidly transmitted by myelinated A-d fibers to the CNS, while slow pain is carried on unmyelinated C fibers. Slow pain is described as duller and more diffuse.

70) Bowel and bladder control involve spinal reflex pathways that are located in the sacral region of the spinal cord. In both instances, two sphincter muscles, an inner sphincter of smooth muscle and an outer sphincter of skeletal muscle, control the passage of waste out of the body. How would completely severing the spinal cord at the lumbar level affect an individual's bowel and bladder control?

Answer: The person would still exhibit a defecation (bowel) and micturition (bladder) reflex because the spinal reflex is processed at the local sacral level of the spinal cord, which is intact at that level. Efferent impulses from the organs would stimulate specific interneurons in the sacral region that would synapse with the motor neurons controlling the sphincters, thus bringing about emptying when organs began to fill. This is the same situation that exists in a newborn infant who has not yet fully developed the descending tracts necessary for conscious control. The individual with the spinal cord transection would lose voluntary control of the bowel and bladder because these functions rely on impulses carried by motor neurons in the brain that must travel down the cord and synapse with the interneurons and motor neurons that are involved in the reflex. With a lumbar transection, these impulses no longer reach the reflex control center.

25) What is a nocebo effect?

Answer: The phenomenon whereby a patient who has been informed of the side effects of a drug he is taking is more likely to experience some of the side effects than an otherwise similar patient receiving the same drug who has not been so informed.

158) Explain three reasons for the greater visual acuity at the fovea.

Answer: The photoreceptor density is highest at the fovea, and light reaches these photoreceptors directly because they are not blocked by other layers of cells as they are in the rest of the retina. Furthermore, each ganglion cell receives input from only a few photoreceptors in the fovea, compared to a larger number of inputs elsewhere in the retina.

88) Carl is suffering from hypothyroidism. How could his physician determine if his symptoms result from problems at the level of the hypothalamus and pituitary or at the level of the gland itself?

Answer: The physician could order blood tests to determine the levels of TSH, T3, and T4 in the blood. If the condition is due to disease of the thyroid gland, you would expect to see elevated levels of TSH because of a lack of feedback inhibition by thyroid hormones. If, on the other hand, the problem is due to problems at the level of the hypothalamus or pituitary, you would expect to see decreased levels of TSH.

74) A chemistry student accidentally spills chlorine bleach into a dilute acid. The mixture reacts and produces fumes that are inhaled by the student and that reduce his ventilation. Assume that the gaseous chemical produced is a base (i.e., it releases OH- in an aqueous solution), and that the chemical is absorbed into the bloodstream at the alveoli. Explain why ventilation is reduced in the patient.

Answer: The physiological response is opposite that for acidosis. A decreased ventilation would reduce loss of CO2. By maintaining higher PCO2 in the blood, the resulting acidity would help to counteract the alkalinity resulting from the inhalation of a base.

94) Describe the pleura and explain its role in respiration.

Answer: The pleura is a double-walled sac that surrounds each lung. One wall clings to the lung surface, the other to the wall of the thoracic cavity. The pleura secretes and contains pleural fluid, which is a lubricant. Pleural fluid also keeps the wall of the lung in close proximity to the wall of the thoracic cavity, which is critical in the process of pulmonary ventilation.

160) After coming off a ribosome, several things can happen to the newly synthesized protein. Name three.

Answer: The possible (:0)Answers include: 1. folded into various three-dimensional shapes 2. create cross-links between different regions of its amino acid chain 3. be cleaved into fragments 4. attach other molecules or groups 5. assemble with other amino acids into polymeric protein

41) Explain why the prefix homeo- is used in the term homeostasis. Why do some physiologists prefer the term homeodynamics over homeostasis?

Answer: The prefix homeo-, meaning like or similar, is used to indicate that the body's internal environment is maintained within a range of acceptable values rather than a fixed state. Some physiologists argue that the term homeodynamics better reflects the small but constant changes that continuously take place in the internal environment, as opposed to homeostasis, which erroneously implies lack of change.

113) Given what you know about blood pressure at various locations in the circulatory system, what pressure is actually determined during a standard blood pressure measurement? Why is the arm the chosen location for determining blood pressure? Is there a more accurate way to determine blood pressure?

Answer: The pressure in the brachial artery is what is determined, but it is assumed to be very close to the pressure in the aorta, which is the desired information. The arm is chosen for proximity to the heart and convenience; farther from the heart in linear distance, the systolic pressure will be lower and the difference between systolic and diastolic will be less. A more accurate way to determine blood pressure in the aorta would be to place pressure sensors inside the vessel, though this is an invasive and unnecessary procedure given the reliability of the brachial pressure.

57) A person with cirrhosis of the liver has lower than normal levels of plasma proteins and a higher than normal GFR. Explain why a decrease in plasma protein concentration would cause an increase in GFR.

Answer: The primary driving force for GFR is blood pressure, opposed by fluid pressure in Bowman's capsule and osmotic pressure due to plasma proteins. If a person has fewer plasma proteins due to liver disease, the plasma will have a lower osmotic pressure. With less osmotic pressure opposing the GFR, GFR will increase.

74) Design a concept map illustrating the source, regulation of secretion, targets, and effects of the hormones of growth and metabolism. Also incorporate causes and effects of hyper- and hyposecretion, and indicate which hormone(s), if any, is/are not essential for life. Color-coding your chart may be helpful.

Answer: The primary hormones covered are adrenal glucocorticoids, thyroid hormones and calcitonin, growth hormone, parathyroid hormone, and calcitriol. See Ch. 23 tables and figures. Thyroid hormone is not essential for life.

73) You are a technician in a neuroanatomy lab, and it's your first day of training to learn a technique of labeling neurons. Your boss tells you that you will fill individual motor neurons, by injecting horseradish peroxidase (HRP) conjugated to tetanus toxin, into a skeletal muscle. How could this possibly label the neurons?

Answer: The process of labeling neurons by HRP was introduced in Ch. 9; this enzyme is taken up by axon terminals and transported retrogradely, filling most or perhaps all dendrites, as well as the axon and soma. When the substrate of the enzyme is added to tissue slices, a dark reaction product is formed, and this is how the details of the cell are visualized. Toxins such as tetanus and cholera toxin, which are also taken up and transported by axon terminals, enhance the uptake of HRP, if they are attached (conjugated) to the HRP.

140) You are a student intern in the research and development department of a pharmaceutical company. You have discovered a compound that destroys the common cold virus in cultured human cells. Chemical characterization reveals that carbon, hydrogen, and oxygen are present, in a 20:40:4 ratio of C:H:O. What chemical class is this compound? Experiments in rats show that neither oral nor injectable treatment with the compound was effective in destroying the virus. Discuss some possible reasons for this lack of effectiveness.

Answer: The relatively low amount of oxygen and high carbon and oxygen indicate that this compound is probably a lipid. Oral administration may result in digestion of the compound so that none is absorbed into the blood. Lipids are not highly soluble in water, and because blood is a watery medium, the injected lipid may not transport well in the blood. Also, human cells as well as the viruses may behave differently in culture compared to in a real patient.

123) A sudden flash of bright light would A) cause contraction of the pupillary constrictor muscles. B) cause contraction of the pupillary dilator muscles. C) cause relaxation of the ciliary body. D) cause relaxation of the ciliary ligaments. E) increase the size of the iris.

Answer:A

141) If a cell lacked the enzyme RNA polymerase, it could not A) form protein. B) form complementary sequences of DNA. C) link pieces of DNA together. D) form spindle fibers. E) form a new nuclear membrane during cell division.

Answer:A

73) An industrious 10-year-old swimmer has attached an extension to his snorkel tube, so that he can sit on the bottom of the ocean for a longer period of time, watching the fish of the coral reef. What problems would you expect him to have? Assuming he is successful in breathing this way for a little while, what are some of the differences he will experience compared to normal breathing?

Answer: The resistance to air flow increases as a result of adding the extension to the snorkel tube. Assuming he has used an extension of sufficiently large diameter, he is in a shallow location where the total length isn't excessive, and he has the strength and endurance in his respiratory muscles, he may be able to breathe this way for a few seconds longer than he could have held his breath. He will probably notice that he is breathing more deeply and at a higher rate, to maintain ventilation against the higher resistance. After a few exchanges he will start to hypoventilate, despite his efforts to increase pulmonary ventilation, because the presence of dead air space in the tube will result in decreasing PO2 in the alveoli. Dead air space is a problem if the total volume of air in the tube is greater than the volume exchanged with each breath.

113) Draw and label a spirogram showing respiratory activity in a resting person. Draw and label a spirogram in this same person during exercise.

Answer: The resting graph should resemble Fig. 17-6. During exercise, the only things that change are the size of the tidal volume curve, which will become larger as it enters into the inspiratory and expiratory reserve capacities, and these reserve capacities will have the same extreme values but will be smaller.

155) Discuss the structure of the retina, including any distinguishing "spots."

Answer: The retina consists of layers of cells, with the posterior layer containing the photoreceptors (rods and cones). Photoreceptors synapse onto bipolar cells, which in turn synapse onto an anterior layer of retinal ganglion cells, whose axons comprise the optic nerve. Also present are horizontal cells, which synapse with photoreceptors and bipolar cells, and amacrine cells, which synapse with bipolar and ganglion cells. Cones are sensitive to color and are most concentrated in a spot called the fovea. Another spot is the optic disk or blind spot, where the axons of all ganglion cells merge into the optic nerve, and where there are no photoreceptors.

83) The graph below shows an oxygen dissociation curve, with the normal curve on the left. What is the name of the phenomenon that shifts the curve to the right? What causes this change in oxygen dissociation? Is the shift beneficial or detrimental? Explain.

Answer: The shift in the dissociation curve is called the Bohr effect. The decrease in pH may be due to an increase in CO2 and lactic acid during vigorous physical activity, or to metabolic acidosis. The Bohr effect is beneficial, because the unloading of oxygen from hemoglobin increases under such conditions. If the decrease in pH is due to exercise, then the skeletal muscles have an increased demand for oxygen, and thus increased unloading is desirable.

104) The retrovirus HIV frequently undergoes mutations, making it difficult for scientists to develop a single effective vaccine. When flu vaccines are administered, patients are told that they are only protected from one strain of the flu and are still as susceptible to other strains as unvaccinated people. Explain why.

Answer: The specificity of vaccines to a particular antigen is critical in order to mount an appropriate immune response. In order to function effectively, both plasma cells and activated T cells must recognize a particular pathogen. Since HIV frequently mutates, several forms of the virus may be present and therefore a single drug may target one form but not another. The flu vaccine is developed early in the flu season after the virus affecting the first victims is characterized. If a person is infected with a different strain or with a mutated vaccine-related strain, the vaccine is ineffective.

117) Describe the events that must occur in order for sperm to penetrate an egg. What prevents more than one sperm from entering? In the case of twins, how many sperm entered the egg? Why must a man have millions of sperm in a semen sample to be considered fertile? If only one sperm were present in the semen and that sperm managed to find its way to the egg, could it fertilize the egg? Explain.

Answer: The sperm must penetrate the corona radiata and the zona pellucida, which surround the egg, in order to get to its membrane. The acrosome of the sperm contains enzymes that disrupt these barriers. The first sperm to reach the membrane binds to a receptor, the sperm and egg membranes fuse, and the sperm nucleus enters the egg. The cortical reaction ensues, in which granules in the egg spill their contents out of the membrane, which alters it in a way preventing additional sperm from binding. Twins result from normal fertilization, but either two eggs are present or the early embryo splits. Only about 100 sperm manage to find the egg, with the other millions swimming off in the wrong direction. One sperm probably does not contain enough enzymes to allow penetration of the egg barriers.

98) There is only one way to lose weight without actually removing a body part: consume fewer calories than you expend. Is this statement true or false? Explain, citing a fundamental law of physics. Why is it difficult for many people to lose weight? Many weight-loss gimmicks work. Explain why they work, and why they may not work forever.

Answer: The statement is true, and is a principle of physics known as the first law of thermodynamics. It is difficult for many people to lose weight because of a variety of factors, such as overeating due to psychological factors, sedentary lifestyles, and so on. Weight-loss gimmicks work because ultimately they either reduce calories or increase activity, or both. They may be difficult to maintain because the novelty wears off, people feel deprived of food, and a multitude of other factors. (:0)Answers to this question will vary considerably.

55) What general trend in systolic blood pressures is seen as both men and women increase in age?

Answer: The systolic pressure of both genders increases with advancing age.

65) List and describe the three categories of movement. Explain why these categories are NOT always distinct.

Answer: The three categories are reflex, voluntary, and rhythmic. Reflex movements are initiated by sensory input and are integrated primarily in the spinal cord. Voluntary movements are integrated in the cerebral cortex and require no external stimuli. Rhythmic movements are a combination of reflex and voluntary. Voluntary movements improve with practice as do reflexive, voluntary movements can become reflexive once learned, and voluntary movements depend upon input from postural reflexes.

143) How many grams of glucose, m.w. 180 daltons, is necessary to make 1 liter of a 1.0 molar solution? A) 180 B) 360 C) 90 D) 6.02 × 1023 E) 1.0

Answer:A

105) The condition known as shock is characterized by a decreased return of venous blood to the heart. How could an upsetting situation such as the sight of a tragic accident or very bad news produce shock?

Answer: The upsetting stimuli would be processed by the higher centers of the central nervous system and relayed to the hypothalamus. The hypothalamus could suppress the vasomotor center of the medulla, resulting in fewer sympathetic impulses to peripheral blood vessels. This would cause a decrease in sympathetic tone in the smooth muscle of the blood vessels, resulting in vasodilation. The vasodilation would cause blood to pool in the extremities, decreasing the amount of blood returning to the heart and producing shock.

94) List the components of the uterus, including the layers of the wall. Briefly explain the function of each.

Answer: The uterus consists of the cervix or neck, the body, and the oviducts or fallopian tubes. The wall layers are an outer connective tissue, middle myometrium, and inner endometrium. See Fig. 26-12 in the CHAPTER.

128) In some severe cases, a person suffering from stomach ulcers may have surgery to cut the branches of the vagus nerve that innervates the stomach. How would this help the problem?

Answer: The vagus nerve carries neurons of the parasympathetic nervous system. These neurons control gastric secretions, notably the secretion of acid and enzymes. Severing these branches would eliminate neural stimulation from the central nervous system, thus eliminating the release of gastric fluids in response to anxiety and other higher-order stimuli when there is no food in the stomach. Normal digestive function would still occur, governed by various hormones and intramural neural reflexes.

70) Compare and contrast the voltage-gated sodium and potassium channels introduced in Ch. 8, with the acetylcholine receptor/channel. What may be confusing to the beginning physiology student trying to understand the ion specificity of the acetylcholine receptor? What type of change in the cell is produced by ion movement through each type of channel?

Answer: The voltage-gated sodium channel opens in response to threshold voltage and allows sodium to enter the cell, thereby depolarizing the membrane potential. The potassium channel is also opened by threshold voltage and allows potassium to exit the cell, repolarizing or hyperpolarizing the membrane potential. The acetylcholine receptor is a neurotransmitter receptor as well as an ion channel. Because the channel is opened by binding of a chemical, it is a chemically gated channel. This channel is permeable to both sodium and potassium; when the channel opens, sodium diffuses into the cell and potassium diffuses out. The inward positive charge is sufficient to depolarize the skeletal muscle to threshold. Students are often confused by the fact that more than one type of ion moves through the acetylcholine receptor channel, and that the muscle cell is able to depolarize to threshold despite the exit of potassium.

36) What are the labels for the graph axes?

Answer: The x-axis is vitamin dose; the y-axis is finish time in minutes.

170) Based on your knowledge of the structures of the eye and visual pathway, propose several possible causes for blindness. How does the disease glaucoma fit in this scheme?

Answer: There are a large number of structures involved in normal vision; damage to nearly any one of them may result in partial or complete blindness. Starting from the beginning, there must be healthy cornea, lens, retina (including rods and three types of cones, bipolar and ganglion cells, horizontal and amacrine cells), normal pressure in the eye chambers, intact optic nerve, and the remainder of the visual pathway through the visual cortex. Glaucoma results from degeneration of the optic nerve, which in turn is associated with several risk factors.

50) Why is the normal baroreceptor reflex absent during exercise?

Answer: There are several theories. See "The Baroreceptor Reflex Adjusts to Exercise" section of the CHAPTER.

131) You and your lab partners in a human anatomy course have just removed the intestines from an adult cadaver, and cut away connective tissues as necessary to uncoil the intestines into a straight line that could be measured. Your professor told you that the intestinal tract of an adult of this size and gender is typically about 13 ft. Your measurement is 22 ft. One of the lab partners insists that everything is smaller in preserved cadavers due to dehydration. Address this suggestion, and propose some other explanations for the difference.

Answer: There is no doubt of some individual variation, so the measurement is expected to be at least a little different. Most of the difference is due to loss of living muscle tone, which normally keeps the tract contracted. While it is true that preserved tissues may shrink from dehydration, this is more than compensated by the loss of muscle tone.

52) Individuals with McArdle's disease cannot break down glycogen. How would this affect substrate utilization during exercise?

Answer: These individuals cannot rely on glycogen stores during exercise. Therefore, carbohydrates are available to them through only two pathways: gluconeogenesis and exogenous glucose. As a result, they must rely mostly on fat oxidation for energy.

116) Define and describe the role of up-regulation and down-regulation of proteins.

Answer: These terms refer to the net change in the amount of a functional protein present in a cell in response to a signal. Up-regulation is an increase in the amount of the protein, whereas down-regulation is a decrease.

47) What is a hypothesis? What are the steps involved in following the scientific method? How does one distinguish the dependent variable from the independent variable in an experiment? How are each of these represented on a graph?

Answer: This is discussed in "The Science of Physiology" section of the CHAPTER, and in Fig. 1-7.

98) What are some dangers of having coronary artery disease? What are some dangers of having hypertension?

Answer: This is discussed in the "Cardiovascular Disease" section of the CHAPTER.

99) Outline the relationship between hypertension and the development of congestive heart failure.

Answer: This is discussed in the "Cardiovascular Disease" section of the CHAPTER.

30) Why are physiology and anatomy frequently studied together?

Answer: This is discussed in the "Physiological Systems" section of the CHAPTER.

135) Describe the progression of a fertilized egg, from totipotent cell through pluripotent and multipotent stem cells. How might stem cells be of therapeutic value? What is plasticity?

Answer: This is discussed in the "Tissue Remodeling" section of the CHAPTER.

94) The approach to treating patients who secrete abnormal amounts of hormone sometimes depends upon the age of the patient. List the hormones of growth and metabolism for which this statement is true, and explain in each case how and why the approach would differ for a prepubertal patient compared to a postpubertal patient.

Answer: Thyroid hormone and growth hormone are the main hormones of growth and development with differing roles in childhood compared to adulthood. Students may wish to discuss calcitonin as well, but there is little detail in the text and incomplete understanding of its role, especially in adults. 1. Thyroid hormone is essential for normal growth and development in children, although this hormone is not essential for living, regardless of age. Thyroid imbalances must be promptly corrected in children, or else the patients will be developmentally delayed. Thyroid hormone has a permissive effect for growth hormone, is anabolic in children, and is required for normal neural development. These effects are not seen with thyroid hormone imbalance in adulthood, but to maintain a normal quality of life, amount of circulating thyroid hormone should be corrected in adults as well. 2. Children cannot grow normally without adequate amounts of growth hormone, and imbalances should be corrected promptly. Secretion of GH peaks during the teens. Excess GH in childhood can lead to giantism, while lack of GH leads to pituitary dwarfism. Lack of GH in adulthood is not problematic, but excess production stimulates the growth of cartilage, causing acromegaly.

71) Timmy is a cantankerous toddler who has just threatened that he will hold his breath until Mom gives him some chocolate. His mother refuses to be manipulated and watches in amusement as Timmy stubbornly refuses to breathe. To her horror, Timmy loses consciousness and collapses onto the floor. Her cousin, who is enrolled in a course for emergency medical technical (EMT) training, is visiting, and tells her there is no need to call for an ambulance. Why did Timmy lose consciousness? Should his mother trust her instincts and call for help and begin CPR, or should she listen to her well-meaning but young and inexperienced cousin? Explain.

Answer: Timmy loses consciousness due to hypoxia in his brain. There is no need for alarm, because the loss of consciousness indicates mainly that cerebral activity has decreased. The parts of the brain involved in respiratory control are in the brain stem. As long as Timmy was awake and determined, his cerebral signals were able to inhibit brain stem control of pulmonary ventilation. Once he loses consciousness, however, the respiratory control areas are released from inhibition and he starts breathing again. Soon normal oxygen content in the brain will be restored, and Timmy will wake up.

113) The rejection of a transplanted organ by the immune system of the recipient is a result of a cell-mediated immune response, rather than antibody-mediated response. What does this indicate about the nature of the transplanted cells? From the perspective of the T cell, are the cells of the transplanted organ significantly different from virus-infected or cancerous self cells?

Answer: Transplanted cells contain surface molecules recognized as self, as well as some recognized as non-self. As far as the T cell is concerned, the transplanted organ consists of altered self cells that are no different from those related to viruses or cancer.

44) Design a concept map on scientific experimental design.

Answer: Variable. It is recommended that the instructor create a map before evaluating the students' maps, and modify the instructor map as necessary.

45) Provide an example of a control system. Be sure to include the three main components: an input signal, a controller, and an output signal.

Answer: Variable. One example is blood glucose concentration. The input signal is a blood glucose concentration outside of the normal range, the controller is the pancreas, and the output signal is release of either insulin or glucagon.

65) Diagram the events that occur at the neuromuscular junction. Be sure to include the somatic motor neuron, axon terminal, synaptic cleft, synaptic vesicles, motor end plate, and appropriate neurotransmitter(s), ion channel(s), and membrane receptor(s).

Answer: Variable. See Fig. 11-13 in the CHAPTER.

136) You are helping your dad prepare food in the kitchen. Dad has a tablespoon of water in one hand and a tablespoon of vegetable oil in the other, when he trips over the rug and spills both spoons on the countertop. Dad notices that the oil forms a thin film on the countertop, whereas spilled water forms smaller, taller beads. How should you explain the different behavior of these liquids to your dad?

Answer: Water forms beads when it strikes a nonabsorptive surface because of surface tension resulting from the hydrogen bonds between neighboring water molecules. Vegetable oil molecules are nonpolar covalent, therefore there is no hydrogen bonding between the lipid molecules and no bead formation.

77) Drowning is often the introduction of outside water into the alveoli, yet some water is normally present in alveoli. Why is water normally present in the alveoli? What is present in alveolar fluid, besides water, that aids in respiration? What properties of water are detrimental to respiration? What causes drowning? While extremely rare, there have even been documented cases of people walking away from recreational swimming only to die later, on dry land, as a result of drowning (termed "delayed" or "secondary" drowning). Propose an explanation for delayed drowning.

Answer: Water is present everywhere in living tissues, because all living cells require water as a diffusion medium for solutes. Alveolar fluid is present in only a thin layer because it contains surfactant, preventing it from collapsing the alveoli due to surface tension. Oxygen has low solubility in water, but the large total surface area of alveoli combined with the thinness of the water layer in the alveoli allows for sufficient oxygen to enter the blood. Inhalation of water increases the amount of water and decreases the relative amount of surfactant. Less oxygen will reach the alveolar membranes, and alveoli will collapse. Alveolar collapse combined with the low solubility of oxygen in water will lead to hypoxemia and will trigger cardiovascular and respiratory reflexes. Delayed drowning likely results from tissue damage in the lungs and malfunction in other organs resulting from hypoxemia and reflexes related to the water inhalation.

142) Describe what happens to NaCl when placed in water.

Answer: Water molecules break the ionic bonds holding Na+ and Cl- together. Each sodium ion becomes surrounded by polar water molecules, with the electronegative ends of water molecules interacting with the ion. Each chloride ion also becomes surrounded by polar water molecules, but in this case it is the electropositive ends of the water molecules that bind to the ion. A consequence is that sodium and chloride ions can function relatively independently of each other when in solution.

69) While watching Olympic weight lifters on television, Carl notices that on several occasions an athlete would lift the weight to his chest and then suddenly drop it. Can you offer an explanation for what might be happening?

Answer: What Carl is observing is an example of the stretch reflex in action. As the weight lifter lifts the weight, he is contracting the biceps brachii and brachialis muscles and at the same time stretching the triceps brachii muscle. If the mass is great, the amount of flexion necessary to move the mass could begin to overstretch the triceps that is being relaxed so as not to interfere with flexion. Stimulation of the muscle spindles in the triceps produces a reflex arc that brings about the relaxation of some of the motor units in the muscles involved in flexion, like the biceps. When the tension produced by the flexing muscles decreases to a point where they can no longer work against the resistance of the mass, the weight lifter will drop the weights involuntarily.

51) During exercise, blood flow to the active skeletal muscles is increased by autoregulation. Propose a mechanism based on the need for nutrients that would explain how the autoregulation might take place.

Answer: When a skeletal muscle becomes more active, it has an increased demand for nutrients such as oxygen and glucose and quickly removes these from the surrounding interstitial fluid. This leaves fewer nutrients available for other cells that share the same interstitial fluid, such as the smooth muscle cells of the precapillary sphincter. As the available nutrients decline, energy production in the precapillary sphincters cannot keep pace with the need to keep the muscle in a contracted state, and the sphincters begin to relax. As they relax, the opening to the capillary becomes larger so that more blood flows into the capillary network that serves the skeletal muscle.

103) Ralph is taking SCUBA diving classes and is confused as to why he should not hold his breath under water while ascending—his instructor told him he must exhale continuously. What would you tell him?

Answer: When breathing the compressed air from a SCUBA tank, the lungs are virtually completely inflated. Problems would arise if he held his breath and began to ascend. On ascending, the decrease in pressure would cause the gas in his lungs to expand (Boyle's law). Since the lungs are already fully inflated, this would cause overinflation and possible rupturing of the lung. If this occurred, air would leak from the lungs into the pleural cavity, resulting in a pneumothorax and a collapsed lung.

126) Explain the polar character of an ammonia molecule (NH3). What is the cause of the partial charges? What is the overall charge for NH3?

Answer: When chemically bonded with each other, the nitrogen atom is partially negative whereas the hydrogen atoms are partially positive. The nitrogen atom in a molecule of ammonia has a stronger attraction for the electrons participating in the covalent bonds than the hydrogen atoms. The net charge on the molecule is still zero, however.

67) Explain the use of the terms fight and flight in describing sympathetic motor responses. How does sympathetic activity explain your increased "jumpiness" when you are home alone at night, watching a horror movie on TV?

Answer: When presented with a real or imaginary threat, the sympathetic motor system produces changes that prepare the body to oppose the threat ("fight") or run away from the threat ("flight" is the past tense of "flee"). Responses include an increase in heart and respiratory rate, and increased blood flow to cardiac and skeletal muscle. Reading or watching scenes that are threatening to others, even if you could not possibly be at risk, can produce the same sympathetic response in you as if you were actually present in the scene. If the phone rings or there is a knock at the door, you are likely to react to that very quickly, because you are primed for action.

74) Mary, a nursing student, has been caring for burn patients. She notices that they consistently show elevated levels of potassium in their urine and wonders why. What would you tell her?

Answer: When tissues are burned, cells are destroyed and the contents of their cytoplasm leak into the interstitial fluid and then move into the plasma. Since potassium ion is normally found within the cell, damage to a large number of cells would release relatively large amounts of potassium into the blood. The elevated potassium levels would stimulate the cells of the adrenal cortex that produce aldosterone. The elevated levels of aldosterone would promote sodium retention and potassium secretion by the kidneys, thus accounting for the elevated levels of potassium in the patient's urine.

185) If the stroke volume of the left ventricle is 65 mL/beat and the stroke volume of the right ventricle is 68 mL/beat, what will happen to the relative distribution of blood in the systemic and pulmonary circulation after 15 beats?

Answer: With each heartbeat, the left side will get "behind" by 3 mLs, with this volume contributing to congestion in the pulmonary circuit; after 15 beats, the deficit will total 45 mL of extra fluid in the pulmonary circuit. This could be the beginning of congestion that sets the stage for congestive heart failure.

120) Radiation emitted by radioisotopes creates an image on a special photographic plate, in the oldest method of imaging used in medicine, __________.

Answer: X-rays

108) Jerry is in an automobile accident that severs his spinal cord at waist level. After his recovery, he wonders if he will still be able to have an erection, enjoy sex, and father a child. What would you tell him?

Answer: Yes, he would still be able to have an erection, though he would lack the pleasurable sensations that normally accompany erection. Erection is controlled primarily by a parasympathetic reflex originating in the sacral region of the spinal cord (inferior to the injury). Tactile stimulation of the penis would initiate the parasympathetic reflex that controls erection. Jerry would also be able to experience an erection by a sympathetic route, since this would be controlled in the T12 to L2 area of the cord (superior to the injury). Stimulation by higher centers could produce a decreased sympathetic tone in the vessels to the penis resulting in an erection. His injury should have no effect on sperm production and ejaculation should still be possible, so he should be able to father a child.

123) Which statement is NOT true regarding cardiac muscle? A) Cardiac muscle cells must obey the all-or-none law of contraction. B) Cardiac muscle cells rely on a sarcoplasmic reticulum for storage of Ca2+ ions. C) Calcium-induced calcium release is the process by which extracellular Ca2+ triggers the release of stored Ca2+ from the sarcoplasmic reticulum. D) Intercalated disks contain both desmosomes for firm attachment and gap junctions to allow communication. E) The oxygen demand of cardiac muscle cells is high because they have many mitochondria.

Answer:A

172) In a recent accident, your favorite uncle lost one of his arms just below the elbow; he sustained no other injuries. The wound has completely healed, yet he is plagued by pain he claims is coming from a hand that is no longer there. Afraid to confide in his doctor, he asks you if he is going insane. What should you tell him? What principles are illustrated by this phenomenon?

Answer: Your uncle still has the surviving parts of the sensory pathway that once served the entire arm. This may include the primary sensory neurons, since the severed axons could have regenerated (but even if these cells degenerated, the remainder of the pathway did not). Anything that stimulates the sensory nerve in the stump may cause signals to go to the area of the cerebral cortex that once received inputs only from the hand. Your uncle is not crazy, rather he is experiencing phantom limb pain, which is very real. This phenomenon illustrates labeled line coding and coding of stimulus location.

...

Answer: [Note to instructor: If students have not yet studied the reproductive system, they may not be able to (:0)Answer the final question, therefore it could be omitted.] Necrosis is cell death as a result of damage from toxins, physical trauma, or lack of oxygen; damaged cells release chemicals that may damage neighboring cells. Apoptosis is programmed cell death, which is an internally regulated process and affects only the cell it occurs within. Menstruation may result from necrosis if the causative factor is changes in blood flow to the endometrium such that the oxygen supply is inadequate. Menstruation may result from apoptosis if it results from hormonal changes that directly kill the cells. While menstruation is a normal event and removes unneeded cells (characteristics of apoptosis), it results directly from the decrease in circulation, causing the cells to die from lack of oxygen; thus it is an example of necrosis.

105) Susan's doctor has detected benign uterine fibroid tumors. Jane's doctor has detected early- stage malignant vaginal tumors. Sarah's doctor has detected late-stage malignant ovarian tumors. Which branch of the immune system would respond to these conditions? What would analysis of the nearby lymph nodes of these women reveal? What is the prognosis for each woman? Explain.

Answer: [Note to instructor: Students must understand the difference between benign and malignant tumors, and that staging of tumors is related to prognosis. This question may be better for a comprehensive final exam or as an out-of-class assignment.] Uterine fibroid tumors are not cancer because they do not spread, hence the designation "benign." Nearby lymph nodes will contain no sign of uterine fibroids, and prognosis is independent of this condition because cancer is not indicated. No antigens are present, so there is no immune response. The other two women have cancer, and the cell-mediated immune response would be stimulated. Early-stage cancers are often completely treatable, and nearby lymph nodes may show no sign of the cancer; prognosis is good. Late-stage ovarian cancer has likely spread, and lymph nodes would contain ovarian cancer cells. Prognosis is poor in this case, because presence of cancer cells in lymph nodes often indicates that the cells have spread to other organs, as well.

148) If immunizations were available to prevent cancer, would you want your children to receive them? Explain. Is it likely that one immunization would prevent all cancers? Explain.

Answer: [Note to instructor: Students will need to have had some background on vaccinations to (:0)Answer this question.] (:0)Answers will vary, but students may want to discuss the possibility of adverse reactions to vaccines due to presence of albumin or mercury, the possibility of vaccines containing live virus to actually cause the disease they are supposed to prevent (as in some forms of the polio vaccine), and other possible negative effects compared to the benefit of avoiding the disease. Appropriately applied statistics can help put the cost:benefit ratio into perspective. For example, if there is a family history of certain cancers, a vaccination may be warranted despite the risks. Because there are many types of cancers and vaccinations are highly specific treatments, one vaccination would not prevent most cancers, but only the type it was designed to prevent.

76) In the science fiction movie The Abyss, a diver is able to breathe while his head is immersed in a specially prepared liquid pumped into a water-tight helmet attached to a suit, thus allowing him to dive without an air tank and at greater depths than possible for scuba divers. Is this purely fiction, or is it at least theoretically possible? Explain, considering the nature of alveoli and alveolar gas exchange. Assuming it is possible, describe the composition and nature of the liquid in terms of its oxygen pressure and concentration.

Answer: [Note to instructor: Such experiments were done in the 1960s with rodents, and were partially successful but ultimately deemed impractical and too risky for humans.] It is theoretically possible. Oxygen dissolves in a thin film of fluid in the alveoli, thus even in normal breathing a liquid containing dissolved oxygen is involved. Obviously the respiratory gases would have to be soluble in this liquid and in concentrations appropriate to maintain the partial pressures necessary to drive diffusion.

42) Explain why animals are used in research. Are there any limitations to the application of animal data to human physiology? Could these limitations be addressed using cell or tissue culture, or computer simulations?

Answer: [Note to instructor: This may be a good question to ask early in the semester, then again toward the end, after the organ systems have been covered.] There is a brief discussion of using humans or animals in research in the CHAPTER. This question is intended to stimulate students to think about how science is done, how data are generated, and how the process is challenged by social issues. Generally, there are limitations to the usefulness of computer simulations and cell/tissue culture systems for the same reason that nonhuman animal data are not 100% applicable to human physiology. How human organ systems perform may be different in very subtle ways from corresponding systems in other species. Cells in culture are in an artificial environment, and while much has been learned from such systems, it has also been noted that the behavior of cells in culture is not identical to cells in a living body. Furthermore, cells cultured from established lines can change over time, becoming less like the original cells from which they were derived, and presumably less like normal cells. Computer simulations are valuable, but are only as good as the data entered, and given that we don't know everything there is to know about physiology, we can't write a perfect computer program. All three approaches are useful, but for different reasons, and therefore one research system does not completely substitute for another, nor is it appropriate to abandon one entirely.

125) These thin, flattened cells allow the rapid passage of O2 and CO2 through the cells of the lungs and of certain blood vessels.

Answer:A

155) Cell membranes consist of lipid, protein, and carbohydrate in relative amounts that vary according to cell type. Describe the relative proportions of these substances in three structures, and relate these differences to cell function where possible.

Answer: [Note to instructor: This may be a good question to use on a comprehensive final exam, as it ties together basic membrane composition with cell functions revealed in later CHAPTERs.] See Table 3-1 in the CHAPTER. Red blood cells have nearly equal amounts of protein and lipid, with a small amount of carbohydrate, in this ratio of protein:lipid:carbohydrate: 49:43:8. Myelin is almost all lipid, followed by protein and carbohydrate in this ratio of lipid:protein:carbohydrate: 79:18:3. In later CHAPTERs on the nervous system, it will be seen that this preponderance of lipid results in electrical insulation, which is one of the main functions of myelin. The inner mitochondrial membrane is mostly protein, in this ratio of protein:lipid:carbohydrate: 76:24:0. This reflects the function of this membrane in chemical synthesis that relies on a variety of protein enzymes.

182) Sonja is pregnant during the year she is taking human physiology. At each check-up, she expects to be told that her blood pressure has increased. Yet into the final weeks of her pregnancy, her blood pressure is unchanged from its prepregnancy value. Why did she expect her blood pressure to increase? Propose some explanations for why it didn't. (Hint: Sometimes women adopt healthier habits during pregnancy, there are large blood-filled sinuses in the maternal portion of the placenta, and there are many hormonal effects that are unique to pregnancy.)

Answer: [Note to instructor: You may want to alter the hint, providing more or less help, depending on what you have covered in class.] Sonja assumes that her pressure will elevate with the increased blood volume and increased peripheral resistance that accompany the tissue growth she experiences. It is possible that she has adopted healthier diet and exercise habits during her pregnancy, offsetting an increase in pressure. It is also possible that the changing hormonal environment includes regulation of blood pressure so that it remains normal. It is also the case that the placenta forms a low-resistance reservoir for the maternal blood, offsetting the expected increase in total peripheral resistance and blood pressure.

68) Parturition is the process of __________.

Answer: giving birth 69) Human gametes contain __________ chromosomes. (:0)Answer: 23 70) Human somatic cells contain __________ chromosomes. (:0)Answer: 46 or 23 pairs of 71) The fertilized egg is properly referred to as a __________. (:0)Answer: zygote 72) The male gametes are called __________. (:0)Answer: sperm 73) The female gonads are called __________. (:0)Answer: ovaries 74) While still undifferentiated, gametes are called __________. (:0)Answer: germ cells 75) Besides sterilization, the only sure way to avoid pregnancy is __________. (:0)Answer:A bstinence 76) Birth control methods are also referred to as __________. (:0)Answer: Contraception 77) The inability for a male to obtain or sustain an erection is called __________. (:0)Answer: impotence (or erectile dysfunction) 78) The inability to conceive a child despite having intercourse is __________. (:0)Answer: infertility

40) PCO2 tends to be __________ in tissues than in systemic capillaries.

Answer: higher 41) __________ is the enzyme that converts CO2 into bicarbonate ions. (:0)Answer: Carbonic anhydrase 42) The __________ group of neurons contains mostly inspiratory neurons. The __________ group of neurons controls muscles used for active expiration and some inspiratory muscles. (:0)Answer: Dorsal respiratory; ventral respiratory group 43) Fear and excitement may affect the pace and depth of respiration by stimulation of portions of the __________. (:0)Answer: limbic system 44) Specialized __________ in the carotid and aortic bodies are activated by a decrease in PO2 and pH or an increase in PCO2. What do they trigger? (:0)Answer: glomus cells; they trigger a reflex increase in ventilation. 45) The output of the __________ group of inspiratory neurons controls the __________ muscle(s) by way of the __________ nerve. (:0)Answer: Dorsal respiratory, diaphragm (or internal intercostal), phrenic (or intercostal) 46) Inappropriate relaxation of the __________ muscles during sleep contributes to __________, a sleep disorder associated with snoring. (:0)Answer: mouth and throat (larynx, pharynx, and tongue), obstructive sleep apnea 47) The carotid and aortic bodies contain specialized __________ cells, which can increase ventilation in response to changes in PO2, PCO2, or pH. These cells release the neurotransmitter __________. (:0)Answer: glomus; dopamine 48) What force(s) move(s) carbon dioxide from the blood into the alveoli? (:0)Answer: Carbon dioxide moves into the alveoli as a result of the pressure gradient.

85) Molecules that readily dissolve in water are called __________.

Answer: hydrophilic (most are also polar) 86) Molecules that do not dissolve in water are called __________. (:0)Answer: hydrophobic (most are also nonpolar) 87) __________ is any molecule that binds to another molecule. (:0)Answer: Ligand 88) Two methods of protein activation include __________ and __________. (:0)Answer: proteolytic (removal of portions of the molecule), cofactor binding 89) The molecule DNA contains the five-carbon sugar __________. (:0)Answer: Deoxyribose 90) The molecule RNA contains the five-carbon sugar __________. (:0)Answer: ribose 91) The purines found in DNA are __________ and __________. (:0)Answer:A denine, guanine 92) The pyrimidines found in DNA are __________ and __________. (:0)Answer: thymine, cytosine 93) In a chemical reaction, __________ between atoms are broken as atoms are rearranged in new combinations to form different chemical substances. (:0)Answer: Bonds 94) The reaction rates of many chemical reactions that occur in the body are controlled by special protein molecules called __________. (:0)Answer: Enzymes 95) List the following in order of increasing mass: atom, molecule, proton, neutron, electron. (:0)Answer: Electron < proton = neutron < atom < molecule

126) Drugs known as calcium channel blockers can be used to A) decrease the force of cardiac contraction. B) increase blood pressure. C) constrict the coronary arteries. D) increase sympathetic stimulation of the myocardium. E) increase stroke volume.

Answer:A

13) A physician basing clinical decisions on primary research published in biomedical literature is doing __________ medicine. A) evidence-based B) traditional C) alternative D) whimsical E) holistic

Answer:A

13) An enzyme reacts with only limited, selected substrates; this characteristic is referred to as the enzyme's A) specificity. B) common bond. C) active site. D) isozyme. E) product.

Answer:A

13) Chief cells secrete A) pepsinogen. B) gastrin. C) mucus. D) hydrochloric acid. E) intrinsic factor.

Answer:A

13) In resistance exercise, such as strength training, muscles rely heavily on __________ energy production. A) anaerobic B) aerobic

Answer:A

57) An increase in the rate and depth of breathing is known as __________.

Answer: hyperventilation 58) The volume of air moved in a single respiration is called the __________. (:0)Answer: tidal volume 59) The volume of air that can be forcefully expelled from the lungs following a normal exhalation is called the __________. (:0)Answer: Expiratory reserve volume 60) The volume of air that can be forcefully inhaled on top of a normal inspiration is called the __________. (:0)Answer: inspiratory reserve volume 61) The volume of air that remains in the lungs after a forced expiration is called the __________. (:0)Answer: residual volume 62) A typical residual volume (in milliliters) for a healthy, 70 kg male is __________. (:0)Answer: 1200 mL 63) The __________ consist of light, spongy tissue whose volume is occupied mostly by air- filled spaces. (:0)Answer: lungs 64) The opposing layers of pleural membrane are held together by a thin film of __________. (:0)Answer: pleural fluid 65) The larynx contains the __________, connective tissue bands that tighten and vibrate to create sound when air moves past them. (:0)Answer: vocal cords 66) The primary function of the alveoli is __________. (:0)Answer: the exchange of gases between themselves and the blood 67) The pressure contributed by a single gas in a mixture of gases is known as __________. (:0)Answer: partial pressure

67) The condition where the skin and the whites of the eyes appear slightly yellow is called __________.

Answer: jaundice 68) The condition where the skin and the whites of the eyes appear slightly yellow is due to high blood levels of the substance __________. (:0)Answer: Bilirubin 69) Plasminogen is activated by an enzyme called __________. (:0)Answer: tissue plasminogen activator 70) In hemostasis, vasoconstriction is rapidly followed by mechanical blockage of the hole by a __________. (:0)Answer: platelet plug 71) The dissolution of fibrin by plasmin is known as __________. (:0)Answer: fibrinolysis 72) The two main components of plasma are __________ and __________. Name two other components found in plasma. (:0)Answer: water, proteins; other components found in plasma include: dissolved organic molecules, ions, trace elements and vitamins, and dissolved oxygen and carbon dioxide. 73) Red blood cells in the circulation live for about __________. The short life span is due to __________. (:0)Answer: 120 days; increasing fragility of the older red blood cells, which may rupture as they squeeze through capillaries 74) __________, __________, and __________ are called granulocytes because __________. (:0)Answer: Basophils, eosinophils, neutrophils, they contain cytoplasmic inclusions that give them a granular appearance 75) Name two cytokines involved in hematopoiesis. What is the name of the cytokine that plays an important role in the immune system? (:0)Answer: See Table 16-2 in the CHAPTER. Interleukins play an important role in the immune system.

68) Smooth muscle fibers can sustain a contraction without fatigue, even under aerobic conditions, by allowing dephosphorylated myosin to remain attached to actin in what is called the __________.

Answer: latch state 69) The gap junctions in cardiac muscle are part of specialized cell junctions called __________. (:0)Answer: intercalcated disks 70) The structures in smooth muscle that are analogous to the Z disks of sarcomeres are __________. (:0)Answer: Dense bodies 71) The enzyme of clinical importance that becomes elevated in the event of damage to either skeletal or cardiac muscle tissue is __________. (:0)Answer: Creatine phosphokinase (CPK) 72) Store-operated calcium channels open in response to __________. (:0)Answer: Depleted intracellular stores of calcium 73) Smooth muscle cells contain stretch-activated calcium channels that open when pressure or other force distorts the cell membrane. This activity is called __________ contractions and are common in __________. (:0)Answer: myogenic, blood vessels 74) __________ and __________ are striated muscles. Why are these muscles classified as striated muscles? (:0)Answer: Skeletal, cardiac muscle; they are called this because of their alternating light and dark bands (See Figs. 12-1a and 12-1b in the CHAPTER.)

47) The beating of the cilia of the respiratory passages in the direction of the pharynx forms a __________.

Answer: mucociliary escalator 48) When the inspiratory muscles relax, the rib cage returns to its original position as a result of __________. (:0)Answer: Elastic recoil 49) The ease with which the lungs stretch in response to changes in pressure is termed __________. (:0)Answer: Compliance 50) The ability of a lung to recoil, or recover from stretch, is called __________. (:0)Answer: Elastance 51) In the disease __________, many symptoms are due to destruction of elastic fibers in the lung. (:0)Answer: Emphysema 52) The substance produced by the lungs to reduce surface tension is called __________. (:0)Answer: surfactant 53) The cells of the lung that produce the substance that lowers surface tension are __________ (:0)Answer: type II alveolar cells 54) Because of their smooth muscle component, the structures of the lower respiratory system that can most alter airway resistance are the __________. (:0)Answer: Bronchioles 55) A powerful bronchoconstrictor released by mast cells is __________. (:0)Answer: histamine 56) Ongoing diseases in which air flow during expiration is diminished are known as __________. (:0)Answer: Chronic obstructive pulmonary diseases (COPDs)

33) The tension maintained in a muscle at rest is known as __________.

Answer: muscle tone 34) In order to flex a limb, the extensors must first be relaxed and vice versa. This is done by a process called __________. (:0)Answer: reciprocal innervation 35) Relaxation of skeletal muscle results from __________ input by the somatic motor neuron. (:0)Answer: the absence of excitatory 36) Rhythmic reflexive motor activities, such as breathing or walking, are controlled by networks of neurons in the central nervous system called __________. (:0)Answer: Central pattern generators 37) __________ monitor the position of skeletal muscles and joints. (:0)Answer: Proprioceptors 38) Autonomic reflexes are also called __________ reflexes. (:0)Answer: visceral 39) __________ reflexes involve skeletal muscles. (:0)Answer: Somatic 40) In a __________ reflex, a sensory neuron synapses directly on a motor neuron. (:0)Answer: monosynaptic 41) A __________ reflex has at least one interneuron placed between the sensory and motor neuron. (:0)Answer: polysynaptic 42) Most reflexes are regulated by __________. (:0)Answer: negative feedback 43) The __________ is essentially a group of interneurons that run from the motor cortex to the spinal cord. (:0)Answer: Corticospinal tract

132) If a myocardial infarction results in the formation of scar tissue along the pathway of the left bundle branch, A) cardiac arrhythmias may occur. B) blood flow to the lungs will decrease. C) the ventricle will contract more forcefully. D) conduction through the left ventricle would remain normal. E) the right ventricle will fail to contract.

Answer:A

133) The protein in the mitochondria that captures the kinetic energy of moving H+ ions and converts it to the stored energy of ATP is called A) ATP synthase. B) lactate dehydrogenase. C) oxidative phosphorylase. D) glucokinase.

Answer:A

59) The thick filament of the myofibril is composed of __________ molecules whereas __________ is a protein that makes up the thin filaments.

Answer: myosin, actin 60) T-tubule membranes contain __________ receptors called __________, which are mechanically linked to __________ in the adjacent sacroplasmic reticulum. (:0)Answer: voltage-sensing, dihydropyridine or DHP receptors, calcium-release channels (also called ryanodine receptors or RyR) 61) Muscles require __________ constantly; they can only store enough of it for about eight twitches. The backup energy source for muscle is __________, a molecule with high-energy phosphate bonds. (:0)Answer:A TP, phosphocreatine 62) Most skeletal muscles are attached to __________ and control __________, whereas cardiac muscle is found only __________ and is responsible for __________. (:0)Answer: Bones of the skeleton, body movement, in the heart, moving blood through the circulatory system 63) Skeletal muscles are usually attached to bones by __________. The __________ of a muscle is the end of the muscle that is attached closest to the trunk. The __________ of the muscle is the end that is more distal. (:0)Answer: tendons; origin; insertion 64) An alternate term for a muscle cell is __________. (:0)Answer: muscle fiber or myofiber 65) A muscle that maintains a measurable level of tension all the time is said to have __________. (:0)Answer: tone 66) For a muscle to change from incomplete tetany to complete tetany, the stimulus frequency must __________. (:0)Answer: increase 67) In single-unit smooth muscle, the force generated in graded contractions varies with the amount of __________ ion entering the cell. (:0)Answer: Calcium

51) The collection of pathways controlling a single joint is known as __________. The simplest reflex in this collection of pathways is the __________, which involves only two neurons: __________ neuron from the __________ and the __________ neuron to the muscle.

Answer: myotatic unit; monosynaptic stretch reflex, sensory, muscle spindle, somatic

35) The two types of cholinergic receptors are __________ and __________.

Answer: nicotinic, muscarinic 36) Cholinergic receptors respond to the neurotransmitter __________. (:0)Answer:A cetylcholine 37) Cholinergic nicotinic receptors are found in the __________ of the ANS. (:0)Answer:A utonomic gangliz 38) Cholinergic muscarinic receptors are found at the __________ in the ANS. (:0)Answer: parasympathetic neuroeffector junctions 39) The __________ is the major source of parasympathetic output. (:0)Answer: vagus nerve 40) __________ are swellings that contain vesicles filled with neurotransmitter. (:0)Answer: varicosities 41) Autonomic neurotransmitters are synthesized in the __________. (:0)Answer:A xon

119) When a nitrogenous base is bonded to a pentose sugar and a phosphate, a __________ is formed.

Answer: nucleotide

118) The __________ of a solution is the negative logarithm of the hydrogen ion concentration, expressed in moles per liter of solution.

Answer: pH

154) What is the pH of a 0.005 M HCl solution? Assume complete dissociation.

Answer: pH = 2.3. If pH = - log [H+] and HCl is a strong acid, we can assume complete dissociation will occur in solution.

92) A molecule positioned on the high-concentration side of a concentration gradient stores __________ energy.

Answer: potential 93) A chemical reaction that can proceed in both directions is called a __________. (:0)Answer: reversible reaction 94) Metabolism is often divided into __________, energy-producing reactions, and __________, energy-utilizing reactions. (:0)Answer: Catabolism, anabolism 95) Chemical reactions that require an input of energy are said to be __________. (:0)Answer: Endergonic 96) Chemical reactions that release energy are said to be __________. (:0)Answer: Exergonic 97) A(n) __________ is a covalent bond that stores an unusually large amount of energy. (:0)Answer: high-energy bond 98) The rates of chemical reactions that occur in the human body are controlled by special molecules called __________. (:0)Answer: Enzymes 99) The enzymatic reactions of metabolism form a network of interconnected biochemical __________ or reactions. (:0)Answer: pathways 100) A certain molecule that participates in more than one biochemical pathway and acts as a branch point for channeling substrate in one direction or another is called __________. (:0)Answer: key intermediate

77) Nucleoli function in the production of __________.

Answer: ribosomes / rRNA 78) Cell membranes are said to be __________ because they allow some substances to pass but not others. (:0)Answer: selectively permeable 79) The nucleus is surrounded by a(n) __________. (:0)Answer: nuclear envelope or double membrane 80) Communication between the nucleus and cytosol occurs through __________. (:0)Answer: nuclear pores 81) Cells are transformed into specialized units during __________. (:0)Answer: Differentiation

166) What terms are used to distinguish the two strands of the DNA molecule?

Answer: sense strand and nonsense strand

40) The principal ions in the ECF are __________.

Answer: sodium, chloride, and bicarbonate 41) A(n) __________ consists of a combination of a weak acid and its dissociation products. (:0)Answer: Buffer system 42) The normal pH range for most body fluids is __________. (:0)Answer: 7.35 to 7.45 43) When the pH rises above 7.45, a state of __________ exists. (:0)Answer:A lkalosis 44) Abnormal fat and amino acid metabolism may lead to the condition called __________. (:0)Answer: ketoacidosis

56) The thyroid gland is composed of many __________ that produce and store thyroid hormone.

Answer: thryroid follicles 57) Thyroid hormones are structural derivatives of the amino acid __________. (:0)Answer: tyrosine 58) __________ is a result of deficient thyroid hormone secretion in infancy. (:0)Answer: Cretinism 59) In bone, the calcium phosphate forms crystals of __________. (:0)Answer: hydroxyapatite 60) The hypothalamic-pituitary-adrenal pathway begins with secretion of __________ into the __________ and transported to the __________. (:0)Answer: Corticotropin-releasing-hormone (CRH), hypothalamic-hypophyseal portal system, anterior pituitary (See Fig. 23-3 in the CHAPTER.) 61) ACTH is synthesized from a large glycoprotein called __________, which also produces __________ in the pituitary. (:0)Answer: pro-opiomelanocortin (POMC), beta endorphin 62) GH stimulates the secretion of __________ from the liver and other tissue. This hormone has __________ feedback effect on growth hormone secretion. (:0)Answer: insulin-like growth factor; negative 63) List four factors that affect normal body growth. (:0)Answer: 1. growth hormone and other hormones 2. adequate diet 3. absence of stress 4. genetics 64) Name four physiological functions of calcium. (:0)Answer: 1. signal molecule 2. part of intercellular cement that holds cells together at tight junction 3. cofactor in the coagulation cascade 4. affects the excitability of neurons See Table 23-4 in the CHAPTER.

68) A __________ is an instrument that measures the volume of air moved with each breath.

Answer: spirometer 69) Air flows into lungs because of __________ created by __________. (:0)Answer: pressure gradients, pumps 70) List the muscles of inspiration and expiration. (:0)Answer: Inspiration: diaphragm, external intercostals, scalenes, and sternocleidomastoids. Expiration: internal intercostals, abdominal wall muscles. 71) Define lung capacity, and give a specific example. (Note: there is a specific physiological definition; don't just define "capacity" in general terms.) (:0)Answer:A lung capacity is the sum of two or more lung volumes. The lung capacities are total lung capacity, vital capacity, functional residual capacity, and inspiratory capacity. 72) List and describe the lung volumes. (:0)Answer: residual volume (quantity of air that cannot be expelled), expiratory reserve volume (quantity of air that is expelled only with forced expiration), tidal volume (quantity of air that is inhaled and exhaled under a specific condition, usually normal resting breathing), inspiratory reserve volume (quantity of air that can be inhaled with a forceful inspiration) 73) Explain how the upper airways and bronchi condition the air, and why this conditioning is important. (:0)Answer:A ir is warmed to body temperature to protect alveoli and avoid disrupting body temperature, air is humidified so that the exchange epithelium will not dehydrate, and foreign material is filtered out so it will not reach the alveoli. 74) During normal expiration, A) elastic recoil of stretched muscles helps return the thorax to its resting volume. B) the internal intercostal muscles are required. C) the abdominal muscles become involved. D) A and B E) A, B, and C (:0)Answer:A

50) Muscle spindles are __________ receptors and cause reflex __________, whereas Golgi tendon organs respond primarily to __________ and cause a __________ reflex.

Answer: stretch, contraction, tension, relaxation (Golgi tendon organs are relatively insensitive to muscle stretch.)

30) Diffusion rate is directly proportional to __________ and __________.

Answer: surface area, concentration gradient 31) Diffusion rate is indirectly proportional to __________. (:0)Answer: membrane thickness 32) Too little oxygen in cells is called __________. (:0)Answer: hypoxia 33) Too little oxygen in cells is often accompanied by too much __________, which is called __________. (:0)Answer: Carbon dioxide, hypercapnia 34) __________ is characterized by a decreased surface area for gas exchange in the lungs. (:0)Answer: Emphysema 35) __________ is characterized by a thickened alveolar membrane, slowing respiratory gas exchange. (:0)Answer: Fibrotic lung disease 36) In __________, fluid accumulates in the interstitial spaces of the lungs, slowing gas exchange. (:0)Answer: pulmonary edema 37) __________ is characterized by an increased airway resistance and decreased ventilation. (:0)Answer:A sthma 38) Generally, PO2 in arterial blood is __________ than PO2 in venous blood. (:0)Answer: higher 39) Generally, PCO2 in arterial blood is __________ than PCO2 in venous blood. (:0)Answer: lower

40) The rise in body temperature triggers two thermoregulatory mechanisms: __________ and __________.

Answer: sweating, increased cutaneous blood flow 41) In addition to the number and type of muscle fibers in a muscle, peak athletic performance requires A) a good blood supply and system of blood delivery. B) a well-developed respiratory system. C) adequate numbers of muscle mitochondria. D) good supplies of nutrients. E) all of the above (:0)Answer: E

111) Amino acids are carried to the ribosomes to be incorporated into polypeptide chains by __________.

Answer: tRNA 112) The enzyme __________ and __________ ions are required for the synthesis of mRNA. (:0)Answer: RNA polymerase, magnesium or manganese 113) A molecule of __________ consists of all the codons needed to produce a specific polypeptide chain at the ribosome. (:0)Answer: mRNA 114) Ribosomes are composed of protein and __________. (:0)Answer: rRNA 115) RNA is usually __________ -stranded. (:0)Answer: single 116) Two types of RNA block translation of mRNA. They are __________ RNA and __________ RNA. (:0)Answer: small interfering, micro 117) Compare and contrast potential energy with kinetic energy. (:0)Answer: Potential energy is the energy an object has stored in its position while kinetic energy is energy associated with a moving object / movement. 118) Define the law of mass reaction. (:0)Answer: When a reaction is at equilibrium, the ratio of the substrate to the products is always the same. 119) What are the advantages of using ATP as the energy-transferring molecule in a cell? (:0)Answer:A TP easily acquires the energy stored in complex biomolecules such as carbohydrates; its bonds are readily broken and it releases just the right amount of energy for cell needs so that the cell is protected from excessive energy release.

56) The male gonad is called a __________.

Answer: testis (testicle) 57) The ovaries produce eggs also known as __________. (:0)Answer: ova 58) Sperm cells are produced by the process of __________. (:0)Answer: spermatogenesis (gametogenesis) 59) The __________ of the sperm contains microtubules and mitochondria. (:0)Answer: midpiece 60) The tail of the sperm is actually a(n) __________. (:0)Answer: flagellum 61) A normal sperm count ranges from __________ spermatozoa per milliliter. (:0)Answer: 20 to 100 million 62) The __________ is the part of the sperm that contains the enzymes necessary for fertilization. (:0)Answer:A crosomal cap(or acrosome) 63) The __________ are fingerlike projections at the end of the uterine tube. (:0)Answer: fimbriae 64) The __________ is the inner lining of the uterus. (:0)Answer: Endometrium 65) __________ is the process of sloughing off the old functional layer of the endometrium. (:0)Answer: menstruation 66) __________ is the time when female menstrual cycles stop completely. (:0)Answer: menopause 67) __________ is the time spent in prenatal development. (:0)Answer: gestation

84) Three tissues—__________, __________, and __________—do NOT have traditional capillaries. Instead they have modified vessels called __________ that are (wider/narrower) than a capillary.

Answer: the bone marrow, the liver, the spleen; sinusoids, wider

78) A sensory receptor is a __________ that converts a stimulus into electrical graded potentials.

Answer: transducer 79) A stimulus below the __________ will stimulate sensory neurons, but the signal will not reach the conscious brain because higher neurons dampen it. (:0)Answer: perceptual threshold 80) The brain's association of a receptor to a particular sensation is called __________. (:0)Answer: labeled line coding 81) Sensory neurons that respond to continuous stimulation are called __________. (:0)Answer: tonic receptors

136) In which situation would the stroke volume be the greatest? A) when venous return is increased B) when venous return is decreased C) when the force of contraction is decreased D) when the difference between the end-diastolic volume and the end-systolic volume is small E) when calcium channel blockers are present

Answer:A

36) When fluid flow through the distal tubule increases as a result of increased GFR, the macula densa cells send a chemical message to the neighboring afferent arteriole. The afferent arteriole constricts, increasing resistance and decreasing GFR. This type of autoregulation involving both the kidney tubule and the arteriole is known as __________.

Answer: tubuloglomerular feedback 37) The __________ branch off the __________ and supply blood to the kidneys. (:0)Answer: renal arteries, abdominal aorta 38) The __________ carry blood from the kidneys back to the __________. (:0)Answer: renal veins, inferior vena cava 39) Eighty percent of the nephrons in a kidney are contained within the __________, but the other 20%, called the __________ nephrons, dip down into the __________. (:0)Answer: Cortex, juxtamedullary, medulla 40) The __________ are the long peritubular capillaries that dip into the medulla. (:0)Answer: vasa recta 41) The nephron begins with a hollow, ball-like structure called __________. (:0)Answer: Bowman's capsule 42) The __________ lie between and around the glomerular capillaries. (:0)Answer: mesangial cells 43) The specialized cells found in the capsule epithelium are called __________. These cells have long cytoplasmic extensions called __________. (:0)Answer: podocytes; foot processes 44) Neural control of GFR is mediated by __________ that innervate __________ receptors on vascular smooth muscle causing __________. (:0)Answer: sympathetic neurons, alpha, vasoconstriction 45) The excretion of glucose in the urine is called __________. (:0)Answer: glucosuria or glycosuria

45) The endothelium of the blood vessels and its adjacent connective tissue make up the __________, which is surrounded by layers of smooth muscle and connective tissue called __________.

Answer: tunica intima or intima, tunica media 46) Capillaries lack smooth muscle and elastic or fibrous tissue reinforcement. Instead, their walls consist of __________, supported on a cellular matrix called __________. (:0)Answer:A flat layer of endothelium, the basal lamina. 47) Cerebral capillaries are surrounded by __________ and glial cells and have tight junctions that create the __________. (:0)Answer: pericytes, blood-brain barrier 48) Venules are similar to capillaries: they have __________ epithelium and little __________. They are distinguished from capillaries by __________. (:0)Answer:A thin exchange, connective tissue (Fig. 15-2); their convergent pattern of flow

31) After it is formed, urine is temporarily stored in the __________.

Answer: urinary bladder 32) Urine is carried to the external environment by the __________. (:0)Answer: urethra 33) The plasma concentration at which all of the renal carriers for a given substance are saturated is the __________. (:0)Answer: renal threshold 34) The amount of filtrate entering the proximal tubules of the kidneys each minute is the __________. (:0)Answer: GFR or glomerular filtration rate 35) The percentage of total plasma volume that filters is called the __________. (:0)Answer: filtration fraction

76) The chambers of the heart that pump blood into the arteries are the __________.

Answer: ventricles 77) The chambers of the heart that receive blood from the veins are the __________. (:0)Answer:A tria 78) The most proximal arteries to branch from the aorta are the __________. (:0)Answer: Coronary arteries 79) An increase in blood vessel diameter is known as __________. (:0)Answer: vasodilation 80) The contraction phase of the cardiac cycle is termed __________. (:0)Answer: systole 81) A resting heart rate of less than 60 beats per minute is identified as __________. (:0)Answer: Bradycardia 82) A resting heart rate above 100 beats per minute is identified as __________. (:0)Answer: tachycardia 83) A heart with cells contracting rapidly in a disorganized manner, with no effective pumping action, is said to be in __________. (:0)Answer: fibrillation 84) A tracing of the electrical activity of the heart, monitored by electrodes placed on the skin, is called a(n) __________. (:0)Answer: Electrocardiogram

10) During meiosis, the chromosome number is halved A) during the first meiotic division. B) during the second meiotic division. C) prior to the first meiotic division. D) prior to the first mitotic division.

Answer:A

10) The H zone contains A) thick filaments. B) thin filaments. C) an area of overlapping filaments. D) all of the above E) none of the above

Answer:A

10) The hormone that directly controls water excretion by the kidneys is A) ADH. B) aldosterone. C) epinephrine. D) ANP. E) angiotensin.

Answer:A

10) When a chemical reaction is in equilibrium A) there is no net change in the amount of reactants or products. B) the reaction is proceeding at its maximum rate. C) the reaction has stopped. D) there are equivalent amounts of reactants and products. E) A and D

Answer:A

105) Lipids are hydrophobic, and do not usually dissolve in water. Because blood is water-based, the lipid cholesterol is combined with __________ so that it can be transported by blood. A) a hydrophilic molecule B) a hydrophobic molecule C) nothing; cholesterol is not transported in blood D) a cation E) an anion

Answer:A

107) In a pancreatic cell producing digestive enzymes, you would expect to find an elaborate A) rough endoplasmic reticulum. B) smooth endoplasmic reticulum.

Answer:A

109) Chemical reactions that occur in the human body proceed at a faster rate due to special catalytic molecules called A) enzymes. B) cytozymes. C) cofactors. D) activators. E) none of the above

Answer:A

11) Cells of the mononuclear phagocyte system include A) macrophages and monocytes. B) neutrophils and eosinophils. C) lymphocytes and plasma cells. D) A and B E) A, B, and C

Answer:A

11) The "normal" contractile fibers of the muscle are also called the A) extrafusal fibers. B) intrafusal fibers. C) muscle spindle fibers

Answer:A

11) When you are dehydrated, your kidneys alter their function to retain more water than they do when you are normally hydrated. This change in function is an example of a __________ mechanism. A) compensatory B) mandatory C) pathophysiological D) pathological E) teleological

Answer:A

11) Which parameters are associated with increased resistance? A) reduced flow B) reduced friction C) increased blood vessel diameter D) A and B E) A, B, and C

Answer:A

110) blocks cholesterol absorption

Answer:A

113) Close examination of an organ reveals a lining of several layers of cells. The layers do not contain any blood vessels, and one surface of the cells faces the internal cavity of the organ. This tissue is probably A) epithelium. B) muscle tissue. C) connective tissue. D) neural tissue. E) fat tissue.

Answer:A

146) During fasting, some capillary beds in the digestive tract are closed and therefore have no blood flow. Others are open to meet the minimal need of the tract for blood flow. During eating and as long as food is present in the tract, all capillary beds open to flow. What happens to total cross-sectional area of vessels in the digestive tract?

Answer:A

148) If total cross-sectional area of vessels in an organ remains the same but blood flow to that organ increases, what happens to velocity of blood?

Answer:A

15) Chronic pain A) is a pathological pain also known as neuropathic pain. B) is easily treated by analgesic drugs. C) does not reflect long-term changes in the nervous system. D) all of the above E) none of the above

Answer:A

15) Each of the following is an example of a nonmembranous organelle except one. Identify the exception. A) lysosome B) cilia C) centriole D) ribosome E) cytoskeleton

Answer:A

15) In the intestine, the epithelial cells have A) leaky junctions. B) tight junctions. C) electrical gap junctions.

Answer:A

15) Leukocytes recognize molecules that are unique to pathogens; thus, the molecules are A) pathogen-associated molecular patterns. B) pattern recognition receptors. C) toll-like receptors. D) phagosomes. E) antigens.

Answer:A

15) The pituitary hormone that triggers the release of thyroid hormone from the thyroid gland is A) TSH. B) ACTH. C) FSH. D) TRH. E) CRH.

Answer:A

16) A technique used to resolve contradictory results in scientific studies is A) meta-analysis. B) retrospective analysis. C) prospective analysis. D) cross-sectional analysis. E) longitudinal analysis.

Answer:A

16) During mild to moderate exercise, arterial pH, PO2, and PCO2 are A) nearly normal. B) nearly half of normal. C) almost twice as high as normal.

Answer:A

16) The thickest protein fibers from the following group are A) microtubules. B) neurofilaments. C) microfilaments. D) myosin molecules. E) keratin filaments.

Answer:A

17) As ATP binds to the myosin head at the beginning of a muscle contraction cycle, A) the myosin head detaches from actin. B) the myosin head initiates binding with actin. C) the myosin head tightens its bond to actin. D) ATP does not bind to the myosin head. E) None of these complete the statement correctly.

Answer:A

17) D cells in the islet of Langerhans secrete A) somatostatin. B) amylin. C) insulin. D) glucagon. E) pancreatic polypeptide.

Answer:A

17) If cardiac output increases and resistance in arterioles does NOT change, what happens to arterial blood pressure? A) increases B) decreases C) is unchanged

Answer:A

17) In the condition known as cryptorchidism, A) the testes fail to descend into the scrotum. B) the testes are surgically removed. C) the foreskin has been surgically removed. D) sperm cells are not produced. E) the prostate gland is enlarged.

Answer:A

17) The motor end plate is A) a folded area of muscle cell membrane with ACh receptors clustered at the top of each fold. B) the same as the neuromuscular junction. C) the same as the synaptic cleft. D) formed by the membrane of enlarged axon terminals, or boutons, that lie on the surface of skeletal muscle cells. E) a special fibrous matrix whose collagen fibers hold the axon terminal in proper position.

Answer:A

17) When the diaphragm and external intercostal muscles contract, A) the volume of the thorax increases. B) the volume of the thorax decreases. C) the volume of the lungs decreases. D) the lungs collapse. E) expiration occurs.

Answer:A

18) Diseases that involve the presence of too many white blood cells have been identified as well as diseases that involve too few white blood cells. A) true B) false

Answer:A

18) Mature, less active bone cells are termed A) osteocytes. B) osteoblasts. C) osteoclasts. D) chondrocytes. E) osteons.

Answer:A

18) When blood glucose levels rise, as in the "fed" state, A) insulin is released. B) glucagon is released. C) peripheral cells take up less glucose. D) protein synthesis decreases. E) all of the above

Answer:A

19) Air moves into the lungs because A) the gas pressure in the lungs is less than outside pressure. B) the volume of the lungs decreases with inspiration. C) the thorax is muscular. D) contraction of the diaphragm decreases the volume of the pleural cavity. E) all of the above

Answer:A

19) An allosteric modulator binds to A) a region of the enzyme other than the active site. B) the active site. C) the substrate. D) the product. E) the surrounding tissue.

Answer:A

19) Cold receptors have been shown to respond also to A) menthol. B) capsaicin. C) alcohol. D) water.

Answer:A

19) Elevated levels of calcium ion in the blood stimulate the secretion of the hormone A) calcitonin. B) thyroid hormone. C) parathyroid hormone. D) growth hormone. E) testosterone.

Answer:A

19) The action potential in a cardiac contractile cell causes A) opening of L-type calcium channels. B) opening of ryanodine receptor calcium channels. C) activation of sodium-potassium ATPase. D) activation of NCX transporters. E) opening of HCN channels.

Answer:A

19) The force for glomerular filtration is the A) blood pressure in the glomerular capillaries. B) osmotic pressure in the glomerular capillaries. C) fluid pressure produced by the displacement of the fluid in the lumen of the tubules. D) ATP-dependent processes in the nephron. E) none of the above

Answer:A

2) Perfusion is A) blood flow through an organ. B) the driving force behind blood flow. C) delivery of oxygen to cells. D) the connection between capillaries and other vessels. E) movement of blood through a shunt.

Answer:A

2) Plasma is mostly A) water. B) blood cells. C) proteins. D) organic molecules. E) ions.

Answer:A

2) Ventilation is also known as A) breathing. B) inspiration. C) expiration. D) air conduction. E) blowing.

Answer:A

2) Which function(s) is/are NOT continuously regulated by the GI tract? A) absorption B) motility C) secretion D) All of the above are regulated. E) None of the above are regulated.

Answer:A

20) When the concentration of sodium ion in the ECF increases, A) osmoreceptors are stimulated. B) there is a decreased thirst. C) ADH secretion decreases. D) aldosterone secretion increases. E) there is an increase in the volume of urine produced.

Answer:A

21) The structure that carries sperm from the epididymis to the urethra is the A) ductus deferens. B) rete testis. C) seminal vesicle. D) ejaculatory duct. E) corpus cavernosum.

Answer:A

22) Which of the following statements about autoregulation is NOT true? A) Myogenic response is the intrinsic ability of vascular smooth muscle to respond to pressure changes. B) Myogenic response is a paracrine signaling mechanism. C) In tubuloglomerular feedback, stretch-sensitive ion channels open, resulting in depolarization of smooth muscle cells. D) In myogenic response, the macula densa cells send a paracrine message to the neighboring afferent arteriole.

Answer:A

23) Air entering the body is filtered, warmed, and humidified by the A) upper respiratory tract. B) lower respiratory tract. C) lungs. D) alveoli. E) all of the above

Answer:A

23) An enzyme that adds or subtracts water molecules is A) hydrolase. B) lipase. C) lyase. D) ligase. E) kinase.

Answer:A

23) Sweat glands contain A) cholinergic receptors. B) alpha-1 receptors. C) alpha-2 receptors. D) beta receptors. E) all of the above

Answer:A

23) __________ are pacemakers for slow wave activity. A) Interstitial cells of Cajal B) Intrinsic neuron cells C) Extrinsic neuron cells D) G cells E) Chief cells

Answer:A

24) Most preganglionic neurons originate in the thoracic and lumbar regions of the spinal cord.

Answer:A

24) Reactive hyperemia is triggered by A) local accumulation of paracrines due to reduced blood flow. B) reflex contraction of smooth muscle following a period of dilation. C) stress and the hormones released during stress. D) toxins or allergens, such as bee venom or pollen. E) none of the above

Answer:A

24) The flattening of the action potentials of myocardial contractile cells, called the plateau phase, is due to a combination of increasing Ca2+ __________ and decreasing K+__________. A) influx, efflux B) efflux, influx C) influx, influx D) efflux, efflux

Answer:A

24) The function of hemoglobin is to A) carry oxygen. B) protect the body against infectious agents. C) aid in the process of blood clotting. D) carry nutrients from the intestine to the body's cells. E) all of the above

Answer:A

25) The elevated blood pressure that sometimes accompanies pregnancy is known as A) pre-eclampsia. B) vasovagal syncope. C) infarction. D) reactive hyperemia. E) orthostatic hypotension.

Answer:A

25) __________ is the neurotransmitter that excites gustatory neurons. A) Serotonin B) Gustducin C) Acetylcholine D) Glutamine E) Muscarine

Answer:A

26) CRH stimulates the release of A) ACTH. B) cortisol. C) DHEA. D) MSH. E) none of the above

Answer:A

27) During exercise, skeletal muscle A) vasodilation decreases peripheral resistance to blood flow. B) vasodilation increases peripheral resistance to blood flow. C) vasoconstriction decreases peripheral resistance to blood flow. D) vasoconstriction increases peripheral resistance to blood flow. E) none of the above

Answer:A

27) Passive immunity is a type of __________ immunity. A) acquired B) innate

Answer:A

27) Under normal circumstances, aged and damaged erythrocytes are broken down by the A) spleen. B) yellow bone marrow. C) kidneys. D) digestive tract. E) thymus gland.

Answer:A

27) excite both alpha and gamma motor neurons

Answer:A

27) increased pH

Answer:A

28) In the digestive system, HCl is released by __________, whereas HCO3- is secreted primarily from the __________. A) parietal cells of the stomach, pancreas B) the pancreas, parietal cells of the stomach C) parietal cells of the stomach, liver D) the liver, parietal cells of the stomach

Answer:A

28) Myocardial cells can generate action potentials spontaneously because they have A) unstable ion channels. B) permanently open channels for Na+ and K+. C) a net influx of Na+. D) A and B E) A, B, and C

Answer:A

28) The ganglia are found in a chain that runs close to the spinal cord or along the descending aorta.

Answer:A

28) Which of the following consists of a network of intracellular membranes with attached ribosomes? A) rough endoplasmic reticulum B) smooth endoplasmic reticulum C) mitochondria D) nucleoli E) Golgi apparatus

Answer:A

29) Bitter and sweet taste transduction A) involves a specialized G protein, gustducin. B) involves a specialized G protein, glutamin. C) is a relatively simple and straightforward pathway. D) A and C E) B and C

Answer:A

29) Releases norepinephrine at the neuroeffector synapse.

Answer:A

29) Stem cells that will form B lymphocytes are found in the A) bone marrow. B) liver. C) spleen. D) thymus. E) kidneys.

Answer:A

29) The enzyme renin is responsible for the activation of A) angiotensin. B) cortisol. C) erythropoietin. D) atrial natriuretic peptide. E) adrenaline.

Answer:A

29) have gamma motor neurons that innervate internal fibers

Answer:A

3) Capillaries are best described as A) microscopic vessels in which blood exchanges material with the interstitial fluid. B) thick walled vessels that convey blood away from the heart. C) thin walled vessels that convey blood toward the heart. D) thick walled vessels that carry blood rich in oxygen. E) thin walled vessels that carry blood deficient in oxygen.

Answer:A

3) In the body, 80% of all lymphocytes, a type of immune system cell, are thought to be present in the A) small intestine. B) large intestine. C) appendix. D) stomach. E) mouth.

Answer:A

3) The characteristic yellow color of urine is attributed to the presence of A) urobilinogen. B) uric acid. C) urea. D) renin. E) bile.

Answer:A

3) The division of the autonomic nervous system that prepares the body for intense levels of activity and stress is the A) sympathetic division. B) parasympathetic division. C) craniosacral division. D) intramural division. E) somatomotor division.

Answer:A

3) The origin of a particular skeletal muscle is defined as A) the attachment site to the less movable bone. B) the attachment site to the more movable bone. C) the embryonic structure from which it initially developed.

Answer:A

30) Muscle fatigue is known to be associated mainly with A) depletion of glycogen stores. B) depletion of ATP stores. C) depletion of Na+ stores. D) A and B E) A, B, and C

Answer:A

30) Total cross-sectional area __________ with each division of the airways. A) increases B) decreases C) does not change

Answer:A

30) eventually synapses with alpha motor neurons that innervate extrafusal muscle fibers

Answer:A

30) small plasma proteins

Answer:A

31) Amylin A) is co-secreted with insulin. B) is co-secreted with glucagon. C) helps regulate glucose homeostasis by speeding up gastric emptying. D) A and C E) B and C

Answer:A

31) Blood vessels cover approximately __________% of the alveolar surface. A) 80-90 B) 10-20 C) > 90 D) < 10 E) 50

Answer:A

31) The depolarization of the pacemaker action potential spreads to adjacent cells through A) gap junctions. B) tight junctions. C) chemical synapses. D) desmosomes.

Answer:A

31) The osmolarity in the deepest part of the loop of Henle is __________ mOsM. A) 1200 B) 100 C) 300 D) 900 E) none of the above

Answer:A

31) Which form of contraception allows fertilization but prevents implantation? A) IUD B) pill C) female condom D) diaphragm E) sponge

Answer:A

32) Bile is A) secreted by hepatocytes. B) manufactured by the gall bladder. C) released into the stomach. D) A and B E) A, B, and C

Answer:A

32) Decreased ECF volume causes A) sympathetic output from the cardiovascular control center to increase. B) parasympathetic output from the cardiovascular control center to increase. C) the force of ventricular contraction to decrease. D) arteriolar vasodilation. E) B and D

Answer:A

32) In the lungs, the A) blood flow rate is higher and the blood pressure is lower, respectively, than the blood flow rate and the blood pressure in other tissues. B) blood flow rate is higher and the blood pressure is higher, respectively, than the blood flow rate and the blood pressure in other tissues. C) blood flow rate is lower and the blood pressure is lower, respectively, than the blood flow rate and the blood pressure in other tissues. D) blood flow rate is lower and the blood pressure is higher, respectively, than the blood flow rate and the blood pressure in other tissues. E) blood flow rate and the blood pressure are the same as in other tissues.

Answer:A

32) Most excess iron in the body is A) eliminated in the urine. B) stored in the bones. C) stored in the liver. D) eliminated in the feces. E) stored in fatty tissues.

Answer:A

32) Polymers are a typical formation of __________ molecules. A) organic B) inorganic C) either organic or inorganic

Answer:A

32) The fibrous skeleton of the heart is important because it A) forces electrical activity to be conducted through the atrioventricular node. B) transmits electrical activity from the atria to the ventricles. C) helps guide the blood into the proper chambers in sequence. D) directs the flow of blood into the arteries.

Answer:A

32) The ion channels for hearing are in or on the A) stereocilia. B) tectorial membrane. C) helicotrema. D) cochlear duct. E) cochlear neurons.

Answer:A

33) Important during stress or emergencies (fight-or-flight).

Answer:A

33) Peroxisomes A) use an enzyme to destroy H2O2 that is toxic to the cell. B) are a type of lysosome. C) are responsible for the atrophy of unused muscles. D) are sites for synthesis of fatty acids, steroids, and phospholipids. E) All of the above accurately describe peroxisomes.

Answer:A

33) Sperm cannot fertilize an egg until they A) undergo capacitation. B) undergo activation. C) undergo decapitation. D) lose their acrosome. E) are in the vagina for 3 days.

Answer:A

33) The distance between the alveolar air space and capillary endothelium is __________, allowing gases to diffuse __________ between them. A) short, rapidly B) long, rapidly C) short, slowly D) long, slowly E) None of the above—distance does not affect diffusion rate.

Answer:A

34) A motor unit consists of A) one neuron and the muscle fibers it controls. B) one nerve and the skeletal muscles it controls. C) one muscle fiber and all the neurons that control it. D) one nerve and the body region it controls. E) one axon terminal branch and the myofibrils it controls.

Answer:A

34) A parasitic condition resulting in extreme enlargement of one or both legs is called A) elephantiasis. B) elephantitis. C) elephantosis. D) ascites.

Answer:A

34) Flow of air A) is directly proportional to a pressure gradient, and flow decreases as the resistance of the system increases. B) is directly proportional to the resistance, and flow decreases as the pressure of the system increases. C) is directly proportional to a pressure gradient, and flow increases as the resistance of the system increases. D) is directly proportional to the resistance, and flow increases as the resistance of the system increases. E) none of the above

Answer:A

34) Most heat is lost from the body through the process of A) radiation. B) conduction. C) convection. D) evaporation. E) concentration.

Answer:A

34) The ossicles connect the A) tympanic membrane to the oval window. B) tympanic membrane to the round window. C) oval window to the round window. D) cochlea to the tympanic membrane. E) cochlea to the oval window.

Answer:A

34) mean arterial pressure during exercise

Answer:A

35) An increase in PCO2 would cause A) the bronchioles to dilate and the systemic arterioles to dilate. B) the bronchioles to dilate and the systemic arterioles to constrict. C) the bronchioles to constrict and the systemic arterioles to dilate. D) the bronchioles to constrict and the systemic arterioles to constrict. E) none of the above

Answer:A

35) The control center for cellular operations is the A) nucleus. B) mitochondria. C) Golgi complex. D) endoplasmic reticulum. E) ribosomes.

Answer:A

35) relative depression of immune system with change from moderate to strenuous exercise

Answer:A

37) Blood pressure and flow to the brain are monitored by receptors located in the wall of the A) carotid artery. B) aorta. C) venae cavae. D) cerebral arteries. E) cephalic arteries.

Answer:A

37) Electrical shock to the heart is usually used to treat A) ventricular fibrillation. B) atrial fibrillation. C) heart block. D) heart murmur. E) myocardial infarction.

Answer:A

37) The term secretion refers to A) the process by which a cell releases a substance into the extracellular space. B) synthesis of a protein for export from the cell. C) the manufacture and assembly of a material. D) storage of a material, until it is time for it to leave the cell. E) None of the above describe secretion.

Answer:A

37) Which fibers generate more force? A) fast-twitch fibers B) slow-twitch fibers

Answer:A

38) A person who suffers from emphysema will exhibit signs of A) respiratory acidosis. B) respiratory alkalosis. C) metabolic acidosis. D) metabolic alkalosis. E) none of the above

Answer:A

38) Which fibers fatigue faster? A) fast-twitch fibers B) slow-twitch fibers

Answer:A

39) During gestation, powerful uterine contractions are suppressed by elevated levels of A) progesterone. B) estrogen. C) oxytocin. D) prostaglandins. E) all of the above

Answer:A

39) Which of the following statements is true about GI muscle contractions? A) Phasic contractions last only seconds and occur in the stomach and small intestine. B) Tonic contractions are sustained for minutes and occur in the small intestine. C) Cycles of smooth muscle contraction and relaxation are associated with fast wave potentials. D) Contractions of the smooth muscle do not depend on calcium. E) none of the above

Answer:A

4) According to the second law of thermodynamics, A) the amount of entropy increases during natural processes. B) entropy decreases in living systems as a result of cellular processes. C) energy can be converted, but not be created nor destroyed. D) the total amount of energy in the universe never changes.

Answer:A

4) Energy input equals energy output. This statement is known as the A) first law of thermodynamics. B) second law of thermodynamics. C) law of mass balance.

Answer:A

4) Which organ is NOT known to include a special portal system for blood? A) heart B) kidney C) liver D) brain E) all of the above have portal systems

Answer:A

4) Which reflex is NOT controlled by the brainstem? A) urination B) vomiting C) sneezing D) swallowing E) salivating

Answer:A

40) Which of the following statements is true? A) Glucose and galactose absorption uses an apical Na+-glucose SGLT symporter. B) Fructose moves across the apical membrane by active transport. C) Glucose and galactose use different transporters in absorption. D) A basolateral GLUT5 transporter moves glucose out of the intestinal epithelial cell. E) none of the above

Answer:A

41) Immunoglobulins that are primarily found in glandular secretions are A) IgA. B) IgD. C) IgE. D) IgG. E) IgM.

Answer:A

42) In the body, the bones act as __________ whereas the joints form the __________, so that the muscles attached to the bones can create force when they contract. A) levers, fulcrums B) fulcrums, levers C) levers, pulleys D) fulcrums, pulleys

Answer:A

42) Tissue factor is released in the A) extrinsic pathway. B) intrinsic pathway. C) common pathway. D) retraction pathway. E) fibrinolytic pathway.

Answer:A

42) the amount of air taken in during a single normal inspiration

Answer:A

43) This particle has a charge of +1 and a mass of 1.

Answer:A

44) the sum of all the lung volumes

Answer:A

45) caused by growth hormone excess in adulthood

Answer:A

45) located at the ventral end of the cecum

Answer:A

47) The primary calcium release channel in smooth muscle sacroplasmic reticulum is A) an IP3 receptor channel. B) a caveolae. C) a DHP receptor. D) a ryanodine receptor. E) none of the above

Answer:A

48) organ that adds exocrine secretions to the duodenum via a duct

Answer:A

49) The drug Chantix, which treats nicotine addiction, is an __________ for the nicotine receptor. A) agonist B) antagonist

Answer:A

49) mature sperm or egg

Answer:A

49) phosphorus

Answer:A

5) Pulmonary ventilation refers to the A) movement of air into and out of the lungs. B) movement of dissolved gases from the alveoli to the blood. C) movement of dissolved gases from the blood to the interstitial space. D) movement of dissolved gases from the interstitial space to the cells. E) utilization of oxygen.

Answer:A

5) The atomic number of an element is equivalent to A) the number of protons in the nucleus. B) the number of neutrons in the nucleus. C) the number of protons plus neutrons in the nucleus. D) the number of electrons in the outer shell. E) the electrical charge of an atom.

Answer:A

5) X-linked genes are expressed A) more often in males. B) more often in females. C) at equal rates in the two genders.

Answer:A

50) Every substance that enters or leaves the internal environment of the body must cross an epithelium. A) true B) false

Answer:A

50) If blood flow through the afferent arterioles increases, A) stretch reflexes trigger vasoconstriction to reduce the flow. B) the smooth muscle in the vessel walls stretches to accommodate the increased flow. C) the stretch triggers further relaxation of the arteriolar wall, lessening blood pressure. D) B and C

Answer:A

50) The term that describes the volume of blood circulated by the heart in one minute is A) cardiac output (CO). B) heart rate (HR). C) end-diastolic volume (EDV). D) stroke volume (SV). E) end-systolic volume (ESV).

Answer:A

50) Which structures monitor rotational movements of the head? A) cristae and the semicircular canals B) maculae of the saccule C) maculae of the utricle D) B and C E) all of the above

Answer:A

51) Due to the fact that they divide frequently, epithelia are prone to the genetic mutations associated with cancer. A) true B) false

Answer:A

51) During ventricular ejection, A) the ventricles are in systole. B) the QRS complex is just starting. C) the semilunar valves are closed. D) A and B E) A, B, and C

Answer:A

51) secondary spermatocyte or oocyte

Answer:A

52) In the lumen of the proximal tubule, Na+ concentration __________ the Na+ concentration inside the cells of the tubule wall. A) is much higher than B) is slightly higher than C) is about the same as D) is slightly lower than E) is much lower than

Answer:A

52) Often called immunocytes, these cells direct the activities of the immune system.

Answer:A

52) decreased activity in digestive tract

Answer:A

53) C

Answer:A

55) Renal tubule cells in the kidney medulla are constantly exposed to high extracellular osmolarity. How do they maintain normal cell volume? A) They synthesize organic solutes as needed to match the osmolarity. B) They synthesize water molecules through increased metabolism to offset volume loss. C) They maintain a water-impermeable membrane. D) They add or remove aquaporins as needed.

Answer:A

55) testosterone

Answer:A

56) The darkest band, with the most overlapping filaments; its abbreviation derives from its property of scattering light unevenly.

Answer:A

58) Two hours before major surgery is to begin, the patient experiences "jitters," an elevated heart rate and blood pressure, increased rate of breathing, cold sweats, and an urge to urinate. These symptoms are the result of A) sympathetic activation. B) decreased levels of epinephrine in the blood. C) decreased activity of sympathetic centers in the hypothalamus. D) increased parasympathetic activity. E) all of the above

Answer:A

59) Dan has been lost in the desert for two days with very little water. As a result, you would expect to observe A) elevated ADH levels. B) decreased blood osmolarity. C) normal urine production. D) increased blood volume. E) cells enlarged with fluid.

Answer:A

59) The P wave of an ECG corresponds to A) the depolarization of the atria. B) the progressive wave of ventricular depolarization. C) the repolarization of the ventricles. D) atrial repolarization. E) none of the above

Answer:A

6) "Glucose is transported from blood into cells by transporters in response to insulin." This type of explanation is A) mechanistic. B) theological. C) teleological. D) metalogical. E) scatological.

Answer:A

6) The cell that is the progenitor of all the types of blood cells is called the A) pluripotent hematopoietic stem cell. B) committed progenitor cell. C) megakaryocyte. D) reticulocyte. E) progenitor.

Answer:A

6) Which of the following terms is NOT used to define the structure that separates the contents of a human cell from its surrounding medium? A) a cell wall B) a cell membrane C) plasma membrane D) plasmalemma E) all of the above

Answer:A

6) __________ is made of multiple globular molecules polymerized to form long chains or filaments. A) Actin B) Tropomyosin C) Troponin D) Myosin E) Titin

Answer:A

60) A codon is A) a meaningful series of three nucleotides found in mRNA. B) a meaningful series of three amino acids within a protein. C) three pairs of nucleotides in a sequence in mRNA. D) any three bases in RNA. E) any three bases in DNA.

Answer:A

60) The term meaning a mitotic population of cells that persists into adulthood is A) stem cells. B) totipotent cells. C) apoptosis. D) nurse cells. E) gametocytes.

Answer:A

61) A pigment molecule synthesized from vitamin A is A) retinal. B) opsin. C) rhodopsin. D) transducin. E) cGMP.

Answer:A

64) Transcription occurs in/on the __________ of the cell. A) nucleus B) ribosome C) Golgi apparatus D) smooth endoplasmic reticulum E) plasma membrane

Answer:A

64) the sequence and number of amino acids in the chain

Answer:A

65) Over time, mRNA is eventually broken down as its usefulness ends. Which enzymes destroy it? A) ribonucleases B) RNA polymerases C) DNA polymerases D) ATP synthases E) lactate dehydrogenases

Answer:A

67) Drugs used to treat diabetes may A) stimulate beta-cell secretion of insulin. B) stimulate digestion and absorption of carbohydrates in the intestine. C) stimulate hepatic glucose output. D) decrease responsiveness of target tissue to insulin. E) none of the above

Answer:A

68) In the fasted state, which of the following statements is FALSE? A) Skeletal muscle will increase gluconeogenesis. B) Pyruvate and lactate are transported to liver to make glucose. C) Some amino acids will be deaminated. D) Adipose breaks down its store of triglycerides.

Answer:A

68) These are probably the oldest sensory receptors as evidenced by their existence in primitive animals that do not have formalized nervous systems.

Answer:A

69) Increased blood calcium levels result in increased A) secretion of calcitonin. B) secretion of parathyroid hormone. C) retention of calcium by the kidneys. D) osteoclast activity. E) excitability of neural membranes.

Answer:A

69) The level of ketone bodies in the blood increases when high levels of __________ are being metabolized. A) fatty acids B) glucose C) proteins D) amino acids E) carbohydrates

Answer:A

7) Blood flow through the kidney includes a feature seen in only a few organs. What is it? A) portal system B) arterial shunts C) vascular sinuses D) veins containing highly oxygenated blood E) anastomoses

Answer:A

7) Cortisol secretion peaks between __________ and __________. A) early morning, noon B) midnight, early morning C) noon, early evening D) early evening, midnight

Answer:A

7) Differences between arterioles and metarterioles include the fact that arterioles A) have a continuous smooth muscle layer in their walls. B) allow blood to bypass capillary beds. C) have an endothelial lining. D) A and B E) A, B, and C

Answer:A

7) The hepatic portal vein carries blood away from the A) digestive tract. B) liver. C) kidneys. D) spleen. E) pancreas.

Answer:A

7) The largest lymphoid tissue(s) is/are the A) spleen. B) thymus. C) GALT. D) lymph nodes. E) tonsils.

Answer:A

7) The respiratory quotient is __________ for consumed carbohydrates compared to proteins and fats. A) highest B) lowest C) the same

Answer:A

70) Bill does not begin puberty until he is 16. What effect would you predict this will have on his stature? A) Bill will probably be taller than if he had started puberty earlier. B) Bill will probably be shorter than if he had started puberty earlier. C) Bill will probably be a dwarf. D) Bill will have bones that are denser than normal. E) The late onset of puberty will have no effect on Bill's stature.

Answer:A

71) structural components

Answer:A

73) covalent modulator

Answer:A

74) Malnutrition can cause edema because A) there are not enough nutrients for plasma protein synthesis. B) interstitial protein levels are lower than plasma protein levels. C) the resulting anemia increases blood pressure. D) A and B E) A, B, and C

Answer:A

74) Meghan thinks she has an abscessed tooth. If she does, what type of white blood cell would you expect to see in elevated numbers in a differential count? A) neutrophils B) eosinophils C) basophils D) lymphocytes E) monocytes

Answer:A

74) chondrocyte

Answer:A

77) Which of the following is greater? A) blood pressure when sympathetic stimulation to the heart increases B) blood pressure when parasympathetic stimulation to the heart increases

Answer:A

77) various touch and pressure stimuli

Answer:A

79) Complete the following sentence to describe the state of relaxed muscle fibers: myosin heads are A) weakly bound to actin, and contain ADP and Pi. B) at a 45 degree angle to actin filaments. C) bound to ATP and detached from actin filaments. D) tightly bound to actin and neither ATP nor ADP are bound. E) none of the above

Answer:A

79) Increasing the alveolar ventilation rate will A) increase the partial pressure of oxygen in the alveoli. B) decrease the rate of oxygen diffusion from the alveoli to the blood. C) increase the partial pressure of carbon dioxide in the alveoli. D) decrease the rate of carbon dioxide diffusion from the blood to the alveoli. E) have no effect on either the partial pressure or diffusion rate of gases.

Answer:A

79) Persons who suffer from hemophilia A fail to produce a functional factor VIII; as a result, A) they do not have a functional intrinsic pathway. B) they do not have a functional extrinsic pathway. C) they do not have a functional common pathway. D) their clotting times are shorter than normal. E) all of the above

Answer:A

79) wound healing, recovering from surgery

Answer:A

8) The layer of loose connective tissue beneath the digestive epithelium is the A) lamina propria. B) muscularis mucosae. C) submucosa. D) submucosal plexus. E) myenteric plexus.

Answer:A

8) When allowed to develop, the Mullerian ducts become the A) fallopian tubes, uterus, and upper vagina. B) distal vagina, clitoris, and labia. C) epididymis, vas deferens, and seminal vesicles. D) penis and scrotum. E) A and B

Answer:A

8) Which of the following is one of Cannon's "internal secretions"? A) hormones B) nutrients C) water D) inorganic ions E) none of the above

Answer:A

81) Meghan thinks she has an abscessed tooth (a bacterial infection). If she does, what type of white blood cell would you expect to see in elevated numbers in a differential count? A) neutrophils B) eosinophils C) basophils D) lymphocytes E) monocytes

Answer:A

81) carrying a pregnancy to term

Answer:A

82) An important difference between single-unit and multiunit smooth muscle is A) numerous gap junctions in single-unit smooth muscle, which allow many cells to work together as a sheet. B) longer actin and myosin filaments in multiunit smooth muscle, which allow coordination of contraction. C) the ability of single-unit fibers to change into multiunit fibers when advantageous. D) closely controlled individual fibers in single-unit smooth muscle to allow fine control and graded contractions by selective activation. E) all of the above

Answer:A

84) An embryo with a genital tubercle, urethral folds, and urethral groove must be A) under seven weeks old. B) seven weeks old. C) nine weeks old. D) unable to determine the age

Answer:A

85) carbonic anhydrase

Answer:A

85) increased respiratory rate and/or volume due to increased metabolism

Answer:A

86) In the late follicular phase of the ovarian cycle, LH is __________ FSH. A) greater than B) less than C) equal to

Answer:A

86) neutrophils

Answer:A

87) During the luteal phase of the ovarian cycle, progesterone is __________ estrogen. A) greater than B) less than C) equal to

Answer:A

9) Swelling and soreness of the lymph nodes indicate that A) immune cells are fighting infection. B) the immune system is incapable of working. C) dangerous cancerous changes are occurring in the immune system. D) too much or improper exercise has been undertaken. E) none of the above

Answer:A

90) Sperm cannot fertilize an egg unless they have been in the female reproductive tract for several hours. This enhanced ability of the sperm to fertilize the ovum is called capacitation. In vitro studies indicate that the sperm can fertilize the ovum if they are first washed before being introduced to the ovum. These observations suggest that A) capacitation involves the removal of some inhibiting substance(s) from the sperm. B) capacitation is temperature-dependent. C) in capacitation, secretions of the female reproductive tract are taken up by the sperm. D) pH is an important factor in the process of capacitation. E) only sperm that come into contact with vaginal secretions are capable of fertilizing an ovum.

Answer:A

91) Put the following steps of fat digestion and absorption in order: 1. Bile salts coat fat droplets. 2. Cholesterol is transported into cells. 3. Chylomicrons are formed. 4. Chylomicrons are released into lymphatic system. 5. Monoglycerides and fatty acids move out of micelles. 6. Pancreatic lipase and colipase break down fats. A) 1, 6, 5, 2, 3, 4 B) 1, 2, 3, 4, 6, 5 C) 2, 6, 5, 1, 3, 4 D) 6, 3, 4, 1, 2, 5 E) none of the above

Answer:A

94) In response to the arrival of acidic chyme in the duodenum, the blood levels of A) secretin rise. B) cholecystokinin fall. C) gastrin rise. D) enterocrinin fall. E) all of the above

Answer:A

97) In response to the hormone cholecystokinin, the pancreas secretes a fluid A) rich in enzymes. B) rich in bicarbonate. C) rich in bile. D) that contains only proteinases. E) that contains only amylase.

Answer:A

99) The designation Ca2+ means A) calcium has space for two more electrons. B) calcium has gained two more electrons. C) calcium has space for two more protons. D) calcium has gained two more protons. E) none of the above

Answer:A

Human Physiology, 5e (Silverthorn) CHAPTER 17 Mechanics of Breathing 1) Which is NOT considered to be a primary function of the respiratory system? A) regulation of water balance B) regulation of pH balance C) protection against pathogens D) vocalization E) gas exchange

Answer:A

Human Physiology, 5e (Silverthorn) CHAPTER 18 Gas Exchange and Transport 1) Of the factors that influence diffusion of respiratory gases, the most variable and, therefore, important factor to consider is the A) concentration gradient. B) diffusion distance. C) membrane surface area. D) membrane thickness. E) electrical charge.

Answer:A

Human Physiology, 5e (Silverthorn) CHAPTER 24 The Immune System 1) Which is NOT a pathogen? A) cancer B) hookworm C) athlete's foot fungus D) chicken pox virus E) meningococcus bacterium

Answer:A

Match each class of biomolecules to the correct statement about it. A. carbohydrates B. lipids C. proteins D. nucleotides 59) Glucose and ribose are examples; these molecules provide energy or structure.

Answer:A

Match each step in urine formation to its description. A. excretion B. filtration C. reabsorption D. secretion 23) movement from the nephron lumen to the external environment

Answer:A

Match each term with its definition. A. cardiac output (CO) B. heart rate (HR) C. end-diastolic volume (EDV) D. stroke volume (SV) E. end-systolic volume (ESV) 68) the volume of blood circulated by the heart in one minute

Answer:A

Match the change in gas composition with the response (disregard weak responses). A. PO2 increases. B. PCO2 increases. C. PO2 decreases. D. PCO2 decreases. 88) Arterioles: systemic constrict, pulmonary dilate.

Answer:A

Match the condition with its description. A. giantism B. kyphosis C. moon face D. myxedema E. goiter 48) caused by growth hormone excess in childhood

Answer:A

Match the following plasma proteins with the correct function. A. albumins B. globulins C. fibrinogen D. transferrin E. antibodies 43) contribute(s) significantly to osmotic pressure of plasma

Answer:A

Match the hormone with the correct statement. A. gastrin B. cholecystokinin C. secretin D. motilin E. gastric inhibitory peptide 102) secreted by cells in the stomach

Answer:A

Match the nutrient to its primary fate. A. carbohydrate B. protein C. fat 37) used immediately for energy

Answer:A

Match the response with the type of chemical. A. sympathetic agonist B. parasympathetic agonist 50) pupil dilation

Answer:A

Match the structure to its function. A. mouth B. stomach C. rectum D. small intestine E. large intestine 51) Carbohydrate digestion begins here.

Answer:A

You want to display data on the finish times of the 10 fastest race horses in a single race at the Kentucky Derby. 31) What type of graph would be best to display this information? A) bar graph B) line graph C) scatter plot

Answer:A

156) Nutrients undergo the last stages of digestion by enzymes located on the cells of the small intestine; then the nutrients are absorbed by these same cells by way of various membrane transport processes. Adipose cells absorb and store excess food energy in the form of fat. You and the other students in the physiology lab you are taking are doing an analysis of cell membrane composition, on unknown animal tissue samples labeled A and B. All lab groups determined that sample A contained, on average, 81% protein, 18% lipid, and 1% carbohydrate. Sample B contained 85% lipid, 10% protein, and 5% carbohydrate. Sketch a graph of the class data. You now have to make a logical conclusion as to which sample is more likely to be intestine and which is adipose tissue. What do you conclude, and why?

Answer:A bar graph would be appropriate, as in the figure below. The presence of digestive enzymes and membrane transporters in small intestine cells indicate there should be a significant amount of protein present. Adipose cells, on the other hand, are relatively inactive and can passively absorb lipids by way of simple diffusion through the membrane phospholipids. Adipose tissue is expected, therefore, to consist primarily of lipids. Sample A is most likely small intestine, and sample B adipose.

154) Explain what a concentration gradient is and how it assists a cell in doing its work.

Answer:A concentration gradient is a difference in concentration of a particular molecule between the cytosol of a cell and the interstitial fluid. A concentration gradient establishes potential energy because of the tendency of molecules to diffuse from areas of high to areas of low concentration.

39) Why is it necessary to label the axes of a graph?

Answer:A graph with no axis labels is meaningless.without knowing what trend is being illustrated, there is no communication of scientific information.

72) Upon testing the knee-jerk reflex during a routine physical, a nurse notices that the reflex is hyperactive. What could be the explanation for this? If instead the reflex was hypoactive, what may the nurse conclude?

Answer:A hyperactive knee-jerk reflex suggests a lack of descending inhibition. This could indicate injury or disease in the spinal cord above the reflex arc, or in a brain area involved in motor control. A hypoactive reflex could indicate damage to any of the components of the local reflex arc, including the muscle, the spinal nerve, or that level of the spinal cord. Alternatively it is possible the patient is deliberately inhibiting the reflex by descending inhibition. Reflexes can also be adversely affected by metabolic disorders.

96) Describe a ketogenic diet. What are the pros and cons of such a diet? How is ketosis related to diabetes mellitus?

Answer:A ketogenic diet is low in carbohydrate and high in protein and fat. Ketone bodies are produced as fat is metabolized. Pros: rapid initial weight loss and caloric restriction. Cons: initial weight loss represents loss of glycogen and water, rather than fat; dehydration, vitamin and mineral deficiency; and kidney problems. Untreated diabetics cannot metabolize carbohydrates normally, so their diets are functionally ketogenic.

93) Carmen has just given birth to her third child, and as with the first two, she stayed home and used the services of a nurse-midwife. While her recovery from the births of the first two children was normal and she was able to breast feed both for two years, she is not lactating this time and is experiencing other difficulties. Her symptoms also include vomiting, fatigue, cold intolerance, and myxedema, which worsen over the coming weeks. She ignored her midwife's suggestion to see her doctor the day after giving birth, but has finally gone to an OB-gyn during the third week postpartum. Her newborn baby is healthy and thriving on human breast milk purchased from a milk bank. Diagnostic tests indicate decreased amounts of several hormones, as well as edema and poor blood flow into the pituitary. What do her symptoms indicate? Which hormones are likely decreased? What may be the root cause?

Answer:A lack of several pituitary hormones is apparent: PRH stimulates prolactin secretion and lactation, vomiting and fatigue could indicate lack of ACTH and consequent cortisol, and cold intolerance and myxedema indicate lack of TSH and consequent thyroid hormone. Abnormal function of the anterior pituitary can result from lack of hypothalamic-releasing hormones, pituitary adenoma, or pituitary infarction related to pregnancy (Sheehan syndrome; this syndrome is not described in the text, but the student may be able to reason decreased pituitary function due to decreased blood flow).

106) Mike has classical hemophilia, a clotting disorder, but his case is considered "mild." His levels of factor VIII are 28% of normal. He is usually okay going about the activities of normal daily living. He plays slow pitch softball and enjoys swimming and horseback riding, and is an exceptionally active individual who believes "why walk when you can run?" He has noticed that once he has one injury, he will have "bleeds" more readily, and has to restrict his own activities to give himself time to heal. Why would he have to "slow down" longer than a person without hemophilia?

Answer:A level of 28% suggests that Mike's body synthesizes factor VIII much more slowly than normal. Thus once he is injured, he depletes his limited supply more quickly and takes longer to replenish it.

104) In the disease mononucleosis ("mono"), the spleen enlarges because of increased numbers of cells, phagocytic as well as others. Common symptoms of this disease include pale complexion, a tired feeling, and a lack of energy sometimes to the point of not being able to get out of bed. What might cause these symptoms?

Answer:A major function of the spleen is to destroy old, defective, and worn-out red blood cells. As the spleen increases in size, so does its capacity to eliminate red blood cells, and this produces anemia. The decreased number of red blood cells decreases the blood's ability to deliver oxygen to the tissues and thus metabolism is slowed down. This would account for the tired feeling and lack of energy. Because there are fewer red blood cells than normal, the blood circulating through the skin is not as red, and so the person has a pale skin coloration.

42) The two varieties of adrenergic receptors are __________ and __________.

Answer:A lpha, beta 43) The signal molecule __________ elicits the stronger response from alpha receptors. (:0)Answer: norepinephrine 44) The signal molecule __________ elicits the stronger response from beta2 receptors. (:0)Answer: Epinephrine 45) __________ postganglionic sympathetic neurons secrete __________ onto their target cells. A) All, norepinephrine B) Most, norepinephrine C) All, acetylcholine D) Most, acetylcholine (:0)Answer: B

173) Months after the accident, your amputee uncle's phantom limb pain subsides. One afternoon he is sleeping and you see a fly walk across his upper lip. He wakes up and tells you he just had the funniest feeling. It was as if someone was touching his missing index finger. You decide to experiment with your uncle and you blindfold him. As you touch his cheek he tells you that you are touching his cheek and his missing thumb! As you move around his face you map out his fingers and forearm on his face. Hypothesize how this could be possible.

Answer:A map of the somatosensory cortex would show that the area corresponding to the phantom hand and forearm is adjacent to the area corresponding to the face. What has happened is the hand cortex has been invaded by the sensory fibers that normally only activate the face area of the cortex. Since the territory for the hand is vacated (not receiving any stimulation), those facial sensory fibers have begun to activate cells there causing the phantom limb sensation.

91) The crew of an ocean-going racing sailboat must adhere to strict weight limits for both their bodies and the food they eat, yet their activity levels are very high, and weather conditions may vary to extreme cold, so they must consume many kilocalories. What kinds of nutrient-dense foods can you suggest to them to bring? In general, what type of energy nutrient should they choose to consume?

Answer:A nutritionist's nightmare, this crew gets to eat foods everyone else is trying to avoid. Fat is high in kilocalories, but lightweight and doesn't occupy much space. Candy bars, granola with lots of nuts, and peanut butter are all good foods to have available, while vegetables, salads, and regular breakfast cereals are too bulky for the calories they provide.

37) What is the difference between a peer-reviewed article and a review article?

Answer:A peer-reviewed article describes original research by one author (or group of authors working together) that has gone through a screening process in which a panel of qualified scientists evaluate the work. A review article is a summary (usually a collection of published research that was previously peer-reviewed, usually from more than one independent lab) that discusses a particular topic in the field.

72) What is a permissive hormone effect? Provide an example, explaining what would result if the permissive hormone was absent.

Answer:A permissive hormone effect is the dependence of one hormone on the presence of another in order to fully exert its effects. An example is that cortisol is required for full glucagon effect. Without cortisol, glucagon cannot adequately perform its role in glucose regulation, and death results.

155) Explain what a portal system is and give an example. How is a portal system different from the typical arrangement of vessels?

Answer:A portal system is a special circulation in which a capillary bed connects to another capillary bed before returning to the general circulation. An example is the hepatic portal system, in which capillaries of the digestive tract are connected to capillaries of the liver by way of the hepatic portal vein. Typically blood in a capillary bed flows through the venous system and back to the heart.

179) What is a normal heart rate for healthy adults at rest, and what neural inputs contribute to the achievement of that range? How is heart rate affected when all nervous system input to the heart is blocked, and why is this effect seen?

Answer:A typical adult heart rate is 70 beats per minute. This normal resting value results from the constant parasympathetic inhibition of the higher endogenous rate of the SA node. Without neural input, resting heart rate is higher, matching the intrinsic rate of the SA node, 90-100 beats per minute.

102) Describe how a virus is eliminated in an immune response.

Answer:A virus in the plasma may be eliminated due to antibody action (see the "Antibodies Have Multiple Functions" section in the CHAPTER). Host cells containing viruses are killed by cytotoxic T cells, through the action of perforin, granzymes, and a "death receptor".all of which lead to apoptosis.

144) What are the advantages of using ATP as the energy-transferring molecule in a cell?

Answer:A TP easily acquires the energy stored in complex biomolecules such as carbohydrates; its bonds are readily broken and it releases just the right amount of energy for cell needs so that the cell is protected from excessive energy release.

109) Brian, 53, finally visited his doctor, after 6 months of being winded after climbing the two flights of stairs to his apartment. He kept blaming himself for being "out of shape," and thought he would be okay after his spring yard work started, but he wasn't. The doctor wasn't surprised when Brian's blood pressure was 160/110 mm Hg. The doctor prescribed some beta-blockers and more tests, including an ECG and blood tests. Why did the doctor want this additional information? What are some items he would check in the blood tests, and why?

Answer:A bnormalities on the ECG would reveal evidence of scarring or damage to the heart and the myocardium; blood tests would be checked for electrolyte levels, especially calcium, sodium, and potassium, as well as for changes associated with volume fluctuation. Heart enzymes would not be elevated unless damage was very recent.

132) You are studying the toxic effects of a newly discovered tropical plant extract. From preliminary studies, it appears that the toxin prevents H+-dependent membrane transport. Explain which digestive processes may thus be impaired.

Answer:A bsorption of some amino acids, dipeptides, and tripeptides may be impaired, as well as iron and sodium absorption, because these are dependent upon an H+ exchanger or cotransporter. HCl secretion into the stomach, which acidifies the lumen for protein digestion, may also be impaired.

37) The adaption of the body to repeated exercise in hot environments occurs through a process called __________.

Answer:A cclimatization 38) What two effects does sympathetic stimulation have on the heart during exercise? (:0)Answer: 1. increases contractility 2. increases heart rate 39) During exercise, skeletal muscle receives approximately __________% of blood flow. Where is this blood redistributed from? (:0)Answer: 88; this blood is redistributed from the brain, kidney, and GI mostly. See Fig. 25-7 in the CHAPTER.

110) Julia is breathing 14 times per minute, with a tidal volume of 520 mL and a dead space of 152 mL. Lyle is breathing 15 times per minute, with a tidal volume of 400 mL and a dead space of 175 ml. Which patient has better alveolar ventilation?

Answer:A lveolar ventilation = ventilation rate × (tidal volume - dead space volume). Julia's ventilation is 5432 mL/min, which is better than Lyle's, which is 3375 mL/min.

103) The process by which the eye adjusts the shape of the lens to keep objects in focus is known as __________.

Answer:A ccommodation 104) Retinal signal processing is modulated by input from cells that synapse with photoreceptors and bipolar cells, called __________, and with __________, which modulate the flow of information between bipolar and ganglion cells. (:0)Answer: horizontal cells, amacrine cells 105) In nearsightedness, the focal point falls in front of or behind (choose one) the retina. (:0)Answer: in front of 106) Receptive fields in the somatic sensory system are similar to __________ in the visual system. (:0)Answer: visual fields 107) As an object is moved closer to the eye, the lens must become more __________ in shape, in order to keep the object in focus. (:0)Answer: round 108) A molecule that blocks substance P from binding to its receptor would A) stimulate a painful sensation. B) inhibit the perception of pain. C) alter the perception of hot and cold stimuli. D) interfere with transduction from chemoreceptors. E) heighten the perception of touch. (:0)Answer: B

166) Acetylcholine is excitatory at the neuromuscular junction, but inhibitory at the SA node. Explain why this is true.

Answer:A cetylcholine receptors in heart muscle are muscarinic, whereas they are nicotinic in skeletal muscle. These receptors are linked to different types of ion channels. At the nicotinic receptor, the response is excitatory due to a sodium influx. At the muscarinic receptor, the response is inhibitory due to a potassium efflux.

127) Erica came down with the flu. She experienced severe vomiting, determined to be primarily stomach contents, for three days, and now is having chest pains. She is calling one of her symptoms "heartburn," but reported that it felt as if she had swallowed a small apple whole. Certain that she was having a heart attack, Erica rushed to the doctor. The doctor took her history and symptoms, and reassured her that she only had esophagitis. How did the physician conclude this? Is her blood pH higher or lower than normal?

Answer:A cid reflux irritated the esophageal lining, causing it to swell. Her pH is higher than normal due to loss of H+ from her stomach.

134) Chronic inflammation of the pancreas, or pancreatitis, impairs the digestive functions of this organ. What would be the effect on digestion and absorption of carbohydrates, fats, and proteins if acinar cells are impaired? What if duct cells are impaired? Which hormones may be used to treat pancreatitis?

Answer:A cinar cells secrete digestive enzymes for all three classes of molecule, therefore nutrient digestion and consequent absorption would be decreased. Duct cells secrete bicarbonate, which counteracts the acidic nature of the chyme entering from the stomach and allows the pancreatic enzymes to function. Thus loss of these cells would also decrease digestion and absorption of the three classes of nutrient. Cholecystokinin stimulates pancreatic enzyme secretion and secretin stimulates bicarbonate secretion.

92) Precocious puberty in girls can result from hypersecretion of nongonadal steroids. In such girls, there is early breast development, onset of menses, and ovulation of viable eggs. This can occur by age 7 or even earlier. Which hormone(s) and nongonadal endocrine gland(s) may be responsible? Can the responsible gland(s) be safely removed? Explain. How might this condition be treated without surgery? What permanent nonsexual consequences may result from not treating this condition?

Answer:A drenal sex hormones include estrogens. Hypersecretion of estrogen would stimulate puberty. The adrenal glands can be removed, if cortisol and aldosterone are replaced. Drugs such as aromatase inhibitors, which block estrogen production, may be appropriate. An effect of estrogen is stimulation then cessation of skeletal growth, which after precocious puberty could result in shorter final stature due to premature closure of growth plates in long bones.

141) Describe and distinguish between the techniques of X-ray, CT scan, PET scan, MRI scan, and ultrasound.

Answer:A ll of these techniques exploit differences in density or activity of tissues, which alter the way in which electromagnetic or sound waves are transmitted, absorbed, or reflected. The data are converted to an image, resulting in a picture of the internal organs, in which areas differ in appearance based on differences in density or activity. These techniques can be used to visualize tumors, blood flow, metabolic activity, damaged organs, gall or kidney stones, fetuses, and so on. X-rays are electromagnetic radiations, which penetrate tissues to varying degrees based on their density. A photographic film is placed behind the body, producing an image. CT (cat) scans are a series of X-rays that are analyzed by a computer instead of exposing film; CT scans provide images in thin planes or slices, for more precise localization of dense areas. PET scans utilize positrons emitted from radioisotopes, which computers can analyze to provide information about metabolic activity and blood flow. In an MRI (NMR or nuclear magnetic resonance) scan, a magnetic field is used to alter the energy level of the nuclei of hydrogen atoms, which emit radiation when they return to their normal state; a computer analyzes the data, which are of relatively high contrast. Ultrasound utilizes ultrasonic (high frequency sound) waves, which produce echoes that vary with tissue density, and are analyzed by computer; this technique is especially useful for observing fetal development because it does not risk harming the fetus with radiation. None of these techniques is as informative as actually visualizing the area of interest by performing surgery, but they are noninvasive.

60) Describe the different types of adrenergic receptors in the autonomic nervous system. Which neurotransmitter binds to each type? For those that bind more than one neurotransmitter, how does the response to the neurotransmitters compare?

Answer:A lpha 1, alpha 2, beta 1, beta 2, and beta 3 receptors are described in Table 11-2. Most of these receptors are more sensitive to norepinephrine than to epinephrine, but they will all respond to either neurotransmitter. These neurotransmitters are nearly identical structurally, and the response of the target tissue to the binding of either is the same.

60) Explain alpha-gamma coactivation.

Answer:A lpha motor neurons control the contraction of skeletal muscle fibers. Gamma motor neurons adjust the stretch sensitivity of the muscle spindle, so that the spindle is active even when the muscle shortens. Coactivation of both sets of fibers causes the tension on the muscle spindles to be maintained as the muscle shortens; thus sensitivity to stretch is maintained.

39) Distinguish between alpha, beta, and gamma radiation.

Answer:A lpha radiation is composed of fast-moving protons and neutrons. Beta radiation is composed of fast-moving electrons. Both of these types are emitted by unstable atoms. Gamma radiation is composed of high-energy waves, not particles.

111) What nutrients are required for red blood cell synthesis and why? What is blood doping? How is it detected? How is blood doping different from simply good nutrition?

Answer:A mino acids for globin synthesis, iron for heme synthesis, folic acid and vitamin B12 for DNA synthesis are among the nutrients required. Blood doping is using a treatment to increase the oxygen-carrying capacity of blood, such as receiving a transfusion of red blood cells or administering hormones that stimulate red blood cell production. Hemoglobin concentration in the blood is elevated above normal in dopers, as is hematocrit. Reticulocyte count indicates an accelerated rate of red blood cell production, which would be expected with use of erythropoietin or similar drugs. These drugs/hormones can be detected in the blood. Good nutrition should ensure that hematocrit and hemoglobin are at their maximum, but nutrition is not sufficient to elevate these parameters above normal.

178) Amy is a premed student with a summer job as an animal research technician. One of her duties is to withdraw blood by syringe from the anterior vena cava of pigs, to be analyzed for various hormones. During her training she noticed that most blood samples were very dark in color, but occasionally a sample was bright red. Propose an explanation.

Answer:A my misdirected her needle to a nearby artery instead of withdrawing blood from the vena cava. Arterial blood is more oxygenated than venous blood, thus not as dark.

142) Distinguish between adequate stimulus, threshold stimulus, and perceptual threshold.

Answer:A n adequate stimulus is the form of energy to which a receptor is most responsive. A threshold stimulus is the minimum depolarization required to trigger an action potential in a sensory neuron. Perceptual threshold is the minimum stimulus required for a person to be aware that peripheral receptors were stimulated.

49) Explain the advantages and disadvantages of glycolytic metabolism.

Answer:A n advantage to glycolytic metabolism is that it is anaerobic; that is, ATP production can continue in the absence of oxygen. Another advantage is that ATP production by this method is very rapid. Disadvantages include low ATP yield per molecule of glucose and metabolic acidosis from buildup of lactic acid.

98) Explain each of these terms: antibiotic, antibody, antigen, antihistamine, antiretroviral drug, antiviral drug.

Answer:A n antibiotic is a drug that kills bacteria. Antibodies are proteins produced by lymphocytes that help lead to destruction of invading pathogens. Antigens are molecules that stimulate an immune response. Antihistamines are drugs that block secretion of histamine from immune cells. Antiretroviral and antiviral drugs are drugs that destroy specific kinds of viruses.

43) You conduct an experiment on twenty 18-year-old male subjects to see how various levels of exercise influence heart rate. Explain why only 18-year-old males were used as subjects.

Answer:A n important part of scientific inquiry is to remove sources of variation from among subjects. By choosing subjects of one gender in a particular age group, it is easier to determine that the dependent variable (heart rate, in this case) depends only on the independent variable, level of exercise. This also allows a study to have fewer participants, assuming that subjects were randomly assigned to a level of exercise. If subjects were of random ages and genders, data would have to be collected from many more individuals.

99) Describe anorexia nervosa and diabetes mellitus, and how they resemble each other.

Answer:A norexia nervosa is lack of adequate caloric intake due to distorted body image. Diabetes mellitus results from lack of insulin and consequent inability to utilize glucose. Both diseases result in starvation if left untreated.

169) It is autumn, and the fall colors are at their best. An orange, black, and tan cat is sitting quietly at the edge of your yard, watching you. You don't see her until she decides to run for cover, even though she was only 20 feet away. With respect to visual receptors, why don't you see her until she runs? What parts of your brain and visual system work together to determine what this moving object is, and whether or not it will harm you?

Answer:A nswers should describe the differences in rods and cones and the "target" shaped visual areas near the fovea. Once movement is perceived, the head is swiveled and the intrinsic and extrinsic eye muscles go to work to focus the image on the fovea, and send the information to the visual cortex in the occipital lobe, where memories of cats and their appearance is housed. One's limbic system then may assess if there is danger if one has had a previous bad experience with a cat (most students wouldn't follow this line of logic much further, if at all).

85) Compare and contrast insulin and glucagon.

Answer:A nswers will vary but should include information in Tables 22-3 and 22-5 in the CHAPTER.

63) Table 1-3 shows data on various factors that may or may not be related to resting pulse rate, time to recovery to resting pulse rate after a few minutes of exercise, and reaction time measured by how quickly a student could press a keyboard key after seeing a computer-generated prompt. For each question below, write a testable hypothesis, identify the dependent and independent variables, sketch an appropriate graph of the results, and draw a conclusion from the data presented in the table. Discuss your results. A. Does caffeine consumption have an effect on resting pulse rate? B. Does age play a role in resting pulse rate? Does weight? C. Is there a relationship between eating breakfast and recovery time? D. Is there a relationship between reaction time and height? E. Do women who smoke show differences in their resting pulse rates compared to female nonsmokers or to male smokers and nonsmokers? F. Does regular exercise have an effect on resting pulse rate?

Answer:A nswers will vary, but examples follow (conclusions written here are based on cursory examination of graphed data.no statistical tests of significance were performed). A. Hypothesis: Caffeine consumption increases heart rate. Independent variable: caffeine consumption. Dependent variable: resting pulse rate. Conclusion: Mean pulse rates between caffeine-drinking (68 bpm) and control subjects (73 bpm) are similar; hypothesis rejected. B. Hypothesis: Pulse rate is lower in older people and is higher in heavier people. Independent variables: age and weight. Dependent variables: resting pulse rate. Conclusion: Pulse rate was similar in all groups; hypothesis rejected. C. Hypothesis: People who ate breakfast have a faster reaction time. Independent variable: breakfast consumption. Dependent variable: pulse rate. Conclusion: People who ate breakfast had a faster reaction time (168.7 msec vs. 180.5 msec); hypothesis supported. D. Hypothesis: There is no relationship between height and reaction time. Independent variable: height. Dependent variable: reaction time. Conclusion: Reaction time did not vary with height; hypothesis supported. E. Hypothesis: Smokers of both genders have a higher resting pulse rate than nonsmokers of either gender, and males and females are affected equally. Independent variables: smoking and gender. Dependent variable: pulse rate. Conclusion: There was no difference in pulse rate in any of the groups (70.4 bpm in nonsmokers vs. 70.3 bpm in smokers); hypothesis rejected. F. Hypothesis: People who exercise regularly have a lower resting pulse rate. Independent variable: exercise. Dependent variable: pulse rate. Conclusion: Regular exercise had no effect on resting pulse rate (68.9 bpm in nonexercisers vs. 71.8 bpm in exercisers); hypothesis rejected. Discussion may cover issues such as the effect of small sample size, use of adults of limited age range, lack of control over treatments (were the subjects honest about age, eating breakfast, consuming caffeine, smoking, and exercising?; were the quantitative data of height and weight determined in the lab using the same equipment and same data collector?), the value of statistical analysis, and so on. It is likely that students will be surprised by some of the results and could make erroneous conclusions. For example, pulse rate may vary with age, but without including children and senior citizens in the sample population, this trend would be missed.

130) What would be the nutritional consequence of excessive antacid use?

Answer:A ntacids make the stomach pH higher than normal, which will interfere with protein digestion because the enzymes would not be as efficient.

79) ________ is a hormone secreted by both the Sertoli cells of the testes and the granulosa cells of the ovaries.

Answer:A nti-Mullerian hormone 80) The peptide __________ may play a role in the positive feedback of estrogen on GnRH secretion. (:0)Answer: kisspeptin 81) Identify the FALSE statement. A) Androgens control the development of reproductive organs in the male fetus. B) Estrogens control the development of the reproductive organs in the female fetus. C) Females have one less active chromosome than males. D) Gametogenesis begins in the female fetus. E) Meiosis is never completed in most oocytes. (:0)Answer: B

101) Describe the various effects of antibodies in an immune response.

Answer:A ntibodies enhance phagocytosis through clumping and opsonization, neutralize bacterial toxins, activate complement, mediate inflammation, and activate immune cells.

54) List three ways that neural reflex pathways can be classified. Name one of the reflexes in each category. What additional information may be gained from the name of the reflex? Name a specific reflex and identify its classification in all of the categories you listed.

Answer:A ny three of the following four (:0)Answers are correct classifications. Further, the effector or action is sometimes evident by the reflex name, e.g., flexor reflex. (:0)Answers to specific reflexes will vary, e.g., knee-jerk reflex is somatic, spinal, innate, and monosynaptic. 1. by the efferent division of the nervous system that controls the response: somatic reflexes and autonomic reflexes 2. by the CNS location where the reflex is integrated: spinal reflexes and cranial reflexes 3. by whether the reflex is innate or learned: innate reflexes and learned or conditioned reflexes 4. by the number of neurons in the reflex pathway: monosynaptic reflexes and polysynaptic reflexes

96) The inferior point of the heart is called the __________.

Answer:A pex 97) The __________ is a wall that separates the two sides of the heart. (:0)Answer: septum 98) The muscle layer of the heart wall is the __________. (:0)Answer: myocardium 99) The cells responsible for establishing the rate of a cardiac contraction are the __________. (:0)Answer: nodal cells 100) Abnormal patterns of cardiac activity are known as __________. (:0)Answer:A rrhythmias 101) The __________ is the amount of blood in a ventricle at the beginning of systole. (:0)Answer: End-diastolic volume 102) The __________ is the amount of blood in a ventricle after it has contracted and before it begins to refill. (:0)Answer: End-systolic volume 103) The amount of blood returning to the heart is the __________. (:0)Answer: venous return 104) The term for reduced blood flow to the cardiac muscle is __________. (:0)Answer: Coronary ischemia 105) The primary function of the cardiovascular system is __________. (:0)Answer: to transport material to and from all parts of the body 106) The cardiovascular system is made up of __________, __________, and __________. (:0)Answer: the heart, blood vessels, blood ((:0)Answers can be in any order.)

49) Blood pressure is highest in the __________ and decreases continuously as blood flows through the circulatory system, due to __________.

Answer:A rteries, the energy lost as a result of the resistance to flow offered by the vessels. 50) The rapid pressure increase that occurs when the left ventricle pushes blood into the aorta can be felt as __________. (:0)Answer: pulse 51) Venous return to the heart is aided by __________ and the __________. (:0)Answer: the skeletal muscle pump, respiratory pump 52) A rupture of a blood vessel in the brain is called __________ and may cause the loss of neurological function commonly called __________. (:0)Answer:A cerebral hemorrhage, a stroke 53) Mean arterial pressure is (indirectly/directly) proportional to __________ and (indirectly/directly) proportional to __________. (:0)Answer: Directly, cardiac output, directly, resistance of arterioles 54) __________ is an increase in blood flow that accompanies an increase in metabolic activity. (:0)Answer:A ctive hyperemia 55) Norepinephrine binding to alpha receptors on vascular smooth muscle causes __________ and epinephrine binding to alpha receptors on vascular smooth muscle causes __________. (:0)Answer: vasoconstriction, vasoconstriction

110) Siegfried experiences a hemorrhage in an artery, and now blood is leaking out of the resulting hole. His blood pressure decreases. Draw a flowchart illustrating the cardiovascular reflex responses that follow, to compensate for the drop in blood pressure.

Answer:A s blood exits the vasculature, there is a loss in blood volume, which in turn causes the systemic blood pressure to decrease. Baroreceptors (stretch receptors) in the carotid and artery walls are less stretched, and therefore their action potential firing frequency decreases. The baroreceptors project to the cardiovascular control center in the medulla oblongata. In response to decreased baroreceptor input, the cardiovascular control center stimulates sympathetic output and inhibits parasympathetic output. Sympathetic activity causes vasoconstriction and consequent increases in peripheral resistance and blood pressure. Decreased parasympathetic activity causes the SA node firing rate to increase and therefore heart rate increases. Combined with the sympathetic stimulation of the ventricles and consequent increase in contraction force, cardiac output increases as a result, also helping to increase blood pressure.

73) Fred suffers from chronic emphysema. Blood tests show that his pH is normal but his bicarbonate levels are elevated significantly. How can this be?

Answer:A s long as the ratio of bicarbonate ion to carbonic acid is 20:1, the pH of body fluids will remain normal. Since Fred's condition is chronic (long term) his body has compensated for the excess carbonic acid (the result of hypercapnea due to poor ventilation) by increasing the amount of bicarbonate to match the elevated level of acid. This process involves the kidneys where some of the excess carbon dioxide is converted into carbonic acid and the carbonic acid is allowed to dissociate. The hydrogen ions are secreted and the newly formed bicarbonate is conserved to maintain a proper buffering capacity.

89) Todd is 13 and lives in an urban apartment. He spends most of his time watching TV and eating "junk food" (low in nutrition, high in calories). One afternoon during gym class, he falls on the playground and breaks his leg. Although he appears to be healthy, his leg takes longer to heal than expected. What might be the cause of the longer healing time?

Answer:A ssuming that there is no other disease process involved, Todd's problem is probably related to his poor diet. Good nutrition is important for promoting the healing process. Insufficient quantities of protein, vitamin C, vitamin A, and vitamin D3 would cause the normal healing process to occur at a much slower rate. Todd's inactivity may also contribute to his slower rate of healing.

5) A bomb calorimeter measures A) the carbon dioxide produced when a food sample is completely combusted. B) the oxygen consumed when a food sample is completely combusted. C) only the food calories available to the human body. D) the heat produced when a food sample is completely combusted. E) two of the

Answers above. (:0)Answer: D

35) In muscles used for fine actions, such as controlling eye movement or use of the hand, a motor unit will have __________ muscle fibers when compared to a motor unit in muscles used for power and strength. A) hundreds more B) very few C) approximately the same numbers of D) Any of these

Answers could be correct, as this is impossible to predict. (:0)Answer: B

Match the correct subatomic particle with the statement about it.

Answers may be used once, more than once, or not at all. A. proton(s) B. neutron(s) C. electron(s) 40) An ion has gained or lost __________. (:0)Answer: C

68) Anne Frank wrote a diary about the years she and a few others spent in hiding from the Nazis during World War II. These people lived in the upstairs rooms of a shop, accessible only by a hidden stairway; Anne called these quarters the Secret Annex. One day Nazis raided the shop below, but were unsuccessful in locating the hideaways or finding proof of their presence during this particular raid. Clearly hearing what was occurring downstairs, Anne and her cohorts cowered in silence, for they feared being transported to "death camps." Later, Anne wrote that most of the residents of the Secret Annex experienced diarrhea shortly after this close call. Explain this response by their digestive systems.

Answer:A ssuming there is no pathogenic or dietary cause, diarrhea probably resulted from the fight-or-flight response of the sympathetic nervous system. Digestive tract secretion and motility are inhibited during a sympathetic response. When the threat had passed and the sympathetic nervous system released the digestive targets from inhibition, there may have been a rebound effect in which the parasympathetic division overstimulated the motility of the tract. Increased motility with decreased time for reabsorption of water can result in diarrhea. It is also thought that the sympathetic response actually stimulates the lower gastrointestinal tract though it inhibits the upper tract. This could also account for the effects. Students may need to consult other sources to (:0)Answer this question correctly.

100) A dead human body has been discovered in the woods near your home. No, you are not a homicide suspect; rather you are an expert hematologist, and the body is still warm. Implausibly, this adult corpse has no gender cues (no sexual organs, and skeletal features that are in between those expected for males or females). How can you assist the detectives in determining the gender of this person, using the hematology lab in your garage?

Answer:A ssuming you can collect and centrifuge the blood soon, prior to clotting and decay, a hematocrit may provide clues. While a differential white cell count is independent of gender, hematocrit, hemoglobin concentration, and red cell count are all slightly lower in females.

113) Hormone replacement therapy is often used to treat symptoms of menopause. Typically, estrogen and progesterone are used rather than GnRH, FSH, and LH. Can you explain why?

Answer:A t menopause, the ovaries are no longer functioning normally and thus do not respond to GnRH, FSH, or LH. Since these hormones are no longer available, direct regulation of estrogen and progesterone can be achieved through supplemental hormones.

75) Mr. Gregory comes to the doctor with high blood pressure. Tests show that he also has elevated levels of renin in his blood and atherosclerotic plaques that have nearly blocked blood flow through his renal arteries. Mr. Gregory is puzzled. Explain to him how decreased blood flow in his renal arteries could cause renin secretion to increase. Map the pathways through which elevated renin causes high blood pressure for Mr. Gregory.

Answer:A therosclerotic plaques block blood flow, which decreases GFR and decreases pressure in the afferent arteriole. These are both stimuli for renin release. Renin secretion starts a cascade that produces angiotensin II, a vasoconstrictor. Vasoconstriction increases blood pressure, and the medullary control center responds to Ag II by also increasing ADH, aldosterone secretion, and thirst, collectively increasing blood pressure even further.

92) Damien, in training for the Boston marathon, finds his plasma cholesterol concentration is somewhat elevated. What other test results should Damien review before changing his training regime or diet?

Answer:A thletes often have elevated cholesterol levels due to their heavy training, since that is how fats travel throughout the body in order to be mobilized and used. Better numbers to depend on would be his HDL levels and the ratio of HDLs to LDLs.

76) The smallest unit of an element is a(n)__________.

Answer:A tom 77) When two or more atoms are chemically linked, the smallest unit of the resulting material is referred to as a(n) __________. (:0)Answer: molecule 78) A(n) __________ is a substance that consists entirely of atoms with the same atomic number. (:0)Answer: Element 79) The center of an atom is called the __________. (:0)Answer: nucleus 80) Electrons travel around the center of the atom at high speed forming a(n) __________. (:0)Answer: shell (or orbital) 81) A combination of two or more atoms that has physical and chemical properties that differ from the atoms that compose it is called a(n) __________. (:0)Answer: Compound 82) Ions with a positive charge are called __________. (:0)Answer: Cations 83) Ions with a negative charge are called __________. (:0)Answer:A nions 84) A(n) __________ is a homogeneous mixture containing a solvent and a solute. (:0)Answer: solution

62) Which tissues contain both cholinergic and adrenergic receptors, and how does this relate to their autonomic control?

Answer:A list is provided in Fig. 11-5 in the CHAPTER. For most target organs listed, acetylcholine and (nor)epinephrine have opposite effects on the target organ.

103) Baroreceptors have stretch-sensitive ion channels in their cell membrane. Increased pressure on the membrane opens the channels and allows ion flow that initiates action potentials. What kind of ion probably flows through these channels and in which direction?

Answer:Action potentials are created when a cell depolarizes to threshold. Depolarization results from net influx of positive ions or efflux of negative ions. The most likely ion is Na+, moving into the receptor cell.

109) Joey is 6 months old and requires a blood transfusion; his blood type is B. What type of blood may he receive? Is his age a factor in your

Answer? What would happen if he received a "wrong" blood type? If Joey were younger, is it possible that there is no "wrong" blood type? Why may it surprise a beginning physiology student, familiar with the basics of immunology, that a person could mount an immediate immune response to transfused blood? Explain all of your (:0)Answers. (:0)Answer: Joey may receive type B or type O blood. Type O lacks antigens to stimulate an immune response, and type B blood will not trigger an immune response in a type B person. It is possible that if Joey were newborn, he could receive any type, because he may not yet have antibodies to foreign blood, however this would not be a reasonable risk to take. The transfusion of incompatible blood would result in a humoral immune attack on the transfused blood, and this could be fatal. A beginning student may logically believe that there are no antibodies to blood antigens in the blood of a person who has never received a transfusion. Antibodies develop in the first few months of life, however, because of the presence of similar antigens on gut bacteria and food.

Match the type of cell junction with the best description.

Answers may be used once, more than once, or not at all. A. gap junction B. tight junction C. anchoring junction 68) simplest cell-cell junction (:0)Answer:A

42) Estimates of the maximum possible ATP yield for one molecule of glucose total either 30 or 32 ATP. Why are there two different results? A) Sometimes the electron transport system is more efficient than other times. B) The NADH molecules produced during glycolysis are in the cytoplasm; occasionally, electrons are carried by the lower energy FADH2 instead of NADH. C) Individual variation is normal among cells. D) Any of these

Answers may work under different cellular conditions. (:0)Answer: B

102) A newborn infant is found dead, abandoned by the road. Among the many questions that the police would like to have

Answers to is whether the infant was born dead or alive. After an autopsy, the medical examiner tells them that the infant was dead at birth. How could the medical examiner determine this? (:0)Answer: Unless the infant was suffocated immediately at birth, the first breath that it took would start to inflate the lungs and some of the air would be trapped in the lungs. By placing the lungs in water to see if they float or not, the medical examiner can determine whether or not there is any air in the lungs. Other measurements and tests could also be used to determine if the infant had breathed at all (air in the lungs) or was dead at birth (lungs collapsed and a small amount of fluid in the alveoli).

Match the

Answers to the questions. A. true only for the sympathetic division B. true only for the parasympathetic division C. true for both divisions 23) The neural pathway from the spinal cord to the target tissue has two neurons, the preganglionic neuron and the postganglionic neuron. (:0)Answer: C

110) The HIV blood test detects anti-HIV antibodies. Why do viruses stimulate antibody production? How effective are antibodies against viruses, as compared to bacteria? How are viral infections normally eliminated in the body? Why does a person's immune response fail to eliminate life-long viral infections such as HIV and herpes? Explain all of your

Answers. (:0)Answer: Viruses have antigens, thus they stimulate the humoral immune response as well as the cell-mediated response. Antibodies are less effective against viruses because viruses "hide" inside cells, and antibodies do not enter cells. Viral infections can be eliminated by the cell- mediated response, which destroys virus-infected self cells. Life-long virus infections are possible because some viruses persist inside of cells without altering them in a way that stimulates immune attack.

Following is a table of data collected from one section of an 8 a.m. physiology lab. There were 20 students present, 10 men and 10 women. Information collected from the students included their height, weight, age, and resting pulse rate. In addition, the students were surveyed to see if they smoked cigarettes, considered themselves "regular exercisers," if they had consumed caffeine the morning of the lab, and if they had eaten breakfast that day. A "y" or "n" (yes or no) was recorded to indicate their

Answers. Each student did "jumping jacks" for 5 minutes and recorded the time required to regain their resting heart rate, which is listed on the table as "recovery time." Finally, each student participated in an exercise designed to measure their reaction time (in milliseconds) in catching an object dropped by a lab partner according to specified criteria. Use this table to (:0)Answer the following questions. Ignore statistical problems caused by small sample size, and so on. Table 1-3

191) You have two tubes: Tube A and Tube B. Tube A has 110 mm Hg at one end and 80 mm Hg at the opposite end. Tube B has 55 mm Hg at one end and 25 mm Hg at the other end. A. If you pour the same liquid through both tubes, which tube will have the greatest fluid flow? Explain. B. If you pour an 8 oz cup of a milk shake through Tube A and an 8 oz glass of water through Tube B, which will flow faster? Explain. C. What parameters in Tube A would you change to make flow equal in (A.)? D. What parameters in Tube A would you change to make flow equal in (B.)? Defend your

Answers. Show calculations where appropriate. (:0)Answer: A. There would be no difference in flow because the pressure difference in Tube A and Tube B is the same (30 mm Hg). B. Water through Tube B because water has a lower viscosity than a milk shake. Resistance is directly proportional to viscosity. Flow is inversely proportional to resistance. Therefore, decreased viscosity = decreased resistance = increased flow. C. No changes because flow is equal in (A.). D. You can change the parameters that would reduce resistance to flow. You could make Tube A shorter or increase its radius. Changing the radius would have a greater impact on increasing flow (Poiseuille's Law).


Set pelajaran terkait

MGMT 3000 Exam 2 - Chapter 9 Slides Review

View Set

Steps for Physical Assessment Study Guide

View Set

Prep U: Assessment of Digestive & Gastrointestinal Function

View Set